You are on page 1of 373

2021 托福阅读真题

The Famine of the Early Fourteenth Century in Northern Europe..................................................................................3


Nesting Challenges of the Great Bustard........................................................................................................................7
Water and Life on Mars.................................................................................................................................................11
The Roots of Economic Transformation in England.....................................................................................................15
Chondrites......................................................................................................................................................................19
The Role of Dingoes in the Australian Ecology............................................................................................................23
Australian Thunderbirds................................................................................................................................................27
Bicknell's Thrush...........................................................................................................................................................31
The Postwar Economic Boom......................................................................................................................................35
Hidden Life in Antarctic Nematodos............................................................................................................................39
The Impact of Farming..................................................................................................................................................43
The Domestication of Dogs...........................................................................................................................................47
The Sound Revolution in American Film......................................................................................................................51
The Global Industrial Revolution..................................................................................................................................55
Grasslands as Agents of Transformation.......................................................................................................................59
Did Sauropods Live in Swamps....................................................................................................................................63
Evidence of Zooxanthellae in Fossilized Corals...........................................................................................................66
The Economics of Academic Tenure.............................................................................................................................71
Abandoning Hunting and Gathering.............................................................................................................................75
Early Iron Metallurgy....................................................................................................................................................79
Examining the Problem of Bycatch...............................................................................................................................83
Naturalism and Nature in Art........................................................................................................................................87
Northwest Coast Art......................................................................................................................................................91
Guam and the Brown Tree Snake..................................................................................................................................95
The Chaco Phenomenon...............................................................................................................................................99
Challenge of Dendrochronology.................................................................................................................................103
The Process of Domestication.....................................................................................................................................107
Ancient Mapmaking....................................................................................................................................................111
Agriculture and Religion.............................................................................................................................................115
Temperate Plant Phenology.........................................................................................................................................119
The River Nile in Ancient Egypt.................................................................................................................................123
Martian Volcanoes.......................................................................................................................................................127
A Mutualistic Fungus of Tall Fescue Grass.................................................................................................................131
The Nile River Valley..................................................................................................................................................135
Glacier Flow and Surging............................................................................................................................................139
Defenses of the Eleodes Beetle...................................................................................................................................143

第 1 页 共 373

2021 托福阅读真题

Habitable Planets.........................................................................................................................................................147
European Cities and the Rise of Commerce...............................................................................................................151
Isolation and diversification in the Tropical Rain Forests...........................................................................................155
Programming Computers to Play Games....................................................................................................................159
Increasing Jellyfish Populations..................................................................................................................................163
The Rise of the Maya..................................................................................................................................................167
The Agricultural Revolution.......................................................................................................................................171
In the Darkest Depths..................................................................................................................................................175
Early Civilization........................................................................................................................................................179
Cotton Ginning and Interchangeable Parts: The Legacy of Eli Whitney....................................................................183
The Advent of Printing...............................................................................................................................................187
The Fur Trade and Native Americans.........................................................................................................................192
Jupiter's Moon Io........................................................................................................................................................196
Recognizing Social Play in Animals...........................................................................................................................200
The Most Common Bird on Earth...............................................................................................................................204
The Beginnings and Development of Agriculture in Egypt........................................................................................208
The Radiocarbon Dating of Prehistoric American Sites.............................................................................................212
The Rise of Diamonds in Europe................................................................................................................................216
How Trout Survive in winter......................................................................................................................................220
Changes in the Art Market During the Late Nineteenth Century................................................................................224
Agriculture and the Classic Maya Collapse................................................................................................................228
Flamingo Behavior......................................................................................................................................................232
Turtle Basking.............................................................................................................................................................236
British Agriculture.......................................................................................................................................................240
Evidence for the First Domesticated Rice in China....................................................................................................244
Machines and Manufacturing.....................................................................................................................................248
Fish and the Sense of Smell....................................................................................................................................... 252
Plant Senses.................................................................................................................................................................256
Understanding Insects through Fossils........................................................................................................................260
The Cooperative Nesting of Fairy-wrens....................................................................................................................264
Prairie Fires.................................................................................................................................................................268
Costs and Benefits of Dispersal...................................................................................................................................272
Adaptations of Marine Mammals................................................................................................................................276
The Sistine Ceiling......................................................................................................................................................280
Wool Manufacturing in Europe in the High Middle Ages..........................................................................................284
Silver in North America in the Late Eighteenth Century............................................................................................288

第 2 页 共 373

2021 托福阅读真题

Building a Modern Japan............................................................................................................................................292


Canals and Railroads in the Early Nineteenth Century...............................................................................................296
Birdsong......................................................................................................................................................................300
The origin of life on Earth..........................................................................................................................................304
American Cultural Independence................................................................................................................................308
Shift in Power Relations..............................................................................................................................................312
How Humans First Arrived in North America............................................................................................................316
Insect Colony Movement.............................................................................................................................................320
Competition for Life....................................................................................................................................................324
Hermit Crabs in Snail Shell.........................................................................................................................................328
Forest Management.....................................................................................................................................................332
Grains, Cities, States, and Armies...............................................................................................................................336
Science and Nineteenth-Century Visual Arts..............................................................................................................340
Determining the Source of a Meteorite.......................................................................................................................344
Songbirds....................................................................................................................................................................348
Dead Zones in the Sea................................................................................................................................................352
Grinding Grain............................................................................................................................................................356
Origins of Earth's Salty Oceans..................................................................................................................................360
Characteristics of Sixteenth-Century European Towns..............................................................................................364
The Meaning of Upper Paleolithic Art.......................................................................................................................368

第 3 页 共 373

2021 托福阅读真题

The Famine of the Early Fourteenth Century in Northern Europe

Paragraph 1: In the spring of 1315, a famine began to affect northern Europe that was more devastating than
anything that had occurred over the preceding millennium. The catastrophe began with spring rains that seemed
never to stop, continuing all through the summer and well into autumn, only to be followed, so far as one can tell-the
evidence for weather patterns comes from contemporary chronicles and from tree- ring studies- by one of the worst
winters in the history of the Middle Ages (approximately 400-1400 C.E.). The year 1316 was even worse: 150 days
of uninterrupted rain. Conditions continued to be bad until the end of 1322, with continuously wet summers
succeeded by vicious, persistent winters only to be followed alternately by other abnormally wet summers or,
equally bad, abnormally dry ones.

1. According to paragraph 1, which of the following best describes the climate in northern Europe during the period
between 1316 and 1322?
A. Each year, the winter became more vicious and persistent than the previous winter had been
B. Summers were unusually wet or dry and winters were often very harsh.
C. Summers were abnormally ware and winters were abnormally cold.
D. Rain that usually fell during winter came instead during the spring, summer, and autumn.

Paragraph 2: The immediate consequence was a crisis in the production of crops and, soon after. of work and food
animals. The problems with production were compounded by a series of wars throughout the British Isles, northern
Germany, Scandinavia, and northern France and Flanders--all accompanied by piracy on the seas--that initiated a
prolonged distribution crisis. The effect on the production of all major cereals--wheat, barley, oats, and rye--as well
as or rapes and other crops was nothing short of catastrophic. Selective statistical material indicates a decline in grain
production in parts of northern Europe of about one-third during the years of the disastrous weather cycle.
Considering the population of northern Europe was at its medieval peak on the eve of the ecological catastrophe
caused by the weather from 1315 to 1322, such a decline in crop yields put roughly 30 million inhabitants of
northern Europe at considerable risk of malnutrition.

2. According to paragraph 2 , which of the following was true of northern Europe just before the beginning of the
disastrous weather cycle?
A. It produced significantly more major cereals and other crops than its people needed.
B. It was experiencing a prolonged period of peace.
C. Its population was greater than at any other time during the Middle Ages
D. Roughly 30 million of its people were involved in farming

Paragraph 3: The estimated decline of crop yields by one-third is an aggregate figure the effects of the weather on
various grains differed. Some could tolerate soggy soils better than dry soil; for others, the reverse was the case.

第 4 页 共 373

2021 托福阅读真题

Furthermore, effects varied over regions as well Nevertheless, nearly everywhere one looks, the combination of
weather extremes and the consequences for human labor (it is very hard to plow sodden fields and equally hard to
plow very dry ones) reduced yields of grain by anywhere from 12 percent to 100 percent. There were farms where
peasants sowed 100 bushels of seed corn at the beginning of the growing season and harvested less than that at
the end.

3. Why does the author state that "There were farms where peasants sowed 100 bushels of seed corn at the
beginning of the growing season and harvested less than that at the end"?
A. To compare the crop yield at the beginning of the famine with that at the end of the famine
B. To show how difficult it was for peasants to plow very wet fields
C. To support the claim that extreme weather did more damage to some crops than to others
D. To illustrate how much the extreme weather affected some grain yields

Paragraph 4: The ceaseless wet weather of the early years of the Great Famine corrupted the grape harvest to a
similar degree. Grapes rotted on the vine or became diseased from the downy mildews associated with excessive
dampness. Again, the statistical evidence points to grape production being roughly 80 percent below normal yields
during the famine years. And contemporaries' comments about the small quantities and awful quality of the wine
produced from these diseased and underdeveloped grapes are recorded in numerous texts.

4. The word "ceaseless" in the passage is closest in meaning to


A. unusual
B. severe
C. unending
D. damaging

5. The word "excessive" in the passage is closest in meaning to


A. frequent
B. unhealthy
C. too much
D. unsuitable

6. According to paragraph 4, all of the following statements are true of grapes harvested during the early famine
years EXCEPT
A. They resulted in poor-quality wine.
B. They often rotted while still on the vine.
C. They were often diseased as a result of damp weather.
D. They suffered as much during dry years as they did during wet years

第 5 页 共 373

2021 托福阅读真题

Paragraph 5: The evidence for other kinds of fruit and crop production and what can be inferred from the effects of
this kind of weather on modern farming suggests similar declines in yields. In provinces where the ordinary peasant
beverage was a very nutritious fermented apple beverage called cider, a pathetic apple harvest, let alone the use of
rotting apples in cider making, must have had disastrous consequences for human health. One can add to this list of
catastrophes the severe effects of the weather on industrial crops. Many of these provided products that farmers and
estates could sell to manufacturers. Flax, for example, was used for the production of linen cloth, and wood provided
the raw material for a blue dye used in the cloth-making industry.

7. Why does the author discuss the uses of flax and wood in paragraph 5 ?
A. To illustrate how the weather negatively affected industry
B. To show that industrial crops were even more seriously affected by the disastrous weather cycle than fruit crops
were
C. To provide an example of crops whose yields did not decline
D. To support the idea that the weather had disastrous consequences for human health

Paragraph 6: The effects on animal populations were equally severe, but they differed depending on the type of
animal. Neither horses nor pigs seem to have suffered widespread disease during most of the famine, while sheep
and cattle did. The supply of horses helped ease the effects of the mortality of oxen as beasts of burden. The
prevalence of pigs helped lessen the effects of disease on the sheep flocks and cattle herds. Nonetheless the effects
were extraordinary, and flocks and herds were often reduced in size by as much as 90 parent from the devastating
impact of disease

8. Paragraph 6 supports the idea that farmers' use of horses was affected in which of the following ways during the
famine?
A. Horses carried leads that had been carried by oxen before the famine
B. Horses replaced pigs as a source of meat.
C. Farmers rode horses to herd cattle and sheep, because the animals required larger land areas to feed on than they
had before the famine.
D. Farmers relied less on horses because of widespread disease among horses

Paragraph 2: ■ The immediate consequence was a crisis in the production of crops and, soon after, of work and
food animals. ■ The problems with production were compounded by a series of wars throughout the British Isles,
northern Germany, Scandinavia, and northern France and Flanders--all accompanied by piracy on the seas--that
initiated a prolonged distribution crisis. ■ The effect on the production of all major cereals--wheat, barley, oats, and
rye--as well as or rapes and other crops was nothing short of catastrophic. ■ Selective statistical material indicates a
decline in grain production in parts of northern Europe of about one-third during the years of the disastrous weather
cycle. Considering the population of northern Europe was at its medieval peak on the eve of the ecological

第 6 页 共 373

2021 托福阅读真题

catastrophe caused by the weather from 1315 to 1322, such a decline in crop yields put roughly 30 million
inhabitants of northern Europe at considerable risk of malnutrition.

9.Look at the four squares [■] that indicate where the following sentence could be added to the passage.
But the failure of farms to produce sufficient goods was only one aspect of the crisis.
Where would the sentence best fit? Click on a square to add the sentence to the passage.

10.Directions: An introductory sentence for a brief summary of the passage is provided below. Complete the
summary by selecting the THREE answer choices that express the most important ideas in the passage. Some
sentences do not belong in the summary because the express ideas that are not presented in the passage or are minor
ideas in the passage. This question is worth 2 points.

From 1315 to 1322, a disastrous weather cycle resulted in catastrophic famine throughout northern Europe.

A. Evidence for the catastrophic weather patterns comes from tree- ring studies contemporary chronicles and
statistical materials concerning crop yields and human population sizes.
B. Evidence indicates that roughly 30 million inhabitants of northern Europe most likely died of malnutrition at the
peak of the ecological catastrophe.
C. Disastrous consequences for human health resulted from a shift in peasants' diet to include fewer cereal crops and
more cider, poor- quality wine, and meat from pigs and horses.
D. Crops differed in their ability to tolerate the ceaseless rain and extreme weather, but harvests of grain, fruits. and
industrial crops were all reduced
E. A series of wars resulted in a food distribution crisis that made the effects of crop losses even more serious and
contributed to the risk of malnutrition.
F. Though the survival of pigs and horses helped reduce the effects of the loss of other farm animals, widespread
disease killed many animals during the period, especially sheep and cattle.

第 7 页 共 373

2021 托福阅读真题

Nesting Challenges of the Great Bustard

Paragraph 1: Great bustards are found on grasslands across Europe and Asia, where they feed on insects, seeds, and
vegetation. These birds nest on the ground, which leaves them vulnerable: many eggs and chicks are lost to predators
such as dogs, foxes, ravens, and crows, and the mothers themselves are vulnerable to predators while brooding
(sitting on) their eggs. To minimize these risks female great bustards prefer nest sites that conceal the nest from
predators while at the same time allow the brooding female to survey the surrounding area for danger . If the
vegetation is too tall or dense, the nest can be ambushed. Even if the predator is detected, dense vegetation may
impede escape. Alternatively nesting in open sites such as plowed fields leaves the nest too visible. In the Spanish
populations, agricultural fields planted in cereal crops or that are fallow (left unplanted for a period) seem to provide
the best compromise between visibility and concealment and are preferred as nest sites. Successful females also
select nest sites that offer some protection from inclement weather. The bodies of newly hatched chicks do not have
a well-developed mechanism for maintaining temperature, and cool, rainy weather during the hatching period can
cause hitch chick mortality. In central and northwestern Spain, sites on southeast-facing slopes that catch the
morning sun but are sheltered from the cool, prevailing northwesterly winds are preferred for this reason.

1.According to paragraph 1, all of the following statements about great bustards are true EXCEPT
A. They eat both plants and insects
B. They build nests on the ground
C. They are at risk from being eaten by dogs, foxes, ravens, and crows.
D. They prefer to nest in locations where chicks are exposed to northwesterly winds.

2. In paragraph 1, why does the author discuss the Spanish populations of great bustards?
A. To indicate that cereal crops are a food source for female great bustards
B. To illustrate the best nesting conditions for female great bustards
C. To highlight the effect of precaution on chick mortality
D. To identify situations in which female great bustards need protection

3. It can be inferred from paragraph 1 that female bustards in certain parts of Spain prefer nesting sites on
southeast-facing slopes because
A. such areas protect newly hatched chicks from the sun
B. such areas have fewer predators than other areas do
C. such areas tend to be warmer than other areas
D. such areas provide the best compromise between visibility and concealment

Paragraph 2: Chicks that escape these early dangers face the challenge of obtaining enough toad. Food scarcity
seems to be an ever-present threat for great bustards, and females often fail to obtain enough food to support

第 8 页 共 373

2021 托福阅读真题

themselves and their chicks. Researchers Maine' MO rates, dean Alonso, and aver Alonso conducted an
eleven-year study of female reproductive success in a population of great bustards in northwest Spain, and
their results starkly illustrate the crucial dependence of bustards on their food supply. In this region breeding
females increase their body mass by about sixteen percent just prior to egg laying, and they depend on the spring
flush of annual herbaceous (soft-stemmed) vegetation to do so. When winter rainfall is above average, the density of
herbaceous vegetation is high, and more females attempt to breed. Females breeding in those years also lay more
eggs, and the eggs have success. Abundant winter rain also increases the density of the annual grasses, and these
provide food for the crass snoopers and crickets that become the main food for great bustard chicks. Chick growth
and survival during the critical first three months after hatching depend strongly on food abundance. and so abundant
summer food means more and larger chicks leaving the nest in fall. The net result is that abundant winter rains leak
to more breeding females, more eggs per female, greater hatching success of eggs, and better chick growth and
survival. Conversely, in years of low winter rainfall and hence low food abundance, bustard productivity falls close
to zero with as few as four chicks produced per one hundred adult females

4. Which of the sentences below best expresses the essential information in the highlighted sentence in the passage?
Incorrect choices change the meaning in important ways or leave out essential information.
A. A long-term study in northwest Spain showed that the reproductive success of female great bustards depends
greatly upon their food supply.
B. A research study showed that female great bustards in northwest Spain required more than eleven years of steady
food supply to achieve reproductive success
C. Researchers in northwest Spain conducted a study illustrating that great bustards are particularly successful there
because of the food supply
D. The results of an eleven-year study clearly illustrate the reproductive success of populations of female great
bustards in northwest Spain.

5. The word "Conversely” in the passage is closest in meaning to


A. On the other hand
B. Consequently
C. Of course
D. Similarly

6. According to paragraph 2, why is a high density of herbaceous vegetation important for great bustards in
northwest Spain?
A. Females lay their eggs in nests made of soft herbaceous vegetation.
B. Herbaceous vegetation helps protect eggs from above-average winter rains.
C. Females use herbaceous vegetation to gain weight before they lay eggs.
D. Herbaceous vegetation is the main food eaten in the spring.

第 9 页 共 373

2021 托福阅读真题

Paragraph 3: The energetic costs of producing eggs and rearing chicks probably contribute to the extreme body -
size differences between the sexes in great bustards by favoring relatively smaller body sizes in females than in
males. The core of this argument is simply that females have to provide food for both themselves and their chicks.
and the less they need for their own maintenance, the more is available for egg production or to be fed directly to the
growing chicks. When food is scarce, as it often is for great bustards, larger individuals may not be able to obtain
sufficient calories to both sustain themselves and allocate energy to reproduction. The trade off between
self-maintenance and reproduction is manifest every time a brooding mother forgoes a search for food to sit longer
on her nest or feeds a prey item to her chick rather than eating herself Eating it herself improves her own body
condition and enhances her chances of reproducing in the coming spring but at he cost of diminishing the potential
growth, survival, and future reproductive success of her chicks. As a consequence of this trade off, great bustard
females that successfully rear a chick in on year seldom have sufficient energy reserves to breed successfully the
following spring

7. The word "diminishing" in the passage is closest in meaning to


A. destroying
B. stopping
C. delaying
D. reducing

8. According to paragraph 3, which of the following does the relatively smaller size of female great bustards allow
them to do?
A. Remain hidden while sitting on their nests
B. Use more food for producing eggs and feeding chicks than larger birds could
C. Search for food more efficiently than larger birds could
D. Attract mates more effectively than larger birds could

Paragraph 1: Great bustards are found on grasslands across Europe and Asia, where they feed on insects, seeds, and
vegetation. These birds nest on the ground, which leaves them vulnerable: many eggs and chicks are lost to predators
such as dogs foxes, ravens and crows, and the mothers themselves are vulnerable to predators while brooding sitting
on) their eggs. To minimize these risks female great bustards prefer nest sites that conceal the nest from predators
while at the same time allow the brooding female to survey the surrounding area for danger. If the vegetation is too
tall or dense, the nest can be ambushes. ■ Even if the predator is detected, dense vegetation may impede escape. ■
Alternatively nesting in open sites such as plowed fields leaves the nest too visible. ■ In the Spanish populations,
agricultural fields planted in cereal crops or that are fallow (left unplanted for a period) seem to provide the best
compromise between visibility and concealment and are preferred as nest sites. ■ Successful females also select nest
sites that offer some protection from inclement weather. The bodies of newly hatched chicks do not have a
well-developed mechanism for maintaining temperature, and cool, rainy weather during the hatching period can

第 10 页 共
373 页
2021 托福阅读真题

cause high chick mortality. In central and northwestern Spain, sites on southeast-facing slopes that catch the morning
sun but are sheltered from the cool, prevailing northwesterly winces are preferred for this reason

9. Look at the four squares passage that indicate where the following sentence could be added
But safety from predators is not the only factor in choosing a nesting site.
Where would the sentence best fit? Click on a square to add the sentence to the passage

10. Directions: An introductory sentence for a brief summary of the passage is provided below. Complete the
summary by selecting the 3 answer choices that express the most important ideas in the passage. Some sentences do
not belong in the summary because they express ideas that are not presented in the passage or are minor ideas in
passage. This question is worth 2 points. Drag your choices to the spaces where they belong.

Great bustards living on the grasslands of Europe and Asia face a number of challenges to successful
reproduction

A. Although great bustards can detect and escape predators, they have in recent years lost many eggs and chicks to
predators. and this has affected the population growth of great bustards.
B. An important compromise female bustards must consider is the need for a warm nesting site versus the need for a
site with abundant winter rain.
C. Females must balance the needs for calories to maintain themselves as well as reproduce against the need to
provide energy for their chicks to grow and survive.
D. The best nesting sites provide concealment as well as easy escape from predators and protection against
inclement weather.
E. Food for great bustard females and their young is often scarce. but when winter rain is plentiful, food is abundant
and increases the chances of reproductive success.
F. Studies have found differences in body size between male and female great bustards, but the reasons for these
differences have not been fully explained.

第 11 页 共
373 页
2021 托福阅读真题

Water and Life on Mars

Paragraph 1: The question of life on Mars depends heavily on the characteristics of its air and water. Mars has a
relatively thin and dry atmosphere, with high percentage of carbon dioxide compared to Earth's atmosphere. Mars
tilts about 25°,which leads to seasons like those on Earth. Carbon dioxide freezes into the polar caps during the
winter and evaporates into the air in the summer. This causes a fluctuation in the density of the atmosphere of up to
25 percent over the year. With so little atmosphere, the planet experiences a wide range of temperatures, heating up
in the day and rapidly cooling at night. Temperatures on Mars have been measured between 225°F and 95°F.

1. According to paragraph 1, all of the following are true about the planet Mars EXCEPT:
A. It has as an atmosphere with a stable density year-round.
B. It has a greater amount of carbon dioxide than Earth has.
C. It has number of different seasons.
D. It experiences significant temperature variations.

Paragraph 2: Scientists remain fascinated by the possibility of liquid water on Mars. Life, as we know it, requires
water so it should be good marker for habitability.Current conditions on Mars suggest that liquid water could not
occur on the surface. The low pressure and low temperature mean that ice would sublime--go straight from solid to
gas--rather than melt. Water may be hiding in subsurface reservoirs, however, and stream forth for limited periods.
In 2002 the Mars Odyssey orbiter detected large quantities of water ice near the surface of the southern hemisphere.
Channels on the surface suggest the presence of moving water in large quantities. In 2006 Mars Global Surveyor
provided pictures of gullies (channels) on the surface that could only have been produced by liquid water flowing
within the past seven years. Theories exist for how climate and erosion may reveal underground ice and cause
sudden floods, but specific details are still a mystery. The water probably represents flash floods that quickly
evaporate and may not provide an adequate medium for the development of life. If life arose in the distant past,
though, the selfish floods may have been enough to maintain organisms adapted to such environments.

2. According to paragraph 2, evidence from the 2002 Mars Odyssey orbiter supports which of the following
possibilities about water on Mars?
A. The near surface of the southern hemisphere contains more water in liquid form than in ice form.
B. The amount of moving water on the surface of Mars is probably greater than that hiding in subsurface reservoirs.
C. The occurrence of flash floods on Mars is enough to maintain certain species of organisms.
D. Water may be present under the surface of Mars and flow for short periods of time.

3. In paragraph 2,the findings of the 2006 Mars Global Surveyor suggest which of the following is likely to be true
about Mars?
A. The climate of Mars has changed greatly over time.

第 12 页 共
373 页
2021 托福阅读真题

B. Life could not have existed on Mars during the planet's early history.
C. The surface of Mars experiences erosion and sudden flooding.
D. Water on the surface of Mars is very slow to evaporate.

Paragraph 3: Astrobiologists continue to investigate whether the current situation--frozen water with occasional
flash floods--was always the situation on Mars. Some surface channels on Mars could not have been produced by
flash floods or geologic activity. The Opportunity rover, which landed on Mars in 2004. made two interesting
discoveries in several craters on Mars. Rock layers on the sides of the craters look like the patterns inside
sedimentary rocks on Earth. Sedimentary rocks form gradually at the bottom of bodies of water as particles settle
down to the bottom. Sedimentary rocks on Mars would imply that a body of water was present for hundreds of years
at some point in the past. In addition, Opportunity has found countless tiny spheres of the mineral hematite (a
mineral composed of iron and oxygen) on the surface. These tiny spheres, or "blueberries" as they have come to be
called, strongly resemble accretions ( accumulations) found on Earth in highly acidic stream beds. Together with
rippling patterns in the stone of the surface, the layers and blueberries make a compelling argument. Acidic, salty,
flowing water must once have covered large regions of the plain where Opportunity landed. The Sun has been very
slowly cooling off the past few billions of years. A warmer Sun in the past means a warmer Mars. Planetary
scientists do not think the difference was enough to bring Mars up to Earth like temperatures, but it may have been
enough to support liquid water in the warmer regions of the planet.

4. The word "compelling" in the passage is closest in meaning to


A. imaginative
B. persuasive
C. important
D. clear .

5. All of the following are mentioned in paragraph 3 as discoveries made by the opportunity rover EXCEPT
A. geologic activity in several craters
B. rocks with rippling patterns
C. Earth-like layers of rock
D. numerous pieces of the mineral hematite

6. According to paragraph 3, the discoveries made by the Opportunity rover support which of the following findings
about Mars?
A. The previously acidic, salty soil in the surface layers of Mars became increasingly dry as the Sun warmed.
B. Specific areas of Mars were once covered by moving water.
C. Temperatures on Mars were once very close to those found on Earth.
D. The channels on the surface of Mars were mostly formed from frequent geological activities.

第 13 页 共
373 页
2021 托福阅读真题

Paragraph 4: Overall, Mars looks like a good candidate for past life. Water, carbon, and energy were all present in
useful amounts. If life arises quickly wherever resources allow, then Mars probably had life once. Unfortunately, we
have no way to assess how common the origin of life might be. This is one of the questions scientists were asking
when they explored Mars in the first place.

7. The word "assess" in the passage is closest in meaning to


A. challenge
B. evaluate
C. prove
D. explain

Paragraph 5: Astrobiologists’ another important question to ask in the exploration of Mars has to disagree about
evidence for life in the first billion years of Earth's history. Mars' history must be even more contentious. The most
unambiguous evidence of life on Earth comes in the form of structural fossils, which probably did not start forming
on Earth until life had been around for at least a billion years. If life did arise on Mars, it may not have been around
long enough to leave that kind of evidence.

8. In paragraph 5, why does the author discuss Earth's history?


A. To imply that the history of Mars is probably similar to that of Earth
B. To demonstrate that astrobiologists use standard procedures when exploring life on different planets
C. To demonstrate that life on both Earth and Mars probably started earlier than scientists had thought
D. To emphasize how difficult it would be to find concrete evidence of past life on Mars

Paragraph 2: Scientists remain fascinated by the possibility of liquid water on Mars. Life, as we know it, requires
water so it should be good marker for habitability. ■ Current conditions on Mars suggest that liquid water could not
occur on the surface.■The low pressure and low temperature mean that ice would sublime--go straight from solid to
gas--rather than melt. ■Water may be hiding in subsurface reservoirs, however, and stream forth for limited periods.
■In 2002 the Mars Odyssey orbiter detected large quantities of water ice near the surface of the southern hemisphere.
Channels on the surface suggest the presence of moving water in large quantities. In 2006 Mars Global Surveyor
provided pictures of gullies (channels) on the surface that could only have been produced by liquid water flowing
within the past seven years. Theories exist for how climate and erosion may reveal underground ice and cause
sudden floods, but specific details are still a mystery. The water probably represents flash floods that quickly
evaporate and may not provide an adequate medium for the development of life. If life arose in the distant past,
though, the selfish floods may have been enough to maintain organisms adapted to such environments.

9. Look at the four squares ■ that indicate where the following sentence could be added to the passage.
It is easy to understand why this is the case.

第 14 页 共
373 页
2021 托福阅读真题

Where would the sentence best it? Click on a square[■] to add the sentence to the passage.

10. Directions: An introductory sentence for a brief summary of the passage is provided below. Complete the
summary by selecting the 3 answer choices that express the most important ideas in the passage. Some sentences do
not belong in the summary because they express ideas that are not presented in the passage or are minor ideas in the
passage. This question is worth 2 points.

A. Although water on its surface is an important indication of habitability on Mars, other features of the planet also
suggest the past existence of life.
B. The discoveries of sedimentary rocks and an iron oxide mineral as well as the fact that the Sun has been cooling
gradually suggest that liquid water probably existed in warmer areas of Mars in the past.
C. The duration of past life forms on Mars may have been too brief to provide fossil evidence for use in scientific
study.
D. Scientific explorations have reported the presence of channels and subsurface water, which suggests the presence
of liquid water on Mars and the possibility of life in the past.
E. The Opportunity rover confirmed planetary scientists' belief that frozen water beneath the surface of Mars was
brought to the surface as result of sudden floods and erosion.
F. The presence of necessary elements for life such as carbon, water,and energy in useful amounts has led scientists
to believe that Mars is good candidate for the develop of future life .

第 15 页 共
373 页
2021 托福阅读真题

The Roots of Economic Transformation in England

Paragraph 1: England was the first nation in Europe to develop a social structure that strongly supported the
innovation and economic growth we associate with modern times. England's advantages were many, some of them
deeply rooted in geography and history. This comparatively small realm contained an excellent balance of resources.
The plain to the south and east, where traditional centers of English settlement concentrated, was fertile and
productive. The uplands to the north and west possessed rich deposits of coal and iron, and their streams had
powered flour mills for hundreds of years. Proximity to the sea was another natural advantage. No part of the island
kingdom was distant from the coast. At a time when water transport offered the sole economical means for moving
bulky commodities, the sea brought coal close to iron, raw materials close to factories, and products close to markets.
Above all, the sea gave Britain's merchants access to the much wider world beyond their shores.

1. The word “sole" in the passage is closest in meaning to


A. most
B. common
C. only
D. nearest

2. According to paragraph 1, the British used the sea to do all of the following EXCEPT
A. provide power for mills
B. transport materials and products efficiently
C. reach all parts of England
D. sell their goods to other countries

Paragraph 2: Efficiency of transport was critical in setting the size of markets. During the eighteenth century,
Britain witnessed a boom in the building of canals and turnpikes (roads that could be traveled for a fee). By 1815 the
country possessed some 2,600 miles of canals linking rivers, ports, and other towns. In addition, few institutional
obstructions to the movement of goods existed. United under a strong monarchy, Britain was free of internal tariffs
(payments for goods transported across a border), unlike pre-revolutionary France, Germany, or Italy. English
merchants everywhere counted in the same money, measured their goods by the same standards, and conducted their
affairs under the protection of the common law. By contrast, in France local regions differed in their legal codes and
in weights and measures, which complicated and slowed exchange. As the writer Voltaire sarcastically remarked, the
traveler crossing France by coach changed laws as frequently as horses.

3. The word "critical" in the passage is closest in meaning to


A. essential
B. useful

第 16 页 共
373 页
2021 托福阅读真题

C. desirable
D. customary

4. It can be inferred from paragraph 2 that France differed from England in which of the following ways?
A. France was less unified in its economic structure than England was.
B. France had lower tariffs on its roads than England did.
C. France had a system of common laws for the protection of merchants that England lacked.
D. France, unlike England, was dependent on international trade.

Paragraph 3: The English probably had the highest standard of living in Europe and generated strong consumer
demand for manufactured goods. English society was less stratified (divided into groups based on status) than
elsewhere in Europe, and the aristocracy was powerful but much smaller. Primogeniture--the right of the eldest son
to inherit the family's land--was the rule both among the aristocratic members of the House of Lords and among the
other land-owning classes. Left without lands, younger sons had to seek careers in other walks of life, and some
turned toward commerce. They frequently obtained capital for their ventures from their landed fathers and elder
brothers. English religious minorities, chiefly Calvinists and Quakers, formed another pool of potential businessmen;
denied careers in government because of their religion, many turned their energies to business enterprises.

5. In paragraph 3, why does the author discuss primogeniture?


A. To explain why the aristocracy in England remained powerful even though it was smaller than in other European
countries
B. To emphasize the importance of family unity in the development of English commerce
C. To identify one source of England's large pool of people available to enter careers in business
D. To suggest a reason why England had such a high standard of living

Paragraph 4: A high rate of reinvestment is very important to industrialization; reinvestment, in turn, depends on
the skillful management of money by both individuals and public institutions. Here again, Britain enjoyed
advantages. Early industrial enterprises could rely on Britain's growing banking system to meet their capital
needs, a system which in the seventeenth century was taken over by the goldsmiths of London, who accepted
and guarded deposits, extended loans, and provided other financial services. In the eighteenth century, banking
services became available beyond London; the number of regional banks rose from 300 in 1780 to more than 700 by
1810. English businessmen were familiar with banknotes and other forms of commercial papers, and their
confidence in paper money facilitated the recruitment and flow of capital.

6. Which of the sentences below best expresses the essential information in the highlighted sentence in the passage?
Incorrect choices change the meaning in important ways or leave out essential information.
A. Britain's industrial enterprises met their capital needs from a growing banking system, which in the seventeenth

第 17 页 共
373 页
2021 托福阅读真题

century was run by London goldsmiths.


B. Britain's banking system was created by early industrial enterprises in order to manage deposits and provide other
banking services for groups such as goldsmiths.
C. In the seventeenth century the goldsmiths of London took over most of the functions of the banking system.
D. Britain's banking system began to grow in the seventeenth century in order to meet the financial needs of
goldsmiths.

7. According to paragraph 4, which TWO of the following occurred in England in the eighteenth century? To receive
credit, you must select TWO answers.
A. The accessibility of banking services in areas other than London
B. A willingness to use paper money
C. A reduction in the number of goldsmiths
D. A dependence by businesses on private investment managers

Paragraph 5: The founding of the Bank of England in 1694 marked a distinctive period in the history of European
finance. The bank took responsibility for managing England's public debt, sold shares to the public, and faithfully
met the interest payments due to the shareholders with the help of government revenue, such as the customs duties
efficiently collected on Britain's extensive foreign trade. When the government needed to borrow, it could turn to the
Bank of England for assistance. This stability in government finances ensured a measure of stability for the entire
money market and, most important, held down interest rates in both the public and private sectors. In general, since
the late seventeenth century, England's government was sensitive to the interests of the business classes, who in turn
had confidence in the government. Such close ties between money and power facilitated economic investment.

8. In paragraph 5 the author mentions which of the following as being a result of the Bank of England's stabilizing of
government finances?
A. The government's debt was greatly reduced.
B. Foreign trade became more profitable.
C. More people were able to buy shares in the Bank of England.
D. Interest rates were kept at a low level.

Paragraph 1: England was the first nation in Europe to develop a social structure that strongly supported the
innovation and economic growth we associate with modern times. England's advantages were many, some of them
deeply rooted in geography and history. ■This comparatively small realm contained an excellent balance of
resources. ■The plain to the south and east, where traditional centers of English settlement concentrated, was fertile
and productive. ■ The uplands to the north and west possessed rich deposits of coal and iron, and their streams had
powered flour mills for hundreds of years. ■ Proximity to the sea was another natural advantage. No part of the
island kingdom was distant from the coast. At a time when water transport offered the sole economical means for

第 18 页 共
373 页
2021 托福阅读真题

moving bulky commodities, the sea brought coal close to iron, raw materials close to factories, and products close to
markets. Above all, the sea gave Britain's merchants access to the much wider world beyond their shores.

9. Look at the four squares [■] that indicate where the following sentence could be added to the passage.
A wide variety of geographic regions could be found within it that offered opportunities for agriculture,
industrialization, and transportation.
Where would the sentence best fit? Click on a square [■] to add the sentence to the passage.

10. Directions: An introductory sentence for a brief summary of the passage is provided below. Complete the
summary by selecting the 3 answer choices that express the most important ideas in the passage. Some sentences do
not belong in the summary because they express ideas that are not presented in the passage or are minor ideas in the
passage. This question is worth 2 points.

In the 1700s, England became the economic leader in Europe.

A. Canal transport eventually became more efficient than turnpike transport and also encouraged the settlement of
towns in previously unpopulated areas.
B. The nature of England's geographic characteristics and its various regional resources combined to provide
England with the basis of strong economic development.
C. The spread of banks encouraged more local investment but did not prevent the increasing of the central
government's debt until stabilizing laws were passed in the late seventeenth century.
D. The members of groups with certain religious beliefs who chose careers in business came mainly from families
that owned little land and had few educational opportunities.
E. The government of England adopted policies and laws that favored the unrestricted movement of goods, the
demand for which was supported by the country's relative prosperity and social equality.
F. England's economy was aided by its efficient banking system, which strengthened the government's finances and
encouraged investment by private businesses.

第 19 页 共
373 页
2021 托福阅读真题

Chondrites

Paragraph 1: Meteorites are the keys to understanding how Earth and the solar system formed because,
although superficially many meteorites do not look significantly different from most Earth rocks, close study
shows that they are very different, with those differences holding clues to their distant origin. The most
common type of meteorite–known collectively as the "chondrites"–contains clues about the kinds of material that
went into the construction of our planet.

1. Which of the sentences below best expresses the essential information in the highlighted sentence in the passage?
Incorrect choices change the meaning in important ways or leave out essential information.
O To understand how our solar system formed, it is important to determine the differences between Earth rocks and
meteorites
O Despite their similar outward appearance, meteorites and Earth rocks are really very different, and the differences
help explain how our solar system formed.
O The superficial similarity of meteorites to Earth rocks provides clues about how our solar system formed.
O While meteorites had a distant origin, their presence on Earth is a key to understanding how our solar system
formed.

Paragraph 2: The chondrites are just one of a variety of meteorite types, all ancient, but each with a different
history--and each containing clues about how Earth and other planets formed and evolved. For example, the
members of one group, the iron meteorites, are made up of solid iron metal, alloyed (or mixed) with a modest
amount of nickel. All the evidence indicates that these meteorites are similar to the metallic cores that inhabit the
interiors of planets.

2. According to the information in paragraph2. which of the following is true about iron meteorites ?
O They are much more ancient than the chondrites.
O They are less similar to planets' metallic cores than chondrites.
O They came to Earth from other planets.
O They are solid metal objects.

Paragraph 3: In contrast to the iron meteorites, the chondrites are made up of a jumble of mineral grains, many of
them familiar constituents of rocks on Earth, but also including pure iron metal--which does not occur in rocks on
Earth--and small marble-like spherical objects called "chondrules" (it is from these that the chondrites take their
name). The chaotic texture of chondrites indicates that they were formed in a process that randomly swept together
their different components and cemented them together. The texture is also an instant clue to one of their most
important characteristics: they have never been melted. Furthermore, mineral grains in the chondrites have provided
the oldest ages ever measured by radiometric dating: they date from the very earliest days of the solar system. These

第 20 页 共
373 页
2021 托福阅读真题

two properties have led the geochemists who work on these intriguing objects to conclude that chondrites bring us
something we cannot find on Earth or among other meteorite varieties: an unprocessed sample of the original
material from which Earth and our neighboring planets were made. They appear to be unaltered samples of the solid
matter that was floating around in the solar system just as Earth was being born; there is strong evidence that they
come from small asteroids that never grew big enough to heat up and melt, as larger objects did.

3. The word chaotic in the passage is closest in meaning to


A. characteristic
B. disorganized
C. smooth
D. unusual

4. According to paragraph 3, which of the following is indicated by the texture of chondrites?


O They were formed by the same process as the rocks on Earth.
O They are made of metals and minerals of different ages.
O They have never been melted.
O They are held together by a marble-like material.

5. According to paragraph 3. which of the following can be inferred about the origin of chondrites?
O They resulted from mixing and joining of many of the same materials that also formed Earth and its nearby planets
O They were created when large asteroids broke apart as they neared early Earth
O They resulted from the repeated melting and hardening of other kinds of meteorites during the long process of
Earth's formation.
O They formed from the heating up of small asteroids as they entered early Earth's atmosphere.

Paragraph 4: The primitive nature of the chondrites has made them very important for information about Earth's
overall chemical composition. You might wonder why these rare rocks from space should play such a key role when
the whole Earth is right below our feet, ready for us to analyze. The answer, in brief, is that if we want to understand
how Earth got to its present state, we have to know something about its initial, overall composition. But we only
have access to rocks from our planet's thin outermost skin--which is very different from the inaccessible interior.
However, by using information from the chondrites as a reference point, and integrating that data with direct
measurements on surface rocks and information about the interior obtained using remote sensing methods,
geochemists have been able to work out models for Earth's overall composition that satisfy already obtained
independent evidence such as the density of our planet.

6. In describing our planet's interior as "inaccessible", the author means that the interior
O cannot be reached
O is very deep

第 21 页 共
373 页
2021 托福阅读真题

O has a very high temperature


O formed a long time ago

7. According to paragraph 4, studying chondrites helps scientists learn about which of the following?
O Earth's overall density
O The chemical composition of Earth's surface rocks
O Earth's original composition
O The age of Earth

Paragraph 5: An important concept in formulating these models is that the composition of Earth and the other
"terrestrial planets (the solar system's inner, rocky planets, Mercury, Venus, and Mars) depends on the proportions of
the main minerals found in the chondrites that each planet incorporated. A good example is iron, which is so
abundant and so heavy that the well-known density variations among the terrestrial planets can almost entirely be
attributed to differences in their iron contents. Grains of iron metal are abundant in the chondrites, but different
chondrites contain different amounts. The reason Earth is much denser than Mars, according to the models, is that its
chondrite-like building blocks happened to contain more iron. Similarly, other differences among the terrestrial
planets can be understood in terms of different planets incorporating different amounts of the various constituents of
chondrites. This is undoubtedly an overly simplistic description of what actually happened, but that does not
invalidate the chondrites as a good starting point for understanding the composition of Earth and other planets.

8. In paragraph 5, why does the author mention that different chondrites contain different amounts of iron?
O To suggest that some types of chondrites affected planets more than did others
O To help explain the differences in the densities of the terrestrial planets
O To indicate one important reason why using chondrites as a model for understanding the composition of Earth may
be overly simplistic
O To emphasize the importance of iron in forming the terrestrial planets

Paragraph 4: The primitive nature of the chondrites has made them very important for information about Earth's
overall chemical composition. [■] You might wonder why these rare rocks from space should play such a key role
when the whole Earth is right below our feet, ready for us to analyze. [■] The answer, in brief, is that if we want to
understand how Earth got to its present state, we have to know something about its initial, overall composition. [■]
But we only have access to rocks from our planet's thin outermost skin--which is very different from the inaccessible
interior. [■] However, by using information from the chondrites as a reference point, and integrating that data with
direct measurements on surface rocks and information about the interior obtained using remote sensing methods,
geochemists have been able to work out models for Earth's overall composition that satisfy already obtained
independent evidence such as the density of our planet.

9. Look at the four squares [■] that indicate where the following sentence could be added to the passage.

第 22 页 共
373 页
2021 托福阅读真题

What is more, geochemists have used these ancient rocks to explain the workings of Earth's crust, oceans, and
other geological systems.
Where would the sentence best ft? Click on a square [■] to add the sentence to the passage.

10. Directions: An introductory sentence tor a brief summary of the passage is provided below. Complete the
summary by selecting the THREE answer choices that express the most important ideas in the passage. Some
sentences do not belong in the summary because they express ideas that are not presented in the passage or are minor
ideas in the passage. This question is worth 2 points.

Meteorites known as chondrites contain clues about the materials that went into the construction of the
terrestrial planets.

O There are a number of different types of chondrites, which are distinguished by their different histories and the
amounts of solid iron and nickel metal alloyed in their metallic cores.
O The age of chondrites can be determined by radiometric dating of the small marble-like spheres that, along with
various mineral grains, were cemented together to make them up.
O Various minerals occur in different proportions in different chondrites, suggesting that many differences among
planets stem from their having incorporated different amounts of these minerals.
O Chondrites are made up of ancient grains, including pure iron, that appear to be unaltered samples of the solid
material present in the solar system when Earth was forming.
O Information from chondrites has enabled geochemists to develop models for Earth's initial, overall chemical
composition despite their lack of direct access to Earth's interior.
O Iron is by far the most abundant of the main minerals found in chondrites, and this fact has been central to the
formation of models of the composition of Earth and the other terrestrial planets.

第 23 页 共
373 页
2021 托福阅读真题

The Role of Dingoes in the Australian Ecology

Paragraph 1: Dingoes first reached Australia about 4,000 years ago, arriving with Southeast Asian sea travelers.
These animals were a barely domesticated variety of the Indian wolf. Unlike domestic dogs in western Asia, which
by then had been intensively bred for a variety of unwolflike characteristics, dingoes looked, and often acted, like
their wild ancestors. Aboriginal people adapted to the new arrivals, keeping them as pets and helpers in kangaroo
hunts. Meanwhile,other dingoes quickly went feral (wild) and began to compete with Aboriginal hunters for
kangaroos and other prey.

1. The word “intensively" In the passage is closest in meaning to


A. vigorously
B. occasionally
C. carefully
D. Gradually

2. According to paragraph 1, which of the following was NOT true of the first Australian dingoes?
A. They were initially brought to Australia by humans thousands of years ago.
B. They resembled the unwolflike domestic dogs of western Asia.
C. Some of them were domesticated by the Aboriginal people.
D. Some of them became wild animals and began hunting animals themselves.

Paragraph 2: When the dingo arrived, long after the mass extinction of Australia's megafauna (large animals), the
continent held one last large marsupial predator, the thylacine. A wolfish-looking beast with striped hindquarters, the
thylacine was also known as the marsupial tiger. It vanished from the mainland soon after the dingo was introduced,
but survived into the twentieth century on the island of Tasmania. Historical records describe thylacine behavior and
document the intense hunting that destroyed the last populations. Students of ecological history have long assumed
that the dingo drove the thylacine to extinction on mainland Australia. The key part of this argument Is a bit of
circumstantial evidence: the thylacine disappeared at about the time the dingo arrived but survived in Tasmania's
dingo-free landscape. For ecologist Chris Johnson, who has made a specialty of studying Australia's mammal
extinctions, this picture is too simple.

PARAGRAPH 3: Twentieth-century accounts of thylacines portray them as solitary creatures that hunted alone. Yet
these are records of the last of their kind, members of an intensely persecuted population. Older reports, from early
in the days of Tasmania's settlement, describe thylacines hunting in groups, "with the tenaciousness of a pack of
wolves on the steppes of frozen Russia ." They were fast, nimble, and big-more massive than a dingo and strong
enough that, in at least one case, seven domestic hunting dogs could not kill a lone individual. A recent analysis of
the bite force thylacines exerted, based on studies of their skull and jaw bones, suggests that thylacines took prey

第 24 页 共
373 页
2021 托福阅读真题

bigger than themselves. All this conflicts with the idea that the dingo pushed the thylacine into oblivion

3. The word "persecuted" in the passage Is closest in meaning to


A. clever
B. specialized
C. hunted
D. efficient

4. In paragraph 3, Why does the author discuss the fast speed, large size, and forceful bite of thylacines?
A. To explain why thylacines were not well suited for domestication.
B. To explain why older reports from the early days of settlement in Tasmania are not reliable.
C. To present evidence against the theory that thylacines were solitary creatures that hunted alone.
D. To present evidence against the idea that dingoes were responsible for the extinction of thylacines.

PARAGRAPH 4: The researcher Chris Johnson suggests that intensified human hunting severely reduced thylacine
populations in northern Australia before the dingo ever became established, pointing out that while both dingoes and
thylacines were often depicted in rock art in the north, the Aboriginal people who first painted dingoes worked in
a distinct style, more recent than any known image of the thylacine. The dingo may have played a minor role in
the thylacine’s disappearance; perhaps dingoes and humans hunting together became a truly formidable enemy.
Regardless of how exactly the thylacine died out, says Johnson, the beast is gone, and the dingo is the only large
predator left on the continent. He argues that native wildlife, and perhaps even outback cattle ranchers, are far better
off with dingoes than without them. (In the absence of dingoes, kangaroo populations can skyrocket, leading to
overgrazing or pastureland.)

5. Why does the author provide the information that the Aboriginal people who first painted dingoes worked in a
distinct style, more recent than any known image of the thylacine.?
A. To compare two distinct Aboriginal painting styles in northern Australia
B. To provide evidence that dingoes became established in Australia only after thylacines had declined in population
C. Te describe changes that occurred in Aboriginal rock art at the time dingoes became established in Australia
D. To support the argument that the dingo played a role in the thylacine’s disappearance

6. Paragraph 4 suggests which of the following about Johnson’s views of the consequences of the thylacine's
disappearance?
A. The thylacine extinction increased the importance of dingoes in the Australian ecology.
B. The thylacine extinction was the last known decrease in large native wildlife predators.
C. The thylacine extinction made the combination of dingoes and humans hunting together a difficult enemy.
D. The thylacine extinction led to an undesirable decline in kangaroo populations.

第 25 页 共
373 页
2021 托福阅读真题

PARAGRAPH 5: In an analysis of threatened and extinct marsupial species across the continent, Johnson found that
small mammals survived much longer in areas with healthy dingo populations. Where dingoes have been removed,
90 percent of ground-dwelling mammal species have gone extinct due to predation by foxes and cats (introduced to
Australia much later by Europeans). Johnson's interest in the protective effect of dingoes began years ago, when he
was studying wallabies on an outback cattle station in northeastern New South Wales. The station was a safe place
for bettongs and rat kangaroos, threatened small marsupials. The landowners believed the bettongs thrived because
of a decision not to poison local dingoes. and the dingoes kept the foxes away. Soon after this experience, one of
Johnson's colleagues was in the Tanami Desert researching a threatened species, the rufous hare-wallaby. Dingoes in
the area were occasionally eating hare-wallabies, so the local parks and wildlife service decided to poison them.
“Within two weeks,” says Johnson, "there were foxes on the site, and they killed off the hard-wallabies. That species
is now extinct on the main land. “ The sad fate of the rufous hare-wallaby is a classic example of what can happen as
a result of Mesopredator Release, the idea that if the top predator is eliminated (in this case, dingoes), middle
predator populations increase, sometimes with harmful consequences.

7. According to paragraph 5, which of the following was the most likely reason why the outback cattle station in
New South Wales was a safe place for bettongs?
A. The area had a healthy population of dingoes.
B. The rufous hare-wallabies in the area became extinct.
C. There were foxes on the site.
D. The local landowners protected the bettongs from the dingoes.

8. According to paragraph 5, which of the following is an example of Mesopredator Release?


A. The extinction of bettongs and rat kangaroos
B. The dingoes’ pattern of occasionally preying on hare-wallabies
C. The dingoes' protection of hare-wallabies and foxes
D. The poisoning of dingoes and the resulting expansion of the fox population.

PARAGRAPH 2: When the dingo arrived, long after the mass extinction of Australia's megafauna (large animals),
the continent held one last large marsupial predator, the thylacine. A wolfish-looking beast with striped hindquarters,
the thylacine was also known as the marsupial tiger. It vanished from the mainland soon after the dingo was
introduced, but survived into the twentieth century on the island of Tasmania. ■Historical records describe thylacine
behavior and document the intense hunting that destroyed the last populations.■ Students of ecological history have
long assumed that the dingo drove the thylacine to extinction on mainland Australia. ■The key part of this argument
Is a bit of circumstantial evidence: the thylacine disappeared at about the time the dingo arrived but survived in
Tasmania's dingo-free landscape. ■For ecologist Chris Johnson, who has made a specialty of studying Australia's
mammal extinctions, this picture is too simple.

第 26 页 共
373 页
2021 托福阅读真题

9. Look at the four squares [■] that indicate where the following sentence could be added to the passage.
But not all researchers agree.
Where would the sentence best fit?

10.Directions: An introductory sentence for a brief summary of the passage is provided below. Complete the
summary by selecting the THREE answer that express the most important ideas in the passage. Some sentences do
not belong in the summary because they express ideas that not presented in the passage or are minor ideas in the
passage. This question is worth 2 points.

The role of dingoes in the Australian ecology has been a subject of debate.

A. When dingoes reached Australia, the powerful thylacine was the only large predator there, but it soon became
extinct on the mainland.
B. Sea travelers brought dingoes to Australia because they were very similar to the domestic dogs of western Asia in
that they had unwolflike features.
C. Ecologists had believed that dingoes were the main cause of the extinction of thylacines, but the researcher Chris
Johnson has shown that there were probably other reasons for their extinction.
D. Studies have shown that the majority or thylacine populations hunted alone, which was probably why dingoes
were able to easily hunt thylacines and eventually cause their extinction.
E. Johnson's research revealed that dingoes have a protective effect on a few species of marsupials, like bettongs, but
not others, a finding confirmed by similar observations in the Tanami desert.
F. Research suggests that the removal of dingo populations from their established environments can have lasting
damaging effects on small ground-dwelling mammals like the hare-wallabies.

第 27 页 共
373 页
2021 托福阅读真题

Australian Thunderbirds

Paragraph 1: Today, Australia is home to two species of giant flightless bird: the emu and the cassowary, but up
until 30,000 years ago, Australia supported another group or giant flightless birds known as thunderbirds. In some
ancient Australian rock paintings, there are birds that appear to represent thunderbirds. The depictions are certainly
too large for emus and cassowaries.

1. Paragraph suggests which of the following about images of birds in ancient Australian rock paintings?
A. They began to appear in the paintings about 30,000 years ago.
B. They were painted at a time when emus and cassowaries generally were not painted
C. They are believed to be thunderbirds because of their size.
D. They were mistaken for images of emus and cassowaries when they were first discovered

Paragraph 2: Since the discovery of the first thunderbird bone in the 1820s, many thunderbird bones have been
found. However, bird skulls are particularly fragile and until recently, no one knew what the head of a thunderbird
looked like. Recent discoveries show that these birds had enormous heads and very impress beaks. The beaks are
very deep, but quite narrow, and some of the species appear to have been equipped with powerful jaw muscles.
Naturally, the impressive biting apparatus of these extinct birds has led paleontologists to speculate about what they
ate. Some paleontologists believe that they were carnivores (meat-eating animals), or perhaps even scavengers
capable of breaking the bones of dead animals that they found. Others believe that the thunderbirds were herbivores
fond of nibbling vegetation and using their terrific bill to crack open seeds and nuts. The image of a giant,
carnivorous duck is an enticing one, especially for the media, but it is highly unlikely that these huge birds were
meat eaters, or even scavengers. They lack the equipment of true predatory animals. The bill may be big, but it
certainly isn’t hooked, a necessary tool for any animal hoping to tear chunks of flesh from a carcass. Also the feet of
the thunderbirds lack the talons (sharp claws) we see in all predatory birds, regardless of their size. Last, the eyes of
the thunderbird are not positioned in a way to provide binocular vision: they are situated on either side of the
head and give good all-around vision but leave blind spots directly in front of and behind the animal. This is the
vision of an animal that is hunted, not a hunter.

2. The word “enticing” in the passage is closest in meaning to


A. easy
B. amusing
C. attractive
D. unusual

3. For which of the following reasons does the author specify that “the eyes of the thunderbird are not positioned
in a way to provide binocular vision”?

第 28 页 共
373 页
2021 托福阅读真题

A. To contrast the way the thunderbird used its eyes with the way it used its feet
B. To indicate one reason that it is unlikely that the thunderbird was a predator
C. To suggest that the thunderbird did not need binocular vision to hunt effectively
D. To help explain why the thunderbird was not able to spot other animals easily

4. According to paragraph 2, all of the following are true of thunderbirds EXCEPT:


A. Their heads were very large
B. Their jaw muscles were extremely strong
C. Their beaks may have been effective at opening seeds and nuts
D. Their skulls were discovered alongside broken bones of other dead animals

PARAGRAPH 3:Chemical analysis of numerous eggshell fragments from one type of thunderbird shows that this
species was undoubtedly an herbivore with a penchant for eating grass. Other common thunderbird fossils also
point to herbivory. Along with the bones of thunderbirds, paleontologists have unearthed numerous polished stones,
known as gastroliths. These were swallowed by the bird and ended up in the gizzard (part of a bird's stomach), where
they helped break up fibrous plant matter. As it s very probable the thunderbirds were herbivorous, the numerous
predators that once stalked Australia must have hunted some of these birds, especially before they reached adulthood.
This is one reason why some of the thunderbirds grew so huge, as large size is an excellent defense against predators.
Their other defense was powerful legs, which probably endowed some of the species with a powerful kick and a
good turn of speed to get them out of harm's way

5. The word “penchant” in the passage is closest in meaning to


A. structure
B. preference
C. strategy
D. requirement

6. According to paragraph 3, all of the following probably protected thunderbirds form predators EXCEPT
A. Thunderbird’s ability to grow and reach adulthood quickly
B. Thunderbird’s enormous body size
C. Thunderbird’s ability to kick with great force
D. Thunderbird’s ability to move rapidly away from danger

PARAGRAPH 4: As well equipped as they were to deal with the rigors of prehistoric Australian life, these giant
birds lacked the adaptability to deal with the combination of humans and the devastation they bring, and with climate
change. Exactly when the thunderbirds became extinct is a cause of dispute among paleontologists, but the last
species is widely thought to have clung to existence until around 30,000 years ago. Scientists have used chemical

第 29 页 共
373 页
2021 托福阅读真题

analysis of ancient eggshells of one species of thunderbird to assess the impact of human activity on these birds and
the Australian landscape in general. It seems that before 50,000 years ago (before the widespread human
colonization of Australia), this particular thunderbird pecked at nutritious grasses. However, only 5,000 years later,
the diet of this species had completely switched to the leaves of bushes and trees. The scientists' theory is that around
45,000 years ago, humans began to have a drastic effect on the fragile Australian landscape by starting bushfires,
which may have burned out of control. With their preferred food up in smoke, the thunderbirds were forced to eat
other plant matter, and it seems that they may not have been able to adapt to this change. It is also likely that
thunderbirds were hunted for food by the first Australians. In the centuries that followed the human colonization of
Australia, the thunderbirds dwindled away to extinction.

7. According to paragraph 4, which of the following is probably true about thunderbirds’ eating the leaves of bushes
and trees?
A. The leaves of bushes and trees were the preferred food of certain thunderbird species throughout their existence
B. thunderbirds ate the leaves of bushes and trees because this was the most nutritious plant food in Australia around
50,000 years ago.
C. Thunderbirds had to start eating the leaves of bushes and trees after fires destroyed the plants that they usually
ate.
D. Thunderbirds ate the leaves of bushes and trees that were planted by the first humans who arrived in Australia.

8. According to paragraph 4 which of the followings is one factor that contributed to the extinction of the
thunderbird?
A. Thunderbirds could no longer obtain the nutrition they needed to from strong eggshells
B. The bushes and trees that thunderbirds needed to hide from predators were replaced by grasses
C. Sudden climate change occurred just before 50,000 years ago
D. Humans arrived in Australia and began eating thunderbirds soon afterward

PARAGRAPH 1: Today, Australia is home to two species of giant flightless bird: the emu and the cassowary, but up
until 30,000 years ago, Australia supported another group or giant flightless birds known as thunderbirds. In some
ancient Australian rock paintings, there are birds that appear to represent thunderbirds. ■The depictions are certainly
too large for emus and cassowaries.■

PARAGRAPH 2: Since the discovery of the first thunderbird bone in the 1820s, many thunderbird bones have been
found. ■However, bird skulls are particularly fragile and until recently, no one knew what the head of a thunderbird
looked like. ■Recent discoveries show that these birds had enormous heads and very impress beaks. The beaks are
very deep, but quite narrow, and some of the species appear to have been equipped with powerful jaw muscles.
Naturally, the impressive biting apparatus of these extinct birds has led paleontologists to speculate about what they
ate. Some paleontologists believe that they were carnivores (meat-eating animals),or perhaps even scavengers

第 30 页 共
373 页
2021 托福阅读真题

capable of breaking the bones of dead animals that they found. Others believe that the thunderbirds were herbivores
fond of nibbling vegetation and using their terrific bill to crack open seeds and nuts. The image of a giant,
carnivorous duck is an enticing one, especially for the media, but it is highly unlikely that these huge birds were
meat eaters, or even scavengers. They lack the equipment of true predatory animals. The bill may be big, but it
certainly isn’t hooked, a necessary tool for any animal hoping to tear chunks of flesh from a carcass. Also the feet of
the thunderbirds lack the talons (sharp claws) we see in all predatory birds, regardless of their size. Last, the eyes of
the thunderbird are not positioned in a way to provide binocular vision: they are situated on either side of the head
and give good all-around vision but leave blind spots directly in front of and behind the animal. This is the vision of
an animal that is hunted, not a hunter.

9. Look at the four squares [■] that indicate where the following sentence could be added to the passage.
Today, more information about this is available to scientists.
Where would the sentence best fit?

10. Directions: An introductory sentence for a brief summary of the passage is provided below. Complete the
summary by selecting the THREE answer that express the most important ideas in the passage. Some sentences do
not belong in the summary because they express ideas that not presented in the passage or are minor ideas in the
passage. This question is worth 2 points.

Australia was once home to a group of giant birds known as thunderbirds

A. The thunderbird was able to obtain some food through scavenging, but it was probably not a predator because its
large body size made it too slow to hunt other animals.
B. The presence of thunderbirds alongside emus and cassowaries in ancient Australian paintings shows thunderbirds’
importance for the people that colonized Australia thousands of years ago.
C. The thunderbird’s beak was so large and powerful that it led to speculation among scientists that the bird was a
carnivore.
D. Prehistoric evidence indicates that humans added thunderbird eggs to their diet about 5,000 years after reaching
Australia. when climate change was already affecting the birds
E. Several pieces of evidence, including the chemicals found in the shells of thunderbird eggs, support the idea that
the thunderbird mainly ate plants
F. The thunderbird went extinct some time after humans spread in Australia, in large part because humans changed
the thunderbird’s natural environment

第 31 页 共
373 页
2021 托福阅读真题

Bicknell's Thrush

Paragraph 1: The highly secretive wood thrush known as Bicknell's thrush is a strikingly melodious songbird under
threat of extinction. Brooding exclusively at high elevations in northeastern America, it prefers dense, stunted
balsam fir forests above 2,800 feet (850 meters) characterized by a cool, wet, and windy climate, where it can find
an abundance of caterpillars to feed on. Due in part to its recently conferred status as a species of its own, an d the
comparatively little research that has been conducted on it, the little bird is not listed on the United States
endangered species list and does not have legal protections.

1. Paragraph 1 suggests that cool balsam forests are important to Bicknell's thrush because they provide
O protection from wind
O an Important part of its diet
O safety from other birds
O a habitat where It enjoys legal protections

2. According to paragraph 1, which of the following is one reason why Bicknell's thrush is not listed on the United
States endangered species list?
O H has not been studied enough.
O has not been officially confirmed to be its own species.
O It is not scarce enough to be considered under threat of extinction.
O Its primary habitat is not in the United States.

Paragraph 2: Chief among the threats facing Bicknell's thrush is climate change. As the planet warms, habitat tends
to move northward or higher in elevation, but since Bicknell's thrush already nests at the tree line on the top of
mountains, its habitat cannot migrate upward any further. According to biologist Kent McFarland, the bird's
population always has been relatively small because it is a habitat specialist, preferring a very narrow type of habitat
that has never been in large supply. McFarland has created a computer model that tracks the Bicknell's thrush habitat
as average temperature increases, and the results are alarming. "It doesn't take much—just 2 degrees Celsius—to
remove 50 percent of the Bicknell's habitat," he said. "The 50 percent that is lost is the low end of the habitat where
Bicknell's thrushes aren't so dense, but [with an increase of] 6 degrees, (here's none left anywhere in the United
States.... If climate-change scenarios are right, this bird won't be around in a hundred years."

3. In paragraph 2, why does the author provide the information that Bicknell's thrushes nest in a narrow habitat
found on the tree line at the top of mountains?
O To demonstrate that Bicknell's thrushes are able to survive in particularly harsh conditions
O To explain why Bicknell's thrushes cannot simply move to a more suitable location as the climate warms
O To argue that the Bicknell's thrush population level is normal, considering the size of its habitat
O To account for the difficulty scientists have In studying Bicknell's thrushes in the wild

第 32 页 共
373 页
2021 托福阅读真题

4. Paragraph 2 suggests which of the following about Bicknell's thrush and climate change?
O The climate will not change enough to affect the survival of Bicknell's thrush.
O Climate change has already eliminated about 50 percent of the bird's population.
O Within 100 years, Bicknell's thrush may become extinct due to rising temperatures.
O Habitat suitable for Bicknell's thrush may actually grow thanks to climate change.

Paragraph 3: While climate change may be the most significant threat to the species in the long term, its short-term
prospects look somewhat bleak as well due to a series of smaller threats that, when combined, significantly
jeopardize the bird's health. Acid rain, a subject that has seen little public attention in recent years, is one issue that
likely will not threaten the species on its own but may be an important contributing factor to its demise. When highly
acidic rain falls—mostly as a result of pollution from power plants-it leaches the calcium out of the rocks and soil,
reducing the quantity of calcium available to birds. "Birds have to get calcium from somewhere for the formation of
their eggs," noted McFarland, “and we know they can't store it. There was a study done at Cornell that shows a high
correlation between acid rain and populations of wood thrushes that are in trouble." Acid rain may also be a factor in
the decline and degradation of balsam fir and red spruce forests.

5. The word "degradation” in the passage is closes! in meaning to


O removal
O ruin
O replacement
O poisoning

Paragraph 4: Added to this are threats from loss of habitat on the Bicknell's wintering grounds in the Caribbean
(just 8 percent of its original winter habitat remains, and half of that is in jeopardy), ski resort d evelopment on its
breeding grounds, blood parasites that compromise its immune system, and one quite surprising throat: mercury.
The presence of mercury in lakes and streams has been well documented for two decades, which is why numerous
health organizations advise the public to restrict their intake of freshwater fish in many places in the Northeast. The
mercury, which comes primarily from the emissions of coal-burning power plants in the Midwest, drifts eastward
and settles on water bodies, where it is converted to toxic methyl mercury by anaerobic bacteria. The mercury enters
the food chain through tiny creatures on the lake bottom, and its concentration becomes magnified as larger
organisms feed on smaller ones. Common loons, birds that feed entirely on freshwater fish during the breeding
season, have been found to have highly elevated levels of mercury in their bloodstream, reducing their reproductive
rates and making them lethargic (inactive). But no one expected mercury to be found in songbirds, especially not
those living at the tops of mountains.

6. The word “compromise" in the passage is closest in meaning to


O affect
O live in

第 33 页 共
373 页
2021 托福阅读真题

O attack
O weaken

7. According to paragraph 4, all of the following are true about mercury in the Northeast EXCEPT:
O It is most concentrated in bacteria at the water's surface.
O It originally enters the atmosphere in the Midwest.
O It gathers in lakes and streams.
O High levels of it are found in freshwater fish.

8. According to paragraph 4, how does mercury affect common loons in the Northeast?
O It decreases the supply of fish available for loons to feed on.
O It increases the exposure of loons to dangerous anaerobic bacteria.
O It leads to the loss of breeding grounds.
O It lowers loons' activity and their ability to reproduce.

Paragraph 5: McFarland and colleagues, including biologist David Evers, decided to test for it anyway. Evers found
unusually high levels of mercury in the blood and feathers of all 178 woodland birds he tested in New York State in
2005. He believes that the mercury is absorbed by soil and fallen leaves, which are fed on by worms and insects and
which are in turn consumed by songbirds. Of all the species that Evers tested, wood thrushes had the most mercury
in their blood.

Paragraph 3: While climate change may be the most significant threat to the species in the long term, its short-term
prospects look somewhat bleak as well due to a series of smaller threats that, when combined, significantly
jeopardize the bird's health. Acid rain, a subject that has seen little public attention in recent years, is one issue that
likely will not threaten the species on its own but may be an important contributing factor to its demise. ■ When
highly acidic rain falls—mostly as a result of pollution from power plants—it leaches the calcium out of the rocks
and soil, reducing the quantity of calcium available to birds. ■ "Birds have to get calcium from somewhere for the
formation of their eggs," noted McFarland, “and we know they can't store it. There was a study done at Cornell that
shows a high correlation between acid rain and populations of wood thrushes that are in trouble." ■Acid rain may
also be a factor in the decline and degradation of balsam fir and red spruce forests. ■

9. Look at the four squares [■] that indicate where the following sentence could be added to the passage.
The damaging effects of acid rain extend beyond this, however.
Where would the sentence best fit? Click on a square to add the sentence to the passage.

10. Directions: An Introductory sentence for a brief summary of the passage is provided below. Complete the
summary by selecting the 3 answer choices that express the most important ideas in the passage. Some sentences do
not belong in the summary because they express ideas that are not presented in the passage or are minor ideas in the

第 34 页 共
373 页
2021 托福阅读真题

passage. This question Is worth 2 points.

Bicknell's thrush is a North American songbird that is In danger of extinction for various reasons.

A. While the U.S. Fish and Wildlife Service has denied Bicknell's thrush status as its own species, extensive research
and the efforts of biologists are returning it from the edge of extinction.
B. Changes in temperature over time could eliminate the Bicknell’s thrush’s preferred habitat for breeding and make
it difficult for the species to survive.
C. Bicknell's thrushes are seriously threatened by the occurrence of a variety of factors at the same time, such as
habitat loss, blood parasites, and the effects of acid rain on egg formation.
D. Scientists have discovered that mercury, which was previously thought to mainly affect animals that live in or
near water, is also a threat to Bicknell's thrushes and other woodland songbirds.
E. Acid rain negatively affects freshwater fish—an important food source for Bicknell's thrush-and also makes it
impossible for the birds to store the calcium needed to produce eggs.
F. The presence of high levels of mercury in the wintering and breeding habitats of Bicknell’s thrush presents the
most significant danger to the survival of the species over the long term.

第 35 页 共
373 页
2021 托福阅读真题

The Postwar Economic Boom

Paragraph 1: The quarter century or so after the Second World War witnessed the longest period of uninterrupted
growth among the industrialized countries of the world and at the highest rates in history. For the industrialized
countries as a group the average increase in gross domestic product per person employed from 1950 to 1973
amounted to 4.5 percent per year. Rates for individual countries ranged from 2.2 percent for the United Kingdom to
7.3 percent for Japan. Growth was most rapid in those countries that had abundant supplies of labor, such as Japan
and West Germany. Growth in the United States, Canada, and Great Britain, which had the highest individual
incomes at the end of the war, was slower than that of continental Europe and Japan but more rapid than in any
prolonged period in their previous histories. At the same time, countries with relatively low individual incomes
within the industrial group—Italy, Austria, Spain, Greece, and Japan—grew more rapidly than the average.

1. According to paragraph 1, all of the following statements about economic growth in the industrialized countries
during the 25 years after the Second World War are true EXCEPT:
O Countries with comparatively low individual incomes had the slowest growth.
O Great Britain experienced slower economic growth than continental Europe did.
O Those countries that had plenty of available labor had the fastest growth.
O Canada and the United States grew steadily and more rapidly than in the past.

2. It can be inferred from paragraph 1 that the industrialized country with the greatest economic growth during the
25 years after the Second World War was
O West Germany
O Italy
O the United States
O Japan

Paragraph 2: The term--economic miracle--was first applied to the remarkable spurt in growth in West Germany
after 1948. When the high rates of growth continued throughout the 1950s and 1960s, it was used to refer to the
entire era. It was then noted that several other nations, notably Italy and Japan, had growth rates as high as or higher
than West Germany’s. Still, though the high growth rates in most of the industrial countries were certainly
remarkable, and unprecedented in history, they were scarcely miraculous. There were solid reasons for them in
every case.

3. The word “era" in the passage is closest in meaning to


O process
O period
O world

第 36 页 共
373 页
2021 托福阅读真题

O situation

4. Why does the author make the statement that "There were solid reasons for them in every case"?
O To emphasize that the term "economic miracle" correctly refers to every example mentioned in the discussion
about the postwar economic boom in industrial countries.
O To provide support for the claim that the term "economic miracle" more appropriately refers to Italy and Japan
than to Germany.
O To question the appropriateness of using the term "economic miracle” to refer to the strong growth rates in certain
industrial countries after the war.
O To argue that the term "economic miracle” should refer to the economic recovery of the late 1940s but not to the
growth of the 1950s and 1960s.

Paragraph 3: The Marshall Plan (officially called the European Recovery Program and based on investment from
the United States) played a crucial role in sparking the European recovery. When funding from the United States
ended in 1951, Europeans kept economic growth going with high levels of savings and investments. Much of the
investment went into equipment for new products and processes, since during the preceding years a backlog of
technical innovations had built up that only awaited capital and skilled labor to be employed. In effect, the European
economies had stopped growing for an entire generation, operating with obsolete equipment and lagging behind in
technical progress. Thus, technological modernization both accompanied and was an important contributory factor to
the so-called economic miracle.

5. According to paragraph 3. all of the following were true of the European economic recovery EXCEPT:
O It involved purchasing new equipment.
O It received financial support from outside Europe.
O It stopped for a generation after the Marshall Plan ended.
O It was affected in important ways by technological changes.

6. According to paragraph 3, how did investment contribute to the economic recovery of Europe?
O It persuaded Europeans to accept the Marshall Plan.
O It encouraged European consumers to purchase new products.
O It provided funds necessary to modernize European technology.
O It stimulated new ideas about technological improvement by funding research.

Paragraph 4: Other major factors were the attitude and role of governments. They participated in economic life
both directly and indirectly on a much larger scale than previously. They nationalized some basic industries, drew up
economic plans, and provided a wide range of social services. Nevertheless, private enterprise was responsible for by
far the largest part of economic activity. On average, between one-fourth and one-third of national income in

第 37 页 共
373 页
2021 托福阅读真题

Western Europe originated in the government sector. Though this proportion was much greater than it had been
before the war. it was less than half the contribution of the private sectors of the economy. In the mixed economies
that became characteristic of the Western democracies, government assumed the tasks of providing overall stability,
a climate favorable to growth, and minimal protection for the economically weak and underprivileged, but it left the
main task of producing the goods and services desired by the population to private enterprise.

Paragraph 5: At the international level the relatively high degree of intergovernmental cooperation deserves major
credit for the effectiveness of the economic performance. The cooperation was not always spontaneous, and some
promising projects failed for lack of it; but on the whole, the contrast to the prewar years is conspicuous.

7. According to paragraph 4, what impact did governments have on the mixed economies that characterized Western
European democracies after the war?
O Governments were responsible for a larger part of national economic activity than was private enterprise.
O Governments provided overall economic stability and encouraged economic growth.
O Governments controlled most private industries to ensure that they worked together to promote national growth.
O Governments created plans for the national production of goods that private enterprise enforced.

Paragraph 6: Finally, in the long term, much credit must go to Europe's wealth of human capital. Its high rates of
literacy and specialized educational institutions, from kindergartens to technical schools, universities, and research
establishments, provided the skilled personnel and brainpower to make the new technology work effectively. During
the first flush of the success of the Marshall Plan, many observers incorrectly deduced that physical or financial
capital alone would suffice to bring about development, and several grandiose projects based on that false premise
were undertaken elsewhere, only to end in failure and disillusionment.

8. Paragraph 6 suggests that the failure of some large development projects in places other than Europe was largely
caused by
O the inadequate supply of financial capital for investment
O the use of inefficient technology
O the lack of an adequately skilled and educated workforce
O the fact that the projects were not covered by the Marshall Plan

Paragraph 3: The Marshall Plan (officially called the European Recovery Program and based on investment from
the United States) played a crucial role in sparking the European recovery. When funding from the United States
ended in 1951, Europeans kept economic growth going with high levels of savings and investments. ■Much of the
investment went into equipment for new products and processes, since during the preceding years a backlog of
technical innovations had built up that only awaited capital and skilled labor to be employed. ■In effect, the
European economies had stopped growing for an entire generation, operating with obsolete equipment and lagging

第 38 页 共
373 页
2021 托福阅读真题

behind in technical progress. ■Thus technological modernization both accompanied and was an important
contributory factor to the so-called economic miracle.

Paragraph 4: ■Other major factors were the attitude and role of governments. They participated in economic life
both directly and indirectly on a much larger scale than previously. They nationalized some basic industries, drew up
economic plans, and provided a wide range of social services. Nevertheless, private enterprise was responsible for by
far the largest part of economic activity. On average, between one-fourth and one-third of national income in
Western Europe originated in the government sector. Though this proportion was much greater than it had been
before the war. it was less than half the contribution of the private sectors of the economy. In the mixed economies
that became characteristic of the Western democracies, government assumed the tasks of providing overall stability,
a climate favorable to growth, and minimal protection for the economically weak and underprivileged, but it left the
main task of producing the goods and services desired by the population to private enterprise.

9. Look at the four squares that indicate where the following sentence could be added to the passage.
But technological development was not the only element in the European economic recovery.

10. Directions: An introductory sentence for a brief summary of the passage is provided below. Complete the
summary by selecting the THREE answer choices that express the most important ideas in the passage. Some
sentences do not belong in the summary because they express ideas that are not presented in the passage or are minor
ideas in the passage. This question is worth 2 points.

During the 25 years after the Second World War, the industrialized countries of the world experienced strong
economic growth.

A. Economic growth was fastest in the most advanced of the industrialized countries, which had the most
modernized technology and the highest individual incomes at the end of the war.
B. Initial investment through the Marshall Plan was followed by European investment providing necessary financing
for the technological modernization of European economies.
C. The mix of active government involvement with private enterprise in Western European economies as well as
intergovernmental cooperation were major factors in economic growth.
D. The postwar economic miracle in countries such as Italy and Japan enabled them to join the group of
industrialized countries, but their individual incomes remained lower than average.
E. Governments nationalized basic industries and drew up economic plans, becoming directly or indirectly
responsible for a greater proportion of economic activity than private enterprise.
F. An important contribution to economic progress came from Europe's human capital, which provided the skills and
intelligence needed to make the new technology work well.

第 39 页 共
373 页
2021 托福阅读真题

Hidden Life in Antarctic Nematodos

Paragraph 1: A surprising number of organisms, including some microscopic worms called nematodes, have
evolved adaptations for surviving the extreme conditions of Antarctica. Although they are essentially water-dwelling
animals, many nematodes inhabit some of the continent's driest parts. Some spend most of the year frozen solid and
often almost completely dehydrated (dried out), with their metabolism (chemical processes that are necessary for life)
completely—but only temporarily—shut down. Biologists call this process cryptoblosls, or hidden life. It differs
from hibernation, the period of inactivity experienced by bears and other animals during winter. While a hibernating
animal can drop its metabolic rate by 80 per cent of resting levels, cryptobiosls means complete shutdown, with no
measurable activity. The nematode record for cryptoblosis is 39 years.

1. According to paragraph 1. all of the following statements are true about Antarctic nematodes EXCEPT:
O They are generally considered to be water-dwelling animals.
O They switch back and forth between hibernation and cryptoblosls.
O They are completely inactive for much of the year.
O They can shut down their metabolisms for as long as 39 years.

2. What can be inferred from paragraph 1 about hibernation?


O Hibernating animals never completely shut down their metabolisms.
O Hibernation is considered to be a type of cryptobiosis.
O Hibernating animals do not demonstrate any measurable.
O Hibernation may last for a year or more in some animals.

Paragraph 2: The form of cryptobiosis triggered by extremely dry conditions is called anhydrobiosls, or life without
water. The larvae (the youngest form) of a certain species of fly are the biggest organisms known to survive
near-complete desiccation (drying up), but some of the microscopic nematodes are so good at it that biologist David
Wharton and a colleague put them forward as promising candidates for space travel. The worms were booked on a
space shuttle launch as part of a European Space Agency experiment in the mid-1980s, but in the end they did not
survive the preparatory training when they were put into an extreme vacuum. They may have lost their seat on a
space mission, but nematodes are nevertheless among the hardiest creatures when it comes to surviving water
loss and can last for several years in a state of reversible shutdown, reduced to tiny flakes of shrivelled-up
tissue dust after losing more than 99 per cent of their body water. For Antarctic nematodes, desiccation tolerance
is also a tool for dealing with the cold and avoiding the risk of tissue damage from the formation of ice crystals,
which are larger than the equivalent amount of water and cause cell membranes to burst. Dried-up specimen has no
water left to turn into ice, even if everything else around it is frozen.

3. Why does the author mention that nematodes were booked on a space shuttle launch as part of a European Space

第 40 页 共
373 页
2021 托福阅读真题

Agency experiment in the mid-1980s?


O To show that nematodes are adapted specifically for life in Antarctica and do not do well in other places
O To argue that cryptobiosis is not an effective survival strategy in space
O To provide evidence that nematodes can last for several years in a state of complete shutdown
O To emphasize the strong survival abilities of nematodes

4. Which of the sentences below best expresses the essential information in the highlighted sentence in the passage?
Incorrect choices change the meaning in important ways or leave out essential information.
O Nematodes are able to begin (he shutdown process as long as they have at least one per cent of their body water.
O Nematodes may have lost their seat on a space mission because even though they can survive extreme water loss,
it can take several years for them to completely recover.
O Nematodes’ ability to last for years in an almost completely dehydrated, shutdown state makes them one of the
best organisms at surviving water loss.
O After several years of remaining in shutdown, nematodes lose most of their body water and become tiny flakes of
shrivelled- up dust.

5. According to paragraph 2, why are ice crystals so dangerous to nematodes?


O They change the shape of the nematodes' cells.
O They can cause the nematodes' cell membranes to break.
O They lower the temperature of the nematodes' cells.
O They cause the nematodes' tissues to dry out.

Paragraph 3: Surviving the drying is only half the challenge, though. Water plays a crucial role in keeping cell
membranes intact and elastic, and without a protective mechanism, rehydralion (reacquiring water) could cause as
much tissue damage as thawing. As water molecules are sucked out during desiccation, membranes thicken to a gel.
When water becomes available again, the gel rehydrates once more but leaks as it swells up—causing potentially
fatal damage to cells. The main protective agent is thought to be a simple sugar called trehalose, which replaces lh©
water molecules to keep membranes more fluid and stop them from breaking. Once dry, trehalose turns into a glassy
film, in a process similar to the melting of household sugar into caramelized sweets. The sticky sugar glass acts like
a frame to hold tissue in place. Most nematodes can produce trehalose in large quantities, which could explain why
anhydrobiosls is widespread in this group. But some don't survive water loss despite the (act that they produce the
sugar in response to desiccation stress, and Wharton expects that trehalose is part of a more general response to
stress and only one of several, as yet unknown, adaptation strategies to life in extreme environments.

6. The word elastic in the passage Is closes! in meaning Io


O strong
O moist

第 41 页 共
373 页
2021 托福阅读真题

O flexible
O supported

7. According to paragraph 3, how does trehalose help nematodes survive?


O It protects cells from damage during freezing.
O It stops water from entering cells too quickly during rehydration.
O It helps slow water loss during desiccation.
O It prevents cell membranes from breaking due to desiccation stress.

Paragraph 4: Compared with cold-tolerant insects, which have usually evolved one dominant strategy to cope with
freezing temperatures, nematodes are more flexible and rely on several approaches, each depending on the specific
conditions of their environment. Wharton's model nematode has at least four survival options. In a completely
desiccated state, it avoids freezing and survives at minus 80 degrees Celsius (-112°F) for a month or more. If it
cannot avoid contact with ice and the crystals form rapidly, it survives complete freezing, including the Interior of its
cells, by keeping ice crystals small so they do not break the cells. If it is exposed to temperatures just below zero
degrees Celsius (32°F) and ice formation is slow, it dehydrates slowly as it cools and avoids freezing. And finally, as
an embryo or larva, it is protected by an eggshell that allows it to cool down without freezing while encased in ice.

8. According to paragraph 4, under what circumstances can a nematode survive complete freezing?
O It protects cells from damage during freezing.
O It stops water from entering cells too quickly during rehydration.
O It helps slow water loss during desiccation.
O It prevents cell membranes from breaking due to desiccation stress.

Paragraph 2: ■ Tho form of cryplobiosis triggered by extremely dry conditions is called anhydrobiosis, or life
without water. ■ The larvae (the youngest form) of a certain species of fly are the biggest organisms known to
survive near-complete desiccation (drying up), but some of the microscopic nematodes are so good at it that biologist
David Wharton and a colleague put them forward as promising candidates for space travel. ■ The worms were
booked on a space shuttle launch as part of a European Space Agency experiment in the mid-1980s, but in the end
they did not survive the preparatory training when they were put into an extreme vacuum. ■They may have lost their
seat on a space mission, but nematodes are nevertheless among anhydrobiosls creatures when It comes to surviving
water loss and can last for several years in a state of reversible shutdown, reduced to tiny flakes of shrivelled -up
tissue dust after losing more than 99 per cent of their body water. For Antarctic nematodes, desiccation tolerance is
also a tool for dealing with the cold and avoiding the risk of tissue damage from the formation of ice crystals, which
are larger than the equivalent amount of water and cause cell membranes to burst. A dried-up specimen has no water
left to turn into ice, even if everything else around it Is frozen.

第 42 页 共
373 页
2021 托福阅读真题

9. Look at the four squares [■] that indicate where the following sentence could be added to the passage.
Typically, only very small organisms are able to survive such conditions
Where would the sentence best fit?

10. Directions: An introductory sentence for a brief summary of the passage is provided below. Complete the
summary by selecting tho 3 answer choices that express Ihe most important Ideas in the passage. Some sentences do
not belong In the summary because they express Ideas that are not presented in Ihe passage or are minor ideas in the
passage. This question is worth 2 points.

Nematodes have developed special adaptations that allow thorn to survive in Antarctica.

O Although some fly species enter anhydrobiosis when they are larvae, nematodes are the only known species to
enter anhydrobiosls as adults.
O Nematodes can appear to be In a state of complete shutdown for many years, but experiments have actually
measured periodic metabolic activity during this time.
O Nematodes cope with cold, dry conditions by entering an almost completely dehydrated state, as any water left in
their cells can turn to Ice crystals and cause damage.
O A protective mechanism makes It possible for Antarctic nematodes to emerge from the dehydrated state safely
without serious damage to their cells.
O Nematodes have a variety of methods to avoid or survive freezing temperatures, depending on the environmental
conditions and their developmental stage.
O Wharton and others have observed that the production of trehalose is unrelated to survival after drying out, leading
them to suggest that it may not be an adaptation to Freezing temperatures.

第 43 页 共
373 页
2021 托福阅读真题

The Impact of Farming

Paragraph 1: Across the world in the seven thousand years or so after 10,000 BCE, most humans abandoned a
largely mobile way of life and settled in villages to tend their crops and herds. Gathering and hunting groups had few
possessions, and the social structure within their small groups was largely egalitarian. A few people might be experts
at making particular tools or finding types of food; one or two among the elders might be responsible for the
traditions and beliefs of the group and therefore have special status. Land was not owned and neither were its
resources—they were available for whoever chose to take them. Food was shared within the group.

1. The word “ abandon” in the passage is closest in meaning to


A. changed to
B. challenged
C. varied
D. gave up

2. According to paragraph 1, which of the following was true about humans before they adopted farming?
A. They settled in villages located near animal herds.
B. There was no tradition of land belonging to individuals.
C. Their social status was determined by their ability to find food.
D. Each member of the group specialized in at least one task.

Paragraph 2: All this changed with farming. The creation of fields for the cultivation of crops and the herding of
animals meant that land, together with its products, came to be owned, perhaps first by the community but very
rapidly by individuals. The most fundamental change was that although farming required far greater effort than
gathering and hunting, it did produce far more food—more than the farmers and their immediate families could
consume. This food surplus was the foundation for all later social and political change. It could be used to support
people in occupations other than farming—specialist craftspeople, religious functionaries, and eventually political
and military leaders. The key question is how this surplus was taken from the farmer and by whom. At first it
was probably given up voluntarily to support those functions, especially the religious ones, which were regarded as
important by the community. However, even relatively small-scale agricultural societies appear to have developed
hierarchies that were often headed by chiefs and clan leaders with authority to redistribute the food surplus. The
specialists who were not farmers therefore came to rely on these mechanisms in order to survive. Over a long period
of time, the methods of food redistribution within society generally appear to have become coercive.

3. According to paragraph 2, which of the following made social change possible in farming societies?
O The production of surplus quantities of food
O The involvement of entire families in farm labor

第 44 页 共
373 页
2021 托福阅读真题

O The sharing of food resources with nearby communities


O The owning and herding of animals

4. Why does the author mention that "The key question is how this surplus was taken from the farmer and by
whom'?
A. To demonstrate that farmers continued to influence decisions about food sharing
B. To explain the relationship between farmers and religious functionaries
C. To introduce a discussion of how surplus food was transferred from growers to non-farmers
D. To suggest increasing competition between farmers and nonfarming specialists

5. Paragraph 2 implies that leaders in farming societies ensured their authority by


A. forcing farmers to provide food to feed others
B. taking land from some farmers and redistributing it to others
C. continuing to use some hunting and gathering skills
D. performing religious ceremonies to protect harvests

Paragraph 3: The adoption of farming had other important consequences. Although some of the dwellings
constructed by semisedentary gathering and hunting groups could be quite elaborate, once humans settled in one
place, houses became even more elaborate. Across southwest Asia, sun-dried mud-brick houses (usually with flat
roofs) were fairly standard, whereas in central and western Europe and China, timber houses were the norm. New
technologies were also required once mobility was abandoned. After the year's harvest, storage was essential, leading
to the construction of grain bins and pits. Water was also vital, and the making of clay pots, at first by building up
rolls of clay and firing them in open fires, developed everywhere (in southwest Asia about 6000 BCE) Most
gatherers and hunters used digging sticks, and the earliest farmers did the same to plant their crops. Wooden an d
stone-bladed hoes capable of breaking up soft soils followed. The farmers also needed tougher working
edges-ground- and polished-stone tools to harvest crops and axes to clear land. This placed a premium on the best
stones, especially obsidian, and networks to trade them over long distances soon emerged.

6. The word elaborate in the passage is closest in meaning to


A .basic
B. practical
C. original
D .complicated

7. Why does the author mention "the construction of grain bins and pits'?
A.To describe some unusual features found in the newly developed houses
B.To give examples of new technologies that were made necessary by farming
C. To emphasize that farmers were skilled in building as well as in planting crops

第 45 页 共
373 页
2021 托福阅读真题

D. To compare this storage method with one used previously

Paragraph 4: Although farming enabled humans to produce more food and therefore population numbers slowly
rose, it had a number of important drawbacks. The early farmers depended on a much smaller range of plants than
gatherers and hunters, and were therefore much more vulnerable to crop failures due to poor weather, plant diseases,
and wastage during storage. In most cases the harvest was only barely sufficient to last through the winter and spring
until the new crop was available. A couple of bad harvests in succession could be catastrophic. Storage of food also
made it vulnerable to theft and to conflict between different groups. The nonfarming specialists were particularly
vulnerable to food shortages and famines, especially if redistribution within society broke down. Having settled in
one place, early farmers had, within a few generations, lost the knowledge and techniques to revert to a mobile way
of life that enabled gathering and hunting groups to avoid the worst consequences of food shortage. Farmers could
also severely damage the local environment. Cutting down trees to create farming land, to build houses, and to
provide fuel for heating and cooking could have a devastating effect in vulnerable areas.

8. Paragraph 4 mentions all of the following as drawbacks of farming EXCEPT:


A.It was possible for crops to fail.
B.The amount of food harvested was often hardly enough for survival.
C.It prevented workers from performing other necessary tasks.
D.People who were not farmers could easily experience food shortages.

Paragraph 3: The adoption of farming had other important consequences. Although some of the dwellings
constructed by semisedentary gathering and hunting groups could be quite elaborate, once humans settled in one
place, houses became even more elaborate. Across southwest Asia, sun-dried mud-brick houses (usually with flat
roofs) were fairly standard, whereas in central and western Europe and China, timber houses were the norm. New
technologies were also required once mobility was abandoned.■After the year's harvest, storage was essential,
leading to the construction of grain bins and pits.■Water was also vital, and the making of clay pots, at first by
building up rolls of clay and firing them in open fires, developed everywhere (in southwest Asia about 6000 BCE)
■Most gatherers and hunters used digging sticks, and the earliest farmers did the same to plant their crops.■ Wooden
and stone-bladed hoes capable of breaking up soft soils followed. The farmers also needed tougher working
edges-ground and polished-stone tools to harvest crops and axes to clear land. This placed a premium on the best
stones, especially obsidian, and networks to trade them over long distances soon emerged.

9. Look at the four squares [■] that indicate where the following sentence could be added to the passage.
One challenge was to devise ways to deal with the sudden abundance of crops.
Where would the sentence best fit?

10. Directions: An introductory sentence for a brief summary of the passage is provided below. Complete the
summary by selecting the THREE answer choices that express the most important ideas in the passage. Some

第 46 页 共
373 页
2021 托福阅读真题

sentences do not belong in the summary because they express ideas that are not presented in the passage or are minor
ideas in the passage. This question is worth 2 points.

The development of farming led to significant changes in early societies.

A.In the early hunting and gathering societies, those who owned the resources usually shared them with most of the
other members of the group.
B.While people in hunting and gathering societies generally held the same social status, farming brought about a
hierarchical social structure, with a few people holding significant authority
C.Religious leaders decided the amount of food produced in a single season, which gave them authority over other
members of their society.
D.Societies developed new technologies, including advances in tools and house construction, in response to the
intensive work and non-mobile lifestyle associated with farming.
E.The development of farming had some disadvantages for society, including the possibility of famine as well as
substantial environmental damage.
F. Both the necessity of creating open fields for farming and the need to use lumber for building houses resulted in a
significant number of forests being cleared.

第 47 页 共
373 页
2021 托福阅读真题

The Domestication of Dogs

Paragraph 1: Dogs, the first animals domesticated by humans, evolved from wolves. The apparent emergence of
protodogs (a stage between wolf and dog) between 26,000 and 30,000 years ago is followed by a gap of over ten
millennia, with no additional, solid evidence indicating the presence of dogs until 15,000 years ago. It seems highly
likely that in numerous places where humans and wolves were living together more than 15,000 years ago, dogs
evolved in what appear to be independent domestication events. This theory about the multiple origins of dog
domestication is supported by the significant diversity in dogs during the late glacial period (13,000 -10,000 years
ago). Small dogs can be identified in southwestern Europe, large dogs (almost wolf-sized) in eastern Europe, and
medium-sized dogs in the Near East. The possibility that the human-dog relationship developed in apparently
disconnected communities as wolves and humans lived together over millennia is not only intriguing but raises a
much deeper question: What is it about this particular cross-species partnership that is so obviously mutually
beneficial that it happened more than once and in more than one place?

1. Which of the following is given in paragraph 1 as evidence for the theory about the multiple origins of dog
domestication?
A. The amount of time that elapsed between the first appearances of dogs in different geographic areas
B. The many different protodogs identified from the period starting around 30,000 years ago until 15,000 years ago
C. The difference in sizes between dogs in parts of Europe and the Near East
D. The different types of human-dog relationships in communities far apart from each other

Paragraph 2: There are many reasons why these two kindred species would unite. The overlapping ecological
niches, similar diets, and comparable social hierarchies of humans and canines made this pairing a natural fit.
Humans and canines are both opportunistic feeders. Early humans were hunter-gatherers who moved from place to
place in search of their next meal. Wolves, minus the intensive gathering practices, are basically the same. Humans
and wolves were hunting many of the same animals, using differing and complementary skills in this pursuit.
Wolves were better at tracking by scent and could easily outrun humans; however, humans were developing
formidable tools to bring down larger animals. Still, that alone is not a sufficient explanation. Wolves were hardly
the only overlapping hunters. Many other large animals were searching for prey and their next meal. Consider, for
example, the lions and bears of pre-Ice Age Europe (around 40,000 years ago), with whom humans did not develop a
partnership. The crucial element that likely came into play is the similar social and hierarchical structure shared by
humans and wolves. Wolves live in packs with a definite alpha (dominant) male and female pair. The rest of the pack
has a rigid order that is kept in check by the alpha male; the order changes rarely and, when it does so, only with
much drama and some inevitable violence. Humans were able to insert themselves into this order, albeit maybe not
deliberately, basically becoming the alpha wolf in the new pack and taking over that position in the dominance
hierarchy.

第 48 页 共
373 页
2021 托福阅读真题

2. Why does the author provide the information that "Wolves were better at tracking by scent and could easily
outrun humans"?
A. To provide evidence for why wolves were superior to humans as hunters
B. To provide specific examples of how humans could benefit from pairing with wolves
C. To indicate the advantage wolves had in comparison to the intensive gathering practices of humans
D. To state the reason why wolves were not afraid of being around humans

3. The word "that" in the passage refers to


A. the ability of both humans and wolves to move quickly from place to place
B. the intensive gathering practices of humans
C. the different, but mutually beneficial, skills humans and wolves used in hunting
D. humans developing tools for bringing down larger animals

4. The word "inevitable" in the passage is closest in meaning to


A. brief
B. terrible
C. unavoidable
D. minor

Paragraph 3:There are very few opportunistic social hunters who do their primary hunting during the day. But both
humans and wolves do so. This combination of being a social, or pack, animal and hunting when they could see and
be seen made the two species natural cohabitants, sometimes competitors, and, eventually, allies. Then, after this
initial overlap of hunting space was reconciled, certain wolves, like the ones who were not alpha in their own
pack, must have discovered that eating the scraps humans threw away, or eating the bones that humans could
not eat, was an alternative and fairly reliable way to survive. These wolves, having become somewhat
accustomed to living around humans, began to live in even closer proximity to them. Their initial instinct to flee
from this human hunter was overridden by the benefits of the scraps found at the edges of settlements or left behind
on the hunting trail. Humans, protecting themselves, would have either scared off or killed the wolves who were
particularly threatening.

5. Which of the sentences below best expresses the essential information in the highlighted sentence in the passage?
Incorrect choices change the meaning in important ways or leave out essential information.
A. The initial overlap of hunting space was resolved when certain wolves began eating scraps and bones thrown
away by humans.
B. Wolves needed to find an alternative and reliable way to survive when they were not alpha in their own pack and
when their hunting spaces overlapped with those of humans.
C. Humans resolved the initial overlapping hunting space with wolves by throwing away scraps and bones they
could not eat for certain wolves to eat.

第 49 页 共
373 页
2021 托福阅读真题

D. After the overlap of hunting space was reconciled, some wolves, such as those who were not alpha, found a
reliable way to survive by eating what humans threw away or could not eat.

6. The phrase "accustomed to" in the passage is closest in meaning to


A. influenced by
B. exposed to
C. satisfied with
D. used to

7. According to paragraph 3, in which of the following ways was the hunting behavior of humans and wolves
different from that of most opportunistic social hunters?
A. They avoided hunting the same animals as other species.
B. They changed their hunting patterns in order to become allies with competing hunters.
C. They did most of their hunting during the day.
D. They left behind scraps from hunting that other species later found and ate.

Paragraph 4: Eventually these close wolf neighbors who had adapted somewhat to human camps began to breed
with each other, thus overturning their previous dominance hierarchy (since none of them were alpha wolves, which
are usually the only wolves that breed in a pack).The alpha wolves were the ones who, not really needing to eat from
the trash, had chosen to stay away altogether, or whom humans had killed because of their aggression. Some wolves
found new niches close to humans and decided to live there rather than with their pack because it was a better bet for
their own survival. According to this scenario, humans did not domesticate dogs through deliberate selection; wolves
and dogs actively participated in the project.

8. Which of the following can be inferred from paragraph 4 about alpha wolves?
A. They were successful hunters.
B. They ate from the trash as well as hunted.
C. They became less aggressive when near humans.
D. They led their packs in adapting to life near humans.

Paragraph 1: Dogs, the first animals domesticated by humans, evolved from wolves. █The apparent emergence of
protodogs (a stage between wolf and dog) between 26,000 and 30,000 years ago is followed by a gap of over ten
millennia, with no additional, solid evidence indicating the presence of dogs until 15,000 years ago. █ It seems
highly likely that in numerous places where humans and wolves were living together more than 15,000 years ago,
dogs evolved in what appear to be independent domestication events. █This theory about the multiple origins of dog
domestication is supported by the significant diversity in dogs during the late glacial period (13,000 -10,000 years
ago). █Small dogs can be identified in southwestern Europe, large dogs (almost wolf-sized) in eastern Europe, and

第 50 页 共
373 页
2021 托福阅读真题

medium-sized dogs in the Near East. The possibility that the human-dog relationship developed in apparently
disconnected communities as wolves and humans lived together over millennia is not only intriguing but raises a
much deeper question: What is it about this particular cross-species partnership that is so obviously mutually
beneficial that it happened more than once and in more than one place?

9-Look at the four squares █ that indicate where the following sentence could be added to the passage.
Consequently, it is difficult to determine precisely when dogs became a fully separate species from wolves.
Where would the sentence best fit? Click on a square █ to add the sentence to the passage.

10-Directions: An introductory sentence tor a brief summary of the passage is provided below. Complete the
summary by selecting the 3 answer choices that express the most important ideas in the passage. Some sentences do
not belong in the summary because they express ideas that are not presented in the passage or are minor ideas in the
passage. This question is worth 2 points. Drag your choices to the spaces where they belong. To review the passage,
click on View Text.

Dogs resulted from the gradual domestication of wolves over thousands of years.

A. The first attempt by humans to domesticate wolves resulted in the emergence of protodogs, but this new species
was not successful and quickly disappeared.
B. Once humans developed tools to hunt larger animals, they became more effective than wolves hunting the same
animals, leaving the wolves to depend on scraps from humans.
C. Less aggressive wolves found it beneficial to live close to human settlements, making them particularly suitable
for domestication.
D. It appears that when domesticated dogs appeared, they emerged independently in many places.
E. Among several factors contributing to the domestication of dogs, the main ones were probably similar social
structures and hunting behaviors of humans and wolves.
F. After humans successfully killed off alpha wolves, the disruption in the dominance hierarchy caused other wolves
to come closer to humans, resulting in their domestication.

第 51 页 共
373 页
2021 托福阅读真题

The Sound Revolution in American Film

Paragraph 1: In early 1928, almost all movie productions in the United States centered in Hollywood were silent
films, but by the end of the year, virtually every production company in the country was making only sound films.
During that year the studios found it necessary to stop all production for significant periods to convert their facilities
and train technical staff for talking pictures. Even the basic organization of the industry would be changed by the
new technology. Coordinating sound production with film production required far more hands-on management of the
process. As a result, producers who oversaw technical and financial aspects of filmmaking became more powerful in
relationship to directors, who guided actors through their performances. There had never been a uniform pay scale
for actors. Each film production studio had its own employment rules. But sound production necessitated far more
rehearsal time, which some studios paid and others didn't. This led to discontent and the trend toward the formation
of labor unions in the industry. This unionization movement gained force when film projector operators, whose
work (showing films in theaters) was greatly complicated by sound, organized and bargained for better
working conditions and rights of seniority (long service), benefits that other trades soon also sought to acquire
by joining the movement.

1. The word “discontent” in the passage is closest in meaning to


A. inequality
B. greater expense
C. reorganization
D. unhappiness

2. Which of the sentences below best expresses the essential information in the highlighted sentence in the passage?
Incorrect choices change the meaning in important ways or leave out essential information.
A. Film projector operators strengthened the movement to unionize workers because sound had complicated their
work, and they were soon joined in the movement by other trades.
B. The unionization movement used force to organize and bargain for the benefits of film projector operators and
other trades.
C. Film projector operators and later other trades greatly complicated the unionization movement by bargaining for
better working conditions and rights of seniority.
D. The unionization movement was able to help film projector operators and other trades organize their work, which
had become complicated by sound.

3. Paragraph 1 suggests that sound film contributed to the unionization movement because of
A. the need to train additional technical staff
B. the increased and often unpaid time needed for rehearsal
C. the changed relationship between producers and directors

第 52 页 共
373 页
2021 托福阅读真题

D. the pressure on projector operators to do the work of other trades

Paragraph 2:The careers of actors, directors, and writers were disrupted by uncertainty about their ability to adapt.
The motion picture industry's leaders assumed that silent picture screenwriters couldn't write dialogue: that actors
would suffer from" microphone nerves" and become unable to perform; and that directors would be equally
challenged by the new medium. For a brief period the entire industry shifted its attention from Hollywood to New
York, where a pool of talent awaited because of New York's many theaters and radio stations. For New York stage
actors who performed live in theaters, the higher wages offered by the film industry was a boon Playwrights
accustomed to living on the meager wages paid by the stage suddenly found themselves courted by the studios and
offered unheard of sums of money. The cost of buying the rights for successful plays to turn them into films also
soared.

4. According to paragraph 2, which of the following was an assumption of industry leaders with respect to changes
in the motion picture industry?
A. Hollywood would provide more talented actors and writers for movie production than New York would.
B. Playwrights should be paid the same to write films as they were when writing for the stage.
C. Talented actors who worked in theater or radio could not make the adjustment to movies.
D. Screenwriters of silent movies lacked the ability to write dialogue, and silent movie actors could not perform
dialogue

Paragraph 3: In Hollywood, silent actors began to enroll in voice and pronunciation classes. It was assumed that the
voice of talking pictures would sound cultured, imitating the accents of the aristocracy. People with perfectly
acceptable speaking voices, perhaps regionally accented or unaccented, were trained to talk as if they had been to
college at Oxford or Cambridge in England. This rejection of naturalism in talking pictures resulted in films that
were awkward and sometimes unintentionally comedic. One early technique used in dialogue recording was for each
speaker to pause for an instant before beginning their lines in the belief that audiences would need the time to keep
up with what was being said. This effect came off sounding slow and ponderous and artificial. Sound engineers
declared that the letter "s" could not be properly reproduced by the technology. This caused writers to tear out their
hair out trying to avoid the offending letter in dialogue scenes.

5. According to paragraph 3, which of the following made early sound films awkward?
A. The use of actors who had been trained in England
B. The training of actors to speak in unnatural accents that were thought to be sophisticated
C. The fast rate at which actors spoke
D. The addition of artificial pauses into recordings by sound engineers

Paragraph 4: The significant increase in the expense of production caused by sound forced the studios to embark on

第 53 页 共
373 页
2021 托福阅读真题

a campaign of cost cutting. An early casualty of the talking era were theater and studio musicians, who had provided
background music for silent films. However, a new opportunity arose for them at the same time: contributing the
theme song. Theme songs were used extensively in radio to identify programs as well as to promote films in advance
of their release. One studio instructed directors that they were allowed only the expense of printing two takes or
versions of each scene. This put tremendous pressure on everyone, especially camera operators, to get it right the
first time. Following the lead of projectionists, cameramen were soon demanding and receiving better pay and
working conditions as a result of the greater responsibility they were bearing. Another economizing move was to fire
all of the "gag" men, comedy writers who routinely were assigned to movie sets to add little bits of comedy as the
film was being shot. Ordinary actors became restless. They weren't customarily paid for rehearsals, but sound
necessitated they spend much more time in preparation. In September 1928, film industry representatives announced
that actors would henceforth be paid for rehearsals on talking films.

6. The word "routinely" in the passage is closest in meaning to


A. regularly
B. previously
C. interestingly
D. individually

7. According to paragraph 4, which of the following happened to some of the theater and studio musicians as the age
of sound films arrived?
A. They moved from working in the film industry to working in radio.
B. They demanded and received better pay and working conditions.
C. They began to record background music used in sound films when actors were not speaking.
D. They started to produce theme songs.

8. In paragraph 4, why does the author include the information that one studio limited the number of takes of each
scene to two?
A. To point out a development in cost cutting that led cameramen to demand higher pay and better working
conditions
B. To provide an example of the efforts that some studios made to produce sound films earlier than other studios
C. To emphasize that directors were often in conflict with camera operators
D. To introduce an unintended result of using theme songs in movies

Paragraph 3: In Hollywood, silent actors began to enroll in voice and pronunciation classes. It was assumed that the
voice of talking pictures would sound cultured, imitating the accents of the aristocracy. █ People with perfectly
acceptable speaking voices, perhaps regionally accented or unaccented, were trained to talk as if they had been to
college at Oxford or Cambridge in England. █ This rejection of naturalism in talking pictures resulted in films that

第 54 页 共
373 页
2021 托福阅读真题

were awkward and sometimes unintentionally comedic. █ One early technique used in dialogue recording was for
each speaker to pause for an instant before beginning their lines in the belief that audiences would need the time to
keep up with what was being said. █This effect came off sounding slow and ponderous and artificial. Sound
engineers declared that the letter "s" could not be properly reproduced by the technology. This caused writers to tear
out their hair out trying to avoid the offending letter in dialogue scenes.

9. Look at the four squares █ that indicate where the following sentence could be added to the passage.
There were other practices that had similar effects.
Where would the sentence best fit? Click on a square █ to add the sentence to the passage.

10.Directions: An introductory sentence tor a brief summary of the passage is provided below. Complete the
summary by selecting the 3 answer choices that express the most important ideas in the passage. Some sentences do
not belong in the summary because they express ideas that are not presented in the passage or are minor ideas in the
passage. This question is worth 2 points. Drag your choices to the spaces where they belong. To review the passage,
click on View Text.

The introduction of sound into movies had number of different effects on the film industry.

A. The popularity of sound films led to the closing down of studios that could not obtain the technology or staff to
produce them.
B. In response to assumed challenges that sound films would present various industry professionals, the film
industry turned for a time to New York to engage the services of stage actors and playwrights.
C. As the film industry adapted to sound some early techniques made films seem unnatural
D. The greater demands that the production of sound films placed on workers and the studios' cost-cutting
procedures contributed to unionization among laborers in the film industry
E. Instead of using silent film actors in sound films, studios hired other actors who could perform with a variety of
different accents.
F. As a result of film industry workers' demands for better pay, studios cut costs in other ways such as by using fewer
songs for background music and by reducing the number of scenes in films.

第 55 页 共
373 页
2021 托福阅读真题

The Global Industrial Revolution

Paragraph 1: In fewer than a hundred years, the global process of industrialization utterly transformed Great
Britain, most of continental Europe, Japan, and the United States. Its social and economic consequences were
complex and profound, and they continue today to shape the nature of global society. The emergence of modern
industrialism was already well underway in many parts of the world by the end of the eighteenth century. The
Industrial Revolution was largely a product of rapid changes in Great Britain, which was the first nation to have
significant industrial capacity. The factory system took root there in the late eighteenth century and revolutionized
the manufacture of cotton thread and cloth. one invention following another in quick succession. Water, wind, and
animal power continued to be important in the textile industry. But more important was the emergence of steam
power, which began to proliferate after the appearance of James Watt's advanced steam engine in 1769. Cumbersome
and inefficient by modern standards, Watt's engine was nevertheless a major improvement over the earlier
atmospheric engine of Thomas Newcomen. Britain's textile industry quickly became the most profitable in the world,
and it helped encourage comparable advances in other fields of manufacturing as well. Despite the efforts of the
British government to prevent the export of industrial technology, knowledge of the new machines reached other
nations quickly. usually through the emigration of people who had learned the technology in British factories.

1. The word utterly in the passage is closest in meaning to


A. permanently
B. famously
C. usefully
D. completely

2. Paragraph 1 suggests which of the following about the role of steam power in Britain's industrial revolution?
A. It led to far more social consequences than economic consequences.
B. It was less important than water, wind,and animal power.
C. It was abandoned after the development of Newcomen's atmospheric engine.
D. It led to the rapid profitability of the textile industry.

3. According to paragraph 1, Great Britain attempted to maintain its industrial dominance by


A. seeking a less cumbersome alternative to Watt's advanced steam engine
B. promoting the spread of new technology from textiles to other fields
C. attracting skilled workers from other countries to work in British factories
D. limiting the availability of its own technology in other countries

Paragraph 2: The United States benefited the most from British technology, because it received more immigrants
from Great Britain than any other country. But British technology spread quickly to continental Europe as well.

第 56 页 共
373 页
2021 托福阅读真题

Belgium was the first, developing a significant coal, iron, and armaments (military equipment) industry in the early
nineteenth century. France—profiting from the immigration of approximately fifteen thousand British workers
with advanced technological skills—had created a substantial industrial capacity in textiles and metals by the
end of the 1820s, which in turn contributed to a areat boom in railroad construction later in the century.
German industrialization progressed rapidly after 1840, beginning with coal and iron production and then, in the
1850s, moving into large-scale railroad construction. By the late nineteenth century, Germany had created some of
the world's largest industrial corporations. In Japan, the sudden intrusion of American and European traders helped
cause the so-called Meij reforms of the 1880s and 1890s, which launched a period of rapid industrialization there as
well.

4. Which of the sentences below best expresses the essential information in the highlighted sentence in the passage?
Incorrect choices change the meaning in important ways or leave out essential information.
A. The arrival of numerous skilled British workers had benefited France's textile and metal industries by the end of
the 1820s, and this contributed to a later railroad boom.
B. France profited from the immigration of approximately fifteen thousand skilled British workers, who worked with
French workers to construct the first railroad in France.
C. Despite the migration of thousands of its workers to Britain, France built large and profitable industries in textiles,
metals, and other manufacturing by the late nineteenth century.
D. France attracted approximately fifteen thousand British workers during the nineteenth century because it had
developed industries in textiles, metals, and railroad construction.

5. According to paragraph 2, which of the following is true about the industrialization of Japan?
A. It occurred much more gradually than the industrialization of Europe.
B. It was possible thanks to investments by Germany's large industrial corporations.
C. It was indirectly caused by the presence of foreign traders in Japan.
D. It led to the Meiji reforms of the 1880s and 1890s.

Paragraph 3: Industrialization changed not just the world's economies but also its societies. First in Britain and then
in Europe, America, and Japan, social systems underwent revolutionary changes. Hundreds of thousands of men and
women moved from rural areas into cities to work in factories, where they experienced both the benefits and the
costs of industrialization. The standard of living of the new working class, when objectively quantified, was usually
significantly higher than that of the rural poor. Most of those who moved from farm to factory, in other words,
experienced some improvement in nutrition and other material circumstances and even in their health. But the
psychological costs of being suddenly uprooted from one way of life and thrust into a fundamentally different one
could outweigh the economic gains. There was little in most workers' prior experience to prepare them for the nature
of industrial labor. It was disciplined, routinized work with a fixed and rigid schedule, a sharp contrast to the varying
seasonal work pattern of the rural economy. Industrial workers, too, experienced a fundamental change in their

第 57 页 共
373 页
2021 托福阅读真题

relationship with their employers. Unlike rural landlords and local aristocrats, factory owners and managers—the
new class of industrial capitalists, many of them accumulating unprecedented wealth—were usually remote and
inaccessible figures. They dealt with their workers impersonally and the result was a growing division between the
two classes—each lacking access to or an understanding of the other. Working men and women throughout the globe
began thinking of themselves as a distinct class with common goals and interests. And their efforts simultaneously to
adjust to their new way of life and to resist its most damaging aspects sometimes created great social turbulence.
Battles between workers and employers became a characteristic feature of industrial life throughout the world.

6. In describing the standard of living of the working classes as higher when objectively quantified, the author means
that this standard of living was higher when measured
A. carefully
B. directly
C. without bias
D. the same way over time

7. In paragraph 3, why does the author explain that there were improvements in the nutrition, material
circumstances,and health of the new working class?
A. To demonstrate that a worker's standard of living could be objectively quantified
B. To support the claim that city workers generally had a higher standard of living than rural workers did
C. To argue that the economic effects of industrialization outweighed the psychological effects
D. To explain why working people believed they had common goals and interests

8. According to paragraph 3, all of the following are true of workers who moved from rural areas into cities
EXCEPT:
A. They experienced some negative psychological consequences as a result of the move.
B. They were not prepared for the transition from rural life to that of cities.
C. They grew more impersonal in their social relationships outside of work.
D. They began to think of themselves as members of a distinct social group.

Paragraph 1: In fewer than a hundred years, the global process of industrialization utterly transformed Great Britain,
most of continental Europe, Japan, and the United States. ■Its social and economic consequences were complex and
profound, and they continue today to shape the nature of global society. ■The emergence of modern industrialism
was already well underway in many parts of the world by the end of the eighteenth century. ■The Industrial
Revolution was largely a product of rapid changes in Great Britain, which was the first nation to have significant
industrial capacity. ■The factory system took root there in the late eighteenth century and revolutionized the
manufacture of cotton thread and cloth. one invention following another in quick succession. Water, wind, and
animal power continued to be important in the textile industry. But more important was the emergence of steam

第 58 页 共
373 页
2021 托福阅读真题

power, which began to proliferate after the appearance of James Watt's advanced steam engine in 1769. Cumbersome
and inefficient by modern standards, Watt's engqine was nevertheless a major improvement over the earlier
atmospheric engine of Thomas Newcomen. Britain's textile industry quickly became the most profitable in the world,
and it helped encourage comparable advances in other fields of manufacturing as well. Despite the efforts of the
British government to prevent the export of industrial technology, knowledge of the new machines reached other
nations quickly. usually through the emigration of people who had learned the technology in British factories.

9. Look at the four squares ■ that indicate where the following sentence could be added to the passage.
Yet, while it spread globally it originated in a single nation.
Where would the sentence best fit? Click on a square ■ to add the sentence to the passage.

10. Directions: An introductory sentence for a brief summary of the passage is provided below. Complete the
summary by selecting the THREE answer choices that express the most important ideas in the passage. Some
sentences do not belong in the summary because the express ideas that are not presented in the passage or are minor
ideas in the passage. This question is worth 2 points.

The emergence of modern industrialism was well underway in many parts of the world by the end of the
eighteenth century.

A. Industrialization began in Britain and proceeded rapidly after the appearance of Watt's steam engine in 1769,
revolutionizing first the textile industry and then other areas of manufacturing.
B. Emigrants from Britain rapidly spread the new technology, and a substantial industrial capacity soon developed in
America and Europe, and eventually Japan.
C. As a result of industrialization.many rural workers fed to cites where their lives were so changed by factory work
and urban living that they began to see themselves as a distinct class.
D. British industry was able to grow rapidly in part because of its large supply of workers, but this situation
encouraged skilled workers to migrate to countries that were less industrialized.
E. Industrialization occurred more rapidly in European countries that invested in large-scale railroad systems and in
countries. such as Japan, whose economies depended heavily on trade.
F. The hundreds of thousands of new city workers brought varied experiences and ways of living from rural
Britain,leading to tensions in the factories and a failure to recognize common interests.

第 59 页 共
373 页
2021 托福阅读真题

Grasslands as Agents of Transformation

Paragraph 1: The development of grasslands about 30 million years ago facilitated significant transformations on
Earth. Growing quickly after fire and drought, grasslands sustain herds of large plant eaters,and they do so with an
efficiency that early dry land vegetation could not Grass leaves have the unusual ability to continue growing from
near their base. When the top of a grass leaf is destroyed by fire or eaten by cattle, the leaf can keep right on
knowing. Even today, grasslands support our most important food animals. Grasses also provided the grains that
helped build great civilizations.

1. In paragraph 1, why does the author discuss the destruction of grass leaves by fire or cattle?
A. To describe the severe damage that nature can do to vegetation
B. To explain the qualities of grasses that helped grasslands transform the Earth
C. To support the argument that there is a continuous need for dry land vegetation
D. To question whether large plant eaters actually threaten grasslands

Paragraph 2: Grasslands have been expanding ever since their earliest appearance on earth. However, the fact that
non-woody plants rarely fossilize means that we have to study the expansion of grasslands indirectly. The
deep-rooted teeth of fossil plant eaters that grazed sharp grasses that could wear down these teeth provide one source
of evidence of grassland expansion. Fortunately, there are additional sources of evidence paleo-botanist Gregory
Retallack has explored grassland expansion by studying fossil soils.

2. According to paragraph 2, why must the early expansion of grasslands be studied indirectly?
A. Early grasslands on Earth differed significantly from the grasslands of today.
B. Early grasslands were largely consumed by plant eaters.
C. Unlike other plants, grasses do not easily become preserved as fossils.
D. The effects of the sharp teeth of plant eaters make it difficult to identify grass remains as fossils.

Paragraph 3: Analyzing both contemporary and fossil soils in the American Midwest, Oregon, East Africa, and
elsewhere, Retallack has found intriguing patterns. He notes that grassland soils have smaller particle sizes than do
the soils of forest and woodland. These smaller particles give grassland soils much more surface area than the soils
of woodlands. (Think of how you can increase the surfaces of a cubic foot of wood by simply cutting it into lots of
cubic inches.) By having greater surface area, grassland soils can bind more organic materials, an important factor in
making grassland soils so rich.

3. Paragraph 3 answers all of the following questions about grassland soils EXCEPT:
A. What makes grassland soils so rich?
B. What do all grassland soils have in common?

第 60 页 共
373 页
2021 托福阅读真题

C. Why do grassland soils have smaller particle sizes than do forest or woodland soils?
D. Why do grassland soils bind with more organic materials than do soils in forests or woodlands?

Paragraph 4: Earlier, soil scientists had discovered that soils were much deeper under moist grasslands than under
the drier grasslands of steppe (large areas of unforested grassland as in Europe or Siberia) and semideserts. Thus, soil
depth gives paleontologists a clue as to how dry a prehistoric grassland may have been. What Retallack discovered
was that grassland soils were shallow and apparently confined to drier regions before about six million years ago.
Since then, he found grasslands have been expanding into moister regions—as evidenced by their deeper soils.
Grasslands with deeper soils appear for the first time around six million years ago, and have been spreading into
regions of higher rainfall since that time. Based on these findings,Retallack sets forth a bold hypothesis; he suggests
that expanding prairies (large areas of grasslands in North America) helped bring on the Ice Age!

4. The phrase "confined to" in the passage is closest in meaning to


A. expanded into
B. found in
C. limited to
D. replaced by

5. According to paragraph 4, what happened around six million years ago?


A. Rainfall increased all over the world.
B. Grasslands expanded to steppes and semidesert regions.
C. Grasslands began to appear in wetter regions.
D. Grassland soils became progressively less deep.

Paragraph 5: The soils of moist prairies are deeper, and because of their smaller granularity, such soils provide far
more surface area than did the open woodland soils that prairies replaced—and blind far more organic matter than
was held by the earlier woodland soils. Plants of the prairie store large amounts of organic matter in the soils for a
reason. It is this underground biomass that allows them to recover quickly after long droughts, severe fires, and
intense grazing. The organic matter, in turn, holds a lot of carbon derived from the carbon dioxide pulled out of the
air by photosynthesis. Some of the stored carbon is not returned to the atmosphere, with a consequent loss of carbon
dioxide from the air. And with less carbon dioxide to insulate Earth. Retallack claims that the prairies therefore
contributed to the onset of our recent ice age—beginning about two million years ago.

6. According to paragraph 5, which of the following can be inferred about the soils of moist prairies compared to
those of woodlands?
A. They recover better after becoming damaged than do the soils of woodlands.
B. They contain less organic matter than do the soils of woodlands.

第 61 页 共
373 页
2021 托福阅读真题

C. They provide more carbon dioxide to Earth's atmosphere than do the soils of woodlands.
D. They provide better insulation to resist ice ages than do the soils of woodlands.

7. According to paragraph 5, which of the following statements about carbon in the soil is true?
A. It is present in significant amounts only in grassland areas.
B. It is increased by long droughts, severe fires, and intense grazing.
C. It is eventually transferred entirely from the soil to the atmosphere.
D. It is stored in underground biomass.

Paragraph 6: Working in concert with fire, grasses can erode the edges of woodlands. Grasses promote fires; once
dry, they burn easily, and many of them burn hot. Nearby woody plants, unable to withstand such hot fires, succumb,
and the grassland quickly advances. In addition, grasslands are not as dark green as is the forest canopy, so
grasslands reflect more light back into space. Additional reflected light heats the air column above, making it warmer.
Warmer air holds more moisture than cooler air, so grasslands produce a drying effect and lowered
rainfall—increasing the incidence of fire. Then, too, animal browsing destroys young tree seedlings that might
germinate at the forest's edge. Retallack thinks that the combination of grassland, fire, and grazing animals produced
a new dynamic, allowing grasslands to expand their range—and change the climate! In his words: "Unidirectional,
stepwise, long-term climatic cooling drying, and climate instability may have been driven.....by the coevolution of
grasses and grazers."

8. According to paragraph 6, all of the following contribute to the advance of grasslands across the edges of
woodlands EXCEPT
A. Grasslands increase rainfall, thereby benefiting grassland expansion into woodland areas.
B. Animals browsing on grasslands keep tree seedlings at forest edges from germinating.
C. Fires start easily in dry grasslands and tend to burn hot enough to kill woody plants on their edges.
D. Grasses are able to grow quickly in areas that have been cleared by fire.

Paragraph 5: The soils of moist prairies are deeper, and because of their smaller granularity,such soils provide far
more surface area than did the open woodland soils that prairies replaced—and blind far more organic matter than
was held by the earlier woodland soils. Plants of the prairie store large amounts of organic matter in the soils for a
reason. It is this underground biomass that allows them to recover quickly after long droughts, severe fires,and
intense grazing. ■The organic matter, in turn, holds a lot of carbon derived from the carbon dioxide pulled out of the
air by photosynthesis. Some of the stored carbon is not returned to the atmosphere, with a consequent loss of carbon
dioxide from the air. ■And with less carbon dioxide to insulate Earth. Retallack claims that the prairies therefore
contributed to the onset of our recent ice age—beginning about two million years ago.■

Paragraph 6: Working in concert with fire, grasses can erode the edges of woodlands. ■Grasses promote fires; once

第 62 页 共
373 页
2021 托福阅读真题

dry, they burn easily, and many of them burn hot. Nearby woody plants, unable to withstand such hot fires, succumb,
and the grassland quickly advances. In addition,grasslands are not as dark green as is the forest canopy, so grasslands
reflect more light back into space. Additional reflected light heats the air column above, making it warmer. Warmer
air holds more moisture than cooler air, so grasslands produce a drying effect and lowered rainfall—increasing the
incidence of fire. Then, too, animal browsing destroys young tree seedlings that might germinate at the forest's edge.
Retallack thinks that the combination of grassland, fire,and grazing animals produced a new dynamic, allowing
grasslands to expand their range—and change the climate! In his words: "Unidirectional, stepwise, long-term
climatic cooling drying, and climate instability may have been driven.....by the coevolution of grasses and graz ers."

9. Look at the four squares ■ that indicate where the following sentence could be added to the passage.
But the explanation of how the climate may have changed does not address why grasslands expanded into
areas that were previously dominated by woodlands.

10. Directions: An introductory sentence for a brief summary of the passage is provided below. Complete the
summary by selecting the THREE answer choices that express the most important ideas in the passage. Some
sentences do not belong in the summary because the express ideas that are not presented in the passage or are minor
ideas in the passage. This question is worth 2 points.

Evidence from fossil soils shows that grasslands developed about 30 million years ago and have been
expanding ever since.

A. Grassland soils are composed of small particles that allow them to bind lots of organic matter, and they can
recover quickly after being disturbed because they grow from near their base.
B. Fossil evidence shows that the earliest grasses were extremely sharp,so much so that that they wore down the
deep-rooted teeth of the animals that grazed on them at woodland edges.
C. Increased grasslands may have helped set off the last ice age, at least one scientist thinks, by removing carbon
dioxide from the atmosphere and storing it underground.
D. Paleobiologist Gregory Retallack discovered that grasslands have been expanding since they first appeared on
Earth, moving into areas with deep soil before areas with shallow soil.
E. Grasslands began expanding into wetter regions about six million years ago, helping dry the climate in the areas
into which they spread and increase the fires.
F. Increased animal browsing on tree saplings at the edges of forests made it possible for grasslands to continue
expanding even during recent cool periods.

第 63 页 共
373 页
2021 托福阅读真题

Did Sauropods Live in Swamps

Paragraph 1: The sauropod dinosaurs were the largest animals ever to live on land. They had a long neck and tail, a
roundish body, and four stout legs. For many years, it was widely believed they must have spent most of their time in
swamps, where their great bulk was buoyed up by water. In this way, they would have remained deeply submerged,
breathing with only their nostrils poking out of the water. This idea may have originated as a consequence of the
early nineteenth-century paleontologist R. Owen's mistaken notion of the sauropod Cetiosaurus, which he thought to
be an exceedingly large marine crocodile. It was later argued that sauropods were too big and heavy to have lived on
land, that their legs would not have supported them, and that, like a whale, they needed the buoyancy of water to
permit them to breathe. It was said that the long tails were used for swimming. Finally, it was suggested that the
nostrils at the top of the skull were like the snorkel on a submarine, allowing the animal to breathe while remaining
fully submerged (and presumably hidden and protected). Although as early as 1904 E.S. Riggs argued coherently
that these animals were terrestrial, sauropods were persistently viewed as having a fully aquatic existence, with their
bodies submerged 6 meters or more below the surface of the water and their necks craning to keep the nostrils just
above the water's surface.

1. The word "presumably" in the passage is closest in meaning to


A. partially
B. sufficiently
C. safely
D. most likely

2. According to paragraph 1, all of the following were generally believed about sauropods EXCEPT:
A. Their bodies were too heavy for their legs to support them on dry land.
B. Their long tails helped them to swim.
C. They used their legs for support only when they were almost entirely submerged.
D. They could breathe with only their nostrils above water.

Paragraph 2: The idea of aquatic sauropods began unraveling in the 1950s and especially in the 1960s and 70s. In
1951, K. A. Kermack examined the consequences of water pressure on a submerged sauropod. The pressure on a
body increases with the depth at which it is submerged. ■ If a sauropod was fully submerged, the chest—and hence
the lungs—would have been under some 6 meters of water. ■ The pressure at this depth is nearly twice what it is at
the surface and would tend to collapse the chest, pushing whatever air was in the lungs up and out of the body. ■
How a breath might be taken is hard to say, since the sauropod's lungs would have to be expanded
underwater, pushing outward against considerable pressure, a pressure opposing taking a breath that is
greater than that encountered by any vertebrate living today. ■ Unless sauropods had exceedingly powerful
chest muscles, these animals certainly would not have been able to breathe in. For this reason, Kermack argued, it is

第 64 页 共
373 页
2021 托福阅读真题

perhaps better to envision sauropods as terrestrial animals.

3. Which of the sentences below best expresses the essential information in the highlighted sentence in the passage?
Incorrect choices change the meaning in important ways or leave out essential information.
A. It is hard to say how sauropods breathed while they were submerged because there are no vertebrates today that
breathe at the same rate.
B. Once sauropods forced air from their lungs, pushing it out against the considerable pressure of the water, it is hard
to imagine how they had strength to move underwater.
C. The sauropod's lungs expanded under water in order to oppose the pressure from the surrounding water.
D. It is not clear that submerged sauropods could have breathed, since they would have had to overcome a pressure
on the lungs greater than that encountered by any vertebrate today.

4. In paragraph 2, why does the author remark "Unless sauropods had exceedingly powerful chest muscles"?
A. To explain how sauropods were able to survive the intense water pressure when submerged
B. To argue that aquatic sauropods must have had powerful chest muscles
C. To emphasize the unlikely conclusion under which sauropods could have breathed while submerged
D. To distinguish the requirements for breathing in while underwater from the requirements for breathing out while
underwater

Paragraph 3: Sauropods climbed out of the water for good with Robert Bakker's studies in the late 1960s and 1970s.
This research emphasized the sturdy, pillar-like construction of the legs and feet of sauropods, surely strong enough
for walking—at least slowly—on land. Additionally, Bakker pointed to the narrow, slab-sided thorax (chest)—very
unlike the amphibious hippopotamus but like rhinoceroses and elephants—as evidence of terrestrial habits. Not since
these studies have sauropods been thought of as submerged up to their nostrils.

5. According to paragraph 3, Bakker's research supports which of the following statements about sauropods?
A. Their legs could support the full weight of their bodies.
B. They had a thorax more like that of a rhinoceros than that of an elephant.
C. They used their sturdy, pillar-like legs to climb out of the water.
D. They easily made the transition from underwater to terrestrial habits.

Paragraph 4: So what were the long necks for, if not for keeping the head above water? The answer is not clear.
Sauropod necks have been compared to those of giraffes, inviting the inference that they fed in tall trees. For some
sauropods such as Cetiosaurus, Diplodocus, and Apatosaurus, such feeding habits do not appear possible. While for
others, such as Brachiosaurus, it may have been true. While mounting a partial skeleton of Cetiosaurus at the
Leicestershire Museums in England, J. Martin experimented with neck vertebrae to see what sort of restrictions there
were on its range of motion, concluding that this sauropod was unable to upwardly flex its neck as far as had been

第 65 页 共
373 页
2021 托福阅读真题

thought previously. A dozen years after Marin's work, Kent Stevens and Mike Parrish demonstrated—using
computer modeling—that similar limitations are also found in Diplodocus and Apatosaurus. Their research,
manipulation of a three-dimensional virtual model of the neck vertebrae, provided the feeding envelope (maximal
vertical and horizontal range of head movement) of these two sauropods, again indicating that they generally
browsed for vegetation within 2 or 3 meters of the ground.

6. The word "inviting" in the passage is closest in meaning to


A. assuming
B. strengthening
C. encouraging
D. emphasizing

7. What does the author mean by "again" in the remark "again indicating that they generally browsed for
vegetation within 2 or 3 meters of the ground”?
A. The computer models gave the same results about Diplodocus and Apatosaurus each time they were run.
B. The computer models showed that Diplodocus and Apatosaurus were like Cetiosaurus in having a limited range of
motion in the neck.
C. The computer models showed that the neck vertebrae of Diplodocus and Apatosaurus were similar to the neck
vertebrae of Brachiosaurus.
D. The computer models confirmed that sauropods had the same limited neck movements as giraffes.

8. Paragraph 4 implies that Brachiosaurus differed from Cetiosaurus in which of the following respect?
A. It could raise its neck higher.
B. It was a plant eater.
C. Its neck was longer
D. It did not use its long neck to keep its head above water.

Paragraph 2: The idea of aquatic sauropods began unraveling in the 1950s and especially in the 1960s and 70s. In
1951, K. A. Kermack examined the consequences of water pressure on a submerged sauropod. The pressure on a
body increases with the depth at which it is submerged. ■ If a sauropod was fully submerged, the chest—and hence
the lungs—would have been under some 6 meters of water. ■ The pressure at this depth is nearly twice what it is at
the surface and would tend to collapse the chest, pushing whatever air was in the lungs up and out of the body. ■
How a breath might be taken is hard to say, since the sauropod's lungs would have to be expanded underwater;
pushing outward against considerable pressure, a pressure opposing taking a breath that is greater than that
encountered by any vertebrate living today. ■ Unless sauropods had exceedingly powerful chest muscles, these
animals certainly would not have been able to breathe in. For this reason, Kermack argued, it is perhaps better to
envision sauropods as terrestrial animals.

第 66 页 共
373 页
2021 托福阅读真题

9. Look at the four squares [■ ] that indicate where the following sentence could be added to the passage.
Obviously, this would be fatal if the sauropod could not quickly refill its lungs with air.
Where would the sentence best fit?

10. Directions: An introductory sentence for a brief summary of the passage is provided below. Complete the
summary by selecting the THREE answer choices that express the most important ideas in the passage. Some answer
choices do not belong in the summary because they express ideas that are not presented in the passage or are minor
ideas in the passage. This question is worth 2 points.

The sauropod dinosaurs had a long neck and tail and were the largest animals ever to live on land.

A. The function of sauropods' long necks is not clear; in particular, sauropod species could not in general use their
necks to feed like giraffes on treetops.
B. Until the 1950s, sauropods were generally thought to have been aquatic animals that spent much of their time
with only their heads above water.
C. The ability of sauropods to move around both in water and on land caused early researchers to classify them as
alligators.
D. Because sauropods have a slab-sided thorax, Robert Bakker and K. A. Kermack argued that sauropods would not
have been able to expand their lungs when they were submerged.
E. Computer models of the vertebrae of sauropod necks show that probably only Brachiosaurus was able to raise its
neck about 2 or 3 meters.
F. In view of arguments that sauropods had legs that could support their weight and probably could not have
breathed with their bodies submerged, it is now accepted that sauropods were land dwellers.

第 67 页 共
373 页
2021 托福阅读真题

Evidence of Zooxanthellae in Fossilized Corals

Paragraph 1 : Corals are very small animals that live in large colonies called coral reefs, which can grow rapidly.
Corals provide the habitat for the greatest diversity of life found anywhere in Earth's seas. The ability of reef corals
to grow robust external skeletons made of calcium carbonate at a very rapid rate depends on the relationship they
have with algae called zooxanthellae. Corals have a symbiotic relationship with zooxanthellae, meaning that there is
mutual benefit to their living together. The algae live protected within the soft tissues of the coral, in turn providing
the Coral with organic compounds(glycerol, glucose,and the amino acid alanine) for nutrition. While most reef
corals retain the ability to feed on small animals using their stinging cells, some species are more dependent
nutritionally on their symbiotic zooxanthellae. Furthermore, zooxanthellae photosynthesis releases carbon dioxide
which assists the secretion of the Calcium carbonate skeleton made by the Corals, allowing a faster growth rate. Reef
corals live only in warm seas, where the temperature of the water does not fall below 18 C, usually at equatorial
latitudes between 30'N and 30'S.They require clear water less than about 90 m (300 feet) deep. Below this depth
insufficient light penetrates to satisfy the photosynthetic needs of the zo0xanthellae. Another characteristic of coral
reef environments is the low level of nutrients in the water. When placed under environmental stress, some corals
expel their zooxanthellae and go it alone. High temperatures can also bring about death of the zo0xanthellae,leading
to coral bleaching (a process by which coral turns white), an effect of global warming that is causing a great deal of
anxiety.

1. The word anxiety in the passage is closest in meaning to


A. concern
B. debate
C. interest
D. criticism

2. According to paragraph 1, how do Zooxanthellae benefit from living together with corals?
A. The corals enable them to multiply at a very rapid rate.
B. The corals provide them with a protected habitat.
C. The corals give them access to certain important organic compounds.
D. The corals make it possible for them to live at greater depths than they otherwise could.

3. According to paragraph 1, all of the following statements about zooxanthellae are true EXCEPT:
A. They are a type of algae.
B. They provide the coral with glycerol,glucose, and alanine.
C. They protect the soft tissues of coral.
D. They release carbon dioxide.

第 68 页 共
373 页
2021 托福阅读真题

4. According to paragraph 1, why are coral reefs found only in water less than about 90 m deep?
A. Because nutrients are too low at greater depths
B. Because corals cannot withstand the pressure at greater depths
C. Because water temperatures are too low at greater depths
D. Because zooxanthellae cannot perform photosynthesis at greater depths

Paragraph 2: Not all corals contain symbiotic Zooxanthellae. This is true in tropical reefs as well as in corals that
live in colder waters such as the loosely branched colonial coral Lophelia that forms reef-like structures in deep
waters of the North Atlantic. How to recognize which fossil corals had zooxanthellae and which did not is a
contentious issue for paleontologists. Although the morphology(form and structure) of the skeleton can offer helpful
clues, a different line of evidence is provided by the forms of carbon present in well-preserved fossil skeletons.
Because zooxanthellae prefer to use for their metabolism (energy production) the more common, lighter isotope
form of carbon (C12) rather than the less common heavier isotope (C13), the excess C13 becomes incorporated into
the coral skeleton. Slightly higher proportions of this heavy isotope in the skeleton contribute to a so -called vital
effect, which indicates a symbiotic relationship with zooxanthellae and can be detected using precise analytical
equipment. Fossil corals exhibiting a vital effect are thought to have had symbiotic zooxanthellae; those showing no
vital effect are inferred to have lacked a symbiotic relationship. A vital effect implying the former presence of
zooxanthellae has been found in skeletons of Scleractinian corals dating back to the Triassic period, about 200
million years ago.

5.According to paragraph 2, all of the following statements about Lophelia are true EXCEPT
A. Lophelia have branches that do not grow tightly together.
B. Lophelia live in deeper waters than most reef corals.
C. Lophelia are found mostly in tropical reefs.
D. Lophelia do not contain symbiotic zooxanthellae.

6. According to paragraph 2, the morphology of the skeleton of fossil corals provides helpful clues about
A. which isotope of carbon these corals used for their metabolism
B. whether the coral had contained symbiotic zo0xanthellae
C. the depth and temperature at which the coral once lived
D. which geological period the corals date back to

Paragraph 3: There is no consensus as to whether corals living in the Paleozoic era (about 540 to 250 million years
ago) possessed zooxanthellae. One line of evidence suggesting that they lacked zooxanthellae is growth rate. Some
features in coral skeletons record the passage of time, very much as the growth rings of a tree do. In modern corals,
X-rays can be used to detect alternating bands of high and low skeletal density, each set of bands indicating one year
of skeletal growth. ■Modern reef corals with zooxanthellae may grow 10 cm (4 inches) or more per year. ■For most

第 69 页 共
373 页
2021 托福阅读真题

fossil corals, x-rays cannot be used because there is insufficient contrast between the coral skeleton and the calcite
cement typically filling the spaces within the skeleton, so that differences in skeletal density do not stand out.
■Instead, sections are cut to reveal the density banding and allow estimates to be made of growth rate. In
combination with some other methods, this has shown that Paleozoic corals seldom grew by more than 10 mm (0.4
inches) per year. ■Furthermore, the growth rate of Paleozoic corals does not decrease in specimens collected
from rocks inferred to have been deposited at greater depths. This is contrary to expectations for corals
harboring zooxanthellae that would have been inactive in lower light levels, thereby slowing down the growth of the
coral.

7. The information in paragraphs 2 and 3 suggests which of the following can be concluded about Paleozoic corals in
terms of a vital effect?
A. A vital effect caused Paleozoic coral to grow faster than modern-day corals.
B. Paleozoic corals do not show conclusive evidence of a vital effect.
C. Different techniques are necessary to detect a vital effect in Paleozoic coral
D. A vital effect prevents scientists from being able to accurately date fossil corals.

8. In paragraph 3, why does the author include the information that "the growth rate of Paleozoic corals does not
decrease in specimens collected from rocks infer red to have been deposited at greater depths?
A. To present evidence suggesting that Paleozoic corals did not have zooxanthellae
B. To support the claim that Paleozoic Corals usually grew less than 10 mm per year
C. To contrast characteristics of Paleozoic corals with those of modern corals
D. To explain why sections of fossil corals must be cut to reveal these corals' density banding

9. Look at the four squares [■] that indicate where the following sentence could be added to the passage.
In comparison, growth rates of Paleozoic coral were lower, but the difference can not be revealed by X-
radiography.

10. Directions: An introductory sentence for a brief summary of the passage is provided below. Complete the
summary by selecting the THREE answer choices that express the most important ideas in the passage. Some answer
choices do not belong in the summary because they express ideas that are not presented in the passage or are minor
ideas in the passage. This question is worth 2 points.

Reef corals and zooxanthellae are in a symbiotic relationship

A. Corals obtain organic compounds for nutrition and carbon dioxide, which helps them grow faster, from
zooxanthellae.
B. Scientists determine the growth rates of modern corals by comparing their carbon isotope proportions, but this

第 70 页 共
373 页
2021 托福阅读真题

method cannot be used for fossil corals.


C. The issue of whether Paleozoic corals had zooxanthellae is still being debated, but evidence concerning the
growth rate of these corals suggests that they did not.
D. Not all corals have zo0xanthellae, but scientists have determined that some corals dating as far back as the
Triassic did have them.
E. Because corals get carbon for their skeletons from zooxanthellae,high pro portions of 13c to 12c in fossil corals
suggest ancient zooxanthellae used more of this heavy isotope than they do today.
F. X-radiography of modern corals reveals that coral growth rates increase with depth,but X-radiography of ancient
corals does not show the same results.

第 71 页 共
373 页
2021 托福阅读真题

The Economics of Academic Tenure

Passage 1: In some countries, many universities use an employment system for teachers known as tenure. After a
lengthy trial period, a faculty member whose performance meets with the approval of the senior members of the
department and the administration of the institution may be awarded tenure. A tenured faculty member enjoys
considerable job security for the rest of his or her working life and can only be fired for reasons of "moral turpitude"
(bad or evil behavior) or "gross incompetence" or if the financial stability of the institution requires the elimination
of an entire department or program.
1234
1. Paragraph 1 answers which of the following questions about tenure?
A. How long must a teacher work before being considered for tenure?
B. What performance standards do senior faculty and administrators use to decide whether to award tenure?
C. In what situations may a tenured teacher lose his or her job?
D. How frequently are tenured teachers fired?

Passage 2: The high degree of job security enjoyed by tenured faculty members has been the source of complaints
about the tenure system. One issue that has been raised by many, including legislators evaluating the finances and
managerial practices of state universities in the United States, is that tenure shelters faculty from accountability for
poor performance. Another argument is that tenure makes the university inefficient in responding to changing
instructional demands. It is difficult to substitute computer engineering faculty for civil engineering faculty if most
of the latter have tenure. In 1988, the Education Reform Act significantly softened the tenure system in the United
Kingdom, making it easier to fire individual faculty members for financial reasons. More recently, some universities
in the United States have taken steps to give university administrators more control over tenured professors And, in
general, American institutions of higher learning have ended to increase the use of part-time and nontenured
instructors over time. In 1992, just 48 percent of all instructors had tenure on were in a position that was expected to
lead to tenure

2. It can be inferred from paragraphs 1 and 2 that some critics of the tenure system believe that tenure can
A. reduce faculty members' motivation to do their jobs well.
B. cause faculty members to prefer employment in state universities over employment in other universities.
C. remove moral turpitude as a criterion for evaluating a faculty member's performance
D. encourage faculty members to switch from one department or program to another within a university

3. In paragraph 2, why does the author state that it may be difficult for some universities to substitute
computer-engineering faculty for tenured civil-engineering faculty?
A. To help explain why teachers in some fields lack as much job security as teachers in other fields do
B. To suggest a reason for the shortage of qualified professors in some fields but not in others

第 72 页 共
373 页
2021 托福阅读真题

C. To identify a possible reason for poor performance by some academic departments


D. To help make clear the argument that tenure can decrease universities' ability to respond to changes in teaching
needs

4. According to paragraph 2. all of the following steps have been taken in response to complaints about tenure
EXCEPT
A, passing a law that allows universities to fire professors in order to save money
B, providing administrators with more control of professors who have tenure
C, allowing each department to have the same percentage of tenured faculty
D, hiring more part-time teachers

Passage 3: The traditional argument in favor of tenure is based on academic freedom, the freedom to investigate and
teach any area of knowledge without restriction or interference. In this view, tenure protects faculty members from
retaliation for voicing unpopular views. For example, a labor economist might not present a complete examination of
the costs and benefits of worker unions it he or she feared that a rabidly anti-union university leader might seek to
have the economist fired for speaking of the positive aspects of unions. In fact, the American Association of
University Professors (AAUP), a group dedicated to protecting academic freedom, got its start in the wake of a 1901
decision by Stanford University to fire economics instructor Edward Ross at the insistence of the university's
co-founder, Jane Stanford, who objected to his views on economics and other matters.

5. Which of the following can be inferred from paragraph 3 about the 1901 decision by Stanford University to
fire economics teacher Edward Ross?
A, It led to greater awareness of possible conflicts of interest between university administrators and university
founders.
B, It stimulated efforts to protect the jobs of professors against threats to their academic freedom.
C, It revealed how interested American university administrators were in the views of their economics teachers.
D, It encouraged many American universities to begin hiring instructors who opposed the AAUP

Passage 4: Going beyond academic freedom, the economics literature has recently turned to an emphasis on tenure
as a labor-market institution that may have a positive payoff to universities through the incentives (motivation) it
provides. For example, economist Lorne Carmichael's model of an academic department treats tenure as the means
of providing incentives for incumbent (current) faculty to participate in identifying the best candidates for new
positions. If incumbent faculty had to worry that more-able new additions to the department might replace them one
day, they would be less inclined to make hiring decisions that were in the best interests of the university. Incumbents
are much better positioned to judge the talents of potential new hires than is the university administration. Moreover,
the long-term job security they gain through tenure gives incumbents an incentive to hire new faculty who might be
more productive than the existing faculty in a department.

第 73 页 共
373 页
2021 托福阅读真题

6. According to paragraph 4, Lorne Carmichael's model makes which of the following claims?
A, The tenure system benefits universities by making it more likely that the best professors will stay for many years
B, Tenure tends to lead to better relations between the faculty and the university administration.
C, Universities can attract better faculty by preventing low-performing incumbents from obtaining tenure
D, Tenure allows professors to choose the best new faculty without fear of being replaced by them in the future.

Passage 5: The economists Michael McPherson and Morton Shapiro have also emphasized the notion that tenure
has a positive payoff for the university by aligning the self-interest of individual faculty members with the long-run
interests of the institution. They see two valuable economic benefits from the tenure system beyond the incentive to
hire and mentor more productive new faculty. First, job security allows tenured faculty the independence to perform
credibly objective evaluations of students and other faculty. People outside the university who rely on the
information provided by student grades or faculty reviews of papers or proposals can have greater confidence that
these evaluations have not been colored by the faculty member's concern about job security. Second, tenure allows
faculty to make long-run strategic decisions about educational policy and research even if these are in conflict with
the short-run, career interests of administrators. The fact that tenured faculty often are viewed as obstacles to change
by ambitious administrators looking to enhance their records for their next career move might well be a good thing
for the long-run interests of the university

7. The word "notion" in the passage is closest in meaning to


A, expectation
B, fact
C, possibility
D, idea

8. According to paragraph 5, a university may receive which TWO of the following benefits from the tenure
system? To receive credit, you must select TWO answers.
A, Greater trust in the university's grading system and in faculty reviews of papers and proposals
B, Improved ability of the university's students to evaluate their professors accurately
C, Protection of the university's long-term interests from the actions of self-interested administrators
D, Encouragement for faculty to focus more on research than on administrative policy

Passage 3: The traditional argument in favor of tenure is based on academic freedom, the freedom to investigate and
teach any area of knowledge without restriction or interference. █ In this view, tenure protects faculty members from
retaliation for voicing unpopular views. █ For example, a labor economist might not present a complete examination
of the costs and benefits of worker unions if he or she feared that a rabidly anti-union university leader might seek to
have the economist fired for speaking of the positive aspects of unions. █ In fact, the American Association of
University Professors (AAUP), a group dedicated to protecting academic freedom, got its start in the wake of a 1901

第 74 页 共
373 页
2021 托福阅读真题

decision by Stanford University to fire economics instructor Edward Ross at the insistence of the university's
co-founder, Jane Stanford, who objected to his views on economics and other matters. █

9. Look at the four squares █ that indicate where the following sentence could be added to the passage.
Without this security, supporters argue, researchers may avoid publishing results disliked by people in
positions of power.
Where would the sentence best fit? Click on a square █ to add the sentence to the passage.

10. Directions: An introductory sentence for a brief summary of the passage is provided below. Complete the
summary by selecting the 3 answer choices that express the most important ideas in the passage. Some sentences do
not belong in the summary because they express ideas that are not presented in the passage or are minor ideas in the
passage. This question is worth 2 points. Drag your choices to the spaces where they belong. To review the
passage, click on View Text.

Universities in some countries reward their faculty members with tenure.

A, Because tenure makes it difficult to fire faculty members, it has been criticized as being against the interests of a
well-functioning university.
B, The tenure system was at first a response to the opposition of some university administrators to attempts by
professors to form an association in the early 1900s.
C, One benefit of tenure is that it makes sure that faculty members can express unpopular opinions without risking
their jobs.
D, Some institutions in the United States and the United Kingdom have eliminated tenure for newly hired faculty
members because of financial reasons.
E, Tenured faculty are losing some of the protections that earlier tenure faculty enjoyed as universities increase the
number of nontenured and part-time instructors hired.
F, It has been claimed that tenure provides faculty with the freedom needed to make honest evaluations of students,
job candidates, and proposed university policies.

第 75 页 共
373 页
2021 托福阅读真题

Abandoning Hunting and Gathering

Paragraph 1: For much of human history, people survived by hunting and gathering, but in the Neolithic periods,
starting around 10,000 B.C.E, this was gradually given up in favor of growing grains and other plants or domesticating
animals ( pastoralism). Why did early societies in so many parts of the world gradually abandon a way of life based on
food gathering? Some theories assume that people were drawn to food production by its obvious advantages. For
example, it has recently been suggested that people settled in what is now the Middle East so they could grow enough
grains to ensure themselves a ready supply of beer. Beer drinking is frequently depicted in ancient Middle eastern art
and can be dated to as early as 3500 B.C.E.

1. According to paragraph 1, which of the following can be inferred about the transition from hunting and gathering
to agriculture?
A, The transition only took place in a few locations around the world.
B, Historians are not sure why this transition occurred.
C, The transition began long before the Neolithic period.
D, People in the Middle East began this transition long after other groups.

Paragraph 2: However, most researchers today believe that climate change drove people to abandon hunting and
gathering in favor of pastoralism and agriculture. So great was the global warming that ended the great Ice Age that
geologists give the next era which began around 9000 B.C.E. a new name: the Holocene. Scientists have also found
evidence that temperate lands were exceptionally warm between 6000 and 2000 B.C.E., the era when people in so
many parts of the world adopted agriculture. The precise nature of the crisis probably varied. In the middle East taking
up food production may have been a response to shortage of wild food caused by a dry spell or population growth.
Elsewhere, a warmer, wetter climate could have promoted rapid forest growth in former grasslands, reducing the
supplies of game and wild grains.

2. According to paragraph 2, all of the following likely occurred between 6000 and 2000 B.C.E. EXCEPT:
A, In most areas, grasslands were replaced by forests.
B, Dry spells caused food shortages in the Middle East.
C, The great Ice Age ended, beginning the Holocene.
D, People in many places started growing food.

Paragraph 3: Additional support for an ecological explanation comes from the fact that in many drier parts of the
world, where wild food remained abundant, agriculture was not adopted. The inhabitants of Australia continued to rely
exclusively on foraging until recent centuries, as did some peoples in all the other continents. Many Amerindians in the
arid grasslands from Alaska to the Gulf of Mexico hunted bison, while in the Pacific Northwest others took up salmon
fishing. Abundant supplies of fish, shellfish, and aquatic animals permitted food gatherers east of the Mississippi River

第 76 页 共
373 页
2021 托福阅读真题

in North America to become increasingly sedentary. In the equations favored retention of the older ways. The
reindeer-based societies of northern Eurasia were also unaffected by the spread of farming.

3. The word “ retention” in the passage is closest in meaning to


A, modification
B, restoration
C, preservation
D, abandonment

4. In paragraph 3, why does the author discuss particular societies in Australia, North Africa, and Eurasia?
A. To support the idea that there were many early societies that depended on food supplies that were more reliable
than agriculture.
B, To compare the lifestyles of various hunting-gathering communities throughout the world.
C, To provide information on various types of wild food sources available to hunter-gather societies.
D, To emphasize the point that it was the changing climate that encouraged the transition of some early societies to
agriculture.

Paragraph 4: Whatever the causes, the effects of the gradual adoption of food production in most parts of the world
were momentous. A hundred thousand years ago, there were fewer than 2 million people, and their range was largely
confined to the temperate and tropical regions of Africa and Eurasia. During the last glacial epoch, between 32,000 and
13,000 years ago, human populations may have fallen even lower. Then, as the glaciers retreated, people moved into
new land and adopted agriculture. Their numbers gradually rose to 10 million by 5000 B.C.E. and mushroomed to
between 50 million and 100 million by 1000 B.C.E.. This increase brought important changes to social and cultural
life.

5. According to paragraph 4, approximately how many people were alive before the last glacial epoch ended?
A, Between 13,000 and 32,000
B, Fewer than 2 million
C, 10 million
D, 50 million

6. Paragraph 4 strongly suggests that the population boom between 5000 and 1000 B.C.E. was a result of
A, social and cultural changes
B, the movement of glaciers into different regions
C, human migration out of temperate and tropical regions
D, the adoption of agriculture

第 77 页 共
373 页
2021 托福阅读真题

Paragraph 5: The evidence that an ecological crisis may have driven people to food production has led researchers to
reexamine the assumption that people in agricultural societies were better off than foragers. Modern studies
demonstrate that food producers have to work much harder and for much longer periods needed to put in long days of
arduous labor clearing and cultivating the land. Pastoralists had to guard their herds from wild predators, guide them to
fresh pastures, and tend to their many needs. There is also evidence that even though the food supply of early farmers
was more secure than that of food-gathering peoples and pastoralists, the farmers’ diet was less varied and nutritious.
Skeletal remains show that on average Neolithic farmers were shorter than earlier food-gathering peoples and more
likely to die at an earlier age from contagious diseases. People in permanent settlements were more exposed to
diseases from water contaminated by human waste, to disease-bearing vermin and insects that infested their
bodies and homes, and to new diseases that migrated from their domesticated animals ( especially pigs and
cattle).

7. Which of the sentences below best expresses the essential information in the highlighted sentence in the passage?
Incorrect choices change the meaning in important ways or leave out essential information.
A, Permanent settlements contained many sources of disease, such as livestock, insects and other pests, and
contaminated water.
B, In addition to disease, permanent settlements create a number of other problems, including vermin and insect
infestations.
C, Water contaminated by human waste encouraged insects and other pests, and often made both people and
livestock sick.
D, Vermin and insects often migrated from pig and cattle to infest the bodies and homes of people in permanent
settlements.

8. According to paragraph 5, how did the lives of early food producers compare with the lives of food gatherers?
A, Food producers consumed less food
B, Food producers were more likely to die at an early age.
C, Food producers expended less effort overall to acquire food.
D, Food producers had a healthier, more complete diet.

Paragraph 2: However, most researchers today believe that climate change drove people to abandon hunting and
gathering in favor of pastoralism and agriculture. So great was the global warming that ended the great Ice Age that
geologists give the next era which began around 9000 B.C.E. a new name: the Holocene. █ Scientists have also
found evidence that temperate lands were exceptionally warm between 6000 and 2000 B.C.E., the era when people
in so many parts of the world adopted agriculture. █ The precise nature of the crisis probably varied. █ In the middle
East taking up food production may have been a response to shortage of wild food caused by a dry spell or
population growth. █ Elsewhere, a warmer, wetter climate could have promoted rapid forest growth in former
grasslands, reducing the supplies of game and wild grains.

第 78 页 共
373 页
2021 托福阅读真题

9. Look at the four squares █ that indicate where the following sentence could be added to the passage.
For example, warmer temperatures could cause decreased rainfall in some areas and increased rainfall in
other areas, with negative effects in both cases.
Where would the sentence best fit? Click on a square █ to add the sentence to the passage.

10. Directions : An introductory sentence for a brief summary of the passage is provided below. Complete the
summary by selecting the 3 answer choices that express the most important ideas in the passage. Some sentences do
not belong in the summary because they express ideas that are not presented in the passage or are minor ideas in the
passage. (This question is worth 2 points. )

Starting around 10,000 B.C.E. humans began to gradually replace hunting and gathering with agriculture and
pastoralism.

A, Agriculture and pastoralism began in the Middle East and gradually spread to other continents.
B, Although food production has many obvious advantages, most researchers today believe that humans were forced
into adopting this lifestyle because of climate change.
C, Food production created a reliable food supply and resulted in growing populations, but it may have also caused
people to work harder for a less varied and nutritious diet.
D, Recent evidence suggests that grain cultivation began first, and as populations increased, societies later adopted
pastoralism.
E, During a period of global warming, people in wet areas may have taken up agriculture becau se they lost
grasslands, while people in many drier parts of the world continued gathering food.
F, On average, early pastoralists were taller than hunters and gatherers but more likely to die at an earlier age from
disease, suggesting that both lifestyles had their drawbacks.

第 79 页 共
373 页
2021 托福阅读真题

Early Iron Metallurgy

Paragraph 1: Metallurgy--the science of working with metals such as iron in order to give them certain desired
properties--has a long history. The first known objects made from metals date to the ninth millennium B.C.E. Usable
iron is usually obtained from iron ore(raw iron)by smelting, a process by which iron is separated(reduced)from its
ore by heating. Iron-ore smelting requires very high temperatures and good control of the temperature and reduction
conditions. Achieving these was beyond the capabilities of the early metallurgists. Thus, the development of the
earliest metallurgy concerned only other metals-gold, silver, copper, lead, and tin-around which a range of
sophisticated technologies developed, practiced by experts and often managed by elites. The rarity of tin required for
making bronze, played a key part in determining the pattern of trade routes, and the consequent expense restricted
bronze use to important members of society.

1. According to paragraph 1, early metallurgists did not produce iron objects because
A, iron ores had not yet been discovered
B, iron smelting had a very high cost
C, they lacked the technology to smelt iron
D, other metals were available in large quantities

2. Paragraph 1 suggests that the earliest metallurgy techniques were developed around only gold, silver, copper, lead,
and tin because
A, metallurgists could combine those metals to make valuable products
B, metallurgists could earn more because of the expertise required to work with those metals
C, those metals could be easily acquired through trade routes
D, those metals melt at relatively lower temperatures than iron

Paragraph 2: In about 1200 BCE, however, iron smelting began in western Asia. Iron ore needed to be reduced in a
furnace at a high enough temperature to ensure that the slag (the material from which the iron has been separated)
melted and ran off, leaving the iron “bloom”. A blown-air supply, provided by a device called a bellows, enabled the
fuel to burn. The resulting bloom was a mass of metallic iron still containing impurities from the fuel, which had to
be driven out by forging (heating and hammering), leaving wrought iron, a soft inferior metal that could not be
hardened by heating. Carburization, however, transformed wrought iron into useful low-carbon steel by
incorporating 0.5 to 1.5 percent carbon, generally by using charcoal as the smelting fuel Low-carbon steel was
harder and stronger than bronze, and could take and keep an edge better; and its hardness could be varied by heating
and cooling it. Skilled and nuanced iron working developed thereafter--expert smiths in western Europe, for example,
produced swords with razor-sharp edges, hard, but easily broken, on softer, more elastic blades.

3. According to paragraph 2, a blown-air supply helps with which of the following?

第 80 页 共
373 页
2021 托福阅读真题

A. Melting the iron bloom


B. Removing the slag from iron ore
C. Removing impurities produced by the fuel from the iron bloom
D. Hardening iron

4. According to paragraph 2, which of the following is true about carbon?


A. It was used in up to 1.5 percent of wrought iron
B. It had long been used to strengthen bronze
C. It transformed wrought iron into a harder material
D. It was driven out of the iron by heating and hammering

Paragraph 3: The technology of smelting and working blooms was relatively simple, requiring little specialist
equipment: an easily constructed bowl furnace into which the iron and charcoal fuel are placed, and bellows to
enable the temperature to be raised, as well as tongs, hammer, and anvil to turn the bloom into usable wrought iron
and work it into objects. Iron has the convenient property that pieces can be joined strongly by hammering them
together while heated (welding). The technology was therefore accessible to ordinary people with the necessary
skills rather than being confined to specialists, stimulating the emergence of small-scale entrepreneurs, such as the
itinerant smiths of Iron Age Europe. In addition, iron ores are abundant and found everywhere. While some
regions with substantial or high-quality ore sources became wealthy by trading in iron, most regions were able to
obtain iron relatively locally and cheaply, altering the patterns of trade. Bronze supplies were now devoted to the
production of luxury goods, such as fine jewelry, elite tableware, and ceremonial armor Iron objects became
common with metal tools, weapons, and armor now within the reach of everyone. This had a considerable impact on
the efficiency of many aspects of everyday life, such as agriculture, industry, and construction, as well as warfare.
Iron had advantages over bronze because it was harder, more durable, easy to repair, and easy to shape. It is thought
that the advent of iron working also had a democratizing effect, affecting social organization.

5. The phrase “confined to” in the passage is closest in meaning to


A, shared among
B, developed by
C, taught to
D, limited to

6. Why does the author state that iron ores are abundant and found everywhere?
A, To offer an advantage that explains the spread of iron metallurgy
B, To emphasize the amount of wealth that the iron trade created
C, To explain why the Iron Age began in Europe
D, To argue that small-scale entrepreneurs were large in number

第 81 页 共
373 页
2021 托福阅读真题

7. Which of the following can be inferred from paragraph 3 about bronze objects during the Iron Age as compared to
iron objects
A, Bronze objects were used by fewer people
B, Bronze objects were less valuable
C, Bronze objects became easier to make
D, Bronze objects were used for a wider variety of purposes

Paragraph 4: In China the path of development was different. While wrought iron was made there, the Chinese
developed furnaces capable of achieving the temperature required to produce cast iron, which was higher in carbon
content and harder, using a blast furnace to produce a powerful forced draft, something not achieved elsewhere until
many centuries later. This meant that they were able to mass produce iron objects by casting (pouring and allowing
to solidify in a mold). Iron was produced on a large scale by elites in large iron foundries. The development of iron
working, therefore, did not have the same social impact as elsewhere, although it enabled ordinary individuals to
acquire metal tools.

8. According to paragraph 4, which of the following helped make the mass production of iron objects possible in
China?
A, The involvement of ordinary people in iron production
B, The production of wrought iron from cast iron
C, The introduction of new technology from other places
D, The use of blast furnaces for casting

Paragraph 2: In about 1200 BCE, however, iron smelting began in western Asia. Iron ore needed to be reduced in a
furnace at a high enough temperature to ensure that the slag(the material from which the iron has been separated)
melted and ran off, leaving the iron “bloom”. A blown-air supply, provided by a device called a bellows, enabled the
fuel to burn. The resulting bloom was a mass of metallic iron still containing impurities from the fuel, which had to
be driven out by forging (heating and hammering), leaving wrought iron, a soft inferior metal that could not be
hardened by heating. █Carburization, however, transformed wrought iron into useful low-carbon steel by
incorporating 0.5 to 1.5 percent carbon, generally by using charcoal as the smelting fuel. █ Low-carbon steel was
harder and stronger than bronze, and could take and keep an edge better; and its hardness could be varied by heating
and cooling it. █ Skilled and nuanced iron working developed thereafter-expert smiths in western Europe, for
example, produced swords with razor-sharp edges, hard, but easily broken, on softer, more elastic blades. █

9. Look at the four squares █ that indicate where the following sentence could be added to the passage.
The strengthening process inspired attempts to adapt iron to more and more uses.
Where would the sentence best fit? Click on a square (█)to add the sentence to the passage.

第 82 页 共
373 页
2021 托福阅读真题

10. Directions : An introductory sentence for a brief summary of the passage is provided below. Complete the
summary by selecting the 3 answer choices that express the most important ideas in the passage. Some sentences do
not belong in the summary because they express ideas that are not presented in the passage or are minor ideas in the
passage. This question is worth 2 points.

Long before humans learned how to smelt iron, the earliest metallurgists made objects out of gold, silver,
copper, lead, and tin.

A, Before iron smelting techniques were perfected, bronze objects were widely available for common tools and
weapons and were traded along well-defined routes
B, In Iron Age Europe, increased access to quality metal products led to greater capabilities in labor and warfare as
well as to a transformation of social structures
C, China developed the technology to mass-produce cast-iron objects but the production was controlled by elite
members of society.
D, The invention of specialized furnaces and fuels made it possible for iron ore to be turned into low-carbon steel
which could be made into a variety of durable products.
E, Although iron ore was easy to find in Europe. most of it was of low quality, requiring most regions to establish
iron trade with the few areas having high-quality ore
F, China developed iron making later than many other regions, and the methods they used were more complex,
which limited the accessibility of iron objects

第 83 页 共
373 页
2021 托福阅读真题

Examining the Problem of Bycatch

Paragraph 1: A topic of increasing relevance to the conservation of marine life is bycatch-fish and other animals
that are unintentionally caught in the process of fishing for a targeted population of fish. Bycatch is a common
occurrence in longline fishing, which utilizes a long heavy fishing line with baited hooks placed at intervals, and in
trawling, which utilizes a fishing net (trawl) that is dragged along the ocean floor or through the mid-ocean waters.
Few fisheries employ gear that can catch one species to the exclusion of all others. Dolphins, whales, and turtles are
frequently captured in nets set for tunas and billfishes, and seabirds and turtles are caught in longline sets. Because
bycatch often goes unreported, it is difficult to accurately estimate its extent. Available data indicate that
discarded biomass (organic matter from living things) amounts to 25-30 percent of official catch, or about 30
million metric tons.

1. The phrase “relevance to” in the passage is closest in meaning to


A, significance to
B, debate about
C, difficulty in
D, threat to

2. Why does the author provide the information that “Available data indicate that discarded biomass (organic
matter from living things) amounts to 25-30 percent of official catch, or about 30 million metric tons"?
A, To disprove the claim that it is difficult to accurately estimate the extent of the bycatch problem
B, To illustrate the extreme effectiveness of the longline and trawling methods
C, To suggest that uncertainty about the true extent of bycatch does not leave in doubt that it is a problem
D, To indicate that data about bycatch are available only from fisheries having the right kind of gear

Paragraph 2: The bycatch problem is particularly acute when trawl nets with small mesh sizes
(smaller-than-average holes in the net material) are dragged along the bottom of the ocean in pursuit of groundfish or
shrimp. Because of the small mesh size of the shrimp trawl nets, most of the fishes captured are either juveniles
(young), smaller than legal size limits, or undesirable small species. Even larger mesh sizes do not prevent bycatch
because once the net begins to fill with fish or shrimp, small individuals caught subsequently are trapped without
ever encountering the mesh. In any case, these incidental captures are unmarketable and are usually shoveled back
over the side of the vessel dead or dying.

3. The word "acute" in the passage is closest in meaning to


A, common
B, severe
C, complicated

第 84 页 共
373 页
2021 托福阅读真题

D, noticeable

4. According to paragraph 2, why have larger mesh sizes not provided a practical solution to bycatch in shrimp
fishing?
A, Larger openings increase the risk that nets will get tangled on damaged as they are being hauled over the sides of
the vessel.
B, Openings large enough to prevent the capture of juvenile and other undesirable fish would also release the shrimp.
C, Large mesh sizes are more likely to result in fish getting stuck partway through, causing more deaths within the
catch.
D, When nets grow full, they still trap fish that cannot reach the mesh openings.

Paragraph 3: The bycatch problem is complicated economically and ecologically. Bycatch is a liability to shrimp
fishers, clogging the nets and increasing fuel costs because of increased drag (resistance) on the vessel. Sorting the
catch requires time, leading to spoilage of harvested shrimp and reduced time for fishing. Ecologically, high
mortality rates among juvenile fishes could contribute to population declines of recreational and commercial species.
Evidence to this effect exists for Gulf of Mexico red snapper and Atlantic Coast weakfish. Because the near-shore
areas where shrimp concentrate are also important nursery grounds for many fish species, shrimp trawling could
have a profound impact on stock size.

Paragraph 4: Once the dead or dying bycatch is returned to the ecosystem, it is consumed by predators, detritivores
(organisms that eat dead plant and animal matter), and decomposers (organisms that break down dead or decaying
organic matter), which could have a positive effect on sport fish, seabird, crab, and even shrimp populations.
Available evidence indicates that 40-60 percent of the 30 metric tons of catch discarded annually by commercial
fishing vessels, and even more of the noncatch waste (organisms killed but never brought to the surface), does not lie
unused on the bottom of the sea. It becomes available to midwater and ocean-bottom scavengers, transferring
material into their food web and making energy available to foragers (organisms that search for food) that is
normally tied up in ocean-bottom, deep-ocean, midwater, and open-ocean species.

5. According to paragraph 4, how does bycatch sometimes benefit sport fish, seabird, crab, and even shrimp
populations?
A, The discarded fish provide these species with a significant amount of food that would otherwise be unavailable to
them.
B, Fishing eliminates up to 40 to 60 percent of the predators of these species, most of which are caught
unintentionally.
C, These fish and other animals may be caught unintentionally in overcrowded locations and then released into more
favorable environments.
D, Many of the competitors of these species are eliminated by fishing. leaving them with access to more food and

第 85 页 共
373 页
2021 托福阅读真题

other resources.

Paragraph 5: Overfishing and overdiscarding may thus contribute to a syndrome known as "fishing down of
food webs," whereby we eliminate apex (top) predators and large species while transforming the ocean into a
simplified system increasingly dominated by microbes, jellyfish, ocean-bottom invertebrates, plankton, and
planktivores. The strongest evidence for the fishing down phenomenon exists in global catch statistics that show
alarming shifts in species composition from high-value, near-bottom species to lower-value, open-ocean species. In
the last three decades of the twentieth century, the global fishing fleet doubled in size and technology advanced
immeasurably. Despite increased effort and technology, total catch stabilized, but landing rates (rates at which
species are caught) of the most valuable species fell by 25 percent.

6. Which of the sentences below best expresses the essential information in the highlighted sentence in the passage?
Incorrect choices change the meaning in important ways or leave out essential information.
A, Overfishing and overdiscarding of jellyfish, ocean-bottom invertebrates, plankton, and planktivores are
transforming the ocean in a process known as fishing down of food webs.
B, Overdiscarding bycatch simplifies the food web by favoring the kinds of predators that feed on such prey as
jellyfish, ocean-bottom invertebrates, and planktivores.
C, Fishing down of food webs may occur if overfishing and bycatch disposal result in the disappearance of species at
the top of the food web and the dominance of species near the bottom.
D, Overfishing and overdiscarding is a syndrome that affects not only top predators and large species but also
microbes, jellyfish, ocean-bottom invertebrates, plankton, and planktivores.

7. What does paragraph 5 suggest is the reason why landing rates of the most valuable species fell 25 percent in the
last three decades of the twentieth century?
A, Changes in technology led many fishers to shift from a focus on near-bottom species to lower-value open-ocean
species.
B, Around the world, the number of people and ships involved in the fishing trade declined because of changes in the
demand for fish.
C, The total amount of fish in the ocean decreased significantly, leading to a steady decrease in global total catch.
D, The most valuable species make up a much smaller percentage of the total sea population than they used to.

Paragraph 6: Conservation organizations have condemned the obvious and extreme waste associated with bycatch.
█ Public concern over high mortality rates of endangered marine turtles captured in shrimp trawls led to the
development of Turtle Exclusion Devices (TEDs) in the 1980s. █ TEDs were incorporated into the shrimp net design
with the purpose of directing turtles out of nets without unacceptably reducing shrimp catches. █ Marine engineers
and fishers also developed shrimp net designs that incorporate Bycatch Reduction Devices (BRDs), taking advantage
of behavioral differences between shrimp and fish, or between different fishes, in order to separate fishes. █

第 86 页 共
373 页
2021 托福阅读真题

8. According to paragraph 6, which of the following led to the development of TEDs?


A, The desire of fishers to increase their shrimp catch by decreasing turtle bycatch
B, The need to capture endangered turtles to relocate them
C, A request by fishers to develop ways to preserve their nets from turtle damage
D, Public concern over endangered turtles in bycatch

9. Look at the four squares █ that indicate where the following sentence could be added to the passage.
Turtles were not the only marine species to benefit from new catch techniques.
Where would the sentence best fit? Click on a square █ to add the sentence to the passage.

10. Directions: An introductory sentence for a brief summary of the passage is provided below. Complete the
summary by selecting the 3 answer choices that express the most important ideas in the passage. Some sentences do
not belong in the summary because they express ideas that are not presented in the passage or are minor ideas in the
passage. (This question is worth 2 points.)

Many fish and other animals are unintentionally caught during commercial fishing, a problem known as
bycatch.

A. Bycatch occurs in both longline fishing and trawling and affects a range of species, although marine engineers
have developed net devices that have lessened the problem for some species.
B, Trawling with small mesh nets for shrimp results in large amounts of bycatch, especially of small, young, or
unwanted species of fish, causing a range of problems for shrimp fishers and the ecosystem.
C, When bycatch is disposed of in the ocean, the extra food is eaten by predators, detrivores, and decomposers but
may ultimately cause the most valuable species to decline.
D, Female fish are especially likely to become bycatch when they are near the ocean bottom spawning their eggs,
which decreases the populations of commercially desirable fish such as salmon and tuna.
E, Efforts are being made to come up with productive uses for bycatch, such as providing food for fish farms or
being used in agricultural products such as animal feed and fertilizers.
F, Much of the equipment designed to prevent bycatch has so far proven to be largely ineffective, with TEDs failing
to significantly reduce the number of sea turtles captured in fishing nets.

第 87 页 共
373 页
2021 托福阅读真题

Naturalism and Nature in Art

Paragraph 1: In the ancient world, the Greek philosopher Aristotle evaluated works of art on the basis of how
faithfully artists recorded what they saw in the natural world. But we need to be aware that when painters working
in a naturalistic style make images that seem like untouched snapshots of actual objects, their skill can also render
lifelike such fictions as a unicorn or a dragon.

1. The word "faithfully" in the passage is closest in meaning to


A. beautifully
B. vividly
C. accurately
D. simply

Paragraph 2: Like many people today, ancient Greeks enjoyed the work of especially skillful naturalistic artists (the
Greek word for "art," techne, is the same as the Greek word for "craft"). Their admiration for naturalistic depiction is
illustrated in a famous story about a competition between rival Greek painters named Zeuxis and Parrhasius in the
late fifth century B.C.E. Zeuxis painted a picture of grapes so accurately that birds flew down to peck at them. Then
Parrhasius took his turn, and when Zeuxis asked his rival to remove the curtain hanging over the picture, Parrhasius
gleefully pointed out that the curtain was his painting. Zeuxis agreed that Parrhasius won the competition since he,
Zeuxis, had fooled only birds but Parrhasius had tricked an intelligent fellow artist.

2. According to paragraph 2, all of the following are true of the competition between Zeuxis and Parrhasius
EXCEPT:
A. Parrhasius' painting was considered to be better than Zeuxis'.
B. Parrhasius painted a curtain that looked real.
C. Zeuxis took the first turn in the competition.
D. Zeuxis painted a picture of birds pecking grapes.

Paragraph 3: In the seventeenth century, painter Adriaen van der Spelt and his artist friend Frans van Mieris paid
homage to the story of Parrhasius' curtain with their painting of a blue satin drapery drawn aside to show a garland of
flowers (a decoration made from many flowers joined together) in Flower Piece with Curtain (1658). The artists not
only re-created Parrhasius' curtain illusion but also included a reference to another Greek legend that was popular in
the fourth century B.C.E. that told of Pausias, who painted the exquisite floral garlands made by a young woman,
Glykera. This second story raises the troubling and possibly unanswerable question of who was the true artist—the
painter who copied nature in his art or the garland maker who made works of art out of nature. The
seventeenth-century patrons—the people who bought such paintings—knew those stories and appreciated the artists'
classical references as well as their skill in drawing and manipulating colors on canvas.

第 88 页 共
373 页
2021 托福阅读真题

3. According to paragraph 3, which of the following statements is true of the painting Flower Piece with
Curtain?
A. It is not known which artist painted it.
B. It features a young woman named Glykera.
C. It includes references to two Greek stories.
D. It depicts an artist painting a garland of flowers.

4. What can be inferred from paragraph 3 about the people who bought seventeenth-century naturalistic
paintings such as Flower Piece with Curtain?
A, They were familiar with ancient Greek legends.
B, They also bought natural works of art from garland makers.
C, They were often skilled in drawing and painting.
D, They knew a great deal about the natural world.

Paragraph 4: The flower garland in Flower Piece with Curtain also symbolizes the passage of time and the fleeting
(temporary) quality of human riches. The brilliant red and white tulip—the most desirable and expensive flower of
the time—symbolizes wealth and power. Yet insects creep out of the flowers, and a butterfly—fragile and transitory
—hovers above a flower. Today, after studying the painting in its cultural context, we, too, understand that it is much
more than a simple flower piece, such as the type of still life with flowers popular in the Netherlands in van der Spelt
and van Mieris' time.

5. The word "fragile" in the passage is closest in meaning to


A, small
B, delicate
C, attractive
D, alert

6. Paragraph 4 suggests that studying Flower Piece with Curtain in its cultural context helps us understand
A, why human riches are temporary
B, why it was an expensive painting, despite its simplicity
C, why still life paintings were popular in the Netherlands
D, what idea the painters were trying to communicate

Paragraph 5: Just as Dutch flower pieces were perceived as ideal expressions of naturalism in the seventeenth
century, so today modern photography seems like a perfect medium for expressing the natural beauty of plants. In his
photograph Succulent (1930), Edward Weston did just that by using straightforward camera work, without
manipulating the film in the darkroom. But Weston did more than accurately portray his subject. He made

第 89 页 共
373 页
2021 托福阅读真题

photography an expressionistic medium by perfecting the close-up view to evoke an emotional response. He argued
that, although the camera sees more than the human eye does, the quality of the image depends not on the camera but
on the choices made by the photographer--artist. Many people even today think that naturalism represents the highest
accomplishment in art. But not everyone agrees. First to argue persuasively that observation alone produced "mere
likeness" was the Italian master Leonardo da Vinci (1452-1519), who said that the painter who copied the external
forms of nature was acting only as a mirror. He believed that the true artist should engage in intellectual activity
of a higher order and attempt to capture the inner life—the energy and power—of a subject. ■Georgia O' Keeffe,
like van der Spelt and Weston, studied living plants; however, when she painted the canna lily in the painting Red
Canna (1924), she, like Leonardo, sought to capture the flower's essence.■By painting the canna lily's organic
energy, she created a essence.■By painting the canna lily's organic energy, she created a new abstract beauty,
conveying in paint the pure vigor of its life cycle, rather than producing a realistic image.■

7. Why does the author note that Leonardo da Vinci said that "the painter who copied the external forms of
nature was acting only as a mirror"?
A, To illustrate how much the principles of naturalism have changed over the centuries
B, To support the claim that not everyone highly values naturalism in art
C, To point out that naturalism is capable of capturing the inner life of its subject
D, To draw a contrast between naturalism in photography and naturalism in painting

8. According to paragraph 5, Georgia O'Keeffe's painting Red Canna illustrates which of the following views
about art?
A, An object's essence is best shown by attempting to depict its inner energy rather than its external appearance.
B, The inner life of a subject is best captured by accurately copying its visible form.
C, A living organism's inner beauty is best captured by realistically painting all the stages of its life cycle.
D, Naturalism can be achieved only through the highest level of intellectual activity.

9. Look at the four squares [■] that indicate where the following sentence could be added to the passage.
Such an approach requires more from the artist but is reflected in the works of many modern painters as well
as photographers.
Where would the sentence best fit? Click on a square [■] to add the sentence to the passage.

10. Directions: An introductory sentence for brief summary of the passage is provided below. Complete the
summary by selecting the 3 answer choices that express the most important ideas in the passage. Some sentences do
not belong in the summary because they express ideas that are not presented in the passage or are minor ideas in the
passage. This question is worth 2 points.

Naturalism in art, dating back to ancient Greece, involves realistic treatment of objects in the natural world.

第 90 页 共
373 页
2021 托福阅读真题

A, In ancient Greek competitions during the fifth century B.C.E., naturalistic artists sometimes painted fictional
creatures like unicorns or dragons, which their skills could render lifelike.
B, Dutch naturalists van der Spelt and van Mieris often created paintings of expensive flowers that appealed to the
artists' rich patrons by celebrating wealth and power.
C, Photography provides a way for artists like Edward Weston to create naturalistic images that, through the artist's
careful choices, can create an emotional response to a natural object.
D, Some seventeenth-century naturalistic painters made references to ancient Greece in their paintings and created
artwork that combined realistic images with symbolic meanings.
E, Some twentieth-century artists took photographs of plants before painting them in order to ensure that their art
was as realistic and natural-looking as possible.
F, Some modern-day artists have adopted the view of Leonardo da Vinci that true art is more than a mirroring of
nature and that it requires the intellectual vision of the artist.

第 91 页 共
373 页
2021 托福阅读真题

Northwest Coast Art

Paragraph 1: The Pacific Northwest of North America produced stable Native American cultures. This was in great
measure due to the abundance of resources and temperate climate, which figured prominently in the social and
cultural fabric of the peoples. Fishing, hunting, and foraging produced ample means of sustenance, and therefore,
the people of this region had no reason to cultivate crops or domesticate animals. Their cultures tended to share
certain features, probably because they traded and warred with one another. Artistic motifs (recurring artistic
elements) were alike, and they shared a common religion-shamanism (ancient belief systems centered on shamans.
individuals who communicate with invisible forces or spirits). Although their gods and myths differed, the
Northwest peoples all acknowledged the power of shamans to contact the spirits of the forest and waters, to heal the
sick, and to predict the future.

1. The word "ample" in the passage is closest in meaning to


A. plentiful
B. typical
C. reliable
D. necessary

2. According to paragraph 1, Northwest Native Americans believed shamans had the ability to
A. interpret cultural myths
B. create artistic motifs
C. locate food sources
D. cure sick people

Paragraph 2: The rich forests of the area provided Native American artists with a wealth of materials for sculpting.
Huge spruce and cedar trees abounded for many centuries, and when steel knives were obtained from fur traders and
used as tools, Northwest-coast artists excelled in producing magnificent totem poles, carved posts for wooden
dwellings, masks, rattles, and other objects. Carved wooden house posts not only supported the roof but also gave
additional decoration to the interior. When living closer to the sea, artists had access to abalone shells that were used
as inlays to give luster to their sculpted works.

3. In paragraph 2, why does the author point out that some Native American artists of the Pacific Northwest used
abalone shells as inlays on sculpted works?
A. To argue that the carvings of these artists were superior to those of forest dwellers
B. To support the idea that the Pacific Northwest provided a large supply of materials from which to make art

第 92 页 共
373 页
2021 托福阅读真题

C. To explain why it was important for these artists to obtain steel knives to make carvings
D. To identify a specific way in which the carvings of these artists changed over time
Paragraph 3: Although many art objects from the area, such as masks, concern themselves with shamanistic
religious rituals, a fair amount of art was secular. Like some African art, it was used to maintain the social fabric and
bolster a ruler's power. For example, in the Tlingit group--a people who lived in the islands and bays of the upper
reaches of the Pacific Northwest--communities were made up of a number of families, each of which had its own
chief who inherited his rank from his mother. Carefully devised social customs obliged both men and women to
marry outside their own clan. In this way a balance was established in which no one family achieved dominance.
Nonetheless, chiefs competed fiercely with each other in displays of riches. Totem poles formed a part of this
ostentation proclaiming prestige and family pride through genealogy--much like the coats of arms of the European
aristocracy. Totem poles were carved from single tree trunks and often reached a height of 90 feet (27.4 meters).
Probably originating as funerary (burial) monuments, by the nineteenth century they had become fixtures adorning
the exteriors of chiefs' houses.

4. According to paragraph 3, which of the following explains why no one family became dominant among the
Tlingit?
A. Each family had its own chief.
B. Family chiefs inherited their ranks from their fathers.
C. Men and women were obliged to marry outside their clan.
D. All chiefs were equally wealthy and powerful.

5. Which of the following can be inferred from paragraph 3 about the fierce competition between Tlingit chiefs?
A. It became less intense when totem poles began to be made.
B. It encouraged chiefs to build their homes near funerary monuments
C. It was expressed in part through symbols of family pride.
D. It led to the creation of shamanistic rituals involving masks.

6. According to paragraph 3, the original purpose of totem poles was probably to


A. decorate houses
B. record social position
C. honor the dead
D. display chiefs' wealth

Paragraph 4: Eagles, beavers, and whales appear on the totem poles, in essence as symbolic crests that a chief
inherited from his ancestors. The Tlingit did not worship these figures or have any supernatural relationship with
them. They are governed by traditional artistic principles. One of these is that of bilateral symmetry--the designs on
either side of the central axis are identical. Another specifies the transition from one motif to another.Each design

第 93 页 共
373 页
2021 托福阅读真题

must appear to grow from the one below. Thus, not only the overall vertical form of the totem pole, but also the
design of its interior parts, leads the eye upward along its central axis.

7. According to paragraph 4. all of the following are characteristic of the images used on totem poles EXCEPT
A depiction of religious figures
B bilateral symmetry
C closely integrated forms
D structured so as to draw the eye upward

Paragraph 5:As historical documents that record the wealth, social position, and relative importance of the
person who paid for them, totem poles are quite similar to sculptural records from other cultures. The ultimate
function of such art was probably to act as a gift. As the anthropologist Frederick J. Dockstader claimed, "The life
goal of many...involved the belief that the greatest value was to give away all of one's possessions." The actual
working out of such a philosophy created some interesting scenarios. █ The more one gave away, the greater one's
prestige. █ In turn, one's rival was more or less obliged to give back the same or more material wealth in order to
prove greater disdain for possessions. █ As a result, totems and other goods were often burned, thrown into the sea,
or otherwise destroyed. █

8. Which of the sentences below best expresses the essential information in the highlighted sentence in the passage?
Incorrect choices change the meaning in important ways or leave out essential information
A. Totem poles are like the sculptures of other cultures in that they document the social significance of their owners
B. In several cultures, totem poles show the social status of their owners
C. Totem poles look like sculptures from other cultures that are also records of important persons.
D. Historical documents in other cultures are similar to totem poles that record the social history of their owners.

9. Look at the four squares █ that indicate where the following sentence could be added to the passage.
Thus one could increase one's prestige by giving a very valuable gift to a social rival.
Where would the sentence best fit? Click on a square █ to add the sentence to the passage.

10. Directions: An introductory sentence for a brief summary of the passage is provided below. Complete the
summary by selecting the 3 answer choices that express the most important ideas in the passage. Some sentences do
not belong in the summary because they express ideas that are not presented in the passage or are minor ideas in the
passage. (This question is worth 2 points.)

Native American peoples of the Pacific Northwest developed characteristic art forms.

A. Northwest-coast art is exemplified by masks, totem poles, housing posts, and rattles carved from the local spruce

第 94 页 共
373 页
2021 托福阅读真题

and cedar trees.


B. The traditional design principles of Northwest-coast art was significantly influenced by European coats of arms.
C. The art of the Native Americans celebrated the power of shamans to contact the spirits of the forest and waters.
D. Totem poles, consisting of a vertical arrangement of animal images, served to display political power and family
heritage.
E. Some anthropologists believe that totem poles were given to competing groups as gifts in order to avoid violent
confrontations.
F Giving away works of art and other valuable items became a way of gaining prestige and putting rivals at a
disadvantage.

第 95 页 共
373 页
2021 托福阅读真题

Guam and the Brown Tree Snake

Paragraph 1: Guam is the largest island of Micronesia and, like many islands, extinct, and the remaining ones are
threatened with extinction.Although many factors contributed to the loss of species, the principal cause is the
invasive brown tree snake.

Paragraph 2: During the Second World War, the United States Navy established a large naval base on Guam. With
the end of the war in 1945, the base proved useful for recovering abandoned war materials from the region, in
particular, vehicles, aircraft. and other supplies from New Guinea where these items may have sat unused for some
time. The brown tree snake is native to New Guinea and other regions of Australasia. It is typically nocturnal, and,
during the day, rests within crevices and holes that provide good cover.It is commonly found in wheel wells on
airplanes, under the hoods of cars, and in boxes of cargo, and most biologists think the brown tree snake was a
hitchhiker within surplus Navy equipment brought to Guam from New Guinea in the postwar years of 1945-1950.

1. The word "abandoned" in the passage is closest in meaning to


A left behind
B useful
C valuable
D damaged

2. The main purpose of paragraph 2 is to explain


A why the United States Navy established a large naval base on Guam
B why war materials from New Guinea were brought to Guam
C why the brown tree snake likes to rest in wheel wells on airplanes, under car hoods, and in boxes of cargo
D how the brown tree snake reached Guam

Paragraph 3: The brown tree snake was first noticed along Guam's southern shore near Apra Harbor. It then spread,
somewhat slowly, until it occupied the entire island by 1970. Guam has no native snakes and no native predators that
could have controlled tree snake numbers. Instead, the native vertebrates were all small species vulnerable to
predation by the generalist tree snakes. The decline of most native forest animals was immediate and dramatic.
Guam's Division of Aquatic and Wildlife Resources discontinued surveys of native birds and bats in the 1970s,
as there were so few individuals of these species to count. The three native birds and mammals that persisted the
longest were the Mariana fruit bat, Guam rail, and island swiftlet. The fruit bat and rail were relatively long-lived
species that likely persisted simply because some individuals escaped predation and lived out the remainder of their
lives: however, neither species had any reproductive success in the presence of the tree snake. The island swiftlet
persists to this day but is confined to one cave on Guam. The swiftlet builds nests on the walls of caves using its own
saliva and mud to adhere the nest to the cave wall. Despite this unusual habit, swiftlets are still vulnerable to tree

第 96 页 共
373 页
2021 托福阅读真题

snake predation, as tree snakes can easily capture prey in total darkness and climb most surfaces. The one cave
where swiftlets persist is unique in that the cave walls are not textured enough for snakes to support themselves. This
does not inhibit the swiftlets' ability to build nests, but it does prohibit tree snakes from reaching those nests.

3. The word "inhibit" in the passage is closest in meaning to


A improve
B restrict
C involve
D destroy

4. Why does the author provide the information that "Guam's Division of Aquatic and Wildlife Resources
discontinued surveys of native birds and bats in the 1970s. as there were so few individuals of these species to
count”?
A To illustrate how Guam used a government agency to keep track of changes in animal populations
B To give an example of one of the ways in which the extinctions caused by the brown tree snake affected Guam
C To emphasize how rapid and extensive the extinctions on Guam really were
D To make the point that the brown tree snake did not occupy the whole island until 1970

5. According to paragraph 3, what are TWO reasons that swiftlets are able to survive in one particular cave? To
receive credit, you must select TWO answer choices.
A The part of the cave in which they build their nests is in total darkness.
B Brown tree snakes cannot find the cave
C Swiftlets are able to attach their nests directly to the walls of the cave
D The walls of the cave are too smooth for the brown tree snake to climb.

Paragraph 4: There have been many other losses of native forest animals on Guam. Consequently, the food web of
Guam's forests (the dominant native habitat) has shifted dramatically. The most striking change is the reorganization
of the web to one in which most components are nonnative. Beyond the brown tree snake, the Polynesian rat.
Philippine turtledove, and the house mouse, among other nonnative species, have been successively
established.Three native lizards survive in Guam's forests. All the lizards are small and active during daylight. One
would assume that, with the destruction of the native food web, the brown tree snake would suffer from a lack
of food resources, but the available pool of invasive prey and native lizards maintains tree snake
densities.Consequently native species went extinct with no corresponding negative effect on the tree snake. The tree
snake even seems to be adapting to the diurnal habits of the remaining native lizards. since biologists have
documented a shift in the tree snake's previously nocturnal activity pattern to one that increasingly includes
activity during daylight hours.

第 97 页 共
373 页
2021 托福阅读真题

6. Which of the sentences below best expresses the essential information in the highlighted sentence in the
passage? Incorrect choices change the meaning in important ways or leave out essential information.
A. The assumption is that the destruction of the native food web led to the introduction of nonnative invasive prey
that now support the brown tree snake and native lizards
F. Contrary to the assumption that the brown tree snake would suffer from a lack of food. it is able to maintain itself
on available nonnative prey and native lizards.
G. One would assume that the brown tree snake depends on invasive prey and native lizards to prevent a collapse of
the food web
H. One would assume that the available pool of invasive prey and native lizards is not sufficient to provide food
resources for the brown tree snake, if the native food web is destroyed

7. According to paragraph 4, what do the Polynesian rat and the house mouse have in common?
A. They both prey on native Guam lizards.
B. They are both nonnative species in Guam.
C. They were both introduced to Guam at the same time that the brown tree snake was introduced.
D. They are both beginning to suffer from the loss of food resources caused by the dramatic shift in the food web of
Guam's forests

8. Which of the following can be inferred from the fact that there has been a shift in the brown tree snake's activity
pattern "to one that increasingly includes activity during daylight hours"?
A. The snakes are taking over the environmental roles of the native prey that have become extinct.
B. The snakes are shifting their activities to a time when more native lizards are available
C. The snakes are trying to avoid competition for prey from other nonnative predators active at night.
D. There are fewer nonnative prey available at night.

Paragraph 5: The reduction in the complexity of the Guam food web has had consequences beyond the loss of
native animals. The loss of mammalian and avian insectivores presumably increased insect abundances at some cost
to agricultural crops and to crop production. And newly introduced nonnative species may find it easy to invade
Guam given the many "open niches" left by the loss of native species.

Paragraph 3: The brown tree snake was first noticed along Guam's southern shore near Apra Harbor. It then spread,
somewhat slowly. until it occupied the entire island by 1970. Guam has no native snakes and no native predators that
could have controlled tree snake numbers.█ Instead, the native vertebrates were all small species vulnerable to
predation by the generalist tree snakes. █The decline of most native forest animals was immediate and dramatic.█
Guam's Division of Aquatic and Wildlife Resources discontinued surveys of native birds and bats in the 1970s, as
there were so few individuals of these species to count.█ The three native birds and mammals that persisted the
longest were the Mariana fruit bat, Guam rail, and island swiftlet. The fruit bat and rail were relatively long-lived

第 98 页 共
373 页
2021 托福阅读真题

species that likely persisted simply because some individuals escaped predation and lived out the remainder of their
lives: however, neither species had any reproductive success in the presence of the tree snake. The island swiftlet
persists to this day but is confined to one cave on Guam. The swiftlet builds nests on the walls of caves using its own
saliva and mud to adhere the nest to the cave wall. Despite this unusual habit, swiftlets are still vulnerable to tree
snake predation, as tree snakes can easily capture prey in total darkness and climb most surfaces. The one cave
where swiftlets persist is unique in that the cave walls are not textured enough for snakes to support themselves. This
does not inhibit the swiftlets' ability to build nests, but it does prohibit tree snakes from reaching those nests.

9. Look at the four squares █ that indicate where the following sentence could be added to the passage.
As early as the late 1960s, native birds were disappearing from Guam’s forests.
Where would the sentence best fit? Click on a square █ to add the sentence to the passage.

10. Directions: An introductory sentence for a brief summary of the passage is provided below. Complete the
summary by selecting the 3 answer choices that express the most important ideas in the passage. Some sentences do
not belong in the summary because they express ideas that are not presented in the passage or are minor ideas in the
passage. (This question is worth 2 points.)

Guam was once home to a large number of native animal species.

A The brown tree snake probably arrived in Guam from New Guinea between 1945 and 1950 and, finding no native
predators, by 1970 it occupied the entire island.
B. The typically nocturnal tree snake quickly wiped out most of Guam's native animals, and today only one bird
(whose nests in one cave cannot be reached by the snake) and three small, diurnal lizards persist.
C The loss of native species opened many niches for invaders, and today most species on Guam are nonnative, but
these, with the native lizards, provide ample prey to maintain tree snake densities.
D The brown tree snake was first seen along Guam's southern shore, where it caused the extinction of most native
species before being stopped by native vertebrates when it later reached Guam's forests.
E. Swiftlets are the only birds whose numbers have risen on Guam, because they are able to build their nests in dark
caves that brown tree snakes do not enter.
F. Since the arrival of the brown tree snake, the food web on Guam has expanded to include a greater number of
nonnative species that prey on agricultural crops.

第 99 页 共
373 页
2021 托福阅读真题

The Chaco Phenomenon

Paragraph 1: A truly remarkable transformation in settlement patterns occurred in the San Juan basin in
northwestern New Mexico in the late tenth and early eleventh centuries, with small household farmsteads giving way
to aggregated communities centered on communal masonry buildings that are now called “great houses.” These
structures are found throughout the basin but are concentrated in Chaco Canyon, where several examples contained
hundreds of rooms and reached four stories in height. The largest great house is Pueblo Bonito, with over 600 rooms
covering two acres. The entire episode of great house construction in Chaco, the Bonito phase (A.D. 900-1140), was
obviously a time of immense cooperative effort. At least 200,000 wooden beams averaging 5 meters long and 20
centimeters in diameter were brought to the canyon from distances between 40 and 100 kilometers away to build a
dozen great houses, signifying a huge labor investment and a complex production process. The bulk of construction
took place in the eleventh century, but by A.D. 1140 it had ceased abruptly, after which there was a rapid decline in
use of the great houses and apparent abandonment of the canyon in the thirteenth century.

1. The word “signifying” in the passage is closet in meaning to


A. creating
B. indicating
C. initiating
D. requiring

2. According to paragraph 1, all of the following provide evidence that the Bonito phase was a time of immense
cooperative effort EXCEPT
A. the large amounts of material needed
B. the size of the Pueblo Bonito complex
C. the unusual materials used in construction
D. the distance the materials needed to be transported

Paragraph 2: For more than a century archaeologists have struggled to understand the circumstances surrounding
the rise and collapse of Chacoan society—dubbed the Chaco Phenomenon. In particular, research has focused on
determining why such an apparently inhospitable place as Chaco, which today is extremely arid and has very
short growing seasons, should have favored the concentration of labor that must have been required for such
massive construction projects over brief periods of time. Until the 1970s, it was widely assumed that Chaco had
been a forested oasis that attracted farmers who initially flourished but eventually fell victim to their own success
and exuberance, as they denuded the canyon of trees and vegetation to build large great houses. In the 1980s, this
reconstruction was largely dismissed in response to evidence that there had never been a forest in Chaco, and that
canyon soils had poor agricultural potential. As scientific interpretations about Chaco changed, the focus of
explanatory models changed from the attractiveness of the canyon for farmers to the position of the canyon within a

第 100 页 共
373 页
2021 托福阅读真题

regional network of dispersed agricultural communities.

3. Which of the sentences below best expresses the essential information in the highlighted sentence in paragraph 2?
Incorrect choices change the meaning in important ways or leave out essential information.
A. Researchers have tried to establish why an area as dry as Chaco was the site of such large construction efforts.
B. Researchers have tried to establish whether the concentration of massive construction projects in a brief period of
time made Chaco the dry area that it is today.
C. Researchers have established that Chaco’s brief growing season required a concentration of labor to produce large
quantities of food in a short period of time.
D. Researchers have established that the hot, dry climate of Chaco forced workers to complete construction on large
buildings in short periods of time.

4. It can be inferred from paragraph 2 that the pre-1970s theory about the Chaco Phenomenon
A. was based on the widespread farm and tool remains found by archaeologists on the site.
B. was largely reinforced by findings in the 1980s.
C. was not supported by substantial evidence.
D. was so strong that it went unchallenged for many decades.

5. According to paragraph 2, why did scientists change their view about the cause of the collapse of Chacoan
society?
A. They found evidence that Chaco had always lacked trees and good soil.
B. They discovered that Chaco Canyon was much drier than they had previously believed.
C. They learned that the population was not large enough to supply the laborers needed to build the great houses.
D. They found evidence that the farming economy was excessively concentrated in the central canyon.

Paragraph 3: The adoption of a regional perspective in explaining the Chaco Phenomenon was based in part on the
discovery of formal trails connecting many of the great houses in Chaco, as well as linking the canyon to smaller
great houses located throughout the San Juan basin, the latter are referred to as Chaco “outliers.” These trails are
densest around the concentration of great houses in the center, and the canyon itself is roughly at the center of the
basin. Consequently, the canyon occupies the geographical and social center of the network formed by the
connecting trails. The current consensus view is that religion provides the fundamental explanation for this
centrifugal pattern.

Paragraph 4: Archaeologists now describe Chaco during the Bonito phase as a location of high devotional
expression and the pilgrimage center of a sacred landscape. These descriptions emphasize aspects of the
archaeological record presumed to be associated with ritual activity, including caches of turquoise beads and
pendants, unusual ceramic vessels and wooden objects, several rooms with multiple human burials, and especially

第 101 页 共
373 页
2021 托福阅读真题

the large number of kivas (multipurpose rooms used for religious, political, and social functions) found in great
houses. Most of these indicators occur only at Pueblo Bonito, but archaeologists generally assume that all the great
houses had a similar ritual function. In fact, some scholars have suggested that the great houses were temples rather
than residences.

6. According to paragraphs 3 and 4, which of the following best describes how archaeologists arrived at their current
view of the nature of Chaco during the Bonito phase?
A. They discovered a large number of kivas, which probably served as temporary houses for pilgrims on their way to
the main temple.
B. They found a series of paths leading to the outliers, which seem to have been centers of trade for makers of
jewelry and other products.
C. They found turquoise beads and pendants and other valuable objects, leading to the theory that the great houses
were wealthy residences.
D. They discovered many objects and rooms associated with ritual activity, leading to the theory that Chaco was a
religious center.

7. The word “function” in the passage is closet in meaning to


A. center
B. practice
C. design
D. purpose

Paragraph 5: However, new geological field studies in Chaco have produced results that may require a significant
reassessment of the assumption that the canyon was not a favorable agricultural setting. It appears that during the
first half of the eleventh century, during the extraordinary boom in construction, a large volume of water and
suspended sediment flowed into the canyon. A large natural lake may have existed at the western end of Chaco,
near the biggest concentration of great houses. The presence of large quantities of water and, equally important, a
source of sediment that replenished agricultural fields, presumably made the canyon an extremely attractive place for
newly arriving people from the northern San Juan River basin.

8. Why does the author state that “A large natural lake may have existed at the western end of Chaco, near the
biggest concentration of great houses?”
A. To suggest that geological studies are better than archaeological studies in identifying the historical uses of land
B. To demonstrate that large construction projects require a large population of workers
C. To support the idea that Chaco may have been favorable to agriculture during the Bonito phase
D. To show that the Chacoan people preferred to build their homes near water

第 102 页 共
373 页
2021 托福阅读真题

Paragraph 3: ■The adoption of a regional perspective in explaining the Chaco Phenomenon was based in part on
the discovery of formal trails connecting many of the great houses in Chaco, as well as linking the canyon to smaller
great houses located throughout the San Juan basin, the latter are referred to as Chaco “outliers.” ■These trails are
densest around the concentration of great houses in the center, and the canyon itself is roughly at the center of the
basin. ■Consequently, the canyon occupies the geographical and social center of the network formed by the
connecting trails. ■The current consensus view is that religion provides the fundamental explanation for this
centrifugal pattern.

9. Look at the four squares [■] that indicate where the following sentence can be added to the passage.
Scholars have attempted to find a reason for this web-like arrangement of great houses around a central
canyon.

10. Directions: An introductory sentence for a brief summary of the passage is provided below. Complete the
summary by selecting the THREE answer choices that express the most important ideas in the passage.

The population of the Chaco Canyon in New Mexico changed significantly between the tenth and eleventh
centuries, as evidenced by the remains of its great houses.

A. Before the 1970s, scholars believed that the failure of Chacoan society was caused by farmers’ cutting down all
the trees to build their great houses.
B. After discovering trails connecting Chaco to surrounding communities, scholars came to believe that there were
many forested oases to support those communities.
C. Archaeological evidence has led current scholars to believe that Chaco was a religious center during the Bonito
phase.
D. Archaeological findings indicate that Chaco Canyon was completely abandoned by the end of the thirteenth
century.
E. Researchers’ findings in the 1980s revealed that Chaco Canyon had been a fertile agricultural area that caused the
population near the center of the canyon to increase steadily during the Bonito phase.
F. Recent geological studies indicating the presence of water in Chaco Canyon in the eleventh century may alter
scholars’ belief that the area was not favorable for farming.

第 103 页 共
373 页
2021 托福阅读真题

Challenge of Dendrochronology

Paragraph 1: Dendrochronology is the technique of counting tree rings to determine a tree's age and measuring the
width of these rings to determine characteristics of past climates. This might seem simple: each ring represents one
year, and wider rings generally mean better growing conditions-plentiful rainfall, moderate temperatures, and so
forth. But the seasonal growth of a particular tree is affected by factors other than the weather. Trees vary, one from
another, just like people do. The genetic makeup of each individual tree is unique, so one particular tree may grow a
bit more quickly that another. Highly local conditions can also change over time. It is easy enough to see that if part
of the soil near a tree has been eroded, this will impact the tree's root system and limit its growth, at least until
the situation stabilizes. Then again, an infestation of insects may affect a tree in one valley more than the same type
of tree ten miles away. Or one tree may suddenly start to get a lot more sunlight when an old, big tree in the
neighborhood finally falls. These kinds of factors produce significant variations among individual specimens, and
that fact means that researchers need to average together samples from many specimens of a single tree species in
one region over the same time period. Some dendrochronologists think that measuring an average of twenty-five to
thirty tree-ring records in a locale is an essential first step in getting around the problem of individual variability.
While it may be easy enough to find thirty samples in some locations for particular periods, it obviously becomes
less and less likely the more ancient the wood samples are.

1. Why does the author provide the information that "if part of the soil near a tree has been eroded. this will
impact the tree's root system and limit its growth. at least until the situation stabilizes"?
A, To give an example of a factor other than weather that can affect the seasonal growth of a particular tree
B, To call into question the importance of the unique genetic makeup of each individual tree as a factor in tree
growth
C, To argue that the condition of soil at a particular time can be easily determined using dendrochronology
D, To provide an explanation for why a particular tree might be more affected by an insect infestation than other
nearby trees of the same type

2. Which of the following can be inferred from the passage about a tree that "may suddenly start to get a lot
more sunlight when an old, big tree in the neighborhood finally falls"?
A, It will become more vulnerable to insect infestations.
B, It will no longer require as extensive a root system as it once did.
C, It will have rings that reflect its age more accurately than do the rings of trees where soil is eroded.
D, It will have a change in its growth rings

3. The word "essential" in the passage is closest in meaning to


A, useful
B, obvious

第 104 页 共
373 页
2021 托福阅读真题

C, critical
D, possible

4. According to paragraph 1, how do dendrochronologists deal with the problem of individual tree
variability?
A, They only use samples from trees that have been living for at least twenty-five to thirty years
B, They select samples from tree species that are known to show little variation over time and within a particular
location
C, They do not use samples from trees that show evidence of an unknown genetic makeup
D, They average together the measurements from samples of many different trees from the same location and time

Paragraph 2: Another issue is more general. Trees that are fortunate enough to live on good soil and near local
sources of groundwater often grow at steady rates. Such growth translates into attractive trees that are tall and well
formed; they also have rings that are wide and quite uniform in thickness, but their uniform growth rings make them
entirely useless when it comes to inferring anything about past weather patterns. That is why, instead of looking at
superb botanical specimens, dendrochronologists focus their work on wood from trees that are living a tough life due
to poor soil, steep slopes, the absence of local groundwater, or some other challenge. It is these "tortured" trees that
are the most likely to grow very little during years of scarce rains or do poorly after a harsh winter and a late spring.
What this means, of course, is that few trees in the woods are likely to be good samples for the scientist. Indeed, it
may be quite a small fraction that yields useful ring patterns. Again, this increases the challenge of finding enough
good samples to say with much certainty what past conditions were like.

5. The word "scarce" in the passage is closest in meaning to


A, abnormal
B, inadequate
C, damaging
D, delayed

6. According to paragraph 2, which of the following is true of trees that are fortunate enough to grow on good
soil near groundwater sources?
A, They have rates of growth that increase over time
B, They are highly interesting to dendrochronologists because of their height and attractive formation.
C, They are affected more significantly than other trees by harsh winters and the late onset of spring
D, They have tree rings that are not helpful in providing an understanding of weather patterns over time.

Paragraph 3: Another factor of dendrochronology relates to wood itself. In the spring, a tree grows rapidly, creating
new cells on the outside of its trunk and branches, just under the bark. These cells, called "earlywood" or

第 105 页 共
373 页
2021 托福阅读真题

"springwood," are large and have thin cell walls; both these factors contribute to making the wood relatively
lightweight for its volume. In the summer, growth slows. Denser "latewood" is formed, creating the band that is
relatively dark when you look at the end of a piece of lumber. But occasionally the sequence of a perfect pair of
springwood and latewood does not hold up. If conditions-weather or disease-severely test a tree one year, it will not
grow over all its surfaces. That may mean that a particular sample of wood taken by the dendrochronologist will
have a missing ring in it, which will result in the scientist's inferences being off base by a year

7. According to paragraph 3, which of the following is NOT true of earlywood?


A, It is less dense than latewood
B, It makes up the part of the ring that appears dark
C, Its cells are large with thin walls
D, It forms quickly, just under the bark of the tree's trunk and branches.

Paragraph 4: A few trees also may trip up scientists by revealing a "false ring" made of latewood that's in the
middle of springwood. These features, sometimes known as double rings, usually can be distinguished from true
rings because the unusual dark ring is likely to change gradually rather than more abruptly into the springwood that
lies on either side of the false latewood. It is not clear what creates such double rings, although people have
speculated that unusual conditions during the middle of the growing season or even highly local issues might be the
cause

8. According to paragraph 4. which of the following is NOT true of false rings?


A, They are also called double rings.
B, Their exact cause is unknown.
C, They are very often mistaken for true rings even by experienced dendrochronologists
D, They consist of a strip of dark wood in the middle of a section of springwood.

Paragraph 3: Another factor of dendrochronology relates to wood itself. In the spring, a tree grows rapidly, creating
new cells on the outside of its trunk and branches, just under the bark. █ These cells, called "earlywood" or
"springwood," are large and have thin cell walls; both these factors contribute to making the wood relatively
lightweight for its volume. █ In the summer, growth slows. █ Denser "latewood" is formed, creating the band that is
relatively dark when you look at the end of a piece of lumber. █ But occasionally the sequence of a perfect pair of
springwood and latewood does not hold up. If conditions-weather or disease-severely test a tree one year, it will not
grow over all its surfaces. That may mean that a particular sample of wood taken by the dendrochronologist will
have a missing ring in it, which will result in the scientist's inferences being off base by a year

9. Look at the four squares █ that indicate where the following sentence could be added to the passage.
It is also rather light in color.

第 106 页 共
373 页
2021 托福阅读真题

Where would the sentence best fit? Click on a square █ to add the sentence to the passage

10. Directions: An introductory sentence for a brief summary of the passage is provided below. Complete the
summary by selecting the 3 answer choices that express the most important ideas in the passage. Some sentences do
not belong in the summary because they express ideas that are not presented in the passage or are minor ideas in the
passage. This question is worth 2 points.

Dendrochronologists who study tree rings to determine past climates face several challenges

A, Some trees respond to favorable growing conditions primarily by growing in height or thickness, while others
devote resources to production of leaves or fruit, which will not show on growth rings.
B, Most trees' individual growth is consistent from year to year, so only trees growing in severe conditions have
useful variation in ring width, making finding a large enough sample of trees difficult
C, Under certain conditions, a tree may have missing rings or false rings, which can lead to inaccuracies in counting
the tree's age and in inking particular growth rings to the correct year.
D, There is a great deal of difference in how quickly individual tree grow, due to both genetic differences and highly
local conditions which affect only one tree or a small group of trees
E, Trees that live in regions without distinctive seasons do not have different types of growth at different times of
year, and thus do not produce rings that can be used for dendrochronology
F, Because earlywood is relatively lightweight for its volume, it degrades more quickly than latewood, leading to
additional difficulties in counting the rings of ancient trees.

第 107 页 共
373 页
2021 托福阅读真题

The Process of Domestication

Paragraph 1: Domestication was not an innovation restricted to farming societies. The first of all domestic
animals-the dog-was domesticated by hunter-gatherers during the Paleolithic Age. which ended roughly 12,000 years
ago. Analysis of DNA lineages suggests an initial domestication at least 14.000 and perhaps as much as 135.000
years ago. Hunter-gatherers often develop close relationships with key plants as well as with animal species, which
lead to practices verging on domestication. Certain Australian Aborigines. for example, followed a practice of
replanting parts of the yams that they dug up. In northern Australia the practice was to eave the main plant and its
root, and to collect and consume only the side tubers; in western Australia, however, people dug up the tubers, broke
them into pieces, and returned some parts to the ground. Even more elaborate practices occurred elsewhere: the
Owens Valley Paiute tribe of eastern California diverted streams to irrigate natural "fields" of water-meadow root
crops.

1. The phrase "verging on" in the passage is closest in meaning to


A, dependent
B, close to
C, resulting from
D, equal to

2. According to paragraph 1, hunter-gatherers are known to have engaged in each of the following practices
EXCEPT
A, replanting parts of certain plants that they had dug up in order to consume
B, collecting only some parts of certain food plants and leaving other parts in place
C, changing the course of streams to provide water for certain kinds of plants
D, planting food plants in places where water was readily available

Paragraph 2: Two conclusions may be drawn from this. First, that hunter-gatherers were not simply passive
bystanders in the history of plant and animal exploitation, but modified those species on which they relied, both
intentionally and unintentionally. And second, that close relationships between humans and their food sources
did not begin abruptly with the development of agriculture less than 10,000 years ago in the early postglacial
period, but have a much longer history stretching back tens of thousands of years into the Paleolithic.

3. Which of the sentences below best expresses the essential information in the highlighted sentence in the
passage? Incorrect choices change the meaning in important ways or leave out essential information.
A, And second, close relationships between humans and their food sources began in the early postglacial period that

第 108 页 共
373 页
2021 托福阅读真题

stretched back tens of thousands of years into the Paleolithic.


B, And second, close relationships between humans and their food sources began to develop not with the beginning
of agriculture, but tens of thousands of years earlier.
C, And second, agriculture began less than 10,000 years ago in the early postglacial period following the Paleolithic,
which stretched back many thousands of years
D, And second, the development of agriculture led to close relationships between humans and their food sources tens
of thousands of years ago.

Paragraph 3: Domestication involves the removal of species from the wild and their propagation by humans within
a sheltered or manipulated setting. As a result, domesticates are subjected to different selective pressures from their
wild relatives and so undergo morphological and genetic change from their wild ancestors through processes of
natural selection. Domesticated species are also subject to selection by humans, who may prefer smaller and more
docile individuals in a herd, for example, or may breed new forms that have specially valued characteristics, such as
woolly sheep.

4. According to paragraph 3, woolly sheep were a new form that resulted from
A, natural pressures of living in a herd
B, selective pressures favoring smaller size
C, processes of natural selection
D, intentional selection by humans

Paragraph 4: Other consequences of human contact may be unintentional. In a now-classic experiment, Jack Harlan
harvested wild stands of cereals by hand in southeast Turkey and showed that it was possible for a small family
group to gather in only three weeks enough to sustain them for a year. It is important to consider the effect of such
collection on plant communities, in particular on the way in which plants reseed themselves. Those with brittle seed
heads will drop their seeds to the ground as soon as they are touched, so those with tougher seed heads will be
preferentially gathered by human collectors. Should human collectors use the plants they gathered as the basis of
next year's crop, they will be sowing the tougher seed head variety, thus altering the characteristics of the species
overall. It may have been through this sort of process of unintentional selection that domesticated forms of wheat
and barley first developed in Southwest Asia.

5. Paragraph 4 suggests that humans who harvested wild stands of cereals by hand preferred tougher seed
heads because those are
A, more nutritious
B, easier to collect
C, more common
D, larger

第 109 页 共
373 页
2021 托福阅读真题

Paragraph 5: There were other common changes in domesticated species. A reduction in body size among animals
occurred, either through intentional selection or as the unintentional result of breeding conditions. It should be noted,
however, that size reduction is a widespread feature of postglacial mammals and has affected humans as well as
animals. There is evidence also for an increase in size among cereals and tubers, through selective propagation. In
addition, incidental changes, such as twisted horns in goats, or the loss of natural coloring in cows or horses, may be
due to the relaxation of natural selective pressures in the protected humanly controlled environment; black and white
Friesian cows, for example, would be conspicuous to predators and thus have reduced adaptive fitness in the wild.

6. Why does the author mention that "size reduction is a widespread feature of postglacial mammals"?
A, To point out that size reduction was the most common change in domesticated species
B, To indicate that the assumed connection between smaller animals and domestication may be open to question
C, To suggest that size reduction was more likely the result of intentional selection than an unintended result of
breeding conditions
D, To support the idea that domestication affected mammals of all kinds very differently than it affected plants such
as cereals and tubers

7. In paragraph 5, the loss of natural coloring in cows or horses is presented as an example of


A, a change that may have been an accidental effect of domestication
B, a change that is most likely to have resulted from intentional human selection
C, a change that resulted from natural selective pressures
D, a change that creased adaptive fitness in protected environments

Paragraph 6: The eventual result of these changes was the emergence of distinct domesticated species, many
(though not all) of which could no longer survive in the wild without human intervention Furthermore, the success of
the new food-producing economy, based on effective combinations of domestic plants and animals (such as the triad
of maize, beans, and squash in the Americas) led to its relatively rapid expansion at the expense of hunting and
gathering. As a result, species were carried by human action to areas far beyond the geographical range of their wild
ancestors.

8. According to paragraph 6. each of the following came about as a result of the development of domestication
EXCEPT
A, the appearance of species that could not survive in the wild
B, a decline in hunting and gathering
C, the relocation of hunter-gatherer economies to different areas
D, species being carried to areas outside their original geographical range

第 110 页 共
373 页
2021 托福阅读真题

Paragraph 5: There were other common changes in domesticated species. █ A reduction in body size among
animals occurred, either through intentional selection or as the unintentional result of breeding conditions. █ It
should be noted, however, that size reduction is a widespread feature of postglacial mammals and has affected
humans as well as animals. █ There is evidence also for an increase in size among cereals and tubers, through
selective propagation. █ In addition, incidental changes, such as twisted horns in goats, or the loss of natural
coloring in cows or horses, may be due to the relaxation of natural selective pressures in the protected humanly
controlled environment; black and white Friesian cows, for example, would be conspicuous to predators and thus
have reduced adaptive fitness in the wild.

9. Look at the four squares █ that indicate where the following sentence could be added to the passage.
Studies in the dry valleys of Mexico, for example, have demonstrated the dramatic size increase of maize cobs
that can occur through intentional selection.
Where would the sentence best fit? Click on a square █ to add the sentence to the passage.

10. Directions: An introductory sentence for a brief summary of the passage is provided below. Complete the
summary by selecting the 3 answer choices that express the most important ideas in the passage. Some sentences do
not belong in the summary because they express ideas that are not presented in the passage or are minor ideas in the
passage. This question is worth 2 points.

Long before humans stopped being hunters and gatherers, they had begun to influence the history of plant
and animal species.

A, Hunter-gatherers developed close relationships in the wild with the plant and animal species on which they relied,
and, in doing so hanged them both intentionally and unintentionally.
B, Experimental evidence suggests that domestication was slower to occur in locations where small family groups
could quickly gather enough cereal to live on for a year
C, Domesticated species are subject to intentional selection by humans for valued characteristics, but many changes
in domesticated species may be the unintentional results of breeding conditions
D, True domestication involves removing species from the wild and propagating them within a humanly controlled
environment in which they are subject to different pressures than their wild relatives.
E, While some humans gave up hunting and gathering to become part of the new food-producing economy. there
were many others who preferred wild as opposed to domesticated food sources.
F, Humans selected for characteristics such as the twisted horns in goats in order to reduce the adaptive fitness of
domesticated animal in the wild.

第 111 页 共
373 页
2021 托福阅读真题

Ancient Mapmaking

Paragraph 1: Claudius Ptolemy, who lived from approximately 85 to 168 AD, was an ancient mapmaker whose
works were rediscovered in Europe after being lost until the fifteenth century. He lived in Alexandria, Egypt, where
he used Alexandria's famous library to compile existing knowledge of astronomy, geography, and astrology into
three treatises. The astronomy and geography treatises had a long-lasting influence. but they both presented serious
errors that went uncorrected for about 1,300 years. Ptolemy's astronomy treatise, Almagest, rejected the theory
earlier proposed by Aristarchus (approximately 230 BC) that Earth revolves around the Sun.Ptolemy's geocentric
idea-that Earth was the center of the universe -accepted the ideas of Aristotle and formed the main thesis of his
treatise. When Ptolemy's works resurfaced in the fifteenth century, they were accepted as gems of ancient wisdom,
and few had the nerve or the authority to challenge them. Likewise, any sixteenth century maps that altered the
Ptolemy map were regarded with suspicion.

1. According to paragraph 1. which of the following is true of Ptolemy's treatises?


A.They were no longer accepted in the fifteenth century
B.They presented the idea that Earth revolves around the Sun.
C.They rejected the ideas of Aristotle
D.Their inaccuracies had an impact on fifteenth-century thought

2. According to paragraph 1, which of the following is true of Aristarchus?


A.He was an ancient mapmaker like Ptolemy.
B.He proposed a theory that Earth was not at the center of the universe.
C.His theory was based on Ptolemy's astronomy treatise Almagest
D.He largely accepted the astronomical ideas of Aristotle.

Paragraph 2: In his other influential treatise, Geographia, Ptolemy rejected the nearly correct computation of the
distance around Earth-Earth's circumference-made by Eratosthenes in approximately 240 BC Rather, he chose an
erroneous and much smaller distance (about 75 percent of the actual size). Ptolemy did not make any measurements
himself, as Eratosthenes had done, but selectively compiled other information that was known at the time. The
estimate he chose came from the Greek astronomer Poseidonius. Subsequently, however, his choices became known
as Ptolemaic ideas and were considered irrefutable. Also, Ptolemy assumed that the known world's land surface
covered 180 degrees of longitude ranging from the Canary Islands in the west to the easternmost part of Asia (about
20 degrees of longitude too much). This error on his map showed the Atlantic Ocean much too narrow and
connecting western Europe and east Asia, without the American continents in between. This remained the
understanding of the world for 1.300 years.

第 112 页 共
373 页
2021 托福阅读真题

3. The word 'erroneous’ in the passage is closest in meaning to


A.widely accepted
B.recently determined
C.Incorrect
D.Debatable

4. According to paragraph 2, how did Ptolemy determine the circumference of Earth for his treatise Geographia?
A.He made his own measurements.
B.He computed an average of the measurements made by several Greek astronomers.
C.He relied on Eratosthenes' computation.
D.He used an estimate Poseidonius had made.

Paragraph 3: The combination of these two errors-Earth's circumference too small and land area too
large-encouraged mariners of the fifteenth century to assume that a relatively short voyage across the Atlantic Ocean
would take them to Asia. Columbus was the first to promote an expedition on the basis of these errors. When
Columbus reached land, he had traveled as far as he expected to travel to reach Asia and logically assumed that he
had succeeded.

5. The word relatively in the passage is closest in meaning to


A.unusually
B.predictably
C.comparatively
D.conveniently

6. Paragraph 3 suggests that it was logical for Columbus to assume that he had reached Asia because
A.Columbus had calculated Earth's circumference using a new method that, although mistaken, seemed correct
B.other mariners had reported that they had reached Asia by crossing the Atlantic Ocean.
C.the voyage across the Atlantic Ocean took Columbus much longer than expected
D.the distance Columbus had traveled matched the distance between Europe and Asia on Ptolemy's map

Paragraph 4: To his credit, Ptolemy's map introduced some excellent standard to mapmaking. despite the
errors. Though he was not the first to use he idea of a gridded coordinate system, his method of showing attitude and
longitude became a standard for future maps. Also Ptolemy insisted that maps should be drawn to scale. Many maps
of his time were distorted by enlarging the better known places in order to include all the known
information. Unfortunately, many mapmakers of his time failed to adopt his practical approach to scale and location.

第 113 页 共
373 页
2021 托福阅读真题

Paragraph 5: Maps began to proliferate in the sixteenth century. Each voyage of exploration and discovery
provided new information that had to be mapped. In 1507, a German mapmaker, Martin Waldseemoler, produced a
map of the world, Universalis Cosmographia, which was the first to show Columbus's discovery as a separate
continent. But he cautiously made the new continent very narrow-just a long,skinny island-rather than contradict
Ptolemy's erroneous circumference of Earth By the middle of the sixteenth century, enough voyages had been made,
including Magelan's trip around the world, that most mapmakers recognized that Earth's circumference as shown on
Ptolemy's map was wrong.

7. According to paragraph 5, the number of maps notably increased in the sixteenth century because
A.mapmakers were competing to show new continents on their maps
B.so much new information was being learned from explorers voyages
C.mapmakers had finally recognized that Earth's circumference as shown on Ptolemy's map was wrong
D.mapmaking had become very profitable in places such as Germany

Paragraph 6: In 1569, the Flemish mapmaker, Gerardus Mercator, made a map of the world showing all the known
lands using his now famous innovative grid system of latitude and longitude. In 1570, Abraham Ortelus, a Belgian
mapmaker, made the first known atlas of the world in an effort to compile the rapidly accumulating geographic
knowledge. Although these maps still had some remaining traits of the Ptolemy map, they showed great
improvements in detail and accuracy. A major feature retained from the Ptolemy map was the presence of a very
large continent in the Antarctic-large enough to counterbalance the weight of the Northern Hemisphere land. This
belief was based on the Greek concept of symmetry, as well as the idea that Earth needed to be balanced to turn
smoothly.

8. Why did the author mention the idea that Earth needed to be balanced to turn smoothly?
A.To help explain why maps continued to show a very large continent in the Antarctic
B.To demonstrate how using a grid system of latitude and longitude affected mapmaking
C.To contrast the Greek approach to understanding continents with that of later mapmakers such as Mercator and
Ortelus
D.To help clarify how people in the mid-sixteenth century interpreted the Greek concept of symmetry

Paragraph 4: To his credit, Ptolemy's map introduced some excellent standard to mapmaking. despite the errors.
█ Though he was not the first to use he idea of a gridded coordinate system, his method of showing latitude and
longitude became a standard for future maps. █ Also Ptolemy insisted that maps should be drawn to scale. █ Many
maps of his time were distorted by enlarging the better known places in order to include all the known
information. Unfortunately, many mapmakers of his time failed to adopt his practical approach to scale and location.

9. Look at the four squares █ that indicate where the following sentence could be added to the passage.

第 114 页 共
373 页
2021 托福阅读真题

It would, in fact, be many centuries before mapmakers practiced their craft in line with his principles,
producing better maps.
Where would the sentence best fit? Click on a square █ to add the sentence to the passage.

10. Directions: An introductory sentence for a brief summary of the passage is provided below. Complete the
summary by selecting the 3 answer choices that express the most important ideas in the passage. Some sentences do
not belong in the summary because they express ideas that are not presented in the passage or are minor ideas in the
passage. (This question is worth 2 points.)

The ideas of Claudius Ptolemny became influential in Europe after his works were lost for many centuries.

A. Although Ptolemy complied three major works, his treatise on geography was the first to resurface and was the
only treatise that had much influence in the fifteenth and sixteenth centuries.
B. Ptolemy’s highly respected map showed only a narrow ocean between western Europe and eastern Asia, and this
made attempts to sail to Asia seem reasonable to European explorers.
C. Information gained from explorers and a new grid system of latitude and longitude greatly improved the accuracy
of maps, although some of Ptolemy's errors were retained.
D. Ptolemy improved maps by having them drawn to scale using a coordinate system, but he incorrectly represented
the size of Earth and its known land areas.
E. Columbus thought that the land mass he had reached was merely a long, thin island rather than a separate
continent, and maps of the day reflected his view.
F. Mapmakers eventually accepted that the Northern Hemisphere's land area was smaller than Ptolemy estimated, so
they began to draw a smaller continent in Antarctica to keep their maps balanced.

第 115 页 共
373 页
2021 托福阅读真题

Agriculture and Religion

Paragraph 1: For hundreds of thousands of years, humans managed to survive by moving seasonally from
place to place gathering edible plants and hunting, until about 10,000 years ago when, probably as a result of
a fortuitous accident of dropping seeds on fertile ground, the rudiments of agriculture were discovered. The
stages of the transition from hunting and gathering to an agrarian economy and society can be reconstructed from the
archaeological record. The record indicates that after the introduction of agriculture in the Near East, irrigation was
developed as a means to ensure higher crop yields. A limited number of crops (various grains) and animals (mainly
sheep and goats) were domesticated. Certain nomadic or seminomadic groups then became sedentary, settling more
or less permanently in villages, which were often surrounded by earthen on stone walls. Sometimes, these villages
were quite large. For example, the ruins of Catalhöyük in Anatolia, built around 7000 B.c by a group of Neolithic
cattle breeders, give evidence of what can almost be termed a town. The settlement was inhabited for two thousand
years before being deserted for unknown reasons, but during the period of its occupation 10,000 people at a time
lived there. Sculpture and gaily colored frescoes (paintings done on plaster walls or ceilings) indicate that the
residents of Catalhöyük had thoughts that went beyond everyday concerns for physical survival.

1. Which of the sentences below best expresses the essential information in the highlighted sentence in the
passage? Incorrect choices change the meaning in important ways or leave out essential information.
A, For hundreds of thousands of years, humans managed to survive gathering edible plants and hunting.
B, The rudiments of agriculture were discovered about 10.000 years ago following hundreds of thousands of years
during which humans survived by moving seasonally to hunt and find edible plants.
C, A fortuitous accident of dropping seeds on fertile ground about 10,000 years ago led to the discovery of the
rudiments of agriculture.
D, Humans continued to survive by gathering edible plants and hunting for tens of thousands of years after the
accidental discovery of the rudiments of agriculture.

2. The word “ensure" in the passage is closest in meaning to


A, collect
B, maintain
C, produce
D, guarantee

3. According to paragraph 1, which of the following is a correct ordering of stages in the transition from
hunting and gathering to an agrarian economy?
A, Building of walled villages, domestication of cattle, irrigation for higher yields

第 116 页 共
373 页
2021 托福阅读真题

B, First growing of crops, irrigation for higher yields, permanent settlement


C, Discovery of agriculture, permanent settlements, irrigation for higher yields
D, Founding of Catalhöyük, domestication of grains and sheep, domestication of cattle

Paragraph 2: Where conditions for agriculture were particularly favorable, complex societies and specialized forms
of organization developed. River valleys led the way in this development: In Egypt, the Nile River flooded annually,
leaving a narrow strip of fertile mud in the desert that supported a rich harvest and a vibrant civilization. In
Mesopotamia (modern Iraq), the land between the Tigris and Euphrates rivers was another center in which
important civilizations arose. The Tigris and the Euphrates river system not only provided plenty of water for
irrigation, but it also allowed for transport between the two emerging food-producing areas of the Near Eastnorthern
Syria and the southern region of Mesopotamia.

4. Why does the author discuss "Egypt" and "Mesopotamia" in the passage?
A, To support the claim that complex societies developed where river valleys made conditions for agriculture
favorable
B, To identify two river valleys where frequent flooding initially prevented the societies founded there from
flourishing
C, Priests maintained their status as administrators of the religion by virtue of heredity
D, To contrast the different irrigation methods of two major civilizations

Paragraph 3: Soon communities sprang up that based their prosperity both on agriculture and on the manufacture of
products necessary for survival such as tools and weapons made of stone and, later, of bronze, and pottery for
cooking and storage of grain. These communities also made luxury items, such as costly textiles, artful metalwork,
and jewelry set with precious stones. Trade networks developed to distribute these various kinds of goods. Rivers
played a significant role in the trade routes, but there were also overland routes on which camels and donkeys were
used.

5. Paragraph 3 supports which of the following claims about economic development in societies that adopted
agriculture?
A, Manufacturing quickly became economically more important than farming.
B, A middle class emerged that had sufficient wealth to buy luxury goods.
C, There was a diversification and specialization of non-agricultural crafts.
D, Economic development depended on the domestication of camels and donkeys

Paragraph 4: These agricultural civilizations were extremely dependent on water and other natural resources, and
this dependence led to an interest in the heavenly bodies that were systematically associated with the agricultural
cycle of the year. The relationship between agriculturally important variables and the regular movements of the Sun.

第 117 页 共
373 页
2021 托福阅读真题

Moon, stars, and planets could not as yet be analyzed scientifically. but in the minds of the ancient farmers, the
heavenly bodies were important forces.

6. Paragraph 4 provides the answer to which of the following questions?


A, Why were the movements of the heavenly bodies important to ancient agricultural civilizations?
B, What actions did people take in response to the observed movements of the heavenly bodies?
C, How were the movements of the heavenly bodies explained?
D, Which heavenly bodies did ancient farmers consider the most influential?

Paragraph 5: In this situation religions came into existence which worshipped the forces of nature and the heavenly
bodies as gods. █ It was not long before those who purported to have knowledge of these gods' activities and the
power to influence them were especially honored. █ These priests served as intermediaries between the divine and
the human worlds. █ Thus, in the hope that they would gain the gods favor, farmers gave the priests some of their
surplus production █ Frequently. a well-defined caste of priests developed with hereditary claims to power. In these
early religions, people expressed their relationship to the unknown and incomprehensible features of their existence
by creating gods. At first the gods were imagined as animals, reflecting the worldview of a pastoralist-nomadic
society. Later, images of gods with human forms were made in the developing urban agricultural communities.
Deities were worshipped in ever more elaborately built cult sites, often centered around mountain-like structures
reaching up to the heavens, where these gods supposedly lived. The faithful went to the temples with their gifts of
grain or cattle, and from the temples the priests exercised a growing power over society.

7. The word "incomprehensible" in the passage is closest in meaning to


A, disturbing
B, overly complex
C, impossible to understand
D, uncontrollable

8. According to paragraph 5, how did farmers attempt to earn divine help with their harvests?
A, By studying the heavenly bodies
B, By becoming intermediaries to the divine world
C, By giving part of their harvests to the priests
D, By helping the priests build elaborate temples

9. Look at the four squares █ that indicate where the following sentence could be added to the passage.
In practice this meant that to influence the gods, one first had to influence the priests
Where would the sentence best fit? Click on a square █ to add the sentence to the passage.

第 118 页 共
373 页
2021 托福阅读真题

10. Directions: An introductory sentence for a brief summary of the passage is provided below. Complete the
summary by selecting the THREE answer choices that express the most important ideas in the passage. Some
sentences do not belong in the summary because they express ideas that are not presented in the passage or are minor
ideas in the passage. This question is worth 2 points

The adoption of agriculture provided the economic foundation for the development of complex societies in the
ancient Near East.
A, Catalhöyük in Anatolia was a Neolithic town with a population o 10,000 whose inhabitants, as shown by the
town's ruins, had interests beyond the means of their subsistence
B, Gradually, trade routes along rivers using boats and barges began to replace overland routes using camels and
donkeys.
C, The priests who served as intermediaries between the divine and human worlds gradually lost power when the
images of gods were given human forms.
D, The Nile River in Egypt and the Tigris and Euphrates Rivers in Mesopotamia supported prosperous agricultural
societies by providing fertile soil, irrigation, and transportation
E, As agriculture produced surpluses, societies began to invest in the production of non-agricultural goods, including
luxury items, which were traded throughout the Near East
F, In ancient agricultural societies heavily dependent on nature heavenly bodies came to be regarded as gods who
could be influenced through religious rituals.

第 119 页 共
373 页
2021 托福阅读真题

Temperate Plant Phenology

Paragraph 1: Phenology is the timing of growth and reproductive activity within a year, and it can vary greatly
among plant species, populations, and even individuals. In temperate forests (characterized by moderate
temperatures), many forest-floor plants expand their leaves and flower before the trees that form the forest's upper
layer (the canopy) begin leaf expansion. The consequence is that these plants do most of their growing and
reproducing in temperatures that are far colder than those experienced by the canopy trees during the same life
stages. Many studies have shown that this timing is actually advantageous to the forest-floor plants. Once the leaves
of the trees fill in the open spaces, very little sunlight sometimes as little as 1 percent-gets through to the forest floor.
Thus, the forest-floor plants' growth and reproduction would be even more limited by light availability than they are
by cool temperatures

1. According to paragraph 1, which of the following is true about the phenology of temperate forests?
A, Growth and reproduction of forest-floor plants are limited more by cool temperatures than by the availability of
light.
B, Canopy trees grow and reproduce in temperatures far colder than those in which forest-floor plants do at the same
stages
C, Leaf expansion takes place in canopy trees later than in forest-floor plants.
D, The timing of growth and reproductive activity is more advantageous to canopy trees than it is to forest-floor
plants.

Paragraph 2: In most temperate species, it appears that temperature and photoperiod (day length) are the main
factors determining plant phenologies. It is important to realize that temperature plays its role in a particular way. It
is generally the sum of temperatures experienced over some period of time and not the temperatures on a particular
day that determines the timing of leaf expansion. In cold years, then, leaf expansion is delayed. Plants, like other
organisms, often use photoperiod as a reliable predictor of the average temperature. If they relied exclusively on
temperature, a warm spell in midwinter would typically cause many plants to expand their leaves. By using
temperature as a cue, plants can respond to an early spring by expanding their leaves early, but because they also use
photoperiod as a cue, this response is limited. These plants, in other words, respond to environmental fluctuations,
but they do so cautiously.

2. The word "cautiously" in the passage is closest in meaning to


A, carefully

第 120 页 共
373 页
2021 托福阅读真题

B, appropriately
C, partially
D, effectively

3. According to paragraph 2, which of the following aspects of temperature has the greatest influence on the
timing of leaf expansion?
A, The sum of temperatures over a certain number of days
B, The coldest temperature reached during winter
C, The difference between the high and low temperatures on a particular day
D, The amount of temperature fluctuation in a given period

Paragraph 3: As the example of temperate forest-floor plants suggests, there are at least two main types of
factors that act on plant phenologies: non-biological factors limiting growth--such as the timing of killing
frosts (frosts cold enough to kill plants) or seasonal droughts--often determine the beginning or end of growth
episodes (or both), but biological factors--in particular, competition for light or water--are also quite
important for many species.

4. Which of the sentences below best expresses the essential information in the highlighted sentence in the
passage? Incorrect choices change the meaning in important ways or leave out essential information.
A, Plant phenologies are affected both by non-biological factors, which often determine the beginning or end of
growth episodes, and by biological factors such as competition for light or water.
B, The example of temperate forest-floor plants suggests that killing frosts or seasonal droughts, as well as
competition for light or water, can determine the beginning or end of growth episodes.
C, The example of temperate forest-floor plants suggests that there are at least two main types of factors that act on
plant phenologies as well as many other less important factors.
D, The relatively greater importance for many plant phenologies of non-biological factors as compared with
biological factors is indicated by the example of temperate forest-floor plants

Paragraph 4: This observation raises an interesting question: why do the canopy trees in temperate forests wait so
long before expanding their leaves? Why don't they use the early spring to add to their growth, as the forest -floor
plants do? There are a number of selective forces affecting the timing of leaf expansion. First, the canopy is elevated.
Temperatures in the treetops can be considerably colder than those at ground level. Thus, leaf expansion for trees
occurs later on the calendar. but it may not really be much later in terms of average temperature. Second, late frosts
are a common occurrence. Leaves on trees are much more vulnerable to frost damage than are those on forest-floor
plants, because the latter are partly sheltered by the trees and because the ground-level temperatures are higher. Third,
all enzymes (substances that facilitate chemical reactions) used for plants metabolism have different defined
temperature ranges over which they can operate, and they are most efficient at particular temperatures. Enzymes

第 121 页 共
373 页
2021 托福阅读真题

adapted for peak functioning at warm temperatures are unlikely to be efficient in the early spring; it is possible that
earlier leaf expansion might reduce the total year's growth of the trees, not increase it. Finally, many temperate-zone
trees are wind-pollinated, and pollination in most of these species occurs while the trees are leafless. The presence of
leaves earlier in the season would be likely to limit pollen transfer

5. The word "peak" in the passage is closest in meaning to


A, precise
B, normal
C, optimum
D, successful

6. According to paragraph 4. why does the common occurrence of late frosts affect the timing of leaf
expansion in canopy trees but not in forest-floor plants?
A, Because the leaves of forest-floor plants respond better to cold temperatures than the leaves of canopy trees do
B, Because the enzymes of forest-floor plants are not as efficient at warm temperatures as the enzymes of canopy
trees are
C, Because the leaves of forest-floor plants are far more protected from direct exposure to late frosts than the leaves
of canopy trees are
D, Because the leaves of forest-floor plants recover more easily from damage done by late frosts than do the leaves
of canopy trees

7. Paragraph 4 suggests that early leaf expansion could reduce the yearly growth of trees in temperate forests
by
A, decreasing the effectiveness of wind pollination
B, negatively affecting the metabolisms of these trees
C, forcing these trees to expend more energy on leaf production
D, narrowing the range of temperatures over which these trees' enzymes operate

Paragraph 5: The phenologies of temperate forest-floor plants are actually more complex than we have implied so
far. Most of the plants in northeastern North America have phenologies like those described here, but there are others
that are able to capture and use light at other times. Some species can use light flecks on the forest floor to grow
during the summer months. Still others use the additional light available in autumn, when some canopy species
have begun shedding their leaves

8. Why does the author mention that "Some species can use light flecks on the forest floor to grow during the
summer months"?
A, To make the point that phenologies of temperate forest plants differ significantly from phenologies of plants in

第 122 页 共
373 页
2021 托福阅读真题

other settings
B, To explain why growth and reproduction of forest-floor plants occur during different seasons
C, To support the idea that most plants in northeastern North America have phenologies like those already described
D, To give an example of a northeastern North American phenology that is less common than the ones previously
described
Paragraph 2: In most temperate species, it appears that temperature and photoperiod (day length) are the main
factors determining plant phenologies. █ It is important to realize that temperature plays its role in a particular way.
█ It is generally the sum of temperatures experienced over some period of time and not the temperatures on a
particular day that determines the timing of leaf expansion. In cold years, then, leaf expansion is delayed. █ Plants,
like other organisms, often use photoperiod as a reliable predictor of the average temperature. █ If they relied
exclusively on temperature, a warm spell in midwinter would typically cause many plants to expand their leaves. By
using temperature as a cue, plants can respond to an early spring by expanding their leaves early, but because they
also use photoperiod as a cue, this response is limited. These plants, in other words, respond to environmental
fluctuations, but they do so cautiously

9. Look at the four squares █ that indicate where the following sentence could be added to the passage.
Moisture, which is significant in a tropical habitat, is not as significant for temperate species.
Where would the sentence best fit? Click on a square █ to add the sentence to the passage.

10. Directions: An introductory sentence for a brief summary of the passage is provided below. Complete the
summary by selecting the THREE answer choices that express the most important ideas in the passage. Some
sentences do not belong in the summary because they express ideas that are not presented in the passage or are minor
ideas in the passage. This question is worth 2 points

Among temperate forest plant species, phenology-the timing of growth and reproductive activity-can vary
greatly.

A, Most forest-floor plants expand their leaves and flower before canopy trees begin leaf expansion, because
sunlight becomes largely unavailable to them after the trees are covered with leaves
B, Factors such as seasonal droughts and the arrival of killing frosts largely determine growth periods for certain
plant species; for others, only factors such as competition for water are of real importance.
C, Canopy trees, unlike forest-floor plants, depend on enzymes in the chemical reactions associated with
reproduction and leaf expansion but at some temperatures these enzymes reduce growth
D, In most temperate plant species, phenology is determined mainly by temperature combined with day length,
which allows adjustment in response to early or late springs.
E, Canopy trees wait to begin leaf expansion, since treetops are colder than forest floors and more exposed to frost;
trees' metabolisms need warmth to function well, and early leafing can limit pollination

第 123 页 共
373 页
2021 托福阅读真题

F, Because many forest-floor plants flower in summer rather than in spring, and some begin leaf expansion in
autumn, phenologies of forest-floor plants must be more complex than those of canopy trees.

The River Nile in Ancient Egypt

Paragraph 1: The special character of the Nile, which made it central to ancient Egyptian culture, was its annual
inundation (flooding). During June the river began to rise, and a quantity of green water appeared. The color is said
to have resulted from the brief period of reproduction of myriad minute organisms. During August the Nile rose
rapidly and assumed a muddy red color created by the rich red earth brought into its waters by its tributaries. The
Nile continued to rise until mid-September, then remained at that level for two or three weeks. In October it rose
again slightly, then began to fall gradually until May, when it reached its lowest level.

1. The word "minute" in the passage is closest in meaning to


A, tiny
B, ancient
C, distinct
D, simple

2. It can be inferred from paragraph 1 that the Nile reached its highest level in
A, May
B, June
C, October
D, September

3. According to paragraph 1, what was the reason for the change in the color of the Nile River that occurred in
August?
A, The rapid rise of the waters of the Nile
B, The presence of organisms in the Nile
C, The transporting of earth by tributaries of the Nile
D, The seasonal warming of the waters of the Nile

Paragraph 2: The Nile has created a convex floodplain. In convex floodplains, sediments (clays and silts) are
deposited by flood waters, making the land nearest the river have the highest elevation. The convex floodplain is
marked by natural levees that form elevated barriers immediately adjacent to the river. These levees rise a few
meters above the seasonally inundated lowlands. When the Nile floods, the water covers most of the low-lying land

第 124 页 共
373 页
2021 托福阅读真题

up to the edge of the desert. When the floods subside, the waters are trapped behind the levees and prevented from
returning to the river. The benefit of such topography is obvious: the water can be used where it stands or can be
channeled to other areas as dictated by agricultural needs.

Paragraph 3: Ancient records, those preserved both in texts and in the visible evidence on ancient devices for
measuring water levels called nilometers, indicate that a flood of six meters was perilously low and that one o f nine
meters was high enough to cause damage to crops and villages. A flood of seven to eight meters was ideal in that
low-lying areas and basins throughout the whole valley would be flooded up to the edge of the rising ground of the
desert, but towns, villages, and dikes that served as paths and water barriers remained above the water level.

4. Paragraph 3 implies which of the following about the Nile's floods in ancient Egypt?
A, Egyptians managed to protect their crops and villages from floods of nine meters or higher by creating dikes that
served as water barriers
B, Floods of six meters and floods reaching nine meters occurred almost as often as those of seven to eight meters.
C, Floods of nine meters could cover the dikes used as paths and water barriers
D, Even floods of seven to eight meters may have caused significant damage to the towns closest to the river basins

Paragraph 4: The ancient Egyptians fully understood the extent to which their lives and prosperity depended on the
unfailing regularity of the inundation. The occasional low flood and consequent shortage of food were enough to
cause much anxiety among the populace at the beginning of each flood season. Ancient Egyptians, therefore,
never became completely confident about the annual inundation and its gifts, even though it usually brought a
layer of fresh, rich silt and waters for irrigation that made agriculture in the Nile Valley relatively easy. The
generally predictable crops and resulting surplus freed a significant segment of the population from agricultural labor,
allowing for the development of nonfarming occupations, such as full-time craftspeople, bureaucrats. and priests.

5. The word "consequent" in the passage is closest in meaning to


A, expected
B, immediate
C, periodic
D, resulting

6. Which of the sentences below best expresses the essential information in the highlighted sentence in the passage?
Incorrect choices change the meaning in important ways or leave out essential information.
A, The ancient Egyptians had enough rich silt and water for agriculture because they were never overly confident.
B, Since annual flooding provided sufficient silt and water for agriculture, the ancient Egyptians never worried about
the river and its gifts
C, Although the inundations usually made agriculture in the Nile Valley comparatively easy, the Egyptians never

第 125 页 共
373 页
2021 托福阅读真题

took for granted the rich silt and irrigation the floods provided.
D, Although the gifts of the Nile did not make agriculture easy, the Egyptians worried the floods might not come.

7. The author mentions "full-time craftspeople, bureaucrats, and priests' in the passage in order to
A, compare how segments of the Egyptian population contributed to the development of an extremely successful
society
B, specify some of the nonfarming occupations that developed in Egyptian society because of agricultural surpluses
C, demonstrate why the Egyptians reached a higher level of prosperity than other societies with similarly regular
river floods
D, help explain why only a small segment of the Egyptian population pursued occupations other than farming

Paragraph 5: The importance of the Nile to Egyptian civilization is reflected in the role that it played in religion
and the myths that revolved around the river. The Nile was known in antiquity by the Egyptian name Iteru, meaning
"great river." The personification of the inundation was a god named Hapy, who was associated with fertility and
regeneration. The ancient Egyptians had various conceptions of the origin of the inundation. Some texts relate that it
began in a cavern at Philae, while others credit the site Gebel Silsila (about 100 kilometers to the north) as the source.
It was believed that veneration of the gods associated with these sites in the Aswan area could ensure a sufficient
inundation. The Famine Stela, a text carved on rocks at Sehel near Philae, records a famine that was averted by
donations of land and goods to the Temple of Khnum at Aswan. This text was formerly thought to date from the
reign of Djoser (2687-2667 B.c). but in reality it dates to the Ptolemaic period some 2.500 years later

8. According to paragraph 5, views have changed on which of the following aspects of the Famine Stela?
A, When the text was written
B, Whether the text is authentic
C, Why the text was written
D, Whether the text was composed by Egyptians

Paragraph 2: The Nile has created a convex floodplain. In convex floodplains, sediments (clays and silts) are
deposited by flood waters, making the land nearest the river have the highest elevation. The convex floodplain is
marked by natural levees that form elevated barriers immediately adjacent to the river. These levees rise a few
meters above the seasonally inundated lowlands. When the Nile floods, the water covers most of the low-lying land
up to the edge of the desert. █ When the floods subside, the waters are trapped behind the levees and prevented from
returning to the river. █ The benefit of such topography is obvious: the water can be used where it stands or can be
channeled to other areas as dictated by agricultural needs.

Paragraph 3: █ Ancient records, those preserved both in texts and in the visible evidence on ancient devices for
measuring water levels called nilometers, indicate that a flood of six meters was perilously low and that one of nine

第 126 页 共
373 页
2021 托福阅读真题

meters was high enough to cause damage to crops and villages. █ A flood of seven to eight meters was ideal in that
low-lying areas and basins throughout the whole valley would be flooded up to the edge of the rising ground of the
desert, but towns, villages, and dikes that served as paths and water barriers remained above the water level.

9. Look at the four squares █ that indicate where the following sentence could be added to the passage.
Nile floods, however, could be a mixed blessing.
Where would the sentence best fit? Click on a square █ to add the sentence to the passage.

10. Directions: An introductory sentence for a brief summary of the passage is provided below. Complete the
summary by selecting the THREE answer choices that express the most important ideas in the passage. Some
sentences do not belong in the summary because they express ideas that are not presented in the passage or are minor
ideas in the passage. This question is worth 2 points.

The Nile was central to the life of ancient Egyptians.

A, Agriculture was successful in the Nile Valley because of the seasonal patterns of flooding and the geographical
features of the floodplain.
B, Nilometers and ancient records indicate that water levels reached by the annual inundation were very predictable,
usually varying by less than a meter from year to year
C, The annual inundation was so important to Egyptian life that the location of temples was selected based on myths
about where the flood originated.
D, Natural levees protected crops from floods by preventing floodwaters from reaching the lowlands.
E, The Nile floods usually resulted in agricultural surpluses, but very low or high floods could cause food shortages
or damage to villages.
F, Practices of honoring gods associated with flood sites and of making offerings to ensure adequate flooding
demonstrate the great importance of the Nile to Egyptians.

第 127 页 共
373 页
2021 托福阅读真题

Martian Volcanoes

Paragraph 1: Earth is not the only planet to have volcanoes; in fact Mars-though all of its volcanoes have long been
inactive-has the two largest ones in the solar system: Olympus Mons and Alba Patera. Mars has spectacular lava
fields, large craters formed by a volcanic explosion, and shields (broad, low volcanoes shaped like shallow domes).

Paragraph 2: The most striking characteristic of Martian volcanoes is in fact their great size and longevity. The
giant shields of Tharsis and to a lesser extent the volcanoes of Elysium and Hellas pack in many times more lava
than the largest volcanoes on Earth, and yet Mars is a smaller planet. The reason lies in the planets' internal
makeup.Because it is larger and hotter, Earth has a nearly molten (hot liquid) mantle that is overlain by a very thin
rock crust-less than10 kilometers thick under the ocean basins. The hot mantle churns over in great loops, and rips
the overlying rigid crust into great slabs, called tectonic plates, that shuffle around the globe. In those places where
deep-seated "hot spots" send plumes of molten material to the surface, the magma (the trapped lava) starts piling up
on the crust to build a volcano, but because the crust is drifting, the volcano eventually breaks away from its source
and goes extinct. In its place, a new volcano starts to grow. so that a hot spot under a mobile plate creates a string of
medium-sized volcanoes rather than one giant shield.

1. The word "longevity" in the passage is closest in meaning to


A length of life
B level of activity
C composition
D distribution

2. In paragraph 2, why does the author provide the information that Mars is smaller than Earth?
A To clarify a common misconception about Mars
B To help explain the size of Earth's volcanoes
C To emphasize the surprising size of Martian volcanoes
D To indicate essential differences between the two planets

Paragraph 3: On the smaller, faster-cooling Mars, the crust has solidified and thickened to the extent that it remains
stationary with respect to the underlying hot spots. When a plume of magma erupts at the surface, it will build a
single volcano in one place for as long as the supply lasts-typically hundreds of millions of years. Not surprisingly,

第 128 页 共
373 页
2021 托福阅读真题

then, the volume of one Martian shield like Olympus Mons or Alba Patera is comparable to that of an entire
chain of volcanoes on Earth, such as the Hawaiian Emperor chain that stretches across the Pacific and spans
nearly 100 million years of hot-spot activity.

3. Which of the sentences below best expresses the essential information in the highlighted sentence in the passage?
Incorrect choices change the meaning in important ways or leave out essential information.
A A comparison of the Hawaiian Emperor chain. Olympus Mons.and Alba Patera shows they are all about 100
million years old.
B The volume, then. of one Martian shield is comparable to the entire number of volcanoes on Earth, spanning
nearly100 million years
C Logically. then, the volume of a single Martian shield is about the same as a chain of volcanoes on Earth.
D A chain of volcanoes like Earth's Hawaiian Emperor chain would take nearly 100 million years to form on Mars.

4. According to paragraph 3, why do volcanoes grow larger on Mars than they do on Earth?
A The underlying hot spots erupt from much greater depths on Mars than on Earth.
B The magma on Mars is not as thick as on Earth, allowing for faster expansion.
C The interior of Mars has a larger supply of magma than does the interior of Earth.
D The thicker crust on Mars allows volcanoes to grow in the same place for much longer periods.

Paragraph 4: Another characteristic of Martian volcanoes is the large size of their craters and the long run-out
distances of their lava flows. This is principally due to the lower gravity on Mars: 38 percent, or about one third, of
Earth's gravity. Gravity controls to some extent how solid rock behaves under stress. When the crust stretches and
breaks in a low gravity field, as it does on Mars, the fissures that open up are wider than on Earth and can funnel
larger amounts of magma toward the surface. Magma chambers grow correspondingly larger and when they empty
out and collapse, they yield larger craters.

5. According to paragraph 4, the low gravitational field of Mars causes


A fissures in the rocks to be quite large
B magma to flow more slowly to the surface
C the planet's crust to break more often
D magma chambers to last longer

Paragraph 5: Eruption rates also tend to be higher on Mars, since greater quantities of lava can flow out of the
wider fissures. These larger volumes guarantee a better retention of heat. Because it stays hot and molten for a
longer period of time, lava on Mars travels greater distances than it does on Earth before cooling and slowing to a
halt.

第 129 页 共
373 页
2021 托福阅读真题

6. The word"retention" in the passage is closest in meaning to


A intensity
B dispersal
C production
D conservation
7. According to paragraph 5, why does molten lava stay hotter on Mars's surface than on Earth's?
A It comes from hotter sources.
B It has been under the crust longer.
C It travels shorter distances.
D It flows in larger amounts

Paragraph 6: Explosive eruptions are also affected by the unique set of conditions that exist on Mars. The
atmospheric pressure is so low that any gas bubbles trapped in the magma will undergo tremendous expansion upon
reaching the surface. As a result, it will take lesser amounts of dissolved gases in Martian (versus terrestrial) magma
for the mixture to foam and shoot out explosively from the vent. On Earth, the confining pressure of the atmosphere
requires that magma contains close to 1 percent (by weight) of light gases for it to spray upward as a lava fountain.
About 3 to 4 percent is needed for the bubbles to grow large enough to blow the magma to shreds and create an ash
cloud. On Mars, thresholds for such disruptive behavior are much lower: respectively 0.03 and 0.2 percent gases by
weight. Twenty to thirty times less gas is needed on Mars to achieve similar results.

8. Assuming that terrestrial and Martian magma have similar levels of gas, which of the following can be inferred
from paragraph 6 about Martian volcanic eruptions?
A They are less affected by atmospheric pressure levels than by other atmospheric conditions.
B They produce ash clouds more frequently than terrestrial eruptions do.
C They include lava sprays 3 percent to 4 percent of the time
D They produce lava fountains less often than they produce ash clouds.

Paragraph 2: The most striking characteristic of Martian volcanoes is in fact their great size and longevity. The
giant shields of Tharsis and to a lesser extent the volcanoes of Elysium and Hellas pack in many times more lava
than the largest volcanoes on Earth, and yet Mars is a smaller planet. The reason lies in the planets' internal
makeup.Because it is larger and hotter, Earth has a nearly molten (hot liquid) mantle that is overlain by a very thin
rock crust-less than10 kilometers thick under the ocean basins. █The hot mantle churns over in great loops, and rips
the overlying rigid crust into great slabs, called tectonic plates, that shuffle around the globe.█ In those places where
deep-seated "hot spots" send plumes of molten material to the surface, the magma (the trapped lava) starts piling up
on the crust to build a volcano, but because the crust is drifting.the volcano eventually breaks away from its source
and goes extinct.█ In its place, a new volcano starts to grow. so that a hot spot under a mobile plate creates a string
of medium-sized volcanoes rather than one giant shield.█

第 130 页 共
373 页
2021 托福阅读真题

9. Look at the four squares █ that indicate where the following sentence could be added to the passage.
These series of volcanoes will continue to be created until the source of lava dries up.
Where would the sentence best fit? Click on a square █ to add the sentence to the passage.

10. Directions: An introductory sentence for a brief summary of the passage is provided below. Complete the
summary by selecting the 3 answer choices that express the most important ideas in the passage. Some sentences do
not belong in the summary because they express ideas that are not presented in the passage or are minor ideas in the
passage. (This question is worth 2 points.)

The unique characteristics of Mars have created the largest volcanoes in the solar system.

A Although it is smaller than Earth, Mars is hotter, and its mantle, the layer of molten rock beneath the crust, is
larger.
B Volcanoes on Earth are much smaller than those on Mars because Earth's moving crust causes the volcanoes to go
extinct quickly.
C Mars's solid crust and low gravity and atmospheric pressure create large volcanoes that release large amounts of
lava for a long time.
D Mars's volcanoes, like those on Earth, begin when hot spots deep in the molten mantle release streams of magma
up through the surface crust
E The many extremely large volcanoes on Mars have caused the planet's crust to thicken and remain stationary.
F. The release of gases from magma on Mars causes the magma to cool down much more quickly than it does on
Earth.

第 131 页 共
373 页
2021 托福阅读真题

A Mutualistic Fungus of Tall Fescue Grass

Paragraph 1: Mutualism is an interaction between two species in which both benefit. A classic example of this
relationship is between tall fescue grass and a fungus named Epichloe coenophiala . That fungus is completely
internal and grows intercellularly (between the cells) in the above-ground portion of the grass. The grass supplies all
the nutritional needs of the fungus. Infected plants exhibit no external symptoms of the fungus and no disease
symptoms. In contrast, pathogenic (disease-causing) fungi result in symptoms that include wilt, leaf spots, deformed
plant structures, or death of plant tissue. Pathogenic fungi often invade plant host cells and absorb the cells' contents
(thereby eventually killing the cells) or produce chemicals that kill them outright.

1. According to paragraph 1, how does the Epichloe coenophiala fungus benefit from its relationship with tall
fescue grass?
A, It is shaded from potentially damaging sunlight.
B, It grows above the ground, out of reach of harmful chemicals in the soil.
C, It obtains sufficient nutrition
D, It is protected from disease-causing fungi.

Paragraph 2: Tall fescue is extensively used as both a pasture grass and a turf or lawn grass. It is a hardy.
vigorously growing grass but has been associated with various health problems in cattle and horses Beef cattle often
show reduced weight gains when grazing on tall fescue pastures, and dairy cattle experience lower milk yields.
Female horses may be more likely to lose their offspring during pregnancy and may not produce milk following a
live birth. Reports of such adverse effects resulted in microscopic investigation of leaf and stem tissue that revealed
the presence of the Epichloe coenophiala fungus. Surveys of infected pastures found a positive correlation between
the percentage of infected plants and the severity of animal symptoms. The fungus in the grass was found to produce
various alkaloids (toxic nitrogen-containing organic molecules). One type of alkaloid (ergot alkaloids) contributes to
the livestock's symptoms and reduced grazing, whereas another type (loline alkaloids) makes the grass more resistant
to insects. Interestingly. the insect-deterring alkaloids are not produced when the fungus is grown in the laboratory
apart from the plant. Because these chemicals defend the grass as well as the fungus from consumption by both
livestock and insects, the mutualistic relationship has been termed defensive mutualism.

2. In paragraph 2. which of the following is NOT mentioned as one of the adverse effects on livestock that led
to examination of plant tissue?

第 132 页 共
373 页
2021 托福阅读真题

A, Increased frequency of infectious disease


B, Lower body weights
C, Unsuccessful pregnancies
D, Decreased milk production

3. According to paragraph 2, all of the following statements about loline alkaloids are true EXCEPT:
A, They help tall fescue grass avoid being eaten by insects.
B, They are more harmful to grazing animals than are ergot alkaloids
C, They are beneficial for fescue grass.
D, They are not produced when Epichloe coenophiala is grown in the laboratory separately from the plant

Paragraph 3: Because the Epichloe coenophiala fungus is not external on the plant, the usual mechanism of spread
by fungi does not take place. Typical fungi form spores (reproductive units consisting of one or more cells that are
similar to the seeds of plants). Fungal spores spread through the air to susceptible plants and usually require an
extended period of moist conditions for their survival and germination (the beginning of growth of a spore or seed),
and for the subsequent infection of the host plant. The tall fescue fungus, in contrast, penetrates the flower head and
infects the plant embryos in the developing seeds. Therefore, seeds collected from infected plants result in infected
seedlings following germination. As the plant grows, the fungus grows along with it by division and stretching of the
hyphae (threadlike structures that make up the body of a fungus) at the base of the grass, where plant cell division
also occurs. Thus, growth of the fungus is concurrent with growth of the plant, which ensures that new stems and
leaves of the grass are infected with the fungus. Fungus-free tall fescue plants can result it infected seed is stored for
several months before planting. The fungus usually dies out during most long-term storage conditions.

4. The word "penetrates" in the passage is closest in meaning to


A, falls on
B, enters
C, rubs against
D, surrounds

5. Paragraph 3 suggests that it may be possible to prevent Epichloe coenophiala infection of tall fescue grass
by
A, storing fescue seeds for a long time before planting
B, growing fescue plants under particularly dry conditions
C, speeding up the growth rate of fescue plants
D, removing infected stems and leaves from fescue plants

Paragraph 4: The alkaloids produced by the fungus also affect microorganisms in the immediate environment of

第 133 页 共
373 页
2021 托福阅读真题

the plant. Nondisease-causing bacteria, recovered from the surfaces of fungus-infected tall fescue plants, can use
loline alkaloids, released on the leaves, as a source of nutrients. The population size of these bacteria is about
eight-fold greater on infected plants than on fungus-free tall fescue plants or on plants colonized by related
fungi incapable of producing loline. Thus, many components of the fungus-infected tall fescue community are
impacted by the fungus: the grass itself, animals. insects, and microbes.

6. The word "immediate" in the passage is closest in meaning to


A, usual
B, nearest
C, general
D, changing

7. Why does the author provide the information that the population size of certain bacteria " is about eight
fold greater on infected plants than n fungus-free tall fescue plants or on plants colonized by related fungi
incapable of producing loline"?
A, To demonstrate that loline production occurs in fescue grass and not in other plants
B, To provide evidence that microorganisms in fescue's environment use the alkaloids produced by Epichloe
coenophiala
C, To contrast bacteria that live on fescue grass with those that live on other plants
D, To explain why Epichloe coenophiala is more harmful than fungi found on other plants

Paragraph 5: Although the fungus of tall fescue is detrimental to livestock, it is considered beneficial when grass is
grown for other purposes Insect pests are numerous in turf (short, thick grass and the soil it grows in), and tall
fescue's increased resistance to them is advantageous when grass is grown for house lawns or sports fields It may be
possible to develop pasture varieties of tall fescue that are infected with this fungus for the purpose of insect
resistance, but with the deletion of the fungal genes for the toxins that harm livestock

8. According to paragraph 5. it is possible to reduce the number of insects on house lawns and sports fields by
A, removing livestock from fields of fescue grass
B, changing fescue grass's genes
C, growing fescue grass in a special kind of soil
D, using fescue grass infected with a specific fungus

Paragraph 2: Tall fescue is extensively used as both a pasture grass and a turf or lawn grass. It is a hardy.
vigorously growing grass but has been associated with various health problems in cattle and horses Beef cattle often
show reduced weight gains when grazing on tall fescue pastures, and dairy cattle experience lower milk yields. █
Female horses may be more likely to lose their offspring during pregnancy and may not produce milk following a

第 134 页 共
373 页
2021 托福阅读真题

live birth. █ Reports of such adverse effects resulted in microscopic investigation of leaf and stem tissue that
revealed the presence of the Epichloe coenophiala fungus. █ Surveys of infected pastures found a positive
correlation between the percentage of infected plants and the severity of animal symptoms. █ The fungus in the
grass was found to produce various alkaloids (toxic nitrogen-containing organic molecules). One type of alkaloid
(ergot alkaloids) contributes to the livestock's symptoms and reduced grazing, whereas another type (loline alkaloids)
makes the grass more resistant to insects. Interestingly. the insect-deterring alkaloids are not produced when the
fungus is grown in the laboratory apart from the plant. Because these chemicals defend the grass as well as the
fungus from consumption by both livestock and insects, the mutualistic relationship has been termed defensive
mutualism.

9. Look at the four squares █ that indicate where the following sentence could be added to the passage.
The discovery that Epichloe coenophiala produces poisonous substances for self-protection explained these
findings
Where would the sentence best fit? Click on a square █ to add the sentence to the passage

10. Directions: An introductory sentence for a brief summary of the passage is provided below. Complete the
summary by selecting the THREE answer choices that express the most important ideas in the passage. Some
sentences do not belong in the summary because they express ideas that are not presented in the passage or are minor
ideas in the passage. This question is worth 2 points

The fungus Epichloe coenophiala interacts in a mutualistic relationship with tall fescue grass.

A, Epichloe coenophiala protects infected tall fescue grass from animals and insects.
B, Alkaloids produced by the fungus of tall fescue grass can end up affecting many organisms beyond the grass
itself.
C, Bacteria that thrive on fungus-infected tall fescue grass contribute to the negative effects that infected grass can
have on its environment
D, Mutualistic relationships between grass and several kinds of fungi are of the type referred to as defensive
mutualism.
E, The fungal spores that infect tall fescue grass require extended periods of moist conditions in order to survive and
germinate
F, The tall fescue fungus, though harmful to some animals, is well suited for other uses, and new varieties of the
grass might be developed that are safe for all uses.

第 135 页 共
373 页
2021 托福阅读真题

The Nile River Valley

Paragraph 1: Though the verdant green course of the Nile Valley appears to be the very antithesis of the
bleached Saharan sands through which it flows, these two contrasting features have together constituted a
primary force in the development of agriculture and settled human society in Africa. The Sahara acted as a
pump, drawing people from surrounding regions into its watered environments during the good times and driving
them out again as conditions deteriorated (though not necessarily returning them to their point of origin). The
Nile. for its part, was a refuge for people retreating from the desert and then a reservoir from which the desert region
was repopulated as conditions improved.

1. Which of the sentences below best expresses the essential information in the highlighted sentence in the passage?
Incorrect choices change the meaning in important ways or leave out essential information.
A The development of settled societies and agriculture in Africa was heavily influenced by the combination of two
physically contrasting areas, the green Nile Valley and the surrounding Sahara desert.
B The green Nile Valley contributed more significantly to the development of agriculture and settled societies than
the bleached Sahara desert did.
C Agriculture and settled societies were able to develop in the valley created by the Nile River, even though it cuts
through the bleached sands of the Sahara desert.
D Although the Nile Valley appears completely different from the Sahara, the similarities that existed between these
two regions led to the settlement and agricultural development of Africa

Paragraph 2: The Nile is commonly believed to have been a conduit along which the principles of food-crop
cultivation moved from Egypt into more southern regions of Africa along with the wheat, barley, peas and lentils
initially domesticated some 9,000 years ago in Southwest Asia. It is true that the ancient Egyptian civilizations
founded on the Nile over 5,000 years ago were sustained by the exceptional productivity of those crops, but they
were a comparatively recent introduction, arriving long after the cultivation of indigenous African plants had begun
farther south.

2. Paragraph 2 supports the idea that the cultivation of food crops in Africa began
A in the Nile Valley

第 136 页 共
373 页
2021 托福阅读真题

B long before food crops were domesticated in Southwest Asia


C with the arrival of domesticated wheat, barley, peas, and lentils from Southwest Asia
D with the domestication of indigenous African plants

Paragraph 3: Contrary to expectations, the earliest evidence of a shift toward a critical dependence on food
production as opposed to food gathering comes not from the floodplain of the Nile but from sites in what is now the
empty and waterless Sahara. The development was complex, involving the domestication of plants and livestock,
technological innovation, the establishment of villages, and an increasing level of social interdependence.

3. According to paragraph 3, what is true about the shift from food gathering to crop cultivation?
A It probably occurred during a period in which the Sahara was becoming drier.
B It probably occurred much earlier in the Nile Valley than people once believed.
C It probably occurred after the establishment of villages along the Nile
D It probably occurred first in the Sahara and only later along the Nile

Paragraph 4: The Sahara desert had been widely inhabited until the last glacial maximum, when conditions of
increasing aridity drove people (and practically all animals) out of its previously productive wooded grassland and
savanna environments. Essentially nomadic-though exhibiting a tendency to settle at lakesides and other sources of
food and water-the groups moving to the north and south continued to fallow their established hunting and gathering
way of life. The groups that moved east into the Nile Valley, however, adopted a distinctly sedentary lifestyle. Indeed,
they had no choice The narrow strips of green floodplain flanking the Nile were bounded by waterless desert, and
movement along the riverside plains was restricted by the presence of competing groups.

4. According to paragraph 4, what was true about the Sahara during the period before the last glacial maximum?
A It was almost completely covered with trees.
B It contained few sources of water.
C It contained no large animals
D It was home to many hunter-gatherers.

5. Why does the author inform readers that the floodplains of the Nile were "bounded by waterless desert" and that
movement along these riverside plains "was restricted"?
A To argue that the Nile Valley was as affected by increased aridity as the Sahara
B To help explain why some groups became sedentary
C To explain why nomadic groups tended to camp at lakesides
D To explain why some groups moved north and south rather than east

Paragraph 5: The Nile along which people congregated 18,000 years ago was a quite different river from that

第 137 页 共
373 页
2021 托福阅读真题

which flows through Egypt today. It was much smaller and flowed more slowly through a tangle of braided channels
across a wide elevated floodplain rather than along a single massive stream. The river. carried a heavy sediment load,
and the silts deposited along its length steadily raised the level of the river and its floodplain.

6. The word "congregated" in the passage is closest in meaning to


A traveled
B survived
C gathered
D farmed

7. According to paragraph 5, each of the following describes the Nile of 18,000 years ago EXCEPT:
A It was not as large as today's Nile.
B It flowed more slowly than today's Nile.
C It flowed through a single stream
D It contained a great deal of silt and other sediments

Paragraph 6: In effect, the Nile Valley at that time was an elongated oasis: a sharply defined area of inhabitable
territory beyond the boundaries of which human survival.was impossible. The oasis extended all the way from the
Sudan to what is now Cairo-a distance of over 800 kilometers-but it was nowhere more than a few kilometers wide.
Its food resources were varied and nutritious. Quantities and availability, however, were always subject to seasonal
variation and the vagaries of the Nile's annual flood. During the critical two to four weeks when the flood was at its
height each year, people and animals were driven out of the Nile Valley and crowded into the narrow band o f
inhabitable land that lay between the floodwaters and the arid desert beyond.

Paragraph 7: People hunted large mammals for meat and hides, but environmental and territorial constraints meant
that only small numbers of a limited range of species were available. Indeed, large mammals constitute a very small
proportion of the faunal remains found at archaeological sites, and only three species are represented: the hartebeest,
the dorcas gazelle, and a type of wild cattle called the aurochs. Birds-particularly coots and migratory geese and
ducks-were caught regularly. Fish, however, were by far the most important source of protein-primarily catfish. For
carbohydrates, the Nile Valley offered its inhabitants a considerable variety and seasonal abundance. Twenty-five
different seeds, fruits, and soft vegetable tissues have been distinguished among archaeological remains.

8. According to paragraph 7, what did the remains found at archaeological sites reveal about the food eaten by
inhabitants of the Nile Valley?
A Small mammals were one of the most important food sources.
B Large mammals were not a major source of food.
C Birds were eaten more regularly than fish.

第 138 页 共
373 页
2021 托福阅读真题

D Carbohydrates were eaten much less often than protein.

Paragraph 4: The Sahara desert had been widely inhabited until the last glacial maximum, when conditions of
increasing aridity drove people (and practically all animals) out of its previously productive wooded grassland and
savanna environments.█ Essentially nomadic-though exhibiting a tendency to settle at lakesides and other sources of
food and water-the groups moving to the north and south continued to fallow their established hunting and gathering
way of life. █The groups that moved east into the Nile Valley, however, adopted a distinctly sedentary lifestyle.█
Indeed, they had no choice.█ The narrow strips of green floodplain flanking the Nile were bounded by waterless
desert, and movement along the riverside plains was restricted by the presence of competing groups.

9. Look at the four squares █ that indicate where the following sentence could be added to the passage.
People moved away from the desert in various directions.
Where would the sentence best fit? Click on a square █ to add the sentence to the passage.

10. Directions: An introductory sentence for a brief summary of the passage is provided below. Complete the
summary by selecting the 3 answer choices that express the most important ideas in the passage. Some sentences do
not belong in the summary because they express ideas that are not presented in the passage or are minor ideas in the
passage.

Together the Nile River and Sahara desert helped shape the development of agriculture and settled human
society in Africa.

A The regular flooding of the Nile forced people into the deserts, where they survived by hunting and gathering until
they could return to their sedentary lifestyle in the Nile Valley.
B The Nile1Valley could not support agriculture until the last glacial maximum, at which time the slow, narrow Nile
River became the wide, powerful river that we know today.
C Although the Nile Valley supported few large animals and space was limited, the wildlife and seasonal foods
available were sufficient to support populations established along the Nile.
D Long before the founding of ancient Nile civilizations that depended on crops of Southwest Asian origin,
agriculture based on indigenous plants was practiced in the Sahara.
E Increasing aridity drove people out of the Sahara, and the more crowded conditions that this created in the Nile
Valley forced people moving there to become sedentary.
F Recent archaeological evidence suggests that hunting and fishing played a very minor role in the Nile Valley in
comparison with food cultivation.

第 139 页 共
373 页
2021 托福阅读真题

Glacier Flow and Surging

Paragraph 1: Glaciers are ice masses that form from snow accumulated and compressed over years. They appear as
permanent features of the landscape, but they are always in motion. Ice is a solid, rather than a liquid, and ice flow is
through the deformation( change in arrangement)of ice crystals, the glaciers tiny building blocks. Glacier motion
resulting from this slow internal deformation is typically a few meters per year. The ice flows downs-slope under the
influence of gravity. However, ice flow is controlled mainly by the slope of the ice surface rather than the bed(the
ground surface below), and for this reason ice near the base of a glacier can flow uphill over obstructing bedrock
ridges.

1. The word “obstructing” in the passage is closest in meaning to


A, rising
B, changing
C, developing
D, blocking

2. According to paragraph 1, which TWO of the following are true about glacier motion? To receive credit, you must
select TWO answers
A, It is made possible by the deformation of tiny ice crystals that make up the glacier
B, It occurs after the glacier reaches a specific height in meters
C, It is controlled mainly by the bed of the glacier
D, It is usually down-slope but can also be uphill

Paragraph 2: Observations of the speed of glaciers, made both by field surveys and from satellite measurements
show that ice flows at rates ranging from less than a meter to several kilometers per year. A number of large
fast-flowing glaciers drain the interior of the Greenland Ice Sheet, and the Jakobshavns Isbrae glacier in West
Greenland has a flow rate of 7 kilometers per year, the fastest observed anywhere in the world. Speeds such as this
are not produced by crystal deformation alone. It is the presence of liquid water, and sometimes water-saturated
sediments (settled matter at the glacier bed) that provides the necessary lubrication for fast flow.

第 140 页 共
373 页
2021 托福阅读真题

3. Which of the following can be inferred from paragraph 2 about the Jakobshavns Isbrae glacier in West
Greenland?
A, It sits on a layer of liquid water or on very wet sediments
B, It flows at a rate of less than a meter to several kilometers a year
C, It is the first to have its speed measured from a satellite
D, Its speed is mainly the result of crystal deformation
Paragraph 3: But, how can the temperature beneath hundreds and sometimes thousands, of meters of ice reach the
melting point to allow water flow? The coldest place on a glacier or Ice sheet is close to the surface, because the ice
is in contact With the cold atmosphere. With increasing depth through the ice, the temperature actually increases
This is because heat from Earth's interior called geothermal heat, and heat generated by ice crystal deformation, ac o
warm the ice. Friction(rubbing), caused by the glacier sliding over the bed, also plays a role in warming the ice at
depth. The further from the cold surface, the higher the temperature, a phenomenon observed in drill holes several
kilometers deep through the Greenland Ice Sheet. Where the melting point is reached at the glacier bed, water may
be located in several places. It can be in channels cut into the bedrock of the overlying ice, in a thin film, or held
within soft sediments immediately beneath the ice. In each case, friction at the glacier bed is reduced and ice can
then move at speeds much faster than through ice deformation alone.

4. According to paragraph 3, all of the following contribute to the melting of the ice at the bottom of glaciers
EXCEPT
A, heat drawn from the atmosphere through the surface of the glacier
B, heat produced within Earth
C, heat produced when ice crystals within the glacier deform
D, heat generated by the movement of the glacier over the bed

Paragraph 4: In addition to being important components of the landscape and modifiers of the local climate,
glaciers receive attention because of their sometimes unpredictable behavior. Many Arctic glaciers undergo
catastrophic advances known as surges, sometimes o several kilometers in a few months, which are unrelated to
any climatic changes. Surge-type glaciers remain inactive for many decades, punctuated by much shorter periods of
fast flow. The shift from stagnation to rapid flow and advance of the end of the glaciers thought to be associated with
the buildup of water to lubricate the glacier bed. Or with a change in the state of subglacial sediment from frozen to
wet, surging glaciers may dam major rivers or even fiords temporarily. Many other glaciers trap water in the form of
internal reservoirs or ice-dammed lakes the sudden release of which causes catastrophic flooding.

5. Why does the author point out that glacier surges are “unrelated to any climatic changes”?
A, To explain why glacier surges can become catastrophic
B, To contrast the effects on fjords of slow-moving glaciers with those of surging glaciers
C, To argue against the claim that glaciers are modifiers of the local climate

第 141 页 共
373 页
2021 托福阅读真题

D, To support the claim that glaciers sometimes behave unpredictably

6. According to paragraph 4, which of the following is true of surge-type glaciers?


A, They occur as a result of changes in the water content of the glacier bed.
B, They affect the climate more rapidly than do other types of glaciers
C, They are more active near major rivers than near fjords
D, They cause subglacial sediment to change from frozen to wet in a short period

7. According to paragraph 4, glaciers may cause damage by doing which of the following?
A, Changing the course of major rivers
B, Creating temporary fjords
C, Trapping water inside themselves and then suddenly releasing it
D, Filling lakes with a great amount of ice

Paragraph 5: The surface of a glacier bears witness to the forces generated during ice flow, the way in which debris
is transported, and the role played by meltwater. When ice is stretched, as it goes round a bend, over a rock
bump or step in the bed, or on entering the sea, it fractures in brittle fashion, producing large cracks called
crevasses. Crevasses are a major hazard to traveling on glaciers, especially in the accumulation area, where they
may lurk largely hidden beneath snow. Crevasses are commonly several meters wide and a few tens of meters deep.
although few direct measurements have been made. They usually form distinct geometrical sets and where several
intersect, travel is usually impossible. The most intense crevassing occurs when glaciers surge At such times, the
whole glacier surface may break up.

8. Which of the sentences below best expresses the essential information in the highlighted sentence in the passage?
Incorrect choices change the meaning in important ways or leave out essential information
A, Although ice is often brittle, it can be stretched to go around and over bends, rock bumps, or steps
B, As ice travels it may become stretched, causing the formation of large cracks.
C, Ice that forms in a brittle fashion will crack easily into crevasses
D, Ice must stretch as it goes around bends and rock bumps before entering the sea

Paragraph 4: In addition to being important components of the landscape and modifiers of the local climate,
glaciers receive attention because of their sometimes unpredictable behavior. █ Many Arctic glaciers undergo
catastrophic advances known as surges, sometimes o several kilometers in a few months, which are unrelated to any
climatic changes. Surge-type glaciers remain inactive for many decades, punctuated by much shorter periods of fast
flow. █ The shift from stagnation to rapid flow and advance of the end of the glacier s thought to be associated with
the buildup of water to lubricate the glacier bed or With a change In the state of subglacial sediment from frozen to

第 142 页 共
373 页
2021 托福阅读真题

wet. █ Surging glaciers may dam major rivers or even fiords temporarily. █ Many other glaciers trap water in the
form of internal reservoirs or ice-dammed lakes the sudden release of which causes catastrophic flooding.

9. Look at the four squares █ that indicate where the following sentence could be added to the passage.
But what causes the sudden movement of these glaciers?
Where would the sentence best fit? Click on a square (█)to add the sentence to the passage.

10. Directions : An introductory sentence for a brief summary of the passage is provided below. Complete the
summary by selecting the 3 answer choices that express the most important ideas in the passage. Some sentences do
not belong in the summary because they express ideas that are not presented in the passage or are minor ideas in the
passage. This question is worth 2 points.

Glaciers are large masses of ice that move slowly across the landscape.

A. Until field surveys and satellite observations revealed that they are always in motion, glaciers were thought to be
permanent features of the landscape.
B. Surges are sudden. unexpected and very rapid movements, sometimes of several kilometers in a few months
which occur after long periods of inactivity and may cause flooding and other problems
C. The difference in temperature between the surface and depth of a glacier has been used to explain the rate at
which ice flow occurs
D. Ice flow, resulting from the internal deformation of ice crystals typically results in movement of a few meters per
year, but glaciers can move much faster over liquid water or wet sediments.
E. Crevasses are deep cracks in the glaciers surface that appear when the glacier surges or moves over uneven
ground, and which can make it difficult or impossible to cross the glacier.
F. Glaciers undergo changes both on the surface and in the interior as they move, and these changes affect the
landscape in different ways.

第 143 页 共
373 页
2021 托福阅读真题

Defenses of the Eleodes Beetle

Paragraph 1: The color black, you would imagine, is impractical in a desert setting, since it offers little protection
against the heat. Yet as naturalists are quick to note, black insects, mainly beetles, abound in desert settings. In the
deserts of the southwestern United States there exist a multitude of jet-black beetles, ranging from mid-size to large
and belonging primarily to the family Tenebrionidae. These beetles are inactive in midday when the sun is most
intense and are active primarily at dusk and dawn, in the twilight, when they are on the march to or from their
feeding sites. True color is poorly perceived in dim light, and there is no better way to be noticeable on sand at dawn
or dusk than by being black. Add to this fact that dawn and dusk are the times when vertebrates are looking for prey,
and you are faced with the apparent paradox that black desert beetles are most conspicuous when they are most at
risk.

1. According to paragraph 1, desert beetles are most active at that time of day when
A the sun is most intense
B they are most visible
C the fewest vertebrates are out looking for prey
D it is completely dark

Paragraph 2: The truth is, however, that desert tenebrionids are for the most part chemically protected and that it is
to their advantage to advertise themselves. Being black in a desert setting may be the best way for a beetle to tell a
predator: "Watch out. I'm a mouthful you're likely to regret." Notable among North American desert tenebrionids is
Eleodes longicollis, a large, shiny, spectacularly beautiful black beetle. Eleodes, like all members of its genus, has a
pair of large abdominal glands from which it discharges a foul-smelling, irritating secretion when disturbed. The
fluid contains 1,4-benzoquinones, plus several hydrocarbons and caprylic acid. The quinone compounds are
doubtless the major repellents in the mixture, while the other components may serve as solvents of the quinones and
as spreading- and penetration-promoting agents. The beetle has a distinctive behavior when disturbed. which it
shares with many other members of its genus. No sooner is it touched than t comes to a halt and assumes a
headstand. It presses its head against the ground and, by straightening out its hind legs, points its rear skyward. It
typically remains motionless in that stance, ready to use its defensive glands should the disturbance
persist. Glandular discharges may be forcible and involve actual spraying of secretion, but they may also

第 144 页 共
373 页
2021 托福阅读真题

involve no more than the emission of a droplet that clings to the abdominal tip and that the beetle attempts to
spread by use of its hind legs.

2. Which of the following statements most accurately describes the relationship of paragraph 2 to paragraph 1?
A Paragraph 2 provides an explanation for a surprising fact discussed in paragraph 1.
B Paragraph 2 calls into question some of the arguments presented in paragraph 1.
C Paragraph 2 uses an example to support the conclusions reached in paragraph 1.
D Paragraph 2 explains why the facts presented in paragraph 1 can be confusing to naturalists.

3. The word "stance" in the passage is closest in meaning to


A strategy
B time period
C circumstance
D position

4. Which of the sentences below best expresses the essential information in the highlighted sentence in the passage?
Incorrect choices change the meaning in important ways or leave out essential information.
A Glandular discharges vary in the amount of secretion released and in the manner in which the secretion is spread.
B Beetles first discharge strong sprays of secretion and then continue to emit and spread droplets.
C Glandular discharges are forcible so that the beetle may spread the droplets with its hind legs.
D Glandular discharges are made up of no more than a single drop that begins at the abdomen and spreads to the
hind legs.

Paragraph 3: The secretion is highly effective against ants, which in a desert setting are a major hazard, and it is
doubtless deterrent also to other predators, including rodents and birds. It is, however, ineffective against one species
of rodent, the grasshopper mouse, Onychomys torridus, which has the remarkable habit of holding the beetle upright
when it comes upon it and forcing its rear into the sand, thereby causing the secretion to be discharged ineffectually
into the soil. The mouse commences eating the moment it has a beetle in its hold and eventually consumes almost
the entire prey, leaving only the abdominal tip (with the intact glands) and the legs and wing covers (which offer few
nutrients). Such remnants are a frequent sight in desert areas where grasshopper mice abound.

5. The word "remnants" in the passage is closest in meaning to


A events
B remaining parts
C prey
D findings

第 145 页 共
373 页
2021 托福阅读真题

6. According to paragraph 3, why are the defenses of the beetles ineffective against grasshopper mice?
A Grasshopper mice kill the beetles before they are able to release their secretion
B Grasshopper mice do not find the beetles' secretion irritating.
C Grasshopper mice prevent the beetles from remaining in a headstand
D Grasshopper mice release a secretion that makes the beetles' glands inactive.

Paragraph 4: Head-standing is not restricted to Eleodes but occurs also in other beetles. One striking head-stander
is Moneilema appressum.a member of the longhorn beetle family (Cerambycidae). This insect is jet black and
wingless like Eleodes and, like the latter, is active on the desert floor. It too "freezes" and raises its rear when
touched, although it has no defensive glands. Moneilema is evidently a mimic of Eleodes-it is intrinsically
defenseless but imitates the head-standing behavior of Eleodes , and thus relies on being confused with the real thing.
Some tenebrionids (for example, the species of the genus Gonasida) pull essentially the same trick.They lack
defensive glands but stand on their heads like Eleodes when threatened, thereby presumably holding predators at
bay.

7. According to paragraph 4, Moneilema appressum and Eleodes are alike in all of the following ways EXCEPT
A Both lack wings.
B Both are black.
C Both release a secretion when attacked.
D Both assume a headstand when disturbed.

8. Paragraph 4 suggests that predators may stay away from species of the genus Gonasida because these species
A have defensive glands similar to those of Eleodes
B are often confused with the truly dangerous beetle Moneilema
C are able to attack predators while standing on their head
D fool predators into thinking that they produce an irritating secretion

Paragraph 1: The color black, you would imagine, is impractical in a desert setting, since it offers little protection
against the heat. Yet as naturalists are quick to note, black insects, mainly beetles, abound in desert settings.█ In the
deserts of the southwestern United States there exist a multitude of jet-black beetles, ranging from mid-size to large
and belonging primarily to the family Tenebrionidae. █ These beetles are inactive in midday when the sun is most
intense and are active primarily at dusk and dawn, in the twilight, when they are on the march to or from their
feeding sites.█ True color is poorly perceived in dim light, and there is no better way to be noticeable on sand at
dawn or dusk than by being black.█ Add to this fact that dawn and dusk are the times when vertebrates are looking
for prey, and you are faced with the apparent paradox that black desert beetles are most conspicuous when they are
most at risk.

第 146 页 共
373 页
2021 托福阅读真题

9. Look at the four squares █ that indicate where the following sentence could be added to the passage.
This behavior helps protect them from the heat but may seem to expose them to other risks.
Where would the sentence best fit? Click on a square █ to add the sentence to the passage.

10. Directions: An introductory sentence for a brief summary of the passage is provided below. Complete the
summary by selecting the 3 answer choices that express the most important ideas in the passage. Some sentences do
not belong in the summary because they express ideas that are not presented in the passage or are minor ideas in the
passage.

Many desert beetles are black.

A The black color helps desert beetles protect themselves from predators by blending in with their surroundings at
night, when most desert vertebrates are active.
B Beetles such as Eleodes release a repellent-containing secretion when touched by predators, a strategy that is
effective against ants, birds, and most other animals.
C Beetles often lie with their abdomens up when touched by predators, a defense that is often successful in
convincing the predator that the beetle is dead and harmless.
D Being black makes desert beetles more noticeable to predators at certain times of day, but the color also indicates
to predators that they should not disturb the beetles
E Many rodents such as the grasshopper mouse have developed resistance to the poisons in the secretion of Eleodes ,
and some of these rodents eat the beetle whole
F Not all desert beetles discharge irritating chemicals as Eleodes does, but some keep predators away by adopting
other behaviors similar to those of Eleodes

第 147 页 共
373 页
2021 托福阅读真题

Habitable Planets

Paragraph 1: What makes a world habitable? Being the right size is probably the first requirement. If a planet were
more than about ten times Earth's mass, it would accumulate a huge atmosphere and become a gas giant like Jupiter
or Saturn with no solid surface. Any living organisms trying to form in the dense atmospheres of these gas giants
would be carried alternately to frozen heights and overheated depths by the strong circulating air currents, making
survival difficult. Thus, despite the detection in Jupiter's atmosphere of simple amino acids-the organic compounds
necessary for life scientists think gas giants are unlikely to make safe harbors for life. At the other end of the scale, if
a planet is too small, that causes trouble too. It would not be able to hold onto a substantial atmosphere, so the
oceans would boil off. What's more, a small planet would not have a stable climate over billions of years. The reason
has to do with plate tectonics, which involves the movement of large chunks of the planet's crust (outermost solid
surface of the planet). Where these plates run into each other, mountain ranges-like the Himalayas on Earth-build up.
The process also enables long-term recycling of substances like carbon dioxide among the atmosphere, the ocean,
and the crust, making a stable climate possible. A rocky planet would have to be at least one-third as massive as
Earth to enable plate tectonics. Mars, for example, falls below that threshold. Being in a fairly circular orbit (path
around a star) is also essential, to ensure even heating from the parent star during the course of a year. A planet in a
highly elongated orbit would suffer from extreme swings in temperature as its distance from the star varies.

1. The word "detection" in the passage is closest in meaning to


A, presence
B, discovery
C, formation
D, possibility

2. According to paragraph 1, which TWO of the following are reasons that a small planet would not be
habitable? To receive credit, you must select TWO answers.
A, It would not have oceans.
B, Its surface would be continually damaged by plate tectonics.
C, Its atmosphere would prevent the surface from warming up
D, Its climate would not be stable.

第 148 页 共
373 页
2021 托福阅读真题

3. According to paragraph 1, all of the following help a planet maintain a climate suitable for life EXCEPT
A, an orbit around the planet's star that is circular
B, large changes in temperature during the year
C, the movement of large sections of the planet's crust
D, the recycling of carbon dioxide among the atmosphere, the ocean, and the crust

4. In paragraph 1, why does the author discuss the consequences of a planet's being in a highly elongated
orbit?
A, To explain why Mars does not have plate tectonics
B, To illustrate why a circular orbit results in more constant temperatures on a planet
C, To give another example of a difference between small and large planets
D, To show that the shape of a planet's orbit depends on how rocky a planet is

Paragraph 2: Beyond those basic requirements, it is all about location. Being located around the right kind of star is
a must. What makes a good parent star? It should live long enough for life to have sufficient time to develop and
evolve. Massive stars live fast and die young; they exhaust their hydrogen fuel supply within hundreds of millions of
years. Low-mass red dwarfs. the smallest types of stars, on the other hand, have incredibly long lives-hundreds of
billions of years. With a lifetime of ten billion years, the Sun falls somewhere in the middle of the range. As in real
estate, neighbors matter, too. If the parent star has a companion, as many stars do, it should be either very close so
that a planet can have a stable orbit encircling both. or very far so that a planet around one star is not affected much
by the other. Luckily, most such star systems appear to be safe. Overcrowded neighborhoods may be hazardous to
life: Within dense. star clusters, for example, close encounters between neighboring stars could disrupt each other's
planetary systems. The Sun probably formed in a loose group of stars that has since dispersed. Today, stars in Earth's
neighborhood are safely separated, by a few light-years on average.

5. Which of the following can be inferred from the fact that today the stars near the Sun are "safely separated.
by a few light-years on average"?
A, Life on Earth is not affected by interference from those other stars.
B, Those stars were once much farther apart from one another than they are now.
C, If those stars had been closer to each other, our Sun would have used up its fuel before life could develop on
Earth.
D, Most of those stars probably have planets orbiting around them that are capable of supporting life

6. According to paragraph 2, a habitable planet is more likely to orbit a small star for which of the following
reasons?
A, The lower mass of a small star requires the planet to use less energy.

第 149 页 共
373 页
2021 托福阅读真题

B, A small star provides more time for life to develop.


C, The orbit around a small star tends to be more stable.
D, A small star is less likely to have companion stars

Paragraph 3: Astronomers usually define a habitable planet as one that can sustain liquid water on its surface. That
means the planet must not be too close to its star or too far from it. The not-too-hot, not-too-cold region around a star,
where the temperature is just right for liquid water, is called the habitable zone. Earth is safely within the Sun's
habitable zone. Venus, about 30 percent closer in, is not-probably because the scorching heat boiled off is
surface water early on-while Mars, about 50 percent farther from the Sun that Earth, is much colder but still
barely inside the present-day habitable zone. Indeed, Mars appears to have had flowing water and large lakes in
the past, and probably even has some surface water today. "The problem is that Mars is too small to recycle carbon
and oxygen," explained Jim Lasting, a planetary scientist at Pennsylvania State University. "An Earth-mass planet at
Mars distance would be habitable, because carbon dioxide would accumulate in its atmosphere, warming the planet
through the greenhouse effect.

7. Which of the sentences below best expresses the essential information in the highlighted sentence in the
passage? Incorrect choices change the meaning in important ways or leave out essential information.
A, Because Venus is closer to the Sun than Mars is, and therefore much hotter. its surface water is too hot for life
B, The cold temperature of Mars allowed it to have surface water unlike the much hotter Venus.
C, Venus is too close to the Sun to be in the habitable zone, but the colder Mars is just inside the zone.
D, Both Venus and Mars were probably in the habitable zone early on. but then Venus's surface water boiled away.

8. Paragraph 3 suggests that Mars is not habitable because


A, it has too much carbon and oxygen in its atmosphere
B, it is too far away from the Sun
C, it is too close to Earth
D, its atmosphere does not keep the planet warm enough

Paragraph 2: Beyond those basic requirements, it is all about location. Being located around the right kind of star is
a must. What makes a good parent star? It should live long enough for life to have sufficient time to develop and
evolve. Massive stars live fast and die young; they exhaust their hydrogen fuel supply within hundreds of millions of
years. Low-mass red dwarfs. the smallest types of stars, on the other hand, have incredibly long lives-hundreds of
billions of years. █ With a lifetime of ten billion years, the Sun falls somewhere in the middle of the range. █ As in
real estate, neighbors matter, too. █ If the parent star has a companion, as many stars do, it should be either very
close so that a planet can have a stable orbit encircling both. or very far so that a planet around one star is not
affected much by the other. █ Luckily, most such star systems appear to be safe. Overcrowded neighborhoods may
be hazardous to life: Within dense. star clusters, for example, close encounters between neighboring stars could

第 150 页 共
373 页
2021 托福阅读真题

disrupt each other's planetary systems. The Sun probably formed in a loose group of stars that has since dispersed.
Today, stars in Earth's neighborhood are safely separated, by a few light-years on average.

9. Look at the four squares █ that indicate where the following sentence could be added to the passage.
But age is not the only issue.
Where would the sentence best fit? Click on a square █ to add the sentence to the passage.

10. Directions: An introductory sentence for a brief summary of the passage is provided below. Complete the
summary by selecting the 3 answer choices that express the most important ideas in the passage. Some sentences do
not belong in the summary because they express ideas that are not presented in the passage or are minor ideas in the
passage. This question is worth 2 points.

Scientists have identified various conditions that a planet must meet in order to be capable of supporting life.

A, To develop the kind of plate tectonics needed for habitability, a planet must be at least as large as Earth but not as
large as the gas giants
B, The size of a planet determines whether it has an atmosphere capable of maintaining a stable climate and a planet
surface suitable for life.
C, A planet's habitability requires that its parent star live sufficiently long and be located far enough away from other
stars that the planet is not disrupted.
D, A very large star is more likely to have habitable planets than a small star because it has a much larger supply of
hydrogen fuel, allowing more time for planetary life to evolve.
E, Even if a planet, like Mars, had enough liquid water in the past, it probably lacked the organic compounds
necessary for forming life.
F, Whether a planet has liquid water depends on its distance from its star and on its size, both of which determine the
temperature of the planet's surface

第 151 页 共
373 页
2021 托福阅读真题

European Cities and the Rise of Commerce

Paragraph 1: After the collapse of the Western Roman Empire in the fifth century A.D, most of Europe endured
centuries of lower population, weaker governments, and a more feeble economy. But by the eleventh century, as
populations rose, cities, long-distance trade networks. local markets. and new business arrangements meshed to
create a profit-based economy. With improvements in agriculture and more land in cultivation, great estates
produced surpluses that helped feed-and therefore make possible-a new urban population.

1. Paragraph 1 answers which of the following questions about European history?


A Why did the Western Roman Empire collapse?
B What evidence exists that European governments became weaker after the fifth century?
C How were local markets affected by long-distance trade in the new profit-based economy?
D What developments allowed larger numbers of people to live in cities?

Paragraph 2: Commerce was not new to the history of the West, of course, but the commercial economy of
medieval Europe spawned the institutions that would be the direct ancestors of western businesses: corporations,
banks, accounting systems, and. above all, urban centers that thrived on economic vitality. Whereas ancient cities
had primarily religious, social, and political functions, medieval cities were primarily centers of production and
economic activity.Wealth meant power: it allowed city dwellers to become self-governing.

2. According to paragraph 2, which of the following was a consequence of economic activities in medieval cities?
A Cities acquired many more religious, social, and political functions.
B Cities began to govern themselves
C Cities' religious and social functions became more important than their political functions.
D Cities' wealth became concentrated in the hands of only a few city dwellers.

Paragraph 3: Commercial centers developed around castles and monasteries and within the walls of ancient
towns. Great lords (powerful landowners) in the countryside were eager to take advantage of the profits that their

第 152 页 共
373 页
2021 托福阅读真题

estates generated. In the late tenth century, they had reorganized their lands for greater productivity, encouraged their
peasants to cultivate new land, and converted the services and dues (traditionally owed to them by peasants and
vassals) to money payments. Now with ready cash, they not only fostered the development of sporadic markets
where they could sell their surpluses and buy luxury goods but even encouraged craftspeople and traders to settle
down near them. The lords gained at each step. Their purchases brought them an enhanced lifestyle and greater
prestige. But they also collected tolls and sales taxes from merchants, in this way profiting even more from trade.

3. The word "sporadic" in the passage is closest in meaning to


A occasional
B established
C large
D successful

4. According to paragraph 3, the development of commercial centers around their castles benefited great lords in
each of the following ways EXCEPT:
A It made it easier to sell the lords' surplus agricultural products
B It brought in luxury goods for the lords to buy.
C It increased the services and dues owed to the lords by peasants and vassals.
D It gave the lords the opportunity to collect tolls and sales taxes from merchants.

Paragraph 4: Trade did not benefit only great lords. The vassals who lived with them enjoyed a better standard of
living. Peasants too participated in the new economy, selling their meager surpluses at local markets. Commerce
sometimes opened up unexpected opportunities for enrichment. Former servants of an important church official in
Mácon (today in France), for example, set up a bakery near the bridge of the city and sold bread to travelers. They
soon grew prosperous.

5. Why does the author discuss the establishment of "a bakery near the bridge of the city"?
A To support the idea that great lords were not the only ones to benefit from trade
B To make the point that travelers were extremely important to the new economy
C To give an example of a development that helped improve everyone's standard of living
D To show how professions like baking played a significant role in the new economy

6. The word "prosperous" in the passage is closest in meaning to


A important
B very skillful
C independent
D wealthy

第 153 页 共
373 页
2021 托福阅读真题

Paragraph 5: The immediate stimulus for the formation of a city varied. At Bruges (today in Belgium), the local
lord's castle became the magnet around which a city formed. Other commercial centers clustered around a number of
monasteries that by the eleventh century had become large communities with many needs to supply.Still other
markets formed just outside the walls of older cities; these gradually merged into new and enlarged urban
communities as town walls were built around them to protect their inhabitants.Sometimes informal country markets
might eventually be housed in permanent structures. To the north, in places like Frisia, the Vikings originally raiders
down from the Scandinavian lands-had already established centers of wealth and trade, and these settlements became
permanent, thriving towns. Along the Rhine and other river valleys, cities sprang up to service the merchants who
traversed the route between Italy and the north. And at Reims, the middle of a forum (a public meeting place) dating
back to the Roman Empire became a new commercial center. Around the marketplace at Reims grew a network of
streets whose names (many of which still exist) revealed their essential commercial functions: Street of the Butchers,
Street of the Wool Market, Street of the Wheat Market.

7. According to paragraph 5, medieval European cities developed in all of the following types of locations EXCEPT
A near monasteries that had large populations
B in areas along the Rhine that provided protection against Viking raiders
C around established centers of wealth
D along major trade routes

Paragraph 6: The look and feel of such developing cities varied enormously.Nearly all included a marketplace, a
castle, and several churches however. And most had to adapt to increasingly crowded conditions;for example, in one
English town at the end of the eleventh century, city plots were still large enough to accommodate houses parallel to
the street, but the swelling population soon necessitated destroying these houses and building instead long, narrow,
hall-like houses constructed at right angles to the thoroughfare.

8. Paragraph 6 suggests that houses in an English town were eventually constructed at right angles to the street
because
A doing so made it possible to house more people on the street
B most of the houses parallel to the street collapsed
C doing so improved the look and feel of the developing cities
D houses built parallel to the street tended to be narrow and hall-like

Paragraph 4: Trade did not benefit only great lords. The vassals who lived with them enjoyed a better standard of
living. █Peasants too participated in the new economy, selling their meager surpluses at local markets. Commerce
sometimes opened up unexpected opportunities for enrichment. █ Former servants of an important church official in
Mácon (today in France), for example, set up a bakery near the bridge of the city and sold bread to travelers. █ They

第 154 页 共
373 页
2021 托福阅读真题

soon grew prosperous.

Paragraph 5: The immediate stimulus for the formation of a city varied. █ At Bruges (today in Belgium), the local
lord's castle became the magnet around which a city formed. Other commercial centers clustered around a number of
monasteries that by the eleventh century had become large communities with many needs to supply.Still other
markets formed just outside the walls of older cities; these gradually merged into new and enlarged urban
communities as town walls were built around them to protect their inhabitants.Sometimes informal country markets
might eventually be housed in permanent structures. To the north, in places like Frisia, the Vikings originally raiders
down from the Scandinavian lands-had already established centers of wealth and trade, and these settlements became
permanent, thriving towns. Along the Rhine and other river valleys, cities sprang up to service the merchants who
traversed the route between Italy and the north. And at Reims, the middle of a forum (a public meeting place) dating
back to the Roman Empire became a new commercial center. Around the marketplace at Reims grew a network of
streets whose names (many of which still exist) revealed their essential commercial functions: Street of the Butchers,
Street of the Wool Market, Street of the Wheat Market.

9. Look at the four squares █ that indicate where the following sentence could be added to the passage.
These possibilities were often provided by trade routes.
Where would the sentence best fit? Click on a square █ to add the sentence to the passage.

10. Directions: An introductory sentence for a brief summary of the passage is provided below. Complete the
summary by selecting the 3 answer choices that express the most important ideas in the passage. Some sentences do
not belong in the summary because they express ideas that are not presented in the passage or are minor ideas in the
passage.

A The commercial institutions of the Western Roman Empire survived its collapse and formed the basis of the
medieval European economy.
B Improved agriculture allowed the production of enough food to support growing cities, which became centers of
economic activity.
C Ancient towns expanded to include outside markets, and commercial centers in a variety of locations turned into
thriving cities
D The growth of cities made it possible for peasants and vassals to leave their lords and thus avoid having to provide
the traditional services and dues
E The great lords profited from the new economy in various ways, but the economy opened up new possibilities for
many others as well.
F The new wealth resulted in larger houses being built in the urban centers, which often limited the space for public
meeting places.

第 155 页 共
373 页
2021 托福阅读真题

Isolation and diversification in the Tropical Rain Forests

Paragraph 1: The dense vegetation of the tropical rain forest prevents living organisms from moving easily. This is
the way the rain forest limits human movement. For example, each valley in New Guinea, an large tropical rain
forests, has its own tribe—each with language manner of dress, and child-rearing tendency. This diversification is so
great that New Guineans have developed about 1000 different languages in contrast to the fifty that occur throughout
all of Europe. Often people born in these isolated tribes never venture further than 10 miles from their birthplace.

1. In paragraph 1, why does the author mention the 50 languages of Europe?


A, To emphasize the extent of diversification in New Guinea
B, To help explain why Europeans often share the same cultural behaviors
C, To suggest that diversification of language occurs everywhere and not just in tropical rain forests
D, To suggest a reason why Europeans are willing to travel farther from their birthplace than New Guineans are

2. According to paragraph 1, all of the following statements about the tribes that live in the tropical rain forests of
New Guinea are true EXCEPT
A. They can be found in valleys
B. They each dress in a distinctive way
C. They share a common approach to child-rearing
D. They have little contact with people live from other New Guinea tribes

Paragraph 2: It is not unreasonable to surmise that the same cause of the great diversity of tribes and languages in
the rainforests of New Guinea—the extreme hindrance to movement caused by ecological congestion—may also
have contributed to the great diversity in plants and animals in rainforests worldwide. In open continental regions,
over grasslands or dry stretches, birds or insects may travel many miles before they come upon a new likely
residence. But beneath the rainforest canopy, dispersal potential becomes drastically severed, and no bird, animal or

第 156 页 共
373 页
2021 托福阅读真题

insect could move very far before encountering a wealth of edible opportunities and available habitats. Jungle
populations that become separated from each other by just hundreds or even tens of miles may be as effectively and
permanently isolated from each other as if they were on opposite sides of the globe.Indeed, this is one of the
evolutionary reasons for the very bright colors and loud calls of many rain forest plants and animals. Consider the
vocalizations and vibrancy of parrots and toucans (tropical birds with typically brightly covered feathers) or consider
the howler monkey, the world’s loudest land animal. These noisy or highly visible shows help the animals discover
the whereabouts of other members of the same species within the lavish display of green.

3. The word “congestion” in the passage is closest in meaning to


A. growth
B. overcrowding
C. damage
D. interdependent

4. According to paragraph 2, what happens when two jungle population separated from each other?
A. They will often travel long distances in search of each other
B. They continue to move farther and farther apart
C. They may never come into contact with each other again
D. They may be unable to find edible opportunities in their habitats

Paragraph 3: Thus for many jungle organisms, new species can begin to occur at very short distances, and like a
wildfire, this great diversity led to spread and fuel itself, for the different plants and animals would provide great
variations in the survival benefits and drawbacks within all the small neighboring micro-territories. These extreme
isolating effects of course would only occur below or at the level of the canopy, which is that upper, oceanic layer of
leaves situated some 45—50 meters above the ground that forms the unbroken living roof of the rain forest.

5. The passage suggests that which of the following helps make it possible for new species to begin to occur at very
short distances" from each other in the jungle?
A. The protection offered by the unbroken living roof of rain forest leaves
B. The fact that the jungle canopy is high above the ground at 45-50 meters
C. The separation of plants and animals from other by thick level of canopy
D. The isolation constantly defends against of other jungle organisms for their territory

第 157 页 共
373 页
2021 托福阅读真题

Paragraph 4: But there is another less crowed layer, the emergent layer. The emergent layer is made up of the
occasional enormous tree that grows 60-70 meters high, like the giant kapok, or silk cotton, tree, which often
breaks through the darkness of the canopy and extends upward into the open sunshine. In South America,
these more sparsely situated leafy treetops have become the nesting sites of the harpy eagle, the largest eagle in the
world, with a wing span reaching 2 meters. The happy eagle uses its advantageous position and view to prey upon
the monkeys resulting through the meadow of canopy leaves below—a hunting system that is similar to that of North
American birds of prey that sit in tress looking down for animals moving about the fields and grasslands. In the rain
forest, thought this system begins at 50 meters off the ground.

6. Which of the sentences below best expresses the essential information in the highlighted sentence in the passage?
Incorrect choices change the meaning in important ways or leave out essential information
A. The less crowded emerged layer is made up of enormous trees often at rise above the darkness of the canopy
B. The emergent layer containing the occasional enormous tree which are 60 70 meters high, often breaks through
the darkness of the canopy and extends upward into the open sunshine.
C. Tropical forests have an emergent layer containing occasional enormous tree which often breaks through the
darkest
D. As like giant kapok that grow 60-70 meters high, the occasional enormous trees compose of the emergent layer
are extremely tall

7. According to paragraph 4, which of the following statement about harpy eagles is true?
A. They build their nests up to 50 meters above the ground
B. They prey on animals outside of the canopy
C. They use a hunting technique like that of North American birds of prey
D. They eat the North American birds

Paragraph 5: Unlike the more than 40 species of toucans that inhabit the Amazon rainforest, the range of the harpy
eagle extends from Central America and all across the South American jungles, yet it has not diversified. Clearly, the

第 158 页 共
373 页
2021 托福阅读真题

freedom of movement offered above the canopy has allowed a continuous flow of genes and has not encouraged
diversification of the species. The giant kapok trees, in which these eagles so often build their nests are similarity
unconfined by their neighboring trees and have retained their powers of dispersal. Their seeds, surrounded by fluffy
cotton fibers.

8. According to paragraph 5, why does the author mention the more than 40 species of amazon rain forest toucans?
A. To explain that extreme diversification extends to the inhabitants living in the forests worldwide
B. To support the idea that some birds species, like toucans, are diversified
C. To contrast that the number of species across the forest does not related tone diversification
D. To illustrate that the flow of genes does not always lead to diversification

Paragraph 2: It is not unreasonable to surmise that the same cause of the great diversity of tribes and languages in
the rainforests of New Guinea—the extreme hindrance to movement caused by ecological congestion—may also
have contributed to the great diversity in plants and animals in rainforests worldwide. ■In open continental regions,
over grasslands or dry stretches, birds or insects may travel many miles before they come upon a new likely
residence. ■But beneath the rainforest canopy, dispersal potential becomes drastically severed, and no bird, animal
or insect could move very far before encountering a wealth of edible opportunities and available habitats. ■Jungle
populations that become separated from each other by just hundreds or even tens of miles may be as effectively and
permanently isolated from each other as if they were on opposite sides of the globe.■Indeed, this is one of the
evolutionary reasons for the very bright colors and loud calls of many rain forest plants and animals. Consider the
vocalizations and vibrancy of parrots and toucans (tropical birds with typically brightly covered feathers) or consider
the howler monkey, the world’s loudest land animal. These noisy or highly visible shows help the animals discover
the whereabouts of other members of the same species within the lavish display of green.

9. Look at the four squares [■] that indicate where the following sentence can be added to the passage.
They are less restricted in these landscapes and can move easily
Where would the sentence best fit?

10. Directions: An introductory sentence for a brief summary of the passage is provided below. Complete the
summary by selecting the THREE answer choices that express the most important ideas in the passage. Some
sentences do not belong in the summary because the express ideas that are not presented in the passage or are minor
ideas in the passage. This question is worth 2 points.

Inside tropical rain forest, dense vegetation severely limits the movement of organisms

A. Aside from those in New Guinea. there are few rainforests worldwide in which a pattern of extreme
diversification extends to humans as well as plant and animal populations.

第 159 页 共
373 页
2021 托福阅读真题

B. Although rain forest birds. animals and insects never need to travel far to find food, small groups temporarily
move into neighboring micro-territories to take advantage of more favorable conditions
C. Because few trees other than the giant kapok extend above rain forest canopies, the harpy eagle, despite its wide
range and freedom of movement, almost always nests in kapok trees
D. In open regions, organisms may travel many miles to find suitable habitats, but in rain forests such dispersal is
unnecessary, and new species can begin to occur in populations in nearby micro-territories
E. Rain forest organisms are extremely diverse, and many rain forest animals have evolved very distinctive colors
and calls to help them identify other members of the same species
F. Inside the Amazon rain forest. more than 40 species of some birds are found, whereas the harpy eagle, which lives
above the canopy where movement is unrestricted. shows no diversification

Programming Computers to Play Games

Paragraph 1: Backgammon is the oldest board game in the world. It was first played in ancient Mesopotamia,
starting around 3000 B.c. The rules of backgammon were codified in the seventeenth century, and the game has
changed little since. The same can't be said about the players of the game. One of the best backgammon players in
the world is now a software program. In the early 1990s. Gerald Tesauro, a computer programmer at IBM, began
developing a new kind of artificial intelligence (Al). At the time, most Al programs relied on the brute computational
power of microchips. This was the approach used by Deep Blue, the powerful set of IBM mainframes that managed
to defeat chess grand master Garry Kasparoy in 1997. Deep Blue was capable of analyzing more than two hundred
million possible chess moves per second, allowing it to consistently select the optimal chess strategy. (Kasparov's
brain, on the other I hand. evaluated only about five moves per second.) But all this strategic firepower consumed a
lot of energy: while playing chess. Deep Blue was a ire hazard and required specialized heat-dissipating equipment
so that it didn't burst into flames. Kasparov, meanwhile, barely broke a sweat. That's because the human brain is a

第 160 页 共
373 页
2021 托福阅读真题

model of efficiency: even when it's deep in thought, the cortex consumes less energy than a light-bulb.

1. According to paragraph 1, Gerald Tesauro was involved in


A increasing the computational power of microchips
B inventing the approach to computation that was used by Deep Blue
C creating the computer mainframes that defeated Garry Kasparov
D designing a computer program that worked differently from previous programs

Paragraph 2: While the popular press was celebrating Deep Blue's stunning achievement-a machine had outwitted
the greatest chess player in the world!-Tesauro was puzzled by its limitations. Here was a machine capable of
thinking millions of times faster than its human opponent, and yet it had barely won the match. Tesauro realized that
the problem with all conventional Al programs, even brilliant ones like Deep Blue's, was their rigidity. Most of
Deep Blue's intelligence was derived from other chess grand masters, whose wisdom was painstakingly programmed
into the machine. (IBM programmers also studied Kasparov's previous chess matches and engineered the software to
exploit his recurring strategic mistakes.) But the machine itself was incapable of learning. Instead, it made decisions
by predicting the probable outcomes of several million different chess moves. The move with the highest predicted
value was what the computer ended up executing. For Deep Blue, the game of chess was just an endless series of
math problems.

2. The word "conventional" in the passage is closest in meaning to


A advanced
B existing
C traditional
D experimental

3. What can be inferred from paragraph 2 about Deep Blue?


A Its programming was significantly more rigid than that of most other Al programs at the time
B It took the same approach to making decisions all the time, regardless of the situation.
C It made fewer correct predictions when it processed information very quickly.
D It caused Kasparov to make several strategic mistakes during their match.

Paragraph 3: Of course, this sort of artificial intelligence isn't an accurate model of human cognition. Kasparov
managed to compete on the same level as Deep Blue even though his mind had far less computational power.
Tesauro's surprising insight was that Kasparov's neurons were effective because they had trained themselves. They
had been refined by decades of experience to detect subtle spatial patterns on the chessboard. Unlike Deep Blue,
which analyzed every possible move. Kasparov was. able to instantly isolate his best options and focus his mental
energies on evaluating only the most useful strategic alternatives.

第 161 页 共
373 页
2021 托福阅读真题

4. The word "evaluating" in the passage is closest in meaning to


A using
B judging
C selecting
D planning

5. According to paragraph 3, the fact that Deep Blue barely won the match against Kasparov led Tesauro to which of
the following unexpected realizations?
A No form of Al will ever be able to accurately model human cognition.
B Kasparov had far less computational power than Deep Blue had.
C The self-training of neurons can lead to effective decision-making.
D Even Deep Blue could never analyze every possible move.

6. According to paragraph 3, Kasparov was able to compete on the same level as Deep Blue in part because
A his computational abilities were almost as strong as those of Deep Blue
B even subtle patterns on chessboards are comparatively easy to detect
C he trained himself to analyze all possible moves
D he could instantly identify and focus on the few moves that were likely to succeed

Paragraph 4: Tesauro set out to create an Al program that acted like Garry Kasparov. He chose backgammon as his
model and named the program TD-Gammon. (The TD stands for temporal difference.) Deep Blue had been
preprogrammed with chess acumen, but Tesauro's software began with absolutely zero knowledge. At first, its
backgammon moves were entirely random. It lost every match and made stupid mistakes. But the computer didn't
remain a novice for long; TD-Gammon was designed to learn from its own experience. Day and night, the software
played backgammon against itself, patiently learning which moves were most effective After a few hundred
thousand games of backgammon, TD-Gammon was able to defeat the best human players in the world.

7. According to paragraph 4, which statement about TD-Gammon is true?


A It was preprogrammed with the knowledge of expert chess players.
B It started with a basic knowledge of how to play the game.
C It did not use any game plan at all when it first started playing.
D It won a few of its first games by random chance.

Paragraph 5: How did the machine turn itself into an expert? Although the mathematical details of Tesauro's
software are numbingly complex, the basic approach is simple. TD-Gammon generates a set of predictions about
how the backgammon game will unfold. Unlike Deep Blue, the computer program doesn't investigate every possible

第 162 页 共
373 页
2021 托福阅读真题

permutation. Instead, it acts like Garry Kasparov and generates its predictions from its previous experiences. The
software compares these predictions to what actually happens during the backgammon game. The ensuing
discrepancies provide the substance of its education, and the software strives to continually decrease this error signal.
As a result, its predictions constantly increase in accuracy, which means that its strategic decisions get more and
more effective and intelligent.

8. All of the following are mentioned in paragraph 5 as ways in which TD-Gammon plays backgammon like a
human expert EXCEPT:
A It makes predictions in a given game based on its experience from previous games.
B It investigates the most complex options but not simpler, more obvious options.
C It compares its predictions about what will happen with what actually does happen.
D It improves its performance by learning from its mistakes.

Paragraph 1: Backgammon is the oldest board game in the world. It was first played in ancient Mesopotamia,
starting around 3000 B.c. The rules of backgammon were codified in the seventeenth century, and the game has
changed little since. █ The same can't be said about the players of the game. █ One of the best backgammon players
in the world is now a software program. █ In the early 1990s. Gerald Tesauro, a computer programmer at IBM,
began developing a new kind of artificial intelligence (Al). █ At the time, most Al programs relied on the brute
computational power of microchips. This was the approach used by Deep Blue, the powerful set of IBM mainframes
that managed to defeat chess grand master Garry Kasparoy in 1997. Deep Blue was capable of analyzing more than
two hundred million possible chess moves per second, allowing it to consistently select the optimal chess strategy.
(Kasparov's brain, on the other I hand. evaluated only about five moves per second.) But all this strategic firepower
consumed a lot of energy: while playing chess. Deep Blue was a ire hazard and required specialized heat-dissipating
equipment so that it didn't burst into flames. Kasparov, meanwhile, barely broke a sweat. That's because the human
brain is a model of efficiency: even when it's deep in thought, the cortex consumes less energy than a light bulb

9. Look at the four squares █ that indicate where the following sentence could be added to the passage.
With the rise of computer technology, playing was no longer limited to humans.
Where would the sentence best fit? Click on a square █ to add the sentence to the passage.

10. Directions: An introductory sentence for a brief summary of the passage is provided below. Complete the
summary by selecting the 3 answer choices that express the most important ideas in the passage. Some sentences do
not belong in the summary because they express ideas that are not presented in the passage or are minor ideas in the
passage. (This question is worth 2 points.)

Multiple artificial intelligence (Al) programs have been developed to play games.

第 163 页 共
373 页
2021 托福阅读真题

A Al game programs were initially developed as experiments to test how great the computational power of
computers could become.
B Deep Blue was able to beat chess grand master Kasparov in 1997, but it used a very inefficient method of
calculating which moves to make
C Gerald Tesauro developed an Al program based on the model of human cognition in which individuals begin by
knowing nothing and acquire knowledge through experience.
D TD-Gammon's knowledge of backgammon is based on an analysis of the strategies of backgammon champions.
E Tesauro's Al program had even more computational power than Deep Blue had, performing more calculations in a
shorter time than Deep Blue could.
F Because TD-Gammon improves through self-training, its playing skills continually improve over time

Increasing Jellyfish Populations

Passage 1: Scientists have had a tough time trying to discover whether recent increases in jellyfish populations are
really worth worrying about. On the one hand, jellyfish are known to proliferate rapidly in response to positive
changes in prey abundance or environmental conditions such as water temperature and sunlight. The size of
these "blooms" can vary from year to year. On the other hand, these population explosions are occurring in many
places on a scale now widely viewed as unprecedented. In the Sea of Japan, for instance, Nomura's jellyfish are
known to have drifted in from the south in large numbers three times during the twentieth century: in 1920 1958 and
1995. Beginning in 2002, however, they have turned up every summer but one, and in astonishingly high numbers.
In 2005, one of the worst years, up to 500 million Nomura's jellyfish were reported to be drifting into the sea each
day

第 164 页 共
373 页
2021 托福阅读真题

1. What is the author's purpose in mentioning that "jellyfish are known to proliferate rapidly in response to
positive changes in prey abundance or environmental conditions such as water temperature and sunlight"?
A, To argue that under normal environmental conditions, jellyfish can spread more rapidly than other marine species
B, To explain why there were only a few cases of population increase in jellyfish before the twentieth century
C, To provide a reason why increases in jellyfish populations might not necessarily be of concern
D, To explain why recent increases in jellyfish populations worldwide are comparable to population increases in
other marine species

Passage 2: Several factors have now been identified as possible contributions to the increased success of jellyfish
worldwide. One of the leading suspects is the human exploitation of fish and other marine resources. something that
has intensified in recent decades partly because of advances in large-scale seafood harvesting and processing
techniques. Only a handful of species are thought to prey directly on jellyfish, and most of these predators-including
giant sea turtles-are becoming increasingly rare. The main impact of overfishing, though, may stem from the
reduction of filter-feeding fish such as sardines and anchovies, which eat the same food as jellyfish. In the southern
Atlantic waters off Namibia. where overharvesting has resulted in the complete collapse of a once-thriving sardine
fishery, unusually large numbers of jellyfish are now a permanent feature of the near-shore marine ecosystem.

2. According to paragraph 2. which of the following may be an important contributor to increased jellyfish
populations?
A, The disappearance of the giant sea turtles
B, Overfishing by humans
C, A reduction in large-scale seafood harvesting and processing
D, Efforts to preserve near-shore ecosystems

3. Paragraph 2 suggests that which of the following most likely accounts for the unusually large numbers of
jellyfish in the waters off Namibia?
A, Jellyfish moved closer to the shore after sardine fisheries were abandoned.
B, Jellyfish have more food because of the disappearance of their competitors
C, Humans stopped attempting to control jellyfish populations after the collapse of sardine fisheries
D, Overharvesting has resulted in the elimination of one of the main predators of jellyfish.

Passage 3: At the same time, jellyfish seem to thrive under conditions that are becoming increasingly widespread
because of human-associated activities. Although the theory is highly speculative and still under debate, global
warming and acidification of the oceans a result of more carbon dioxide dissolving in the water-may be two such
factors. Jellyfish love warm water, for one thing. And at least one study, from the North Sea, has reported finding a
connection between greater jellyfish abundance and higher acid levels.

第 165 页 共
373 页
2021 托福阅读真题

4. It can be inferred from paragraph 3 that the results of the study in the North Sea are important because
A, they contradict other findings about the effects global warming has had on jellyfish populations
B, they demonstrate that the acidification of the oceans and its effects on jellyfish populations is not as widespread as
it was once thought
C, they may prove that in areas where carbon dioxide has been in decline, jellyfish populations increased
D, they may provide concrete evidence of the role of human activity in the increase of jellyfish populations

Passage 4: Another environmental change, and one that is more firmly linked to expanding jellyfish populations, is
eutrophication, the process by which water becomes enriched with dissolved nutrients, resulting in more aquatic
plant life and, subsequently, reduced levels of oxygen. Eutrophication occurs in near-shore ocean waters close to
large human population centers and the mouths of large rivers. A strong correlation exists between blooms of algae
and other plankton, caused by excessive nutrients from sewage and fertilizer runoff, and jellyfish eruptions. One
example is the waters off the southern United States coast. Nutrient-enriched waters from the Mississippi River have
created in the Gulf of Mexico a massive dead zone (ocean area with low oxygen levels).

5. Paragraph 4 indicates that all of the following are characteristics of waters where eutrophication occurs
EXCEPT
A, decreased oxygen levels
B, large areas with little to no algae and other plankton
C, large amounts of dissolved nutrients
D, nearness to highly populated areas

Passage 5: While fish and other aquatic life-forms are finding survival increasingly challenging, jellyfish such as
moon jellyfish and sea nettles are becoming increasingly numerous. This is not really surprising. A checklist of
jellyfish traits-good survival rates during periods of starvation, rapid reproduction, diverse diet, ability to
feed in murky water, capacity for surviving under low oxygen conditions typical of dead zones-reveals that
these seemingly fragile organisms are actually tough life-forms ideally suited to survive in disturbed
environments. According to the National Science Foundation, the main science-funding agency in the United States
there are currently 400 known ocean dead zones where almost nothing lives except jellyfish.

6. Which of the sentences below best expresses the essential information in the highlighted sentence in the
passage? Incorrect choices change the meaning in important ways or leave out essential information.
A, Jellyfish surviving in disturbed environments have traits that make them tougher than jellyfish living in other
environments
B, Jellyfish are fragile organisms that must undergo a long checklist of adaptations in order to survive in disturbed
environments.
C, Jellyfish reproduce most rapidly and feed most effectively in low-oxygen environments that cannot support other

第 166 页 共
373 页
2021 托福阅读真题

life-forms
D, Though they appear fragile, jellyfish possess a number of characteristics that help them to survive in hostile
conditions.

Passage 6: Finally, like many other marine invasive species, jellyfish have benefited from the globalization of
human trade. Many species are hardy enough to survive in the ballast water of ships, and jellyfish polyps can attach
themselves to their hulls. Because of this, jellyfish species are being inadvertently introduced into new aquatic
habitats. One of the best examples involves the warty comb jelly (Mnemiopsis leidyi ), a species that likely made its
way into the Black Sea for the first time in 1982. Since then this highly adaptable organism has been thriving; its
original nonnative range has expanded to include not just the Black Sea but also the Caspian, Baltic and North seas

7. The word "inadvertently" in the passage is closest in meaning to


A, accidentally
B, unnecessarily
C, constantly
D, unnaturally

8. According to paragraph 6. what explains the wide geographic range of the warty comb jellyfish?
A, It was once traded by humans conducting overseas commerce.
B, It has a larger native range than any other type of jellyfish.
C, It has traveled to nonnative regions by attaching itself to ships
D, It is a better competitor than other jellyfish.

Passage 3: At the same time, jellyfish seem to thrive under conditions that are becoming increasingly widespread
because of human-associated activities. Although the theory is highly speculative and still under debate, global
warming and acidification of the oceans a result of more carbon dioxide dissolving in the water-may be two such
factors. Jellyfish love warm water, for one thing. And at least one study, from the North Sea, has reported finding a
connection between greater jellyfish abundance and higher acid levels. █

Passage 4: Another environmental change, and one that is more firmly linked to expanding jellyfish populations, is
eutrophication, the process by which water becomes enriched with dissolved nutrients, resulting in more aquatic
plant life and, subsequently, reduced levels of oxygen. █ Eutrophication occurs in near-shore ocean waters close to
large human population centers and the mouths of large rivers. █ A strong correlation exists between blooms of
algae and other plankton, caused by excessive nutrients from sewage and fertilizer runoff, and jellyfish eruptions. █
One example is the waters off the southern United States coast. Nutrient-enriched waters from the Mississippi River
have created in the Gulf of Mexico a massive dead zone (ocean area with low oxygen levels).

第 167 页 共
373 页
2021 托福阅读真题

9. Look at the four squares █ that indicate where the following sentence could be added to the passage.
That is where agricultural, industrial, and urban activities often generate significant amounts of nutrient
pollution.
Where would the sentence best fit? Click on a square █ to add the sentence to the passage.

10. Directions: An introductory sentence for a brief summary of the passage is provided below. Complete the
summary by selecting the THREE answer choices that express the most important ideas in the passage. Some
sentences do not belong in the summary because they express ideas that are not presented in the passage or are minor
ideas in the passage. This question is worth 2 points

Jellyfish populations have been increasing at a rate that cannot be explained only by naturally -occurring
fluctuations

A, As a result of human exploitation of fish and other marine organisms, jellyfish have both fewer predators and
greater access to food sources
B, Human activity is linked to increased jellyfish populations in the Gulf of Mexico but scientists are still uncertain
about the changes occurring in the North Sea and the Sea of Japan.
C, Eutrophication of the oceans, and possibly acidification, are causing environmental changes that benefit jellyfish
because they can survive in harsh conditions
D, Scientists believe that some jellyfish species have increased their presence worldwide because they were able to
develop adaptations that allow them to prey on a wider variety of species.
E, New, highly invasive species of jellyfish evolved in areas such as the Black Sea, as well as the Caspian and the
Baltic seas, and this has contributed to the spread of jellyfish worldwide
F, Jellyfish are spreading rapidly to areas where they did not exist before as a result of their increased transport
attached to ships.

The Rise of the Maya

Paragraph 1: A hundred and fifty years of painstaking archaeological inquiry allows us today to understand how
the Maya emerged to transform the rain forest of Central America into a scene of urban civilization. By 1000 B.c. the
Maya were settled agriculturalists growing a variety of crops in clearings in the forest, which they turned into
villages They appear to have lived in a society of equals, without clear rulers or ceremonial centers. Then between
800 and 500 B.C, signs of a ruling elite within Maya society start to emerge in the form of elaborate burial
monuments. At Los Mangales in the Salama Valley of highland Mexico, a chief was buried on a special mortuary
platform accompanied by rich grave goods of jade and shell. Slightly later the ceremonial center at El Portón was
built, involving the construction of earth terraces and platforms, on which were located altars and standing stones.
one with a brief written inscription, although too damaged to be read today.

第 168 页 共
373 页
2021 托福阅读真题

1. The word "painstaking" in the passage is closest in meaning to


A, remarkable
B, very thorough
C, cooperative
D, highly creative

2. According to paragraph 1, which of the following was true of the Maya around 1000 B.C ?
A, They had become a largely urban civilization.
B, They had left the rain forest in order to grow a variety of crops.
C, They were beginning to develop elaborate burial customs
D, They lived in villages in the rain forest.

Paragraph 2: In the lowlands of Guatemala and the Yucatán Peninsula of Mexico, vast ceremonial centers appeared
quite suddenly after 600 B.C. At Nakbe in northern Guatemala, the site was swiftly transformed from a modest
village into a city with a major monumental structure at its heart. A massive platform was constructed on the ruins of
the original settlement, on top of which were a series of terraced buildings up to 60 feet tall. Without any doubt we
can see here the development of a more complex society. From 400 B.C. to Ad 250, major ceremonial centers
developed in all parts of the Maya area, many carved out of the tropical rain forest that covered the southern
lowlands of Guatemala. Belize, and Mexico. These cities were dominated by enormous terraced platforms, some
forming giant temple-pyramids. Vast palaces of limestone masonry with vaulted rooms were constructed. set within
architectural layouts that emphasized the most important buildings of a city, arranged around plazas with rows of
standing stones lined up in front of them. A highly sophisticated art style emerged, seen in bas-reliefs, wall paintings,
and beautiful pottery with multicolored fired decorations. Hieroglyphic writing became widespread, and inscriptions
can be dated using the Maya Long Count, an elaborate but incredibly accurate calendarical system.

3. The word "incredibly" in the passage is closest in meaning to


A, unbelievably
B, apparently
C, completely
D, uncommonly

4. In paragraph 2, why does the author describe the Nakbe site in northern Guatemala?
A, To indicate that earlier settlements were commonly destroyed in order for the Maya to build monumental
structures
B, To illustrate the rapid development of a more complex society after 600 B.C.
C, To demonstrate the steps involved in constructing early Maya ceremonial centers

第 169 页 共
373 页
2021 托福阅读真题

D, To contrast the Nakbe ceremonial center with those developed in other regions

Paragraph 3: Earlier generations of archaeologists considered the great Maya cities to be purely ceremonial centers.
In this view. the cities were occupied only by their peace-loving priestly rulers and their attendants except at great
festivals. The British Mayanist Sir Eric Thompson, working at the Field Museum of Chicago, suggested that the
Maya character encouraged the development of religious authority, and that their devoutness, discipline, and respect
for authority would have facilitated the emergence of a theocracy.

5. It can be inferred from paragraph 3 that the earlier generations of archaeologists believed the great Maya
cities
A, evolved over time from urban settlements to ceremonial centers
B, allowed priests and their attendants to enter only during festivals and ceremonies
C, were theocracies that influenced the development of the Maya character
D, were not designed to be occupied by ordinary city inhabitants

Paragraph 4: This was not, however, an age of peace ruled over by unworldly priests living in solitude among the
temples. In the last 25 years, a succession of remarkable breakthroughs has enabled us to read the Maya's
hieroglyphic language. Whereas Thompson and others assumed that inscriptions outside temples concerned complex
matters of astronomy and calendric systems that fascinated the priests, the translations now available show beyond
any doubt that the cities were ruled by an aristocracy that was firmly secular (not based on religion) and indeed
warlike in outlook. Hieroglyphic writing on monuments was mainly used to record the achievements of Maya rulers,
especially in war. Victory stones were erected outside the temples, bearing the names of famous captives. Great
monuments were liberally marked with the names and faces of the rulers who commissioned them.

6. According to paragraph 4, the discovery of which of the following led archaeologists to change their view of
the Mayas?
A, The true topic of the hieroglyphic writing on monuments
B, The way the Mayas' calendar system actually worked
C, The extent of the Mayas' understanding of astronomical matters
D, The actual names of the rulers of the cities

7. According to paragraph 4. what is true about the inscriptions outside Maya temples?
A, They are primarily concerned with astronomical observations.
B, They establish that the priests were fascinated by calendric systems.
C, They mostly record the military victories of Maya rulers
D, They provide important information about Maya religious beliefs

第 170 页 共
373 页
2021 托福阅读真题

Paragraph 5: Excavation has also played its part in overturning the established picture of Maya cities. Vital
evidence that the cities were not just ceremonial centers has now been found at many lowland sites. On the
outskirts of cities such as El Mirador are groups of low, rectangular mounds of earth and stone long ignored,
but which archaeological investigations have now shown were occupied by small wooden houses, raised above
the level of the summer flooding-humble dwellings that housed the ordinary inhabitants who served the
aristocrats living in palaces at the heart of the city.

8. Which of the sentences below best expresses the essential information in the highlighted sentence in the
passage? Incorrect choices change the meaning in important ways or leave out essential information.
A, Long ignored by archaeological investigations. rectangular mounds outside the cities have been found to contain
the remains of small houses, inhabited by people who attended to the aristocracy
B, Archaeologists have now determined through investigations that the groups of low rectangular mo unds of earth
and stone on the outskirts of cities were raised above the level of summer flooding.
C, Because of summer flooding, the ordinary inhabitants living on the outskirts of El Mirador enclosed their small
wooden houses N inside raised rectangular mounds.
D, Rectangular mounds of earth in El Mirador were ignored until recently, when archaeologists discovered they
contained small wooden houses.

Paragraph 3: █ Earlier generations of archaeologists considered the great Maya cities to be purely ceremonial
centers. █ In this view. the cities were occupied only by their peace-loving priestly rulers and their attendants except
at great festivals. █ The British Mayanist Sir Eric Thompson, working at the Field Museum of Chicago, suggested
that the Maya character encouraged the development of religious authority, and that their devoutness, discipline, and
respect for authority would have facilitated the emergence of a theocracy. █

9. Look at the four squares █ that indicate where the following sentence could be added to the passage
Such impressive achievements clearly show that the Maya had become a civilization, but they do not tell us
about the true nature of the cities
Where would the sentence best fit? Click on a square █ to add the sentence to the passage.

10. Directions: An introductory sentence for a brief summary of the passage is provided below. Complete the
summary by selecting the THREE answer choices that express the most important ideas in the passage. Some
sentences do not belong in the summary because they express ideas that are not presented in the passage or are minor
ideas in the passage. This question is worth 2 points.

Archaeological research has shown how Maya society developed from an agricultural society to an urban one.

A, Archaeologists once believed that before 1000 B.C., the early Maya were simple agriculturalists, but evidence

第 171 页 共
373 页
2021 托福阅读真题

from the Mexican highlands demonstrates that it was a complex society


B, The rapid transformation of simple villages into cities dominated by huge, structurally complex ceremonial
centers strongly suggests that the Maya population had grown enormously by AD. 250.
C, Translations of inscriptions now show that Maya cities were ruled by a secular and warlike elite, and were not
merely ceremonial centers occupied mainly by peaceful priestly rulers.
D, Between 800 and 500 B.C. elaborate burial monuments appeared, indicating the emergence of a ruling elite, and
modest villages were transformed into cities with monumental structures.
E, From 400 B.C. to AD. 250, many major ceremonial centers with huge terraced platforms and palaces appeared;
sophisticated art, accurate calendars, and hieroglyphic writing were widespread.
F, Once the established picture of Maya cities was overturned archaeologists realized that ordinary people lived in
simple dwellings throughout the heart of the cities.

The Agricultural Revolution

Paragraph 1: During a relatively warm climatic period between 8000 and 6000 B.C.. humans on the fringes of
Mesopotamia began to shift from a hunting-gathering existence and started to control the animals and plants they
would eat, thus initiating the Agricultural (or Neolithic Revolution. That it was a revolution there can be no dispute.
It transformed the way human beings lived and shattered a tradition over two million years old. However, why the
Agricultural Revolution occurred at this precise time is still largely a matter of conjecture. Why. for instance, did it
not occur during one of the earlier interglacial periods (intervals of relatively warm climate that occurred
periodically between ice ages over millions of years) when, presumably, the same conditions prevailed? It is difficult
to find any uniformly satisfying answers. We know that agriculture developed more or less simultaneously in many

第 172 页 共
373 页
2021 托福阅读真题

different parts of the globe, so it is unlikely that it resulted from any single cause such as climatic change or
population growth, although both have been offered as explanations. We also know that the move to agriculture was
not always permanently successful. In some places it was tried for a while and then abandoned. It is even possible
that certain plants and animals were domesticated more than once and by different peoples

1. The word "uniformly" in the passage is closest in meaning to


A, consistently
B, logically
C, permanently
D, truly

2. Paragraph 1 supports which of the following statements about the hunting-gathering way of life?
A, Humans practiced it for more than two million years.
B, It was most successful during relatively warm periods
C, It was first replaced by agriculture during an ice age.
D, It made significant population growth impossible

Paragraph 2: Most modern explanations of the origins of agriculture tend to emphasize the role of
microenvironments and long-standing human-plant and human-animal relationships. Such factors as changing
climatic conditions, the presence of animals and plants that offered good potential for domestication, and the cultural
and technological levels of achievement of the human populations present undoubtedly played important roles in the
development of agriculture.

3. According to paragraph 2. all of the following have been proposed as contributing to the Agricultural
Revolution EXCEPT
A, cultural diversity within human populations
B, human relationships with plants and animals
C, changing environmental conditions
D, availability of plants and animals suitable for domestication

Paragraph 3: The key to understanding agriculture is the process known as domestication. Domestication was the
essential technological breakthrough that allowed human beings to escape the age-old system of hunting and
gathering and to control the production of food. rather than being at the mercy of what sustenance the terrain might
offer at any given moment.

Paragraph 4: Domestication can be defined as a primitive form of genetic engineering in which certain plants and
animals are brought under human control, their objectionable characteristics eliminated. their favorable ones

第 173 页 共
373 页
2021 托福阅读真题

enhanced. and in the case of animals. inducing them to reproduce in captivity. If wild animals cannot be induced to
breed in captivity, they cannot be domesticated. Modern domesticated cattle, sheep, and pigs, for example, look only
remotely like their leaner. meaner, and faster-moving ancestors. Domestication is best viewed as the creation of an
artificial environment in which the chosen plants or animals come to exist exclusively. Left alone, domesticated
species either die or revert to their original wild forms. Because herds. farms, orchards. and gardens are permanent,
static entities, once they came into being, the old hunting-gathering forms of social organization had to be replaced

4. In saying that domestication is best seen as the creation of an environment in which the chosen plants or
animals exist "exclusively," the author means that the chosen organisms
A, keep other organisms out of their domesticated environment
B, can live only in the domesticated environment
C, live their whole lives in the domesticated environment
D, spend their lives in a single domesticated environment

5. According to paragraph 4, all of the following are true statements about the domestication of plants and
animals EXCEPT:
A, Domestication encourages characteristics in plants and animals that humans prefer.
B, Domestication can result in plants and animals that look somewhat different from their ancestors
C, Plants and animals can no longer reproduce with their wild counterparts after domestication.
D, The success of animal domestication depends on the animals' ability to reproduce in captivity.

Paragraph 5: Hunter-gatherers place a low value on possessions and a high value on mobility. Always on the move,
they carry only a few tools and weapons with them. Agriculture reverses this way of life. It cannot be practiced
without a commitment to permanence and the accumulation of large amounts of material goods. Homes, villages and
storage facilities must be constructed: fields cleared,divided, and fenced; herds built up and maintained; and tools
fabricated. Constant effort is required to maintain all of these. Once settled, farmers may not move again for
generations. Pastoralists (animal herders) are equally committed to their flocks and herds.

6. According to paragraph 5, why do hunter-gatherers place a low value on possessions?


A, Few goods other than tools or weapons can be of use to them.
B, They lack the resources to build storage facilities.
C, They move often and can carry only small loads of necessary goods.
D, They have a limited ability to defend their possessions against other groups.

7. What is the purpose of paragraph 5 in the passage?


A, To show that the adoption of agriculture required a change in the way people lived
B, To emphasize the various challenges of the agricultural way of life

第 174 页 共
373 页
2021 托福阅读真题

C, To describe the steps involved in growing food and domesticating animals


D, To suggest that humans have a natural commitment to permanence

Paragraph 6: For practical purposes, hunting-gathering bands always remained small, in the range of 30 to 50
people. Larger groups would have been difficult to sustain in most environments; smaller groups could not
reproduce themselves. Agriculture, by contrast knew no limits as far as population growth was concerned. Thus,
where hunting-gathering bands restricted their numbers, agricultural communities tended to expand them. People
could be put to work in the fields or gardens at an early age and at harvest time, when it was essential to maximize
the number of people who could be mobilized. Overpopulation was solved by emigrating and opening up new land
for cultivation. By about 6000 B.C., villages with populations in the thousands were common throughout the Middle
East.

8. According to paragraph 6, which of the following was true of agricultural communities in the Middle East?
A, They became overpopulated when they grew to more than 50 members
B, They experienced their highest population levels at harvest time
C, When they became too crowded. some of their inhabitants moved to new areas.
D, Many of them had become overpopulated by about 6000 B.C

Paragraph 5: █ Hunter-gatherers place a low value on possessions and a high value on mobility. █ Always on the
move, they carry only a few tools and weapons with them. Agriculture reverses this way of life. █ It cannot be
practiced without a commitment to permanence and the accumulation of large amounts of material goods. █ Homes,
villages and storage facilities must be constructed: fields cleared,divided, and fenced; herds built up and maintained;
and tools fabricated. Constant effort is required to maintain all of these. Once settled, farmers may not move again
for generations. Pastoralists (animal herders) are equally committed to their flocks and herds.

9. Look at the four squares █ that indicate where the following sentence could be added to the passage
This is because finding enough food requires them to search through a sufficiently large territory.
Where would the sentence best fit? Click on a square █ to add the sentence to the passage.

10. Directions: An introductory sentence for a brief summary of the passage is provided below. Complete the
summary by selecting the THREE answer choices that express the most important ideas in the passage. Some
sentences do not belong in the summary because they express ideas that are not presented in the passage or are minor
ideas in the passage. This question is worth 2 points.

Agriculture replaced hunting and gathering between 8000 and 6000 B.C.

A, Glacial periods that followed the agricultural revolution forced agricultural peoples in some parts of the world to

第 175 页 共
373 页
2021 托福阅读真题

go back to hunting and gathering


B, Domestication allowed people to control their food production by bringing certain plants and animals under
control and enhancing their characteristics to benefit humans
C, In contrast to hunting and gathering, the agricultural way of life favored population expansion because there was
field work for many people, especially at harvest time.
D, Hunting and gathering groups lacked a system of social organization sufficient to exploit the food and other
resources that the environment could provide.
E, The agricultural way of life required permanence and the accumulation of material goods, so humans who
adopted agriculture abandoned hunting and gathering.
F, Although various explanations have been proposed for the beginning of the agricultural revolution, scientists agree
that the best explanation is climate change.

In the Darkest Depths

Paragraph 1: Although scientists in the early 1800s were unable to explore the deepest regions of the ocean, they
Knew that sunlight could not penetrate depths greater than 275 meters Without sunlight, there can be no
photosynthesis(the process by which plants convert sunlight into energy), and therefore no plants or algae to serve as
food for other organisms. Since ocean depths can exceed 10, 000 meters, scientists hypothesized that the deepest
areas of the ocean could not support life.

1, According to paragraph 1, all of the following are true of the deepest ocean regions EXCEPT

第 176 页 共
373 页
2021 托福阅读真题

A, They could not be studied in the early 1800S


B, Photosynthesis cannot take place there
C, Scientists in the early 1800s hypothesized that a source of food other than plants or algae existed there
D, They extend to depths greater than 10,000 meters

Paragraph 2: Throughout the nineteenth century,scientists exploring the ocean collected organisms from ever
greater ocean depths. In an 1873 expedition, researchers aboard the ship HMS Challenger dragged an open -sided
box suspended from the ship across the floor of the Atlantic Ocean. This box-known as a dredge-sample the sea floor
in different parts of the ocean at depths of up to 4,57 meters. The scientists were astonished to discover nearly 5,000
previously unknown Species. When it became clear that life flourished at depths beyond the penetration of light,
scientists were forced to reject their earlier hypothesis that no life existed in the deep ocean waters. After discovering
this rich abundance of deep sea life.scientists were faced with the need to understand how it could exist. The lack of
light suggested that deep-sea organisms were somehow sustained by energy that did not come from photosynthesis
on the ocean floor. Scientists had observed that the surface waters of the ocean produced a steady descent of tiny
particles that were produced by the death and decomposition of organisms living there. These particles are known
as"marine snow. Large organisms such as whales also occasionally died and fell to the ocean floor. Scientists
hypothesized that marine snow and the remains of large organisms provided the energy needed to sustain organisms
in the depths of the ocean.

2, In paragraph 2, the author reports the findings of the 1873 expedition of the HMS Challenger in order to
A, demonstrate the creativity involved in the invention of the dredge or studying the ocean floor
B, help explain why scientists no longer accepted the hypothesis at deep ocean waters were unable to support life
C, document the success of nineteenth-century explorations of the ocean's depths
D, establish the fact that the ocean floor was at least 4, 572 meters deep

3, It can be inferred from paragraph 2 that the discovery of abundant deep-sea organisms led scientists to consider
which of the following questions?
A, How were deep-sea organisms able to reach the surface waters of the ocean?
B, Was there enough light on the ocean floor for photosynthesis to occur?
C, What caused "marine snow" to develop in the surface waters of the ocean?
D, What was the source of energy on the ocean floor?

Paragraph 3: In the 1970s, scientists were finally able to send submersibles-small manned submarines-to take a
first-hand look at the deepest ocean areas. Their discoveries were striking. They not only confirmed that much of the
ocean floor supported living organisms but that areas near openings in the floor of the ocean, which later came to be
known as hydrothermal Vents, contained a great diversity of deep-sea species. Hydrothermal vents release plumes of
hot water with high concentrations of sulfur compounds and other mineral nutrients. A tremendous diversity of

第 177 页 共
373 页
2021 托福阅读真题

species surrounded these hydrothermal vents. Indeed, the total amount of life at these depths rivaled that seen in
some of the most diverse places on Earth. It became clear that the amount of energy contained in organic matter
resting on the sea floor was not sufficient to support such a diverse and abundant set of life forms. That earlier
hypothesis now had to be rejected.

4, According to paragraph 3,all of the following are true about hydrothermal vents EXCEPT
A, They were discovered in the 1970s when scientists were able to send submersibles to the deepest areas of the
ocean
B, They increase species diversity by releasing plumes that bring surface organisms to the ocean floor
C, They supply various nutrients to the surrounding water
D, They have as many different species of organisms as are found in the most diverse places on Earth

Paragraph 4: How could so much life exist at the bottom of the ocean? That this life existed near the hydrothermal
vents suggested that the vents were somehow responsible. Scientists had known for a long time that some species of
bacteria could obtain their energy from chemicals rather than from the Sun. The bacteria use the energy in
chemical bonds, combined with carbon dioxide (CO2), to produce organic compounds--a process known as
chemosynthesis--similar to the way that plants and algae use the energy of the Sun and CO2, to produce
organic compounds through photosynthesis. Based on this knowledge, scientists hypothesized that the hot vents,
which release water with dissolved hydrogen sulfide gas and other chemicals, provided a source of energy for
bacteria and that these bacteria could be consumed by the other organisms living around the vents.

5, Which of the sentences below best expresses the essential information in the highlighted sentence in the passage?
Incorrect choices change the meaning in important ways or leave out essential information.
A, Organic compounds are produced in both chemosynthesis and photosynthesis, making the bacteria similar to
plants and algae in the way they use carbon dioxide
B, Carbon dioxide and the Suns energy are combined in the process of photosynthesis, as well as in a process known
as chemosynthesis, to produce chemical compounds
C, Much like plants and algae use the Suns energy in photosynthesis, the bacteria can produce organic compounds
from the energy in chemical compounds through chemosynthesis
D, The bacteria, like plants and algae, require organic compounds that are formed when carbon dioxide and energy
are combined n either chemosynthesis or photosynthesis

6, According to paragraph 4, the information that there were species of bacteria that obtained energy from chemicals
was important because
A, it helped explain how hot vents could support organisms living near these openings
B, it provided a clearer understanding of how photosynthesis occurs in plants and algae
C, it demonstrated that plants, algae, and bacteria were all similar and related to one anther

第 178 页 共
373 页
2021 托福阅读真题

D, it established the role of carbon dioxide in both of the processes of chemosynthesis and photosynthesis

Paragraph 5: After several years of investigations, scientists found that the immediate area around the hot vents
contained a group of organisms known as tube worms. These animals have no digestive system, but possess
specialized organs that house vast numbers of bacteria that live in a symbiotic(mutually beneficial)relationship with
the tube worms. The tube worms capture the sulfide gases and CO2 from the surrounding water and pass these
compounds to the bacteria,which then use the sulfide gases and CO2 to produce organic compounds. Some of these
organic compounds are passed o the tube worms, which use them as food. These bacteria also represent a food
source for many of the other animals that live near the vents. In turn, these bacteria-consuming animals can be
consumed by larger animals.

7, The word immediate in the passage is closest in meaning to


A, nearest
B, active
C, visible
D, large

8, According to paragraph 5,which of the following happens in tube worms specialized organs?
A, The tube worms destroy vast numbers of bacteria
B, The tube worms capture smaller animals for food
C, Bacteria produce organic compounds from sulfide gases
D, Sulfide gases are produced with the help of CO2 that indicate

Paragraph 2: Throughout the nineteenth century,scientists exploring the ocean collected organisms from ever
greater ocean depths. In an 1873 expedition, researchers aboard the ship HMS Challenger dragged an open -sided
box suspended from the ship across the floor of the Atlantic Ocean. This box-known as a dredge-sample the sea floor
in different parts of the ocean at depths of up to 4,57 meters. The scientists were astonished to discover nearly 5,000
previously unknown Species. When it became clear that life flourished at depths beyond the penetration of light,
scientists were forced to reject their earlier hypothesis that no life existed in the deep ocean waters. After discovering
this rich abundance of deep sea life.scientists were faced with the need to understand how it could exist. The lack of
light suggested that deep-sea organisms were somehow sustained by energy that did not come from photosynthesis
on the ocean floor. ■Scientists had observed that the surface waters of the ocean produced a steady descent of tiny
particles that were produced by the death and decomposition of organisms living there. ■These particles are known
as"marine snow. ■Large organisms such as whales also occasionally died and fell to the ocean floor. ■Scientists
hypothesized that marine snow and the remains of large organisms provided the energy needed to sustain organisms
in the depths of the ocean.

第 179 页 共
373 页
2021 托福阅读真题

9, Look at the four squares where the following sentence could be added to the passage.
But these were not the only source of organic material.
Where would the sentence best fit? Click on a square ■ to add the sentence to the passage.

10.Directions: An introductory sentence for a brief summary of the passage is provided below. Complete the
summary by selecting the THREE answer choices that express the most important ideas in the passage. Some
sentences do not belong in the summary because they express ideas that are not presented in the passage or are minor
ideas in the passage. This question is worth 2 points.

A, Bacteria that perform chemosynthesis near hydrothermal vents provide food for animals, some of which
themselves become food for other animals.
B, Explorations of the ocean floor revealed an abundance of species at great depths that scientists at first believed
were sustained by decomposing organisms that fell to the ocean floor.
C, After the explorations of the 1970s, scientists rejected their earlier hypothesis and theorized that hydrothermal
vents could be a source of energy for deep-sea organisms.
D, Scientists in the early 1800s tried to understand how plants and algae could survive at depths below 275 meters,
but it was not until later in that century that explorations provided answers.
E,Because of the almost complete absence of sunlight at the ocean bottom, bacteria pose a greater danger to
organisms there than they do to organisms living near the surface.
F, The use of submersibles showed that the most species-diverse places on Earth are at the bottom of the ocean
where minerals and other chemicals are present in higher concentrations than on land.

Early Civilizations

Paragraph 1: Beginning around 3500 B.c., an urban revolution occurred independently in multiple centers around
the world. The same remarkable patterns of settlements coalescing into centralized kingdoms based on intensified
agriculture occurred at least six times at six different sites around the globe during the Neolithio period: in
Mesopotamia after 3500 B.C., in Egypt after 3400 B.C.,in the Indus River valley after 2500 B.c., along the Yellow
River in China after 1800 B.C.in Central America at about 500 B.C.and in South America after 300 B. The origin
and development of these civilizations were essentially independent and not the result of diffusion (or spreading)
from a single center, and hence they are known as the pristine civilizations.

第 180 页 共
373 页
2021 托福阅读真题

1, According to paragraph 1, the concept of pristine civilizations can be applied to the civilizations of Mesopotamia,
Egypt, China, the Indus River valley, and the Americas because those civilizations
A developed independently of each other
B were all the result of diffusion from a single central civilization
C each improved on earlier patterns of intensified agriculture
D each sought to establish a centralized kingdom that could produce an urban revolution

Paragraph 2: Why did civilizations arise independently and repeatedly on a. worldwide scale after the fourth
millennium B.C. in those particular sites? Several explanations have been proposed. The precise processes involved
in the leap to civilization are research questions actively debated by archaeologists and anthropologists, but
many scholars emphasize the importance of hydrology (the study of water on Earth and in the atmosphere)
and the physical environment, and they recognize that intensified agriculture, made possible by large-scale
hydraulic-engineering irrigation) projects, was a key element in the formation of large, highly centralized
bureaucratic states. The fact alone that pristine civilizations arose in hydrologically distressed regions--that is,
where too little or too much water required hydraulic engineering for the successful practice of intensified
agriculture--gives credence to what is called the hydraulic hypothesis.

2, Which of the sentences below best expresses the essential information in the highlighted sentence in the passage?
Incorrect choices change the meaning in important ways or leave out essential information
A.Although scholars are still debating details related to the formation of civilizations, many stress the importance of
hydrology and the critical role of intensified agriculture.
B. Archaeologists and anthropologists do not yet understand the relationship between the formation of large
centralized states and intensified agriculture.
C. Large-scale hydraulic projects were made possible by the highly centralized bureaucracies that accompanied the
rise of civilizations.
D. Research into the rise of intensified agriculture has shown that the study of hydrology and the physical
environment were key to successful hydraulic-engineering projects.
Paragraph 3: The hydraulic hypothesis links the rise of civilization to the development of technology for large-scale
hydraulic systems Under a hot, semitropical sun, irrigation agriculture is extraordinarily productive, and a yield that
can support large populations becomes possible. Rivers carrying sediment provide water for irrigation and,
especially when controlled artificially, they enrich the soils around them. Irrigation agriculture and flood control
required hydraulic-engineering systems and some level of communal action to build and maintain them and to
distribute water when and where needed: marshes had to be drained; dams, dikes, canals, terraces, and embankments
had to be built; and ditches had to be kept free of debris. Water disputes had to be settled by some authority, and
grain surpluses had to be stored, guarded, and redistributed. The interacting effects of the geographical setting and
the techniques of hydraulic agriculture reinforced trends toward an authoritarian state.

第 181 页 共
373 页
2021 托福阅读真题

3, The word "reinforced" in the passage is closest in meaning to


A reflected
B produced
C strengthened
D were based in

4, Paragraph 2 supports which of the following statements about the hydraulic hypothesis?
A.The hypothesis explains how the rise of civilizations resulted in intensified agriculture and hydraulic engineering
B. The hypothesis focuses on how dramatic changes in the physical environment after the fourth millennium
required more intensified agriculture.
C. Although the hypothesis does not explain the rise of all civilizations worldwide. it does explain the development
of certain important civilizations after the fourth millennium.
D. While the hypothesis has not been accepted by all scholars, the rise of pristine civilizations in hydrologically
distressed areas provides support for it

5, According to paragraph 3, which of the following is true about irrigation agriculture


A. It does not require that people work together in groups.
B. It is easy to maintain once a system has been established.
C. It is less successful in hot, semitropical climates.
D. It produces sufficient food to support large populations.

Paragraph 4: That model described above admirably fits what happened with the first human civilization arising on
the floodplain between the Tigris and Euphrates Rivers in what is present-day Iraq. This was ancient Mesopotamia,
the land "between the rivers."By 4000 B.C.Neolithic villages filled the Mesopotamian plain. Local authorities
drained marshes and installed extensive irrigation works on the flood plain. Great walled cities arose after 3500
B.C.,and the Sumerian dynasty developed fully by 2500 B.C.

6, In paragraph 4, why does the author mention the area between the Tigris and Euphrates rivers?
A. To show that civilizations sometimes developed even in places where irrigation systems were not needed
B. To give an example of a place where the rise of civilization can be explained by the hydraulic hypothesis
C. To identify the location of the Mesopotamian villages that eventually became part of Sumerian civilization
D. To disprove the argument that floodplains are unlikely locations for the development of civilizations

Paragraph 5: The notion of "environmental circumscription" provides a key explanation for this rise in urban
development. Environmental circumscription occurs when an area of land is surrounded by a less productive area
such as mountains, desert, or sea. Civilizations arose in prehistoric river valleys and floodplains that were

第 182 页 共
373 页
2021 托福阅读真题

environmentally restricted agricultural zones beyond which intensive farming was impossible or impractical. In
these constricted habitats, like the Nile River valley, expanding Neolithic populations soon pressed against the limits
imposed by the surrounding desert, waterfalls, and sea, leading to pressures to intensify food production. Warfare
became chronic and developed beyond mere raiding(attack and retreat)to involve conquest and subjugation since, in
a habitat already filled, the defeated peoples could no longer go off and form a new agricultural community. Whereas
previously, defeated groups could generally move on to a new locale, in environmentally restricted areas such as the
Nile River valley, agricultural communities had nowhere to go.

7, Paragraph 5 supports which of the following explanations of the idea of "environmental circumscription"?
A. If the environment was unfavorable farmers moved to another location.
B. Agriculture that began in prehistoric river valleys and floodplains was adopted by farmers in nearby areas as well
C. Urban civilizations could not develop in areas that were surrounded by harsh environments.
D. Food production became most intensified in areas surrounded by land that could not support agriculture

Paragraph 6: Victors not only took over land and smaller irrigation works but subjugated and dominated conquered
groups, sparing their lives in return for their labor as slaves and peasants in maintaining systems of intensified
farming. Once this process started, the historical momentum favoring unifying and centralizing forces was
irreversible. Neolithic communities thus became increasingly stratified, culminating in a dominant elite in command
of are agricultural underclass as regional powers subsumed local ones.Time and again civilization and the state
emerged wherever these environmental and demographic conditions occurred.

8, According to paragraphs 5 and 6, all of the following resulted from environmental circumscription EXCEPT
A. warfare and conquest
B. extensive migration of defeated peoples
C. conquered slaves and laboring peasants
D. the creation of stratified social classes

Paragraph 3: The hydraulic hypothesis links the rise of civilization to the development of technology for large-scale
hydraulic systems. ■Under a hot, semitropical sun, irrigation agriculture is extraordinarily productive, and a yield
that can support large populations becomes possible. ■Rivers carrying sediment provide water for irrigation and,
especially when controlled artificially, they enrich the soils around them. ■Irrigation agriculture and flood control
required hydraulic-engineering systems and some level of communal action to build and maintain them and to
distribute water when and where needed: marshes had to be drained; dams, dikes, canals, terraces, and embankments
had to be built; and ditches had to be kept free of debris. Water disputes had to be settled by some authority, and
grain surpluses had to be stored, guarded, and redistributed. ■The interacting effects of the geographical setting and
the techniques of hydraulic agriculture reinforced trends toward an authoritarian state.

第 183 页 共
373 页
2021 托福阅读真题

9, Look at the four squares ■that indicate where the following sentence could be added to the passage.
Despite its many advantages, irrigation agriculture placed new demands on the societies practicing it.
Where would the sentence best fit? Click on a square ■ to add the sentence to the passage.

10.Directions: An introductory sentence for a brief summary of the passage is provided below. Complete the
summary by selecting the THREE answer choices that express the most important ideas in the passage. Some
sentences do not belong in the summary because they express ideas that are not presented in the passage or are minor
ideas in the passage. This question is worth 2 points.

A One proposed explanation for the rapid rise of civilizations after 3500 B.C. is that the first civilization in
Mesopotamia quickly spread into other areas of the globe.
B Scholars believe that centralized civilizations, like ancient Mesopotamia, developed independently out of the need
for an authoritarian government to manage the expansion of agriculture.
C Population tended to concentrate in geographically restricted areas, leading to continual warfare over the scarce
land favorable to agriculture.
D Neolithic settlements that were located near rivers are not generally considered to be true pristine civilizations
E The great walled cities that arose in Mesopotamia became part of the Sumerian dynasty, but irrigation systems
remained in the control of local authorities.
F Geographical limits on appropriate agricultural sites led to a stratified society, in which a dominant elite class ruled
a working class of conquered slaves and peasants.

Cotton Ginning and Interchangeable Parts: The Legacy of Eli Whitney

Paragraph 1: Cotton is a soft fiber that grows around the seeds of the cotton plant. It was first cultivated in the
Indus Valley about 6,000 years ago and then spread to Egypt, Persia, China, and the Mediterranean. For thousands of
years, the cultivation of cotton was very labor-intensive. Many steps were involved in preparing cotton to be used in
a garment, and many machines were invented to make cotton more valuable as a general-purpose textile material.

1, The word "garment" in the passage is closest in meaning to


A, weaving device

第 184 页 共
373 页
2021 托福阅读真题

B, article of clothing
C, farming method
D, manufacturing process

Paragraph 2: The process of separating cotton fibers from seeds is called ginning, which is depicted as far back as
the fifth century in paintings in the Ajanta Caves in India. Egyptian cotton has the longest fibers and is used to make
premium fabrics. In the American South, the high quality long-staple cotton can be grown only in a narrow band
along the Carolina and Georgia coast and is called Sea Island cotton. The interior areas of the American South can
only grow Upland short-staple cotton. The shorter-length fibers mean reduced yarn and cloth quality, and they have
"fuzzy"seeds because the fibers are tightly attached to the entire seed surface This strong attachment of fibers to
seeds makes it difficult to remove the fibers without damage, which made the process expensive as it was
time-consuming and labor-intensive. These time-consuming tasks were done by slaves in the American southern
states during the 1800s.

2, Which of the following can be inferred from paragraph 2 about cotton grown in the American South?
A, The cotton grown in the American South is of higher quality than Egyptian cotton.
B, Both long-and short-staple cotton can be grown successfully throughout the American South
C, Most areas of the American South are able to produce only low quality cotton.
D, The American South produced only small amounts of cotton during the 1800s.

3, According to paragraph 2, why are short-staple cotton varieties difficult to work with?
A, They produce only small amounts of fiber.
B, Their fibers are tightly attached to the seeds, making them hard to remove.
C, Shorter-length fibers are easily damaged during cloth manufacture.
D, The fibers are especially fuzzy and rough, making them hard to handle

Paragraph 3: The modern cotton gin was invented by the American Eli Whitney (1765-1825), who graduated
from Yale University in 1789 and is known for two great inventions: the cotton gin, which transformed the
southern economy, and interchangeable parts in machinery, which led to the mass-production method of
manufacturing. When he graduated, he was short of funds and went to Georgia to seek his fortune in plantations.
He was introduced to the problem of finding a better way to gin the Upland short-fiber cotton, which led him to
invent the cotton gin in 1794. Whitney's cotton gin made the mechanical separation of fiber from seeds much more
efficient. His machine had spiked sawteeth mounted on a revolving cylinder in a box, which was turned by a crank.
The teeth pulled the cotton fiber through small slotted openings that were too small for the seeds to pass; thus, the
lint was separated from the seeds. Then a rotating brush removed the fibrous lint from the teeth, and the seeds fell
into a hopper. Whitney's machine could separate up to 50 pounds of cleaned cotton daily, making cotton production
profitable for the southern states.

第 185 页 共
373 页
2021 托福阅读真题

4, Which of the following expresses the essential information in the highlighted sentence in the passage?Incorrect
choices change the meaning in important ways or leave out essential information.
A, The American Eli Whitney became an inventor after graduating from Yale University, which was famous for the
inventions produced by its students, such as the cotton gin and interchangeable parts in machinery
B, Eli Whitney, who graduated from Yale University, invented the modern cotton gin, a machine that changed the
southern economy by introducing the idea of interchangeable parts
C, The economy of the South was transformed by the American Eli Whitney's famous invention of the cotton gin as
well as by his new method of producing interchangeable machinery parts
D,The American Eli Whitney is best known for inventing the modern cotton gin and interchangeable parts for
machinery, each of which had large economic impacts

5,According to paragraph 3, Whitney's move to Georgia affected his later professional life by
A, allowing Whitney to make enough money to pursue his new inventions
B, exposing Whitney to the problems with processing short-fiber cotton,which prompted him to invent the cotton gin
C, introducing Whitney to new types of machinery used in cotton ginning
D, showing Whitney that his cotton gin could be more useful in some regions than in others

Paragraph 4: Unfortunately for Whitney, his cotton gin was extremely simple and easy to copy. so he failed to
profit from his invention. Soon after, imitations of his machine appeared, and his 1794 patent for the cotton gin could
not be upheld in court until 1807. Instead, he decided to make money by going into the ginning business, and
manufactured and installed cotton gins through Georgia and the southern states. However, his ginning plants were
resented by the planters, who thought that he charged too much, and hence they found ways to circumvent his patent.

6, The word "installed" in the passage is closest in meaning to


A, repaired
B, sold
C, put in place
D, improved

7, According to paragraph 4. Whitney was unable to profit from his invention of the cotton gin because
A, other inventors quickly fled patents for similar machines
B, he faced heavy competition from other ginning businesses
C, his ginning plants were too large to be practical for most farmer
D, it was easy for people to copy his invention on their own

Paragraph 5: In later years, Eli Whitney cultivated social and political connections through his status as a Yale

第 186 页 共
373 页
2021 托福阅读真题

alumnus and through his marriage. Although he had never made a gun in his life, through those connections he won
a contract from the War Department in 1798 to make and deliver 10,000 muskets(firearms used by soldiers before
the invention of the rifle)for the army in 1800 Treasury Secretary Wolcott sent him a "foreign pamphlet on arms
manufacturing techniques." and Whitney began to talk about interchangeable parts. This concept was a departure
from the usual method of having an individual master machinist make each part of the musket and ft them together,
which is slow and painstaking Whitney's method was to have many less skilled machinists specialize in making only
one part per person to highly specified dimensions, so that the parts would be interchangeable and could then be
assembled. This is the heart of the mass-production method later made more famous by Henry Ford in the
manufacture of the Model T automobile. Whitney ultimately delivered on the army contract in 1809, and he spent
the rest of his life promoting interchangeability.

8, In paragraph 5, what can be inferred about Whitney's persona connections?


A, Whitney's business success in later life resulted in part from those connections.
B, Whitney was employed inside the War Department in the late 1700s because of those connections.
C, Whitney established business contracts with foreign arms manufacturers through those connections
D, Whitney's designs for a new manufacturing method were made famous by those connections.

Paragraph 2: The process of separating cotton fibers from seeds is called ginning, which is depicted as far back as
the fifth century in paintings in the Ajanta Caves in India.■Egyptian cotton has the longest fibers and is used to make
premium fabrics. ■In the American South, the high quality long-staple cotton can be grown only in a narrow band
along the Carolina and Georgia coast and is called Sea Island cotton. ■The interior areas of the American South can
only grow Upland short-staple cotton. ■The shorter-length fibers mean reduced yarn and cloth quality, and they have
"fuzzy" seeds because the fibers are tightly attached to the entire seed surface This strong attachment of fibers to
seeds makes it difficult to remove the fibers without damage, which made the process expensive as it was
time-consuming and labor-intensive. These time-consuming tasks were done by slaves in the American southern
states during the 180Os.

9, Look at the four squares that indicate where the following sentence could be added to the passage.
This is a less desirable variant of the plant.
Where would the sentence best fit? Click on a square to add the sentence to the passage.

10.Directions: An introductory sentence for a brief summary of the passage is provided below. Complete the
summary by selecting the THREE answer choices that express the most important ideas in the passage. Some
sentences do not belong in the summary because they express ideas that are not presented in the passage or are minor
ideas in the passage. This question is worth 2 points.

The inventions of Eli Whitney greatly influenced the development of industry.

第 187 页 共
373 页
2021 托福阅读真题

A, The process of turning raw cotton into garments is thought to have first developed in ancient Egypt, where cotton
has the longest fibers
B, Over the years, Whitney improved his cotton gin, and though many copied this invention, no imitator was able to
equal the quality of Whitney's gin or his success in selling gins
C, Cotton has been cultivated in many areas of the world for thousands of years, but processing it was extremely
labor-intensive in most areas before machine processing became available.
D, Whitney's invention of a machine for ginning cotton allowed farmers to process cotton mechanically. a technique
far more efficient that traditional methods
E, Whitney developed a mass production method for manufacturing weapons that replaced the slow and painstaking
method of work previously performed by individual machinists
F, Whitney's manufacturing innovations were later patented by Henry Ford, who applied a much-altered form of
mass production to the manufacture of automobiles.

The Advent of Printing

Paragraph 1: The invention of movable type in Germany around 1450 was one of the most important developments
of the early modern era in Europe. By 1480 printing presses had been successfully established in all the major cities
of Germany, France, the Netherlands, England, Spain, Hungary, and Poland. It has been estimated that by 1500 these
presses had printed between 6 and 15 million books in 40,000 different editions, and the production of books
increased exponentially thereafter.

第 188 页 共
373 页
2021 托福阅读真题

1. Paragraph 1 answers which of the following questions about European printing?


A.In what major European city did commercial printing originate?
B.How many books had been printed in Germany by 1480?
C.In what country was printing invented?
D.In what European country was the largest number of books printed by 1500?

Paragraph 2: The consequence of this massive dissemination of print was a revolution of knowledge and
communication that affected society from top to bottom. The speed with which books were distributed and the
quantity of books suggest that print cultivated new communities of readers eager to consume the diverse material
that rolled off the presses. The accessibility and relatively low cost of printed books also meant that more people
than ever before had access to books. Printing was a profitable business. It responded to public demand, and the
success and wealth of the great printing houses of Aldus Manutius and Nicolas Jenson in Venice, William Caxton in
London, and Christophe Plantin in Antwerp suggest that this demand was substantial. As more people spoke and
wrote in the European vernacular languages(the languages of everyday use)—German, French, Italian,
Spanish, and English—the printing presses increasingly published in these languages rather than in Latin and
Greek, which were used by scholars and professionals and thus appealed to a smaller audience. Vernacular
languages were gradually standardized and became the primary means of legal, political, and literary communication
in most European states. This encouraged the rise of national consciousness. The mass of printed books in everyday
languages contributed to the image of a national community among those who shared a common vernacular. Over
the centuries this would ultimately lead to individuals defining themselves in relation to a nation rather than a
religion or a ruler, a situation which had profound consequences for religious authority.

2.The word "cultivated" in the passage is closest in meaning to


A.encouraged the growth of
B.require
C.reached
D.brought together

3.Paragraph 2 implies which of the following about the economics of early printing?
A.Expensive scholarly and professional books were the first to be printed in the vernacular languages.
B.The wealth of printing houses depended on selling large numbers of books rather than making a large profit on
each book sold.
C.The early success of the printing industry followed from the fact that when printing developed Europe already had
a large and highly educated population.
D.A few great publishing houses were able to establish an international presence and so dominate the trade in books
throughout Europe.

第 189 页 共
373 页
2021 托福阅读真题

4.Which of the sentences below best expresses the essential information in the highlighted sentence in the passage?
Incorrect choices change the meaning in important ways or leave out essential information.
A.European printing presses that published in vernacular languages rather than in Latin and Greek tended to appeal
only to a small group of scholars and professionals.
B.As the use of European vernacular languages rose, printing presses began to publish more in these languages than
in scholarly languages such as Latin and Greek.
C.As the people who worked in European presses gradually changed from those who spoke Latin and Greek to those
who spoke vernacular languages, printing presses began publishing in those vernacular languages.
D.As more people spoke and wrote European vernacular languages, and printing presses increasingly published in
these languages, the number of scholars and professionals of Latin and Greek gradually decreased.

5.According to paragraph 2, what effect did the publication of books in the vernacular have on Europeans?
A.While non-European authors wrote in the vernacular, European scholars continued to write in Latin and Greek.
B.Europeans began learning to speak and write in the vernacular languages of countries other than their own.
C.Two types of vernacular languages developed, one for popular use and another for legal, political, and literary
communication.
D.Europeans became more aware of their nationalities and began to define themselves by them.

Paragraph 3: Printing permeated every area of public and private life.Initially presses issued religious books, but
gradually, more secular books like romances, travel narratives, pamphlets, broadsheets, and conduct books advising
people on everything from medicine to wifely duties were introduced. By the 1530s, printed pamphlets sold for the
same price as a loaf of bread. A culture based on communication through listening, looking, and speaking gradually
changed to a culture that interacted through reading and writing. Rather than being focused on courts or churches, a
literary culture began to emerge around the semi-autonomous printing press. Its agenda was set by demand and profit
rather than religious orthodoxy or political ideology. Printing houses turned intellectual and cultural creativity into a
collaborative venture as printers, merchants, teachers, scribes, translators, artists, and writers all pooled their skills
and resources in creating the finished product. One print historian has compared the late fifteenth-century printing
press of the Italian printer Aldus Manutius to a factory, boardinghouse, and research institute all in one. Presses
like Manutius' created an international community of printers, financiers, and writers as opportunities for expansion
into new markets emerged.

6.According to paragraph 3, which of the following determined what a printing press published?
A.Religious beliefs
B.Political views
C.Commercial demand
D.The courts

第 190 页 共
373 页
2021 托福阅读真题

7.The author provides information about "the late fifteenth-century printing press of the Italian printer Aldus
Manutius" in order to
A.emphasize the degree to which the work of printers was controlled by craftspeople and other specialists
B.demonstrate the diverse range of functions that took place at printing presses
C.suggest the importance that printers placed on intellectual and cultural creativity
D.help explain how Italy became an international center for publishing

Paragraph 4: Print also transformed how knowledge itself was understood and transmitted. A manuscript
(hand-written document) is a unique and reproducible object, however brilliant its copyist. Print, however, with its
standard format and type, introduced exact mass reproduction. This meant that two readers separated by distance
could discuss and compare identical books, right down to a specific word on a particular page. With the introduction
of consistent pagination, indexes, alphabetic ordering, and bibliographies (all unthinkable in manuscripts),
knowledge itself was slowly repackaged. Textual scholarship became a cumulative science as scholars could now
gather manuscripts of, say, the Politics of the Greek philosopher Aristotle and print a standard authoritative edition
based on a comparison of all available copies. This led to the phenomenon of new and revised editions. Publishers
realized the possibility of incorporating discoveries and corrections into the collected works of an author. As well as
being intellectually rigorous, this was also commercially very profitable as individuals could be encouraged to buy a
new version of a book they already possessed. Pioneering reference books and encyclopedias on subjects like
language and law aimed to reclassify knowledge according to new methodologies of alphabetical and chronological
order.

8.Paragraph 4 mentions all of the following as advantages of printed books over manuscripts EXCEPT:
A.Printed books could be produced in large numbers.
B. Printed books could be revised as necessary.
C..The individual pages of printed books could be uniformly presented.
D.It was easier to find authors for printed books.

Paragraph 3: Printing permeated every area of public and private life.Initially presses issued religious books, but
gradually, more secular books like romances, travel narratives, pamphlets, broadsheets, and conduct books advising
people on everything from medicine to wifely duties were introduced. By the 1530s, printed pamphlets sold for the
same price as a loaf of bread. A culture based on communication through listening, looking, and speaking gradually
changed to a culture that interacted through reading and writing. Rather than being focused on courts or churches, a
literary culture began to emerge around the semi-autonomous printing press. Its agenda was set by demand and profit
rather than religious orthodoxy or political ideology. Printing houses turned intellectual and cultural creativity into a
collaborative venture as printers, merchants, teachers, scribes, translators, artists, and writers all pooled their skills
and resources in creating the finished product. One print historian has compared the late fifteenth-century printing
press of the Italian printer Aldus Manutius to a factory, boardinghouse, and research institute all in one. ■Presses like

第 191 页 共
373 页
2021 托福阅读真题

Manutius' created an international community of printers, financiers, and writers as opportunities for expansion into
new markets emerged.

Paragraph 4: ■Print also transformed how knowledge itself was understood and transmitted. ■A manuscript
(hand-written document) is a unique and reproducible object, however brilliant its copyist. ■Print, however, with its
standard format and type, introduced exact mass reproduction. This meant that two readers separated by distance
could discuss and compare identical books, right down to a specific word on a particular page. With the introduction
of consistent pagination, indexes, alphabetic ordering, and bibliographies (all unthinkable in manuscripts),
knowledge itself was slowly repackaged. Textual scholarship became a cumulative science as scholars could now
gather manuscripts of, say, the Politics of the Greek philosopher Aristotle and print a standard authoritative edition
based on a comparison of all available copies. This led to the phenomenon of new and revised editions. Publishers
realized the possibility of incorporating discoveries and corrections into the collected works of an author. As well as
being intellectually rigorous, this was also commercially very profitable as individuals could be encouraged to buy a
new version of a book they already possessed. Pioneering reference books and encyclopedias on subjects like
language and law aimed to reclassify knowledge according to new methodologies of alphabetical and chronological
order.

9. Look at the four squares █ that indicate where the following sentence could be added to the passage.
But the contributions of print went beyond the creation of communities and the opening of new markets.
Where would the sentence best fit? Click on a square █ to add the sentence to the passage.

10.Directions: An introductory sentence tor a brief summary of the passage is provided below. Complete the
summary by selecting the 3 answer choices that express the most important ideas in the passage. Some sentences do
not belong in the summary because they express ideas that are not presented in the passage or are minor ideas in the
passage. This question is worth 2 points.

Printing with movable type, invented in 1450, soon influenced every area of European public and private life.

A.By 1480, printing with movable type was established in major cities of all European countries except Italy, and
had produced millions of books in thousands of editions.
B.Printing in vernacular languages brought about the decline of Latin and Greek in scholarly writing and led readers
to define themselves in terms of the vernacular languages they read.
C.Printing aided scholarship by making it possible to create authoritative texts with reliable pagination, indexes, and
bibliographies that were identical in each printed copy.
D.Printing made large numbers of affordable books available on topics that reflected customer demand rather than
the interests of institutions such as the church.
E.Printed pamphlets on religious topics cost no more than a loaf of bread, but printing was profitable because advice

第 192 页 共
373 页
2021 托福阅读真题

books and romances about peoples' private lives were more expensive.
F.The printing business encouraged the development of a literary culture that brought together printers, financiers,
artists, and authors in the collaborative process of producing printed books.

The Fur Trade and Native Americans

Paragraph 1: Soon after Europeans made contact with the aboriginal peoples of North America, known as Native
Americans, trading became the focus of European concern. French, British, and Dutch merchants sought animal furs,
especially beaver skins, which were in high demand between the sixteenth and nineteenth centuries in Europe for
making men' s hats. At the same time, Native Americans were superb hunters, but they did not possess metalworking
technology as the Europeans did. They exchanged furs from America's abundant wildlife for iron, copper, and brass
goods such as knives, axes, pots, and needles.

第 193 页 共
373 页
2021 托福阅读真题

Paragraph 2: Many Native Americans were willing, even enthusiastic, to trade for European goods. Over the
centuries, participation in the fur trade increased in volume and in importance in aboriginal economies. The
immediate consequence of trade was the addition of material and technological innovations, but dependence on trade
had negative effects not foreseen by most participants. Since the market for beaver could not be controlled by Native
American fur trappers, they were vulnerable to changes in demand. When demand was high, they abandoned some
aboriginal practices in order to keep pace. Instead of following traditional conservation principles, they excessively
trapped nearby territories so that they could obtain as many animals as possible. This led to the rapid depletion of
beaver in some areas. As a result, men were forced to travel farther from their communities to find the desired
resource, often entering territories of other people who were similarly engaged in trapping and trading, resulting in
conflict. When the demand for furs declined, people were left without the ability to procure the goods that they
wanted. In societies where traditional craft skills had been abandoned once people acquired manufactured tools and
utensils, the loss of European goods was difficult to adjust to or even contemplate.

1.Why does the author provide the information that some areas experienced a "rapid depletion of beaver"?
A.To provide evidence that Native American fur trappers could not control the demand for beaver
B.To point out an effect of Native Americans' abandonment of traditional conservation principles
C.demand for furs
D.To explain why Native American practices changed to help rebuild reduced beaver populations

2.The word "contemplate" in the passage is closest in meaning to


A.defend against
B.recover from
C.accept
D.consider

3.According to paragraph 2, how did high demand for furs contribute to conflict among Native Americans?
A.It strongly disadvantaged Native American groups that did not engage in trapping and trading.
B.It forced Native American groups that practiced traditional conservation methods to compete for furs against
groups that over-trapped animals.
C. It encouraged Native Americans to trap for furs in areas outside their own.
D.It encouraged Native American groups to compete for control of unoccupied territories.

Paragraph 3: In addition to the acquisition of a wide range of imported goods, transformations in aboriginal
societies included shifts in economic activities, changes in gender roles, the development of notions of private
property in goods and especially in land, the emergence of or increases in social differences based on wealth, and the
intensification of warfare caused by competition over access to resources and to trade routes. These transformations

第 194 页 共
373 页
2021 托福阅读真题

were manifested more intensely in some societies than in others, but they were prevalent throughout North America
at different historical periods. These changes occurred earliest in regions of initial European entry and settlement,
that is, along the eastern coasts and nearby inland territories, but they eventually spread to the interior of the
continent, leaving no Native American nation untouched.

4.Paragraph 3 suggests which of the following was true about Native American nations before trade with Europeans
became common?
A.Native American nations usually settled disagreements over land ownership without warfare.
B.Native American nations relied heavily on trade with one another for needed goods.
C.Differences in status in Native American societies had little o do with differences in wealth.
D.The idea of the private ownership of goods had become well established.

5.Which of the following was true of the transformations of Native American nations discussed in paragraph 3?
A.These transformations encouraged good relations among Native American nations by reducing traditional
conflicts.
B.These transformations affected most but not all Native American nations in North America.
C.These transformations increased conflict between Native American nations and Europeans.
D.These transformations occurred in coastal areas before they occurred in the far interior.

Paragraph 4: As early as the seventeenth century in some eastern aboriginal nations, trapping and trading replaced
hunting as men' s central productive activities. Among agricultural people where farming was the responsibility
of women, food supplies were maintained, but among other groups that depended more heavily on meat, fish,
and fowl brought in by hunters, aboriginal food resources were not exploited as fully as had been done prior
to involvement in the fur trade. Many people then traded with Europeans for food, but this led to increased
dependence on traders. Women, too, were involved in the fur trade because their labor was needed to prepare the
pelts (animal skins) for the market. Since they also had to provide food for their families and perform household
tasks, demands on their labor increased as well. As the economic roles of both men and women changed to place
greater focus on the fur trade, people grew more dependent on the trade in order to supply their needs and wants.
This reliance on trade tended to intensify and solidify the productive shifts that supported it.
6.Which of the sentences below best expresses the essential information in the highlighted sentence in the passage?
Incorrect choices change the meaning in important ways or leave out essential information.
A.In some groups involved in the fur trade, food supplies were maintained by relying more heavily on meat, fish,
and fowl brought in by hunters.
B.Some societies depended heavily on meat, fish, and fowl brought in by hunters, but farming was more importan t
for other groups.
C.Among agricultural people, women took on additional responsibilities when men became more involved in the fur
trade.

第 195 页 共
373 页
2021 托福阅读真题

D.After involvement in the fur trade began, groups with female farmers maintained their food supplies better than
groups that depended more on hunting did.

7.According to paragraph 4, participation in the fur trade changed the lives of Native American women in which of
the following ways?
A.It required them to perform more work than they had previously.
B.It led them to create new ways to perform traditional household tasks.
C..It allowed them to take leadership roles in setting up trade arrangements with Europeans.
D.It encouraged them to produce extra food that could be exchanged for European goods.

Paragraph 5: In addition, since European traders dealt with Native American trappers as individuals, a process
began that eventually resulted in a reorientation of ideology away from kin-based, community-based mutual reliance
and support to one that stressed individuals. Over the centuries, notions of personal private property developed that
contrasted fundamentally with beliefs about communal ownership of resources. Although aboriginal societies had
concepts of territorial rights, these rights were held by groups, not by individuals. Strangers in need were permitted
to use local resources, at least temporarily, but the idea that ownership of land and resources could be transferred was
foreign to Native American cultures.

8. According to paragraph 5, before their involvement in the fur trade, Native Americans held which of the following
cultural beliefs?
A.Land was owned collectively by the group and could not be bought or sold.
B.Ideas of personal private property would weaken their societies.
C.Strangers in need could claim ownership of land and resources that were not being used.
D.Communal lands could be transferred to another group.

Paragraph 1: Soon after Europeans made contact with the aboriginal peoples of North America, known as Native
Americans, trading became the focus of European concern. ■French, British, and Dutch merchants sought animal
furs, especially beaver skins, which were in high demand between the sixteenth and nineteenth centuries in Europe
for making men' s hats. ■At the same time, Native Americans were superb hunters, but they did not possess
metalworking technology as the Europeans did. ■They exchanged furs from America's abundant wildlife for iron,
copper, and brass goods such as knives, axes, pots, and needles.■

9.Look at the four squares ■ that indicate where the following sentence could be added to the passage.
Thus both groups benefited by providing the other with goods that the other wanted.
Where would the sentence best fit? Click on a square ■to add the sentence to the passage.

10.Directions: An introductory sentence tor a brief summary of the passage is provided below. Complete the

第 196 页 共
373 页
2021 托福阅读真题

summary by selecting the 3 answer choices that express the most important ideas in the passage. Some sentences do
not belong in the summary because they express ideas that are not presented in the passage or are minor ideas in the
passage. This question is worth 2 points.

Trade between Native Americans and Europeans had lasting effects on Native American societies.

A.Trade between Native Americans and Europeans increased in part because some of the metal goods that Native
Americans obtained through this trade increased their success as trappers.
B.When demand for furs was low, Native American nations supported their communities by trading meat, fish, fowl,
or agricultural crops for European goods.
C.Under European influence, Native Americans developed new attitudes about the roles of men and women and
began placing less emphasis on the welfare of the group and more on the welfare of individuals.
D.When local populations of animals became reduced because of the high demand for furs, Native Americans began
trapping in distant areas, a practice that caused tension with members of other groups.
E.As Native Americans devoted less time to hunting and fishing and more time to trapping and trading, they became
more dependent on trade with Europeans for the things they needed, including food.
F.Possession of European goods became a sign of high status within Native American groups, a development that
strengthened existing Native American ideas about personal private property.

Jupiter's Moon Io

Paragraph 1: Of Jupiter's four largest moons, known as the Galilean moons, lo is the closest to Jupiter and the
densest. lo is the most geologically active object in the entire solar system. In all, more than 80 active volcanoes
have been identified on lo. In contrast to the surfaces of the other Galilean moons, its surface is neither cratered nor
streaked, but is instead exceptionally smooth, mostly varying in altitude by less than one kilometer. This smoothness
is apparently the result of molten matter that constantly fills in any dents and cracks, giving Io the youngest surface
of any known object in the solar system.

第 197 页 共
373 页
2021 托福阅读真题

1.According to paragraph 1, which of the following is NOT true of Io?


A.It is the largest of the four Galilean moons.
B. It is the densest of Jupiter's Galilean moons.
C.It has more than 80 active volcanoes.
D.Its surface is much more even than those of other Galilean moons.

Paragraph 2 : Io's volcanism has a major effect on Jupiter's magnetosphere (the space around a planet affected by
its magnetic field). All the Galilean moons orbit within the magnetosphere and play some part in modifying its
properties, but Io's influence is particularly marked. Although many of the electrically charged particles (ions) in
Jupiter's magnetosphere come from the solar wind, there is strong evidence that Io's volcanism is the predominant
source of heavy ions in the inner regions. Jupiter's magnetic field continually sweeps past lo, gathering up the
particles that Io's volcanoes spew into space and accelerating them to high speed. The result is the lo plasma torus, a
doughnut-shaped region of gas that has been heated to such high temperatures that its atoms are ionized. The plasma
torus completely encircles Jupiter along the path of Io's orbit. The plasma torus is quite easily detectable from Earth,
but before the explorations of the United States' Voyager space probes, its origin was unclear. Later, the Galileo
spacecraft made detailed studies of the plasma torus. Its spectroscopic analysis shows that sulfur is one of the torus's
major constituents, strongly implicating Io's volcanoes as its cause.
2.The word “properties" in the passage is closest in meaning to
A.characteristics
B.movements
C.effects
D.positions

3.According to paragraph 2, Io's volcanoes affect Jupiter in which of the following ways?
A. They produce ions that enter Jupiter's magnetosphere.
B.They speed up the solar winds that surround Jupiter.
C.They increase the surface temperature of Jupiter.

第 198 页 共
373 页
2021 托福阅读真题

D.They produce the sulfur that covers the surface of Jupiter.

4.According to paragraph 2, what did the Galileo spacecraft reveal about the plasma torus?
A.That it sometimes extends beyond Jupiter's magnetosphere
B.That its temperatures are so high that the atoms within it are ionized
C.That sulfur makes up much of it
D.That it matches up exactly with the path of Io's orbit

Paragraph 3: It must be asked what causes such astounding volcanic activity on lo, since, left to itself, lo would
long ago have become geologically inactive, like Earth's Moon, whose once molten interior has cooled below the
temperature needed to support volcanic activity. Before the origin of the plasma torus was understood, some
scientists suggested that the process by which the magnetosphere created the plasma was also heating lo. We now
know that this is not the case. The real source of Io's volcanic activity is gravity- Jupiter's gravity. Two factors are at
issue. First, lo orbits very close to Jupiter. As a result, Jupiter's huge gravitational field exerts strong forces on the
moon. These forces are so large that they pull the surface of lo into a raised area or bulge that points toward Jupiter.
Second, Io's orbit is not perfectly circular. If lo were the only moon orbiting Jupiter, it would long ago have come
into a state in which its rotation on its axis was perfectly synchronized with its revolution around Jupiter. In that case,
lo would move in a perfectly circular orbit with the same side unvaryingly turned toward Jupiter.

5.What does the author imply in paragraph 3 by mentioning that Earth's Moon is now geologically inactive?
A.Io is closer to Jupiter than the Moon is to Earth.
B.Io is much younger than the Moon.
C.Io's volcanoes eventually will disappear.
D.Io's continued volcanic activity is surprising.

Paragraph 4: But lo is not alone. As it orbits, it is constantly pulled by the gravity of its nearest large neighbor,
another Galilean moon, Europa. Although Europa's pull on lo is small, it is sufficient to make Io's orbit slightly
non-circular. In a non-circular orbit, the moon's speed varies from place to place as it moves in its orbit around
Jupiter, but its rate of rotation on its axis remains constant. Thus it cannot keep the same face steadily turned toward
Jupiter. Instead, as seen from Jupiter, lo turns slightly from side to side as it moves. On the other hand, the large
tidal bulge on Io's surface caused by Jupiter's gravity always points directly toward Jupiter, so it moves back and
forth across Io's surface as the moon wobbles. Those conflicting forces result in enormous tidal stresses that
continually flex and squeeze Io's interior.

6.Why does the author state that“as seen from Jupiter, lo turns slightly from side to side as it moves"?
A.To support the claim that Jupiter's gravity is strong enough to make Io's orbit slightly non-circular
B.To argue against the claim that the bulge on Io's surface always points directly toward Jupiter

第 199 页 共
373 页
2021 托福阅读真题

C.To illustrate a result of Europa's gravitational effect on Io's orbit


D. To provide evidence that lo behaves in a similar manner to the other Galilean moons

7.According to paragraph 4, what is the effect of Europa's gravity on lo?


A.It makes Io speed up or slow down during different parts of its orbit.
B.It causes Io's rate of rotation on its axis to vary slightly as it moves around its orbit.
C.It produces a gravitational bulge on lo.
D.It is too small to significantly influence lo.

Paragraph 5: Just as frequent back-and-forth bending of a piece of wire can produce heat through friction, the
ever-changing distortion of Io's interior constantly heats the moon. This generation of large amounts of heat within lo
ultimately causes huge jets of gas and molten rock to squirt out of the moon's surface. Galileo's sensors indicated
extremely high temperatures in the out-flowing material. Researchers estimate that the total amount of heat
generated within lo as a result of tidal flexing is about 100 million megawatts.

8.Which of the following can be inferred from paragraph 5 about the heat within Io?
A. Researchers believe it to be well over 100 million megawatts.
B. It is generated by friction.
C. It has been decreasing over time.
D. It causes the moon's entire surface to be molten.

Paragraph 1: Of Jupiter's four largest moons, known as the Galilean moons, lo is the closest to Jupiter and the
densest. ■lo is the most geologically active object in the entire solar system. ■In all, more than 80 active volcanoes
have been identified on lo. ■In contrast to the surfaces of the other Galilean moons, its surface is neither cratered nor
streaked, but is instead exceptionally smooth, mostly varying in altitude by less than one kilometer. ■This
smoothness is apparently the result of molten matter that constantly fills in any dents and cracks, giving lo the
youngest surface of any known object in the solar system.
9. Look at the four squares █ that indicate where the following sentence could be added to the passage.
Despite this, lo is further distinguished by being relatively featureless.
Where would the sentence best fit? Click on a square █ to add the sentence to the passage.

10.Directions: An introductory sentence tor a brief summary of the passage is provided below. Complete the
summary by selecting the 3 answer choices that express the most important ideas in the passage. Some sentences do
not belong in the summary because they express ideas that are not presented in the passage or are minor ideas in the
passage. This question is worth 2 points.

Io, one of Jupiter's large Galilean moons, is the most geologically active object in the entire solar system

第 200 页 共
373 页
2021 托福阅读真题

A. All four of the Galilean moons have surfaces that are neither cratered nor streaked, indicating that they were
either created relatively recently or have been protected from impacts.
B. Heavy ions expelled by Io's volcanoes create the lo plasma torus, a ring around Jupiter that can be detected from
Earth.
C. A large amount of heat is generated by the conflicting movements of lo and its gravitational bulge, and this heat
is what causes the frequent volcanic eruptions on lo.
D. Jupiter's magnetosphere is especially large due to the size of the planet, and it accelerates and heats all objects that
come within it, from small particles to the large Galilean moons.
E. The moon Europa pulls on lo enough to prevent it from having a perfectly circular orbit, while Jupiter's gravity
creates a raised area on lo that always points toward the planet.
F. Sensors indicate that huge jets of gas and molten rock spew from lo's surface, propelling and accelerating the lo
plasma torus to high speeds.

Recognizing Social Play in Animals

Paragraph 1: Many animals engage in some type of social play--that is, playing with others, including chasing and
fighting. Three functions of social play have been proposed: Social play may (1) lead to the forging of long-lasting
social bonds, (2) help develop much needed physical skills, such as those relating to fighting, hunting, and mating,
and (3) aid in the development of cognitive skills. One cognitively related benefit of social play revolves around the
idea of self-assessment. Here, animals use social play as a means to monitor their developmental progress as
compared to others. For example, in infant sable antelope (Hippotragus niger ), individuals prefer same-age play
partners. While this preference could be due to numerous factors, Kaci Thompson has hypothesized that it is

第 201 页 共
373 页
2021 托福阅读真题

primarily a function of infants attempting to choose play partners that provide them with a reasonable comparison
from which to gauge their own development.

1.The phrase forging of in the passage is closest in meaning to


A.increase of
B. support for
C.creation of
D.desire for

2.According to paragraph 1, young animals can judge how well they are developing by
A. engaging in fighting with other members of their species
B. establishing social bonds with individuals that have successfully developed physical skills
C. participating in play with a large number of different play partners
D.choosing to play with other animals that are close to them in age

Paragraph 2: Since many of the behavior patterns seen during play are also common in other contexts--hunting,
mating, dangerous aggressive contests--how do animals know they are playing and not engaged in the real activity?
And even more to the point, how do they communicate this information to each other? Bekoff has proposed three
possible solutions to this important, but often overlooked, question. One way that animals may distinguish play from
related activities is that the order and frequency of behavioral components of play is often quite different from that of
the real activity. That is, when play behavior is compared with the adult functional behavior that it resembles,
behavioral patterns during play are often exaggerated and misplaced. If young animals are able to distinguish these
exaggerations and misorderings of behavioral patterns by, for example, observing adults that are not involved in play,
a relatively simple explanation exists for how animals know they are playing.

3.According to paragraph 2, which of the following may signal that animals are playing rather than engaging in real
activity?
A.By repeating a single component of a behavior rather than including all of the components
B. Performing actions in a different sequence than they are performed in during real activity
C.Performing actions that are appropriate for younger animals but not the animal performing the actions
D.Using certain behaviors only when adults are available to observe interactions among animals

Paragraph 3: A second, somewhat related, means by which animals may be able to distinguish play from other
activities is by the placement of play markers. These are also known as play signals and can serve both to initiate
play and to indicate the desire to continue playing and to warn adults that the young are playing and not in danger of
injury. In canids (the animal group that includes dogs and wolves), for example, biting and shaking are usually
performed during dangerous activities such as fighting and predation. Yet, biting and shaking are also play

第 202 页 共
373 页
2021 托福阅读真题

behaviors of young canids. Bekoff found that play markers such as a bow (lowering the head) would precede biting
and rapid side-to-side shaking of the head to indicate that they were not dangerous behaviors. The bow would
communicate that this action should be viewed in a new context--that of play. Another play marker might be a
particular kind of vocalization--for example, chirping in a rat, whistling in a mongoose, panting in a wolf or a
chimpanzee-- before or during a play interaction. Or there might be a distinctive smell that indicated that the
animals were engaged in play.

4. Why does the author point out that "biting and shaking are also play behaviors of young canid'"?
A.To provide examples of activities that sometimes function as play markers
B.To indicate that certain play behaviors are identical to behaviors that are unrelated to play
C.To point out that some types of play behavior are performed by several different species
D.To identify activities that are performed to indicate the desire to continue playing

5. The word “distinctive" in the passage is closest in meaning to


A.characteristic
B.pleasant
C.weak
D.common

6. It can be inferred from paragraph 3 that in the absence of play signals, adult animals would
A.perceive young animals as being in danger of injury during play activity
B.be unable to give warnings to young animals that were in danger of injury
C. engage more frequently in dangerous activities such as fighting and predation
D. use side-to-side shaking of the head to indicate that biting was not dangerous

Paragraph 4: Play markers have also been found in primates such as the juvenile lowland gorilla. Juvenile lowland
gorillas play with each other often, and play ranges from what Elisabetta Palagi and her colleagues call gentle play to
rough play. Palagi's team discovered that when juvenile gorillas--particularly males--were involved with rough play,
the play was often preceded by a facial gesture they call the play face. This facial gesture, which is not seen in other
contexts, includes slightly lowered eyebrows and an open mouth. In addition to using this facial gesture during rough
play, juvenile gorillas also displayed it when a play session was in a place that made escape (leaving)
difficult--another context in which it may be important to signal to others that what is about to occur is play.

7.According to paragraph 4, one reason juvenile gorillas use play markers is that juvenile gorillas
A.cannot distinguish between different types of play
B.sometimes play in areas where it is difficult for them TO get away
C. cannot distinguish facial gestures used during play from those used during real activity

第 203 页 共
373 页
2021 托福阅读真题

D.need to indicate when play should become less rough

8.Paragraph 4 suggests which of the following about the use of play markers by lowland gorillas?
A.Lowland gorillas tend to use play markers most often during gentle play.
B.Female lowland gorillas use play markers more often than male lowland gorillas do.
C.Lowland gorillas mainly use play markers to signal that they do not want to play.
D.Lowland gorillas are one of several primate species that use play markers.

Paragraph 5: Yet another way by which young animals may be able to distinguish play from related behaviors is by
role reversal, or self-handicapping, on the part of any older playmates they may have. In role reversal and
self-handicapping, older individuals either allow subordinate younger animals to act as if they are dominant during
play or the older animals perform some act (for example, an aggressive act) at a level clearly below that of which
they are capable. Either of these provides younger playmates with the opportunity to recognize that they are involved
in a play encounter.

Paragraph 3: A second, somewhat related, means by which animals may be able to distinguish play from other
activities is by the placement of play markers. These are also known as play signals and can serve both to initiate
play and to indicate the desire to continue playing and to warn adults that the young are playing and not in danger of
injury. In canids (the animal group that includes dogs and wolves), for example, biting and shaking are usually
performed during dangerous activities such as fighting and predation. Yet, biting and shaking are also play behaviors
of young canids. Bekoff found that play markers such as a bow (lowering the head) would precede biting and rapid
side-to-side shaking of the head to indicate that they were not dangerous behaviors. █The bow would communicate
that this action should be viewed in a new context- -that of play.█ Another play marker might be a particular kind of
vocalization- -for example, chirping in a rat, whistling in a mongoose, panting in a wolf or a chimpanzee- before or
during a play interaction.█ Or there might be a distinctive smell that indicated that the animals were engaged in
play.█

9. Look at the four squares █ that indicate where the following sentence could be added to the passage.
While this and other play markers are meant to be seen, not all play markers are visual.
Where would the sentence best fit? Click on a square █ to add the sentence to the passage.

10.Directions: An introductory sentence tor a brief summary of the passage is provided below. Complete the
summary by selecting the 3 answer choices that express the most important ideas in the passage. Some sentences do
not belong in the summary because they express ideas that are not presented in the passage or are minor ideas in the
passage. This question is worth 2 points.

Many animals participate in social play, which helps develop social relationships and improves physical and

第 204 页 共
373 页
2021 托福阅读真题

cognitive skills.

A.Much social play mimics dangerous behaviors such as hunting and fighting, so animals must have ways to signal
to one another that play is happening, rather than something more serious.
B.Play signals, which animals use to indicate that they want to play, may include facial expressions, gestures or
behaviors (such as bowing), vocalizations, or giving off a certain smell.
C.One of the main benefits of play is that it allows young animals to develop their individual way of communicating
through signals such as vocalizations and facial gestures.
D.Some behavioral elements used in play closely resemble those used during serious aggressive activities, but adults
also engage in role reversal and self-handicapping when playing with youngsters.
E.Young animals must develop the cognitive skills needed to limit social play because play activity can lead to
exaggerations and misorderings of behavior that are dangerous to them.
F.Adults primarily use play markers to indicate to young animals that they are not in danger of injury during play
activity in which the adult engages in biting, shaking, or other rough behavior.

The Most Common Bird on Earth

Paragraph 1: The red-billed quelea is a small mostly brown bird found in the annual grasslands of Africa. Other
than ornithologists and the people who see the bird in its native habitat, almost no one has ever heard of the
little quelea although, with a population estimated at 10 billion, it may be the most common bird on Earth.

1. Which of the sentences below best expresses the essential information in the highlighted sentence in the passage?
Incorrect choices change the meaning in important ways or leave out essential information.
A. The quelea is the world's most common bird, yet even ornithologists and people who have seen it know almost

第 205 页 共
373 页
2021 托福阅读真题

nothing about it
B. Ornithologists and the people who see queleas in large numbers in their native habitat believe the quelea is the
world's most common bird.
C. Although queleas can be seen in great numbers, even outside their native habitat, few people really know that the
quelea population is huge.
D. Few people know about the quelea even though its large population probably makes it the most common bird on
Earth.

Paragraph 2: Given the seemingly stressful conditions in the environments where the birds are found, it is
surprising that the red-billed quelea survives at all, much less that it is so remarkably successful. Queleas feed on
native annual grasses. Any animal that feeds primarily on the seeds of annual grasses has a potentially major
problem—its food supply periodically appears and then disappears. A wide range of adaptations allows annual-seed
eaters to survive. For example, rodents in annual grasslands often store their extra seeds to survive lean times, an
adaptation that was taken up by human populations once they domesticated and became dependent on annual grasses,
such as barley and wheat. If they are to survive, queleas must also cope with the challenging feast- or-famine
situation attending annual-grass feeders. One might expect such species to be regularly wiped out (at least locally)
when fluctuations in the growing season result in seed failures of their food. The red-billed quelea copes well enough
to be the world's most common bird.

2. According to paragraph 2, what problem do queleas have to overcome in their native habitat in order to survive?
A. Humans have turned the native grasslands into fields of barley and wheat
B. Other annual-seed eaters are moving into the same habitat as the queleas.
C. For part of the year, there is little for queleas to eat in any given location.
D. In recent years, periods of famine have occurred more often and have been more severe.

Paragraph 3: Part of the quelea's success arises from its life-history adjustments to its environment. Over the
African dry season, the species has to subsist on the ever-diminishing supply of seeds produced by the annual
grasses at the end of the last wet season. As food becomes scarcer, the species feeds actively and gains sufficient
weight reserves to be able to migrate to more auspicious areas. With the onset of rains in the African wet season, the
seeds that queleas eat germinate (develop) and so are no longer available for food. The resulting severe food
shortage lasts six to eight weeks. The birds are forced to migrate to better situations (if they can find them). For
red-billed queleas, the savannas in which they live are an ever-shifting mosaic of patches of varying suitability
depending on their recent rainfall history. Some areas have dry seeds, others have immature grasses, mature grasses
with abundant green seeds, or old grasses with no seed. By moving over distances of 30 to 120 miles (50 to 200 km),
the birds subsist in an inhospitable universe by moving from one patch of habitat with suitable food to another.

3. The phrase “ever-shifting” in the passage is closest in meaning to

第 206 页 共
373 页
2021 托福阅读真题

A. loosely arranged
B. constantly changing
C. extremely unreliable
D. isolated

4. It can be inferred that which of the following happens within a time span of “six to eight weeks”?
A. The amount of land covered by grasses increases significantly.
B. Queleas gain enough weight to be able to survive the dry season.
C. Grasses go from germinating seeds to seed-bearing, mature plants.
D. Enough rain falls so that by the end of that period, grass seeds germinate.
Paragraph 4: The so-called early-rains migration ends when queleas return to formerly abandoned locations. By
this time the rains have come and grass seeds have germinated. The new grasses mature, and eventually fresh green
seeds become available, ending the local food shortage. Then queleas establish communal breeding colonies in
appropriate locations. They typically nest in thorny acacia trees. Their breeding colonies can contain several million
pairs of breeding birds and can cover tens of hectares. The entire breeding cycle, from nesting colony to independent
fledged young, requires six weeks--an exceptionally short interval for birds. The biological synchrony found among
the birds in a breeding colony can be remarkable. Millions of eggs in millions of nests hatch on the same day. The
fall of eggshells from subsequent dropping of the shells from the nests after these synchronous hatching has been
likened to a snowstorm. Sometimes the dry season comes early and the breeding areas dry out prematurely. In such
times, the breeding colonies are abandoned. In other instances, the rains may sustain a prolonged green period and
several episodes of breeding can occur in the same general area.

5. According to paragraph 4, queleas establish communal breeding colonies when


A. grass seeds are just beginning to germinate
B. mature grasses produce new seeds
C. acacia trees begin to bloom
D. the so-called early-rains migration begins

6. Why does the author mention that the dropping of eggshells "has been likened to a snowstorm?
A. To stress the seasonal nature of an event
B. To verify that people have witnessed an event
C. To further emphasize the huge scale of an event
D. To evaluate the negative consequences of an event

7. According to paragraph 4, Bach of the following is true about the quelea's breeding EXCEPT.
A. Queleas prefer breeding locations where acacia trees are available
B. The breeding cycle is shorter than for most other birds

第 207 页 共
373 页
2021 托福阅读真题

C. Enough rain can allow for a number of breeding episodes.


D. Millions of eggs hatch over a period of six weeks.

Paragraph 5: Since red-billed queleas feed on the seeds of annual grasses, they are preadapted to be effective
feeders on the seeds of domesticated annual grasses as well. As human agriculture has planted more and more cereal
grains across Africa, the numbers of quelea have exploded in response to the abundance of suitable food. The
quelea's magnitude as a pest animal has increased correspondingly. Although the numbers may be overestimated,
the species is certainly capable of destroying 10 to 20 percent of the production of large farms and the entire crop of
small, independent farmers. The red-billed quelea is not only the most abundant bird, it can also be described as the
most destructive pest bird on Earth.
8. By saying that the numbers may be “overestimated” the author means that the numbers used in estimates
A. maybe outdated
B. are difficult to believe
C. may be incomplete
D. may be higher than the actual numbers

Paragraph 2: Given the seemingly stressful conditions in the environments where the birds are found, it is
surprising that the red-billed quelea survives at all, much less that it is so remarkably successful. █ Queleas feed on
native annual grasses. █ Any animal that feeds primarily on the seeds of annual grasses has a potentially major
problem—its food supply periodically appears and then disappears. █ A wide range of adaptations allows
annual-seed eaters to survive. █ For example, rodents in annual grasslands often store their extra seeds to survive
lean times, an adaptation that was taken up by human populations once they domesticated and became dependent on
annual grasses, such as barley and wheat If they are to survive, queleas must also cope with the challenging feast-
or-famine situation attending annual-grass feeders. One might expect such species to be regularly wiped out (at least
locally) when fluctuations in the growing season result in seed failures of their food. The red -billed quelea copes
well enough to be the world's most common bird

9. Look at the four squares █ that indicate where the following sentence could be added to the passage.
More precisely, it is the seeds of those grasses that they feed on.
Where would the sentence best fit? Click on a square █ to add the sentence to the passage.

10.Directions: An introductory sentence tor a brief summary of the passage is provided below. Complete the
summary by selecting the 3 answer choices that express the most important ideas in the passage. Some sentences do
not belong in the summary because they express ideas that are not presented in the passage or are minor ideas in the
passage. This question is worth 2 points.

The world's most common bird, the red-billed quelea, is a small seed-eating bird that is found on African

第 208 页 共
373 页
2021 托福阅读真题

savannas.

A. Unlike other bird species, queleas rarely abandon their breeding colonies, even it the dry season arrives early, and
this has allowed them 10 reproduce successfully year after year.
B. The quelea's breeding cycle, which is uncommonly short, brings millions of birds together in large breeding
colonies, and rains may allow for more than one breeding episode in the same area.
C. The quelea has become a highly destructive agricultural pest as agricultural activity has allowed the quelea's
population to increase dramatically,
D. During times when food is hard to find, queleas sometimes find and feed on seeds that have been harvested and
stored by rodents and humans
E. Queleas can migrate extensively over distances of up to 120 miles in order to deal with local variations in seed
availability.
F. It is calculated that the quelea's population will continue to increase between 10 to 20 percent every year, and that
there will be a corresponding increase in crop destruction

The Beginnings and Development of Agriculture in Egypt

Paragraph 1: It was only between 6000 and 5000 B.C.E. that agriculture emerged in Egypt, substantially later than
in the neighboring Levant (to the east, along the Mediterranean Sea) where people started to live in permanent
farming communities by 7000 B.C.E. The relative richness of wild natural resources may explain why the Egyptians
adopted the new technology later than people in other areas surrounding the Levant. The Nile provided fish and
waterfowl, and in the desert lived game, while wild sorghum and other plants could be harvested. The technology of
farming was clearly an import into Egypt as it involved plants and animals not available in the wild there: the
primary domesticated animals were sheep and goats, and the first cereals cultivated were emmer wheat and
barley, all of which were foreign to Egypt and imported from the Levant. The domestication of cattle may have
been inspired by practices farther west in Africa. The adoption of agriculture had different consequences in Upper
and Lower Egypt.

1. Which of the sentences below best expresses the essential information in the highlighted sentence in the passage?
Incorrect choices change the meaning in important ways or leave out essential information.

第 209 页 共
373 页
2021 托福阅读真题

A. The earliest domesticated animals and plants in Egypt came from the Levant, as did overall farming technology.
B. The main domesticated animals and plants in both Egypt and the Levant were sheep, goats, emmer wheat, and
barley
C. The plants and animals that had been domesticated the Levant were not available in the wild in Egypt.
D. The technology of farming spread from the Levant through the domestication of sheep, goats, and cereals.

2.According to paragraph 1, why did farming emerge later in Egypt than in other nearby areas?
A. Egyptians developed effective harvesting practices later than nearby areas
B. Egypt already had a plentiful and varied food supply
C. Farming communities in the Levant interacted less with Egyptians with other surrounding areas.
D. The plants and animals native to Egypt were not appropriate for domestication.

Paragraph 2: In the Nile Delta and in Fayyum between 6000 and 5000 B.C.E., people started to live sedentary
lifestyle like their neighbors in the Levant. They built villages and obtained most of their food from the cereals they
grew. Few settlements are known, but they show that from around 5400 B.C.E. the northern (lower) Egyptians
practiced farming. In Upper Egypt and Nubia people primarily engaged in pastoralism, the herding of sheep and
goats. This made them more mobile, and we do not find village settlements near the Nile Valley in Upper Egypt and
Nubia. The people spent much time in the desert--more fertile than it is now--and the permanent remains we have of
them are tombs. When we find traces of settlements, they contain ash and debris, but no architecture. The graves
show that people produced nicely polished pottery and the first representations of humans in figurines, and that they
chose to place valuable mineral and metal objects with the dead. Similar burial practices appear from Middle Egypt
to Khartoum in Sudan, which suggests that the people over this large area shared common beliefs. We call their
material culture in Upper Egypt Badarian after the archaeological site of el-Badari.

3.Why does the author mention that settlements found in Upper Egypt and Nubia "contain ash and debris, but no
architecture"?
A.To provide evidence that desert conditions preserved fewer remains than the conditions in the Nile Delta and in
Fayyum did
B. To explain why tombs and pottery are the most useful objects for learning about ancient Upper Egypt
C. To support the claim that pastoralism made people in Upper Egypt more mobile, spending most of their time in
the desert
D. To indicate that long-lasting materials were not used for the buildings within permanent settlements in Upper
Egypt

4. According to paragraph 2, which of the following is NOT true about the people of Upper Egypt and Nubia around
5400 B.C.E.?
A. They shared similar beliefs with others who were part of the Badarian culture

第 210 页 共
373 页
2021 托福阅读真题

B. They placed valuable objects with their dead


C. They produced pottery and small figurines shaped like humans
D. Their lifestyle was similar to that of people living in the Nile Delta and in Fayyum

Paragraph 3: The extensive use of agriculture with permanent settlements only arose in Egypt after 4000 B.C.E.
This development occurred only north of the first cataract (whitewater are of the Nile) , which divides Egypt from
Nubia. Large centers appeared, and people went to live near the zones that the Nile flooded annually to work in the
fields. Until the building of the Aswan Dam (in modern times) , agricultural practices in Egypt were very different
from those in the neighboring Near East and Europe. The country received too little rain to rely on its water to feed
the crops, and the Nile was the farmer's lifeline. That river's cycle provided everything needed, however, and the
Egyptians relied on natural irrigation, or water for crops. The water rose in the summer, washing away salts that
impede plant growth and leaving a very fertile layer of silt on the fields bordering it. The water receded in time for
the crops--all grown in winter--to be sown, and it left the fields so moist that they did not require additional water
while the plants grew. Farmers harvested crops in the late spring, and the fields were ready for a new inundation by
July. The cycles of the river and the crops were in perfect harmony. The only concern was the height of the flood,
which dictated how much land received water. The ideal flood was somewhat more than eight meters above the
lowest river level. If the river rose too much villages and farms would be submerged; if it rose too little not enough
land would be irrigated.

5. The word "dictated" in the passage is closest in meaning to


A. indicated
B. varied
C. determined
D. limited

6. Paragraph 3 suggests that which of the following is one way that areas surrounding Egypt were different from
Egypt itself?
A. Surrounding areas grew crops during the winter.
B. Surrounding areas did not rely on river flooding for watering crops.
C. Surrounding areas received less dependable rainfall
D. Surrounding areas waited for silt to build up before planting crops Paragraph 3 is marked with an arrow

7. According to paragraph 3, the main thing farmers in the Nile valley had to worry about was whether
A. there would be enough rain to support the crops
B. the timing of the floods would interfere with planting or harvesting the crops
C. the dams and dykes would control the floods
D. the floods would be high enough to reach their fields but low enough to avoid their houses

第 211 页 共
373 页
2021 托福阅读真题

Paragraph 4: People could help the river by guiding water in canals and building dykes around fields in order to
regulate when the water reached the crops. Some of the earliest representation of kings from around 3000 B.C.E.
may show such work, but they do not constitute major projects to extend agricultural zones substantially. Artificial
irrigation that used canals and basins to store and direct the water into areas that the river could not reach only
appeared later in Egyptian history, and scholars debate when it started. Probably the increased lack of rainfall after
3000 B.C.E. pushed people into controlling the water more.

8. According to paragraph 4, what is one reason that Egyptians may have started to use artificial irrigation in
agricultural areas?
A. The limited rainfall after 3000 B.C.E. created a need for more careful use of water to meet agricultural needs.
B. People were unable to help guide river water using dykes, so they created basins to collect and store water for
agricultural needs.
C. Canals created in earlier times were ineffective in directing water.
D. Kings wanted to expand agricultural zones substantially.

Paragraph 3: The extensive use of agriculture with permanent settlements only arose in Egypt after 4000 B.C.E.
This development occurred only north of the first cataract (whitewater are of the Nile), which divides Egypt from
Nubia. Large centers appeared, and people went to live near the zones that the Nile flooded annually to work in the
fields. Until the building of the Aswan Dam (in modern times), agricultural practices in Egypt were very different
from those in the neighboring Near East and Europe. ■The country received too little rain to rely on its water to feed
the crops, and the Nile was the farmer's lifeline. ■That river's cycle provided everything needed, however, and the
Egyptians relied on natural irrigation, or water for crops. ■The water rose in the summer, washing away salts that
impede plant growth and leaving a very fertile layer of silt on the fields bordering it. ■The water receded in time for
the crops--all grown in winter--to be sown, and it left the fields so moist that they did not require additional water
while the plants grew. Farmers harvested crops in the late spring, and the fields were ready for a new inundation by
July. The cycles of the river and the crops were in perfect harmony. The only concern was the height of the flood,
which dictated how much land received water. The ideal flood was somewhat more than eight meters above the
lowest river level. If the river rose too much villages and farms would be submerged; if it rose too little not enough
land would be irrigated.

9. Look at the four squares that indicate where the following sentence could be added to the passage.
The floods were predictable, which benefited farmers in several ways.
Where would the sentence best fit? Click on a square to add the sentence to the passage.

10. Directions: An introductory sentence for a brief summary of the passage is provided below. Complete the
summary by selecting the THREE answer choices that express the most important ideas in the passage. Some

第 212 页 共
373 页
2021 托福阅读真题

sentences do not belong in the summary because they express ideas that are not presented in the passage or are minor
ideas in the passage. This question is worth 2 points. Drag your choices to the spaces where they belong. To review
the passage, click on View Text.

Agriculture came to Egypt from the Levant around 5400 B.C.E., with different consequences in different
regions

A.By 7000 B.C.E., Egyptians had adopted agricultural practices similar to those used by their neighbors in the
Levant, domesticating desert game and also plants such as wild sorghum.
B. In Lower Egypt, people began to settle in place more, building villages and living off of the cereals they grew, but
settlements were still fairly sparse.
C. Use of agriculture in the Nile valley became extensive after 4000b.c.e., with populations concentrate arc areas
that experienced crucial annual flooding.
D. Cattle domestication farther west in Africa was adopted later in Upper Egypt as a supplement to fish and
waterfowl provided by the Nile
E. In Upper Egypt and Nubia, people primarily engaged in pastoralism, herding goats and sheep and spending much
time in the desert, with tombs their only permanent structures
F. Egyptians were successful in widely implementing irrigation techniques throughout agricultural zones by 3000
B.C.E, as shown in some of the earliest representations of kings.

The Radiocarbon Dating of Prehistoric American Sites

Paragraph 1: Because all living things contain carbon artifacts and organic remains from archaeological sites can
often be dated by comparing the proportion of carbon-14 (radiocarbon) remaining in them (or in the location where
they were found)to the proportion of non- radiocarbon (carbon-12 and carbon-13) in them, a method called
radiocarbon dating.This technique based on the fact that the amount ofcarbon-12 and carbon-13 stays constant in
organisms, while the amount of carbon-14 declines at steady rate once the organism dies. When an animal or plant
dies, it stops exchanging gases with the atmosphere and its carbon-14 begins to decrease. Thus the amount of
carbon-14 remaining indicates how long ago an organ is died. Radiocarbon is the preferred method for dating the
sites of the earliest Americans. Its range reaches back 50,000 years, which is the period of interest, and its reliability
is well attested since variations in atmospheric carbon levels over time have been well studied and can readily be
adjusted for in age calculations. Moreover the technique can be applied to any material that incorporates
radiocarbon,such as bone, wood,charcoal burned wood) , and even soils, the last by virtue of the organic acids that

第 213 页 共
373 页
2021 托福阅读真题

seep into them.

1. Which of the following can be inferred from paragraph 1 about the amount of Carbon-14 in organisms?
A. It is higher in living organisms than the amount of either carbon-12 or carbon-
B. It is higher when organisms are alive than it is long after they have died
C. It declines more quickly in living organisms than in dead ones
D. It is less variable in organisms than the amounts of carbon-12 and carbon-13 are

2. According to paragraph 1, radiocarbon dating is now the preferred method for determining the age of ancient
biological specimens for all of the following reasons EXCEPT:
A. It can be used to date specimens that are tens of thousands of years old
B. It is considered scientifically reliable.
C. It can be used to date all materials that have radiocarbon
D. It can be used to determine whether 0rganic acids have seeped into materials

3. The phrase "distinguished by" in the passage is closest in meaning to


A. surrounded by
B. studied for
C. covered with
D. noted for .

Paragraph 2: Yet not all of these materials, including charcoal and bone from which radiocarbon ages are most
often obtained, give results that are equally re1aDle:' I wet the sample multiple times, will we get the same result: )
or this situation occur central and North dated to about 1o, of which is distinguished by the remains of bison, the site
being a location for killing and processing these animals, possibly using stone points. In the 1990s, the bison kill was
thought to have dated to 10,890B.P., based on radiocarbon ages of charcoal found with the bones. Yet the subsequent
radiocarbon dating of the bison bones themselves yielded an average age 400 years younger: 10, 490 B.P. Why the
discrepancy?

4. According to paragraph 2,the radiocarbon dating of charcoal suggested which of the following about the Folsom
kill sit
A. Paleo-Indians used the site partly as a place for making stone points.
B. Paleo-Indians were using the site earlier than radiocarbon dating based on bison bones indicated.
C. Paleo-Indians were using the site near the end of the history of the Folsom culture.
D. Paleo-Indians used the site for about 400 years.

5. The word "relatively" in the passage is closest in meaning to

第 214 页 共
373 页
2021 托福阅读真题

A. particularly
B. fairly
C. surprisingly
D. naturally

Paragraph 3: Charcoal has the virtue of being relatively impervious to contamination and thus normally yields a
reliable age. But the age may not be valid since the charcoal may not nave .the bison were the Folsom site. If tnecn
fart f it wilm, say, a lightning-strike forest fire, it will have no bearing on the age of the kill. Even if the charcoal was
from wood that stoked the fires over which the bison meat was Cooked, it could still overestimate the age of the kill
site.If the charcoal was from wood from the oldest part of a long-lived tree, it could be decades or even centuries
older than other parts of the same tree and thus older than the kill site.

6. In paragraph 3, why does the author discuss charcoal from different sources?
A. To present possible reasons why the Charcoal tested may provide a wrong date for the Folsom kill
B. To support the claim that charcoal is relatively impervious to contamination
C. To emphasize the reliability of charcoal in radiocarbon dating
D. To explain why scientists used charcoal to help determine the age of the Folsom kill

Paragraph 4: Conversely, radiocarbon dating of animal bones can more precisely pin down the age of a site, since
animals live shorter lives than trees, though, of course, only bones present as a result of human activity(which is
certainly the case of the Folsom bison are of interest to archaeologists. But dating bone itself can be
challenging,since it is made of multiple constituents, some of which are highly susceptible to Contamination. The
inorganic portion, for example, readily absorbs ancient carbon via groundwater or other contaminants. Before that
was realized, multiple radiocarbon ages on the same skeleton often gave wildly divergent results-in one case, for
example, the oldest and youngest radiocarbon ages on the same mammoth were nearly 3,000 years apart. In the
1980s, after laboratory advances enabled the extraction of protein and individual acids from organic bone collagen,
substances less susceptible to contamination, it became possible to derive ages from bone that were more reliable.
Hence the conclusion that the bone amino acid obtained from the Folsom bison better estimated when that kill
occurred.

7. According to paragraph 4,why has radiocarbon dating of bone been inaccurate at times?
A. Some parts of bones can be easily contaminated
B. Some parts of bones do not absorb radio carbons.
C. Human activity can affect the substances in bones.
D. Some bones came from animals that had no contact with humans.

Paragraph 5: But the handicap of radiocarbon dating which will be felt if the first Americans prove to be far

第 215 页 共
373 页
2021 托福阅读真题

older than supposed, is that it does not provide reliable ages for any samples that might prove to be
significantly more than 50,000 years old. For that, there are number of other dating methods that can go beyond
radiocarbon's limits.

8. Which of the sentences below best expresses the essential information in the highlighted sentence in the passage?
Incorrect choices change the meaning in important ways or leave out essential information.
A. The handicap of radiocarbon dating is that it cannot be used to determine whether samples of the first Americans
are older than supposed
B. Radiocarbon dating is not reliable for samples significantly older than 50,000 years
C. Radiocarbon dating of samples more than 50,000 years old has been proved inaccurate.
D. The handicap of radiocarbon dating is that there is not a significant number of samples more than 50,000 years
old.

Paragraph 4: Conversely, radiocarbon dating of animal bones can more precisely pin down the age of a site, since
animals live shorter lives than trees, though, of course, only bones present as a result of human activity(which is
certainly the case of the Folsom bison are of interest to archaeologists. But dating bone itself can be
challenging,since it is made of multiple constituents, some of which are highly susceptible to Contamination. ■The
inorganic portion, for example, readily absorbs ancient carbon via groundwater or other contaminants. ■Before that
was realized, multiple radiocarbon ages on the same skeleton often gave wildly divergent results-in one case, for
example, the oldest and youngest radiocarbon ages on the same mammoth we re nearly 3,000 years apart. ■In the
1980s, after laboratory advances enabled the extraction of protein and individual acids from organic bone collagen,
substances less susceptible to contamination, it became possible to derive ages from bone that were more reliable.
■Hence the conclusion that the bone amino acid obtained from the Folsom bison better estimated when that kill
occurred.
9.Look at the four squares that indicate where the following sentence could be added to the passage.
Eventually, a way Was found to obtain consistently reliable findings.
Where would the sentence best fit?

10. Directions: An introductory sentence for a brief summary of the passage is provided below. Complete the
summary by selecting the THREE answer choices that express the most important ideas in the passage. Some
sentences do not belong in the summary because they express ideas that are not presented in the passage or are minor
ideas in the passage. This question is worth 2 points. Drag your choices to the spaces where they belong. To review
the passage, click on View Text.

A. Radiocarbon dating can date specimens as old as 50,000 B.P.,but it does not always provide equally reliable and
valid results, as observed at the Folsom site.
B. While charcoal is often useful for radiocarbon dating, the charcoal found at the Folsom site was too damaged by

第 216 页 共
373 页
2021 托福阅读真题

fire and contamination to be tested for its age.


C. Bone dating is reliable when researchers test components of collagen rather than components that are more likely
to become contaminated.
D. Radiocarbon dating s most reliable he organic acids n specimen can be compared 0 known radiocarbon levels in
the atmosphere from a given time.
E. Because animals usually live shorter lives than trees do, researchers can often obtain more precise site ages by
radiocarbon dating bones than by radiocarbon dating wood or charcoal.
F. Removing proteins and individual amino acids from a bone before radio car bond ting helps ensure that the
resulting radiocarbon date for the bone is accurate

The Rise of Diamonds in Europe

Paragraph 1: In the thirteenth and fourteenth centuries. Europeans established new trading links with civilizations
to the east, allowing many new luxury goods to be imported. This, combined with the growing wealth of the
European nobility,led to the increasing use and affordability of precious st ones.Europeans began using gems for all
manner of things, including decorating picture frames, ornamenting statues, an d decorating weapons,as well as for
personal adornment.Yet,throughout this creative explosion in precious stone usage,rubies,emeralds opals,and
sapphires continued to rank above diamonds,as did p earls. Why? Well, anyone who has observed a diamond in
rough form, that is, uncut and unpolished, and thus aesthetically unspectacular, can answer that question. Moreover,
the diamond's legendary hardness actually limited its utility and, thus,its allure.At this time,consumers of precious
stones valued a gem's color and proportion much more than its hardness.("Proportion"refers to the relationship
between the size,shape,and angles of age cuts,all of which affect its ability to reflect light.) Around the 1460s the

第 217 页 共
373 页
2021 托福阅读真题

diamond's position at the pinnacle of the gem hierarchy was set in motion by a Flemishl apidary (a specialist who
works with precious stones named Louis de Berquem.

1. Why does the author include the information that European "consumers of precious stones valued a gem's color
and proportion much more than its hardness"?
A. To show how the public's views of precious stones changed during the thirteenth and fourteenth centuries
B. To provide one reason why most diamonds in the thirteenth and fourteenth centuries were left in their rough form
C. To identify a particular way in which Louis de Berquem influenced gem consumption in Europe
D. To help explain why diamonds ranked below rubies,emeralds,opals,and sapphires in the thirteenth and fourteenth
centuries

Paragraph 2: Although Indian lapidaries had long been working with diamonds, their techniques failed to reveal the
stones' brilliance. Even when these artisans began to revise their methods in the 1400s, the proportioning remained
so poor that much of the stones' latent brilliancy remained "unreleased." De Berquem's recutting of the Beau Sancy
diamond introduced a new era in cutting. De Berquen demonstrated now to successfully cut facets (flat, polished
surfaces), on the face of a diamond, which became known as scientific faceting, The technique rendered the diamond
a period reflector of light and unleashed its interior beauty and, thus,helped establish diamonds as the most coveted
of all gems, which,in turn,further fueled demand.

2. Which of the following can be inferred from paragraph 2 about the diamonds produced by Indian lapidaries?
A. They were similar to Beau Sancy's recut diamond.
B. Their latent brilliancy was maximized by their proportions.
C. Their polished surfaces hid imperfections in the stones' interiors.
D. They reflected light less well than the diamonds cut by de Berquem did.

Paragraph 3: The novel forms of aesthetic activity and appreciation central to the European Renaissance,a period of
cultural and intellectual innovation beginning in the fifteenth century, provided a perfect environment for the
ascendance of the diamond. Two developments, in particular, thrust the diamond into its paramount position. First
was the establishment of a reliable international trade network linking India's diamond-rich Golconda region directly
to Europe. Diamonds were procured and transported by intrepid merchant travelers to eagerly awaiting Europeans,
who were now consuming these stones in formerly unseen quantities. Perhaps the most famous of these was the
Frenchman. Jean-Baptiste Tavernier who, in 1668, sold the legendary Hope Diamond to the French king Louis XIV.
Second was the development of the "rose cut." As Europeanl apidaries gained experience working with ever greater
numbers of stones,their elevated levels of craftsmanship enabled them to improve upon de Berquem's work, a clear
case of practice makes perfect." In turn, they further highlighted, or "revealed, "the diamond's brilliance, which
heightened demand for a stone whose post-cut beauty was quickly outpacing the aesthetics of rival gemstones.

第 218 页 共
373 页
2021 托福阅读真题

3. The word "novel"in the passage is closest in meaning to


A. various
B. special
C. popular
D. new

4. The word "paramount"in the passage is closest in meaning to


A. supreme
B. popular
C. well-deserved
D. present

5. According to paragraph 3, which of the following was true of diamonds during the European Renaissance period?
A. More diamonds were being used in Eu rope than at any time before the period.
B.A new source of diamonds was discovered
C. Indian merchants started to transport diamonds across various reliable transportation links
D.Europeans began to sell diamonds out side of Europe.

6. In paragraph 3, the author mentions Jean-Baptiste Travernier because Jean-Bap tiste Travernier
A. developed the rose cut
B. helped Louis XIV establish a trade net work linking France and India
C. was one of the merchant travelers who brought diamonds from Golconda to Europe
D.encouraged European merchants to participate in the diamond trade

7. According to paragraph 3, which of the following made the development of the rose cut possible?
A. The increased skill that lapidaries had acquired from working with large numbers of diamonds
B. Support from the French king Louis XI V
C. The adaptations of techniques that we re specially created for cutting the Hope Diamond
D. De Berquem's efforts to improve on hi s earlier work

Paragraph 4: The increased demand for Indian diamonds in Europe was further spurred on by the invention of the
"brilliant cut," an improvement on the rose cut. The rapidly expanding diamond market was now sup porting an
industry that stretched across three continents. Caravans had once transported these stones across Arabia and
offloaded them to traders waiting in Aden, Aleppo, and Alexandria (the initial extent of Africa's involvement in the
trade). However, ships were now increasingly employed to convey these stones to Europe, utilizing the Red Sea but
otherwise largely bypassing the Middle East. Upon reaching Europe, these precious imports were purchased by
Jewish diamond merchant s in places as far-flung as Lisbon. Venice, and Frankfurt. Jewish involvement in the

第 219 页 共
373 页
2021 托福阅读真题

burgeoning diamond industry in Europe was logical for a few reasons. For centuries, Jewish people were excluded
from most occupations; many served as moneylenders, and these naturally concerned themselves with assessing,
repairing, and selling the gems offered as security for the repayment of loans. Moreover, the cutting and polishing of
diamonds, initially centered in Lisbon before shifting to Antwerp, was one of the few crafts in which craft and trade
associations permitted Jews to participate.

8. According to paragraph 4, which of the following developments occurred in the diamond trade as demand for
diamonds increased in Europe?
A. The rose cut became especially popular in some parts of the world.
B. Ships largely replaced caravans in transporting diamonds from India to Europe.
C. Aden, Aleppo, and Alexandria became centers of the diamond
D. Diamond merchants in Lisbon, Venice, and Frankfurt began selling diamonds to customers on three continents.

Paragraph 2: Although Indian lapidaries had long been working with diamonds, their techniques failed to reveal the
stones' brilliance. ■Even when these artisans began to revise their methods in the 1400s, the proportioning remained
so poor that much of the stones' latent brilliancy remained "unreleased." ■De Berquem's recutting of the Beau Sancy
diamond introduced a new era in cutting. ■De Berquen demonstrated now to successfully cut facets (flat, polished
surfaces), on the face of a diamond, which became known as scientific faceting. ■The technique rendered the
diamond a period reflector of light and unleashed its interior beauty and, thus,helped establish diamonds as the most
coveted of all gems, which,in turn,further fueled demand.

9. Look at the four squares that indicate where the following sentence could be added to the passage.
This method involved the use of a metal wheel to cut the diamond at specific angles, and the results were
spectacular.

10. Directions: An introductory sentence for a brief summary of the passage is provided below. Complete the
summary by selecting the THREE answer choices that express the most important ideas in the passage. Some
sentences do not belong in the summary because they express ideas that are not presented in the passage or are minor
ideas in the passage.This question is worth 2points Drag your choices to the spaces where they belong. To review the
passage, click on View Text.

From the 1460s, the diamond's status in Europe began to rise.

A. De Berquem developed a way of cutting diamonds that revealed more of their inner brilliance,and later cuts such
as t he rose cut and the brilliant cut improve d the appearance of diamonds even more
B. The establishment of a trade network connecting Europe to a diamond-rich pa rt of India made diamonds much
more easily available in Europe

第 220 页 共
373 页
2021 托福阅读真题

C. As the diamond market in Europe expanded,Europeans began to train Indian lapidaries in European techniques
and established diamond cutting centers in Aden, Aleppo, and Alexandria
D. While rubies,emeralds,,opals,and sapphires were preferred for personal adornment,diamonds began to be used f
or other purposes,such as ornamenting picture frames,statues and weapons
E. Eventually,diamonds were transported to Europe by routes that generally did not pass through the Middle
East,and merchants in many European cities became able to buy and sell diamonds.
F. Because of competition between craft and trade associations,the center of there expanding diamond market shifted
from one European city to another several times.

How Trout Survive in Winter

Paragraph 1: Trout are a group of freshwater fish species that inhabit cool, clear streams. Like all fish, trout are
cold-blooded, which means their body temperature changes with the temperature of the surrounding water. Once the
water becomes colder than abour4 degrees Celsius, trout go into a state of torpor (decreased physiological activity).
Because their metabolism slows, they have limited energy, and saving it becomes crucial. Trout survive winter best
when they stay in slower-moving water in a protected location where they do not need to expend much energy to
move or resist the current.

1. According to paragraph 1, all of the following statements about trout are true EXCEPT:
A. They generally live in cool water.

第 221 页 共
373 页
2021 托福阅读真题

B. They spend more energy during winter than they do during other parts of the year.
C. They adjust their levels of physiological activity according to the surrounding water temperature
D. They survive better in winter in waters where currents are slower.

Paragraph 2: A stream that is entirely ice covered provides both a stable environment and safety from predators.
The smallest trout seek shelter in holes between small rocks. Bigger trout look for areas of slower-moving, warmer
water, typically at the bottom of a deep part of the stream. During warmer months, as they compete with each other
for food, trout guard their individual territories, and trout longer than30 centimeters can prey on smaller trout species,
but in winter. when their main concern is conserving energy, it is common for large numbers of trout of different
sizes to aggregate in a suitable spot.

2. The word shelter in the passage is closest in meaning to


A. nutrients
B. warmth
C. protection
D. stability

Paragraph 3: A layer of snow on top of the ice further improves trout's survival chances because it prevents light
from reaching the water, and darkness helps keep trout inactive. In winter trout should not move even to feed, but if
they see instars, which are tiny aquatic (water-dwelling) insects, they will instinctively go after them. Trout litter
instars out of that water through their gills, but because instars are so small, the gills catch only an insignificant
volume of this food. which provides little nutrition. As a result of spending its energy tying to feed in this way. a
trout may weaken so much that it becomes unable to resist the current, in which case its survival will depend on the
environment downstream.

3. According to paragraph 3, snow covering the stream has which of the following effects on trout?
A. The snow enables trout to catch instars more easily because trout can hide in the darkness created by the snow.
B. The snow limits the types of food available to trout by preventing insects from reaching the water.
C. The snow helps trout save energy because trout tend to move less in dark environments,
D. The snow increases the supply of instars to trout because instars are attracted t0 darkness.

4. Paragraph 3 suggests which of the following about instars as a source of food for trout in winter?
A. Instars are more abundant in downstream environments in winter than they are in other environments.
B. Instars provide trout with less energy than trout spend feeding on them.
C. Instars provide an easy source of food for trout in dark environments
D. Instars provide trout with certain nutrients that trout need to survive in winter

第 222 页 共
373 页
2021 托福阅读真题

Paragraph 4: Trout are most vulnerable when the soft and sticky frazil ice occurs. Frazil ice forms during the
freezing of super-cooled Water (water that is colder than 0 degrees Celsius). Water can become supercooled after a
sharp drop in air temperatures at night, usually all the time of year just before the stream becomes covered by solid
ice or soon after solid ice melts. When frazil ice sticks to riverbanks, it forms shelf ice. Trout hide under shelf ice
like they do under sold ice, but under shelf ice they can become easy prey for mink and other predators on the banks
Because frazi1 ice often melts in the daytime, fish trying to hide under it are frequently forced to move.

5. According to paragraph 4, frazil ice makes trout vulnerable for which of the following reasons?
A. Frazil ice can limit trout's ability to move.
B. Frazil ice can force trout's prey to move, making it difficult for trout to obtain food.
C. Frazil ice attracts trout to locations where the trout are at risk from land-dwelling predators.
D. Frazil ice prevents the formation of the shelf ice that trout live under during winter.

Paragraph 5: Frazil ice can also create ice dams. In a section of South Cottonwood Creek, a large stream in the state
of Wyoming (United States). the Wyoming Game and Fish Department (WGFD) placed metal structures in the water
to create deep pools that were thought to provide a trout-friendly environment. However, these efforts failed to
produce a measurable increase in fish numbers in the long term. Researchers monitoring trout in the stream later
observed that sudden temperature drops at night during winter caused frazil ice to form, which stuck to the artificial
structures and resulted in the appearance of ice dams that would melt during the day. Night after night newly formed
frazil ice restricted the stream's flow, causing water levels upstream to rise dramatically as the ice built up and fall
rapidly as it melted. Because the unstable environment caused energy losses for the trout, some perishd while others
migrated to beaver dams

6. The word artificial in the passage is closest in meaning to


A. large
B. new .
C. solid
D. human-made

7. In paragraph 5, why does the author discuss the WGFD's actions in South Cotton wood Creek?
A. To illustrate one way in which frazil ice endangers trout's survival
B. To show that trout survival in frazil ice requires the presence of beaver dams
C. To support the idea that more trout-friendly environments are needed and should be created
D. To emphasize the need for additional research by state departments

Paragraph 6: Beaver dams are large pools created by beavers. To build their dams, these semi-aquatic plant-eating
mammals cat down trees with their teeth and carry the wood as well as mud and stones to a nearby stream, where

第 223 页 共
373 页
2021 托福阅读真题

they use this material to construct barriers restricting the water's flow.[■] At the bottom of the resulting dam, they
build special chambers to store bark stems, and other food for winter consumption. [■] Trout that spend winter in
beaver dams' deep calm water save valuable energy. [■] Thus beavers inadvertently increase trout's chances of
surviving until spring. [■]

8. According to paragraph 6, which of the following statements most accurately describes the relationship between
beavers and trout?
A. Beavers create deep pools where trout can stay during winter.
B. Beavers prey on trout.
C. Beavers compete with trout for resources
D. Beavers store food that trout can use in winter.

9. Look at the four squares that indicate where the following sentence could be added to the passage.
But the dams are useful not only to the beavers.
Where would the sentence best fit? Click on a square to add the sentence to the passage.

10.Directions An introductory sentence for a brief summary of the passage is provided below. Complete the
summary by selecting the THREE answer choices that express the most important ideas in the passage. Some
sentences do not belong in the summary because they express ideas that are not presented in the passage or are minor
ideas in the passage. This question is worth 2 points. Drag your choices to the spaces where they belong. To review
the passage. click on View Text.

Surviving in winter is not always easy for a trout

A. Because a trout's metabolism slows when the water temperature drops below a certain level, it is crucial for the
trout to spend winter in a place where it can save energy.
B. Dams built by beavers for food storage provide the stable conditions needed by trout in winter, but dams created
by frazil ice do not.
C. Trout are most vulnerable in the middle of winter because supercooled water occurs most frequently during the
coldest winter months.
D. When the stream is covered by solid ice with a layer of snow on top, trout can remain inactive and safe from
predators
E. Because the food available to trout during winter is very limited, trout are forced to feed on instars to obtain the
energy they need to survive
F. By monitoring the movement of trout, researchers have determined that state departments like the WGFD could
improve trout survival by creating trout-friendly environments.

第 224 页 共
373 页
2021 托福阅读真题

Changes in the Art Market During the Late Nineteenth Century

Paragraph 1: Today's market tor European art has its roots in the nineteenth century. In 1882 the passage of the
Settled Land Act reform ed inheritance law in Britain, As a consequence of industrialization and the avail ability in
Europe of large quantities of cheap American wheat, the fortunes of British landowners were in decline. After
bankruptcies began to mount, the inheritance laws were changed, making it possible for landowners to support their
landholdings by selling their art treasures. Until 1882.estates were strictly settled, meaning that the head of the
family did not actually own the land and estate goods.but rather was entrusted to preserve and protect the family
property. This was me ant to hinder large estates from being sold off in sections.The Settled Land Act made it
possible Tao sell individual portions of an estate, such as works of art, in order to save the real estate (land and

第 225 页 共
373 页
2021 托福阅读真题

buildings). The art treasures of the British nobility began to flow into London auction ho uses, shoring up heir
dominant position in the art market.

1. What can be inferred from paragraph 1 about the finance of British landowners in the nineteenth century?
A. Most of them went bankrupt after 1882
B. They made great profits as a result of industrialization
C. They earned less money from selling wheat than they had in the past
D. Other than their land, they possessed little of any value

Paragraph 2: Besides the passage of this law, the early 1880, marked the public acknowledgement of French
Impressionism, a revolutionary art movement that emphasized from everyday life and the accurate depiction of light
and its changing qualities. Prior to this, Impressionist artists had been regarded as radical immoral who had declared
war on beauty. Impressionist paintings were no accepted by the Salon des L'Academiedes Beaux-Arts in Paris,
which had been since 1830 the most important exhibition forum in the world and was historically the an of
traditional standards of French painting. In reaction to the conservative tendencies of this institution, a group of
Impressionist artists in Pars organized their own exhibition in 1874. Reviews of the exhibition were mixed. But in
1881, the Impressionist painter Edouard Manet was named a knight of the Legion of Honor. In 1882, Paul
Cezanne was allowed to exhibit his work tor the first time at the Salon.

2. Why does the author provide the information that "Edouard Manet was named a knight of the Legion of
Honor?
A.To demonstrate that Impressionist artists gained some respect and acceptance in late-nineteenth-century France
B. To give an example of a French artist who was not typical of the Impressionist movement
C. To show the cultural divide in French Society between the state and private art institutions
D. To explain why French impressionist s changed their painting style the early 180s

3. What can be inferred from paragraph 2 about the Salon de L’ Academie de Beaux-Arts in Paris
A. It was one of the few organizations to support the impressionist painting from the very beginning of the
movement
B. Its official exhibition in 1874 contained many important work impressionist painters such as Ceazanne
C. Is main function was to introduce new art movements to the French art world
D. It did not exhibit the works of French Impressionist painters until 1882

Paragraph 3: With the advent of Impressionist he modern art market was born.Since most collectors rejected the
Impressionists, especially early on,the usual paths for distribution were closed to these artists,and the y had to find
other ways to sell their pictures. The rejection however, meant that the artists were not dependent on the tastes of
potential patrons(wealthy supporters),and hence they began to see themselves as autonomous masters. This new

第 226 页 共
373 页
2021 托福阅读真题

approach also awakened a new found respect for the art dealer, since the dealer had access to artistic genius and
could ope n up the path to new art and explain it to potential buyers. In the early twentieth century, the Cubist style
(abstract art based on geometric forms) arose and representational art-an that represented objects and people
realistically-became less significant. This development also strengthened the notion of the artist genius, as well as
that of the art dealer as a go-between. On the one hand, the content of art became more complex, and on the other,it
also became more difficult to understand and communicate

4. The word "awakened" in the passage is closest in meaning to


A. increased
B. Stimulated
C. limited
D. Involved

5. According to paragraph 3, the refusal of collectors to buy Impressionist painting s resulted in which of the
following?
A. Artists painted as they wanted to paint without considering the views of potential patrons
B.Artists became less concerned with money and focused instead on the quality of their art.
C. Collectors discovered that art dealers could help them find works of greater value
D. Collectors more readily accepted other abstract art movements such as Cubism

7. According to paragraph 3, which of the following is an essential role that art dealers performed in the modern art
market?
A.They helped make sure that representational art remained popular.
B.They purchased art they could not sell to help artists make a living
C.They persuaded artists to reject realistic art and adopt new styles, such as Impressionism and Cubism.
D. They helped potential buyers understand and appreciate modem art

Paragraph 4: Of course, artworks by the so-called old masters continued to be sold. The most successful dealer of
such art was probably Joseph Duveen, who arrived in New York City at the age of seventeen in1886.After a brief
apprenticeship under his uncle, the art dealer Henry Duveen,Joseph opened a gallery (a privately owned
establishment for displaying and selling artwork) in proximity to the Waldorf-Astoria Hotel, which was already the
meeting place for the city's elite. Duveen's business strategy entailed the purchase of complete art collections from
their owners. In this way, a buyer would be unable to calculate Duveen's wholesale price for an individual artwork
and hence the percentage that he kept for himself. It he made purchases at auctions where the price he paid would be
public knowledge, he did not shrink from paying record prices, since this practice brought him a reputation and also
raised the value of the pieces already in his inventory. He also stirred up enthusiasm for collecting among many
American industrialists: these millionaires regarded the acquisition of European masterpieces as an opportunity to

第 227 页 共
373 页
2021 托福阅读真题

assure them mortality of their names

7. The word "hence" in the passage is closest in meaning to


A. also
B. Even
C. therefore
D. Especially

8. According to paragraph 4.which of the following is true of Joseph Duveen's business strategy?
A. Joseph Duveen did not like buying and selling the old masters because he earned a smaller percentage on these
works than on never art dealer
B. Joseph Duveen learned is strategy fro m his uncle, Henry Duveen who was the most successful
nineteenth-century art dealer
C. Joseph Duveen earned larger profits by selling complete an collections to wealthy buyer
D. Joseph Duveen tried to buy entire art collections so that he could hide the value of each individual artwork.

Paragraph 2: Besides the passage of this law, the early 1880, marked the public acknowledgement of French
Impressionism, a revolutionary art movement that emphasized from everyday life and the accurate depiction of light
and its changing qualities. Prior to this, Impressionist artists had been regarded as radical immoral who had declared
war on beauty. Impressionist paintings were no accepted by the Salon des L'Academiedes Beaux-Arts in Paris,
which had been since 1830 the most important exhibition forum in the world and was historically the an of
traditional standards of French painting. [■] In reaction to the conservative tendencies of this institution, a group of
Impressionist artists in Pars organized their own exhibition in 1874. [■]Reviews of the exhibition were mixed.
[■]But in 1881, the Impressionist painter Edouard Manet was named a knight of the Legion of Honor. In 1882, Paul
Cezanne was allowed to exhibit his work tor the first time at the Salon. [■]

9. Look at the four squares that indicate where the following sentence could be added to the passage
Many art critics thought the exhibited works looked unfinished.
Where would the sentence best fit? Click on a square [] to add the sentence to the passage

10. Directions: An introductory sentence for a brief summary of the passage is provided below. Complete the
summary by selecting the 3 answer choices that express the most important ideas in the passage. Some sentences do
not belong in the summary because they express ideas that are not presented in the passage or are minor ideas in the
passage. This question is worth 2 points Drag your choices to the spaces where they belong. To review the passage,
click on View Text

The modem art market was born in the late nineteenth century

第 228 页 共
373 页
2021 托福阅读真题

A. Changes in British inheritance law introduced in the 1880s meant that struggling landowners were able to put
their works of art on the London market.
B.The notion of the artist as genius began with the old masters in earlier centuries, but modern buyers lost interest in
the artists and focused instead on the works themselves
C.Art dealers like Joseph Duveen, who helped wealthy Americans become more interested in collecting European art,
continued to sell the paintings of old master S
D.In the early 1900s, Impressionists dominated the art market in London and the old masters dominated the market
in New York
E. New art movements that arose in the l ate nineteenth and early twentieth centuries contributed to the emergence of
markets in which the art dealer had a crucial function
F. London auction houses were more successful than those in New York, thanks to the improved fortunes of the
British nobility following changes in inheritance law.

Agriculture and the Classic Maya Collapse

Paragraph 1: The Classic Maya civilization, which flourished in the lowlands of what is now southern Mexico and
Central America Mesoamerica for some 600 years, collapsed around AD.800. The Classic Maya collapse is one of
the biggest mysteries in archaeology, made particularly perplexing by the combination of the cultural sophistication
attained by the Maya before the collapse and the relative suddenness of the collapse itself. Its cause has been
vigorously debated and has been variously attributed to invasion. social turmoil,drought, environmental degradation.
and the collapse of trade routes, among other factors. These theories have been subjected to exhaustive analysis by
researchers that involves both simulation studies and examinations of trading patterns and demographic and
ecological stresses that could have affected population densities.

第 229 页 共
373 页
2021 托福阅读真题

1. The word "perplexing" in the passage is closest in meaning to


A. complex
B. significant
C. interesting
D. Puzzling

2. The word "exhaustive" in the passage is closest in meaning to


A. Repeated
B. Recent
C. sophisticated
D. thorough

Paragraph 2: The archaeologist Patrick Culbert has examined population densities and the potential for agricultural
production in the southern lowlands. He has shown that because population densities rose to as many as 200 people
per square kilometer (518 per square mile) during the late Classic over such a large area, it was impossible for
people to adapt to bad times by moving to new land or emigrating He believes that the magnitude of the population
loss during the two centuries after AD. 800 was such that social malfunction alone cannot account for it. Failure of
the agricultural base must have been an important component in the collapse equation at the local level.

3. According to paragraph 2. what does Patrick Culbert believe is the reason that People did not move to new land
when conditions in the southern lowlands became difficult?
A. Other areas were less suitable for agricultural production
B. The areas surrounding the lowlands Were already densely populated
C. People miscalculated the potential for agricultural production in the lowlands
D. The extent of social malfunction was so great that people could no longer adapt

Paragraph 3: Maya agriculture became increasingly intense as populations rose, and both terrace (stepped) and
raised-field systems Covered large areas in many parts of the lowlands. At some of the larger sites like Tikal, the
people may have been transporting great quantities oi foodstuffs from distances of between 50 and100
kilometers(31 and 62 miles) in the short term the intensification strategies worked, but they carried the seeds of
collapse. The risks of climatic change, plant disease, erosion, and long-term declines in soil fertility are always
present in such enterprises. To continue functioning efficiently, the newly intensified systems would have had to
have been managed constantly. Just the repair of field systems after floods and rains would have required
watchful effort on a large scale, but there are no signs that the Maya made any social changes that enabled
them to achieve such a level of management, especially when so many people were engaged in public
construction projects and apparently in military activity (perhaps the Maya were under pressure from the
north.)

第 230 页 共
373 页
2021 托福阅读真题

4. Why does the author provide the information that at some sites "the people may have been transporting great
quantities of foodstuffs from distances of between 50 and 100 kilometers (31 and 62 miles)? .
A. To argue that agricultural intensification strategies should have been continued in the long term
B. B To emphasize the degree of intensification reached by Maya agriculture
C. To show why the intensification strategies were not successful in the short term
D. To give an example of people's response to the increasing intensity of agriculture

5. Which of the sentences below best expresses the essential information in the highlighted sentence in the passage?
Incorrect choices change the meaning in important ways or leave out essential information.
A. Nothing indicates that the Maya made any social changes that would have enabled them to achieve the level of
organization necessary to repair their field system
B. The Maya were unable to make any major social changes because so many people were engaged in public
construction projects and in military activity.
C. The repair of field systems after floods and rains required a high level of management and watchful effort on a
large scale
D. Most of the public construction projects were designed to support military activity, so there was little effort made
to repair field systems damaged by floods and rains.

6. According to paragraph 3, which of the following was required for the intensification system to succeed?
A. Increased population growth
B. Construction of terraces and raised-field systems
C. A great deal of management
D. Restriction of intensification practices to small rather than tape cities

Paragraph 4: Culbert believes that long-term environmental degradation was an important element in the scenario,
where short-term gains in productivity were followed by catastrophic declines. For example, as populations rose.
fallow cycles (periods when land is left unplanted) may have been shortened. so that there was increased competition
between crop plants and weeds; this is a problem that can be solved only by constant weeding. a very labor-intensive
activity. Shortened fallow cycles also lead to lower levels of plant nutrients and declining crop yields, and we do not
know whether the Maya tried to counteract these trends by systematic mulching or by planting soil-restoring crops.
The problem of erosion may have been even more acute. There are signs that the people lost much soil to runoff in
the lowlands for they did not build the terraces needed in time to retain the soil. Some of this erosion may have
resulted from extensive deforestation, At the same time, severe drought cycles played havoc with subsistence
agriculture in an environment with only moderately fertile soils and already overly dense populations. phenomena
now well-documented in Mesoamerican lake sediments Such droughts may indeed have been the trigger that brought
about the collapse in the southern lowlands.

7. What does paragraph 4 suggest about the Maya's reaction to environmental degradation?

第 231 页 共
373 页
2021 托福阅读真题

A. They could have delayed the collapse of their agriculture by mulching and planting soil-restoring crops
B. They were far more concerned about the threat posed by erosion than they were about the threat posed by drought
cycle
C. They were aware that they would achieve short-term gains in agricultural productivity only at the expense
long-term degradation.
D. They devoted most of their available labor power the constant weeding of their fields.

Paragraph 5: The sediments of Lake Chichancanab in the Yucatan show a recurring pattern of drought, occurring
about every 208 years.The period between AD 800 and 1000 was the driest of the middle to late Holocene, with two
arid peaks, the first coinciding with the Classic Maya collapse. Such serious droughts were generally devastating to
large-scale agricultural societies depending on surface water and dry agriculture.

8. The information in paragraph 5 supports which of the following conclusions?


A. Droughts were a very common phenomenon during the Classic Maya civilization.
B. Classic Maya agriculture needed to adjust constantly to periods of drought
C. Most of what is known about the Classic Maya period comes from the study of Mesoamerican lake sediments
D. The Classic Maya collapse could have resulted from a period of especially severe drought.

Paragraph 1: The Classic Maya civilization, which flourished in the lowlands of what is now southern Mexico and
Central America Mesoamerica for some 600 years, collapsed around AD.800. ■The Classic Maya collapse is one of
the biggest mysteries in archaeology, made particularly perplexing by the combination of the cultural sophistication
attained by the Maya before the collapse and the relative suddenness of the collapse itself. ■Its cause has been
vigorously debated and has been variously attributed to invasion. social turmoil,drought, environmental degradation.
and the collapse of trade routes, among other factors. ■These theories have been subjected to exhaustive analysis by
researchers that involves both simulation studies and examinations of trading patterns and demographic and
ecological stresses that could have affected population densities.■

9. Look at the four squares that indicate where the following sentence could be added to the passage.
In fact over 80 different explanations or variations of explanations have been identified.
Where would the sentence best fit?

10. Directions:
A. Population densities had increased greatly before the collapse. and archaeologist Patrick Culbert believes that the
huge population loss after AD.800 shows that the agricultural base must have failed.
B. As populations grew agriculture became increasingly intensive and possibly included shortened fallow cycles, a
strategy that temporarily yielded productivity gains but eventually led to catastrophic declines.
C. Analyses by archaeologist Patrick Culbert have provided new evidence for collapse theories that prioritize non
agricultural factors, such as social turmoil, invasions, or disruption of trade routes.

第 232 页 共
373 页
2021 托福阅读真题

D. It was earlier thought that population losses as significant as those that occurred after the collapse could result
from people adapting to bad times by moving to new lands or emigrating.
E. Large-scale intensive subsistence agriculture in moderately fertile soils would have been devastated by the period
of severe droughts that occurred around AD.800
F. The collapse of Classic Maya civilization might have been prevented the Maya had built more terraces while
maintaining their traditional agricultural production strategies.

Flamingo Behavior

Paragraph 1: Flamingos are a group of birds made up of six distinct species.They are one of the most recognizable
birds, thanks to their bright feathers,long legs,and the fact that they are native to five continents. Their pink color is
caused by the presence of car otenoids: pigments produced by plants, algae, and some bacteria, Animals that eat
these organisms acquire their carotenoids, sometimes altering them slightly. Although it is often said that flamingos
are pink because they eat shrimp, this is only true of individuals of larger species, which may teed on small aquatic
animals. The more common sources of caroteno ids are some types of bacteria,algae,and microscopic plants.

1. The word "acquire" in the passage is closest in meaning to


A. receive

第 233 页 共
373 页
2021 托福阅读真题

B. break down
C. enable
D. remove

2. According to paragraph 1.which TWO of the following statements about flamingos' food are true? To receive
credit,you must select TWO answers
A. Smaller-sized flamingo species some times eat small aquatic animals
B.Shrimp is the main source of carotenoids in the diet of most flamingo species
C. Car otenoids in the food ave responsible for the pink color of flamingo's feathers
D.The diet of most flamingos includes bacteria,algae,and microscopic plants.

Paragraph 2: After their stunning color,perhaps the n ext most distinctive thing about flamingos is their propensity
to stand on elegcientists have long wondered about this habit. One suggestion was that it permitted the birds to limit
muscle fatigue to on leg, allowing them to respond more easily to emergencies such as sudden attack s from
predators. Another idea was that the stance facilitated heat retention, in much the same way that folding arms across
the chest maintains warmth in human s.Scientists debated the merit of these two main theories for years, but studies
conducted in 2010 and 2011 provided increasing evidence for the latter. Specifically, researchers have shown that
flamingos stand on one leg more often in cooler temperatures. Furthermore, birds resting on one leg take longer to
initiate forward movement, suggesting that there is no muscular benefit to resting on one leg. Although the birds
seem to handle this tricky position with the grace and confidence of ballerinas, it is probably not very stable: the
length of one-legged resting periods has been shown to decrease on windier days, indicating that, even for flamingos,
it is not easy to maintain balance while "flaming ."

3.The word "initiate"in the passage is closest in meaning to


A. complete
B. begin
C.master
D. speed up

4. Why does the author mention that "there is no muscular benefit to resting on one leg"?
A. To challenge the claim that flamingos usually stand on one leg in cooler temperatures
B.To emphasize that flamingos have no known way of avoiding muscle fatigue while standing
C.To give a reason why scientists now think one of the theories about the flamingos one-legged stance is probably
wrong
D.To support the idea that the real advantage of the min one-legged stance is to be able to respond more quickly to
dangers

第 234 页 共
373 页
2021 托福阅读真题

Paragraph 3: Another aspect of flamingos distinctive stance that has caught the attention of scientists is whether
they curve their necks to the right or the left when laying their heads on their backs.Since there seem to be both
individual-level and flock level preferences toward bending the neck to the right, this suggests that,like humans,
flamingos have a right-side bias. A study of captive birds found that left-handed "flamingos are more likely to
become involved in aggressive encounters.This,along with the fact that birds in the same position can more easily
stand side by side without bumping into each other, indicates that later ality (a preference for one side of the body)
may promote flock cohesion.

5.Which of the following can be inferred from the discussion in paragraph 3 about the side to which a flamingo
curves its neck?
A.There tends to be less aggression in flocks where all the birds curve their necks to the same side.
B. Flamingos that curve their necks to the left are less likely to lay their heads on their backs
C.Flamingos will curve their necks to the left side of their bodies to display aggression to other birds.
D.Flamingos living in captivity are influenced by their human caretakers to have a right-side bias when curving their
necks.

Paragraph 4: Flamingos may keep an eye on the feeding behaviors of their flock-mates in order to determine the
quality of particular food patches.Although use of these sorts of social cues often leads to competition and
aggression in other species, this is not generally an issue for flamingos. In Africa, Europe, and Asia, two flamingo
species,lesser and greater flamingos, can share space peacefully because they have minim lover lap in prey species;
additionally.greater flamingos prefer less alkaline waters and stick closer to the shore. Similar spatial separations
have been observed in the Americas among Chilean, Andean,and napu flamingos. Because flamingo prey are often
found in incredibly high densities, thousands of flamingos can for age (search for food)within a single wetland
habitat without competing for each other food. Furthermore, depleted food stocks can regenerate within just a couple
of weeks, permitting the birds to revisit favorite hunting grounds fairly frequently.

6.Paragraph 4 suggests which of the foll owing about the Chilean, Andean, and puna flamingo species?
A. They prefer more alkaline waters than do both lesser and greater flamingos
B.They generally flock together in higher densities than do both lesser and great er flamingos.
C. They favor prey that live close to the shore.
D.They may share general water areas but tend to stay at different distances fro m the shore

7.Paragraph 4 discusses all of the following as reasons that flamingos generally share space peacefully when feeding
EXCEPT
A. Flamingos use certain social cues that help them avoid aggression.
B.Different species of flamingos differ in what they eat
C. Flamingos' prey is often highly about ant.

第 235 页 共
373 页
2021 托福阅读真题

D.Flamingos' food can reappear in a given area fairly soon after they consume what was available there earlier.

Paragraph 5: The formation of large feeding flocks ma y actually be beneficial, as it allows flamingos house their
own bodies to create cal m patches buffered from the wind. There is also safety in numbers: many flan in go
observers over the centuries have not ed that there is usually at least one bird on alert at all times,ready to use its
trumpet like voice to alert its foraging neighbors to the presence of danger. Surprisingly, the main source of tension
at feeding sites is encounters between younger and older birds.Adult flamingos appear to have little patience for
juveniles and expect the latter to give way: when they do not, these encounters may result in aggression. The one
time that this expectation maybe relaxed is during external disturbance events (such as visitations by humans), since
cautious flamingos tend to draw closer together while feeding.

8.According to paragraph 5 which of the following statements about the behavior of flamingos in large feeding
flocks is true?
A. On windy days,feeding flocks tend to move to calmer patches of water.
B. Adults behave aggressively when disturbed by outsiders near their feeding flock
C. To avoid tensions,juvenile birds remain in a separate flock and rarely enter the adult feeding flock.
D. One or more birds in a large feeding flock watch for danger and warn the other s when necessary.

Paragraph 3: ■Another aspect of flamingos distinctive stance that has caught the attention of scientists is whether
they curve their necks to the right or the left when laying their heads on their backs. ■Since there seem to be both
individual-level and flock level preferences toward bending the neck to the right, this suggests that, like humans,
flamingos have a right-side bias. ■A study of captive birds found that left-handed "flamingos are more likely to
become involved in aggressive encounters. ■This, along with the fact that birds in the same position can more easily
stand side by side without bumping into each other, indicates that laterality (a preference for one side of the body)
may promote flock cohesion.

9. Look at the four squares[■]that indicate where the following sentence could be added to the passage.
Individual birds that do not follow this general pattern may be at a disadvantage.
Where would the sentence best fit? Click on a square to add the sentence to the passage.

10.Directions An introductory sentence for a brief summary of the passage is provided below. Complete the
summary by selecting the THREE answer choices that express the most important ideas in the passage.

A. Evidence suggests that flamingos use their one-legged stance to retain warmth, and that individual birds tend to
curve t heir necks in the same direction to more easily stand close together.
B. Flamingos are generally more aggressive to flamingos of other species than to other birds of their own species,
especially when juveniles are present.

第 236 页 共
373 页
2021 托福阅读真题

C. Greater and lesser flamingos tend to forage in less alkaline waters from the shore, while flamingos in the
Americas prefer wetland habitats on shorelines.
D. Scientists are interested in studying the direction in which flamingos curve their necks,but research on this topic
has be en limited to flamingos held in captivity.
E. Flamingos look for high-quality food sources by observing the feeding behavior of other birds in their
flock,and.unlike in other species, this behavior does, not lead to competition among them
F. Feeding together i-large numbers increases the safety of flamingos and makes the water in which they feed calmer.

Turtle Basking

Paragraph 1: After a chilly night, the easiest way for a turtle to restore its optimum body temperature is to bask (lie)
in the sun. A log crowded with turtles, often of several different species, is a familiar sight is certain parts of the
world. The turtles certainly look as though they are doing their best to warm up often positioning themselves for
maximum exposure, their legs and webbed feet outstretched to soak up the sun. Some turtles will put up with
considerable discomfort to continue basking. Australian zoologist Grahame Webb watched captive Macleay River,
Jardine River, and saw-shelled turtles that appeared to be undergoing heat stress, at least on their heads, but
continued to bask anyway. The turtles gave off a watery discharge from their eyes, panted, and frothed at the mouth
They dipped their front feet into the water and wiped them over their heads in an apparent attempt to cool off. They

第 237 页 共
373 页
2021 托福阅读真题

did everything in fact,except go back into their pool. Did they have to wait until the interior of their bodies
reached the proper temperature, no matter how uncomfortable their extremities were?

1.The word optimum"in the passage is closest in meaning to


A.daytime
B.ideal
C.average
D.original

2.The word "apparent" in the passage is closest in meaning to


A.seeming
B.careful
C.serious
D.final

3.According to paragraph 1,why do turtles stretch out their legs and feet when they are out of the water?
A.To give their limbs a chance to dry out after long periods in the water
B.To ensure that their bodies absorb as much heat from the sun as possible
C.To mark their territory so that other turtle species do not take over their space
D.To relieve the stress felt when they become too warm

4.Why does the author ask the question,"Did they have to wait until the interior of their bodies reached the
proper temperature, no matter how uncomfortable their extremities were"?
A.To suggest a hypothesis about the observed behavior of the turtles
B.To call into question the observations made by Grahame Web
C.To help explain why it took so long for the turtles' bodies to reach the proper temperature
D.To suggest that the turtles behaved as they did because they were being held in captivity
Paragraph 2: What, in fact, are basking turtles doing?Warming themselves up, certainly; but slider turtles will
"bask"in the dark, or in the rain, so warming in the sun cannot always be the purpose of basking. Once a turtle goes
back into the water, it may lose heat more rapidly than it warmed in the sun. Australian zoologists Ben Manning and
Gordon Grigg found, to their surprise, that although Brisbane River turtles spend considerable time basking, once in
the river their body temperature scarcely differs from that of the water around them. The researchers wondered if
basking had to do with body temperature at all.

5.According to paragraph 2, what made researchers question whether the purpose of basking is always to raise body
temperature?
A.Some turtles spend much more time in the water than they do basking.

第 238 页 共
373 页
2021 托福阅读真题

B.Some turtles require more time than others do to become warmed by the sun.,
C.Some turtles bask when the sun is not shining
D.Some turtles cannot raise their body temperature significantly by basking.

Paragraph 3: However, in a recent study of Blanding's turtles in the state of Minnesota in the central United States,
zoologists Todd D. Sajwaj and Jeffrey W. Lang found that basking turtles warmed up quickly to a high body
temperature, sometimes greater than 28C, and kept that temperature for hours each day. On cool overcast days, the
turtles kept to the water, and their bodies stayed close to the water temperature. In spring, more than 90 percent of
the turtles crawled out of the water to bask in the morning, usually on mats of sedge (grass like plants) and in
protected areas where they could be very hard to find. By mid summer, that figure had dropped to 40 percent.

6.According to paragraph 3, which of the following is true of the Blanding's turtles studied by Sajwaj and Lang? .
A.They stop basking when their body temperature reaches 28C.
B.More of them bask in the middle of the summer than in the spring,
C.When the weather is cool and cloudy, they tend to stay in the water.
D.The high body temperature they achieve by basking is quickly lost once they return to the water.

Paragraph 4: Even on sunny spring days, some Blanding's turtles stayed in the water, perhaps because they needed
to feed more than they needed to bask. The basking turtles that Sajwaj and Lang examined all had food in their
stomachs. A hungry turtle may need to spend more time in the water, but for one that has fed, climbing out for a
bask may be a good idea even if it loses that heat when its sunbath is over. Basking after a meal can speed digestion,
especially in herbivores. Digestive enzymes work better at higher temperatures (though common snapping turtles in
Algonquin Provincial Park, Ontario, Canada, did not bask after feeding: instead, they buried themselves in sediment
under logs and organic debris).

7.Why does the author state that "The basking turtles that Sajwaj and Lang examined all had food in their
stomachs"?
A.To support the argument that turtles generally do not bask if they are hungry
B.To demonstrate that turtles are not active after they eat
C.To illustrate that weather conditions are a more important determining factor in basking behavior than hunger
D.To contradict the theory that Blanding's turtles spend mo re time in the water than basking

Paragraph 5: In September and October, female Blanding's turtles basked, but males did not. This sexual difference
is known in other turtles, too. Perhaps female turtles need extra warmth for egg production, while male turtles need
to concentrate on finding mates instead of sitting around on a log. This balance between temperature and a turtle's
other needs may be a delicate one. In the former Yugoslavia, in the north of their range, summers are cool and
Hermann's tortoises must spend longer basking in the morning to reach their preferred temperature of around 34°C.

第 239 页 共
373 页
2021 托福阅读真题

This presents females with a problem. Female Hermann's tortoises may be twice the size o f males, and so take
longer to warm up. However, if they take too long to bask, they may not have time to get the amount of food they
need to produce a full clutch of eggs. In the end, they must give up basking time in favor of feeding time, especially
on cool days,and operate at a lower temperature,trading off the ability to heat up quickly for a large body capable of
producing,and holding, as many eggs as possible.

8.According to paragraph 5, why must female Hermann's tortoises limit their basking time?
A.They need to devote time to looking for mates.
B.They need enough time to feed so that they can produce many eggs.
C.They must make room for males who need more time to warm up, especially on cool days.
D.They must be careful to avoid exceeding their preferred temperature of 34"C.

Paragraph 3: ■However, in a recent study of Blanding's turtles in the state of Minnesota in the central United States,
zoologists Todd D. Sajwaj and Jeffrey W. Lang found that basking turtles warmed up quickly to a high body
temperature, sometimes greater than 28C, and kept that temperature for hours each day. ■On cool overcast days, the
turtles kept to the water, and their bodies stayed close to the water temperature. ■In spring, more than 90 percent of
the turtles crawled out of the water to bask in the morning, usually on mats of sedge (grass like plants) and in
protected areas where they could be very hard to find. ■By mid summer, that figure had dropped to 40 percent.

9. Look at the four squares, that indicate where the following sentence could be added to the passage.
Daily weather and season of the year were important factors that influenced basking behavior of the group as
a whole.
Where would the sentence best fit? Click on a square, to add the sentence to the passage.

10.Directions: An introductory sentence for a brief summary of the passage is provided below. Complete the
summary by s electing the THREE answer choices that express the most important ideas in the passage. Some
sentences do not belong in the summary because they express ideas that are not presented in the passage or are minor
ideas in the passage. This question is worth 2 points.

A. A cold turtle can get much warmer by basking,but the extra heat is lost when it returns to the water,and there is
evidence that turtles sometimes continue to bask even though doing so causes discomfort.
B. Research suggests that,for at least some turtles, one important function of basking may be to help increase the rate
at which food is digested.
C. Extra warmth may benefit egg production, but time spent basking is time lost from finding the food needed to
produce a full clutch of eggs, so females sometimes achieve less than ideal temperatures.
D. The basking behavior of captive turtles appears to differ significantly from that of wild turtles, with captive turtles
continuing to expose themselves to the sun until they develop signs of severe heat stress.

第 240 页 共
373 页
2021 托福阅读真题

E. Researchers have discovered that there are turtles that do not bask at all, but these turtles spend the majority of
their time in sediment, under logs, and in organic debris rather than in the water.
F. Some female turtles can become so large that they ca p-reach their preferred temperature on cool summer days no
matter how much time they spend basking.

British Agriculture

Paragraph 1: England was one of the first nations in the modern period to increase the efficiency of its agricultural
production, making it possible to produce the same or more. While using fewer workers. By the end of the
seventeenth century. England was already in advance of most of continental Europe in agricultural productivity, with
only about 60 percent of its workers involved primarily in food production. Although the actual number of workers
in agriculture continued to grow until the middle of the nineteenth century, the proportion declined steadily to about
36 percent at the beginning of the nineteenth century, to about 22 percent in the mid-nineteenth century (when the
absolute number was at its maximum), and to less than 10 percent at the beginning of the twentieth century.

第 241 页 共
373 页
2021 托福阅读真题

1. Which of the following can be inferred from paragraph 1 about agricultural production in the seventeenth century?
A. England had stopped focusing on increasing its agricultural productivity.
B. In England, the proportion of workers involved in food production increased rapidly.
C. In most of continental Europe, more than 60 percent of workers were primarily involved in food production.
D. In most of continental Europe, the number of workers in agriculture had begun to decrease.

2. In paragraph 1. why does the author present the change over time of the percentage of England's workers who
were employed in agriculture?
A. To explain why workers in Europe sought jobs in agriculture more than in any other sector of the economy
B. To compare the skills of England's agricultural workers with those of the rest of Europe
C. To indicate the rate and extent of improvements in England's agricultural productivity
D. To show how the proportion of workers in agriculture changes as the proportion of land in agricultural use
changes

Paragraph 2: The means by which England increased its agricultural productivity owed much to trial-and-error
experimentation with new crops and new crop rotations Turnips, clover, and other fodder crops (plants used to feed
livestock) were introduced from the Netherlands in the sixteenth century. and became widely diffused in the
seventeenth. Probably the most important agricultural innovation before scientific agriculture was introduced in the
nineteenth century was the development of so-called convertible husbandry involving the alternation of field crops
with temporary pastures in place of permanent cultivated fields and pastures. This had the double advantage of
restoring the fertility of the soil through improved rotation including leguminous crops, and of maintaining a larger
number of livestock, thus producing more manure for fertilizer as well as more meat, dairy produce and Wool. Many
landowners and farmers also experimented with selective breeding of livestock.

3. According to paragraph 2, next to the introduction of scientific agriculture in the nineteenth century, the most
important change in the way the English produced food was
A. the introduction of fodder crops from the Netherlands
B. the recognition that trial-and-error experimentation did not work
C. the selective breeding of livestock
D. the alternating use of land for crop cultivation and animal pasture

Paragraph 3: An important condition for both the improved rotations and selective breeding was the enclosure (the
conversion of common land into private plots) and consolidation of the fields. Under the traditional open field
system it was difficult, if not impossible to obtain agreement among the many participants on the introduction
of new crops or rotation: and with livestock grazing in common herds was equally difficult to manage
selective breeding. The most famous enclosures were those carried out by acts of Parliament (the British governing

第 242 页 共
373 页
2021 托福阅读真题

body) between 1760 and 1815 Enclosure by private agreement,however, had been going on almost continually from
the late Middle Ages it was especially active in the late seventeenth and the first six decades of the eighteenth
centuries. By that time more than half of England's arable land (Land that can be farmed productively) had been
enclosed.

4. Which of the sentences below best expresses the essential information in the highlighted sentence in the passage?
Incorrect choices change the meaning in important ways or leave out essential information.
A. The traditional open field system made it very hard for any new crop practices to be introduced and for selective
breeding to be carried out.
B. Under the traditional open field system, many farmers planted new crops and practiced crop rotation as well as
carrying out selective breeding of the livestock that grazed in common herds
C. Selective breeding of livestock as well as new crops and new rotations were practically impossible to introduce
because the farmers all shared a common distrust of anything new.
D. Because the open fields were shared by all the farmers, any planting scheme had to be agreed to by all before it
could be carried out.

5. The word "emerged" in the passage is closest in meaning to


A. Multiplied
B. Developed
C. Threatened
D. Ceased

6. The word "boosting" in the passage is closest in meaning to


A. influencing
B. Changing
C. Protecting
D. Raising
Paragraph 4: The new agricultural landscape that emerged to replace villages surrounded by their open fields
consisted of fields that were compact, consolidated. and closed-in by walls. fences. or hedges Along with the
processes of enclosure and technological improve me, gradual tendency toward larger farms emerged, further
boosting productivity. By 1851 about one-third of the cultivated acreage was in farms larger than 300 acres: farms
smaller than 100 acres accounted for only 22 percent of the land. Even so, the occupants of the small farms
outnumbered those of the others almost two to one. The reason for this is that the small farmers were
owner-occupiers who farmed the land with the help of farmily labor; the larger farmers rented pieces of their
property to others and hired landless agricultural workers. It used to be thought that the enclosures led to a loss of
population in the countryside, but in fact the new techniques of cultivation associated with them actually increased
the demand for labor. Not until the second half of the nineteenth century With the introduction of such farm

第 243 页 共
373 页
2021 托福阅读真题

machinery as threshers, harvesters, and steam plows did the absolute size of the agricultural labor force begin to
decrease.

7. According to paragraph 4, why did the enclosures lead to an increase in the agricultural workforce?
A. Because a considerable workforce was needed to carry out the process of enclosure
B. Because enclosure brought with it new practices that required more labor
C. Because enclosure encouraged the introduction of farm machinery that required specialized labor
D. Because enclosure allowed entire villages to share the same land

Paragraph 5: In the meantime, the increasing productivity of English agriculture enabled it to feed a burgeoning
population at steadily rising standards of nutrition. For about a century, from1660 to 1760. English farmers produced
surplus for export; after that period, the rate of population growth overtook the rate of increase of productivity. The
relatively prosperous rural population provided a ready market for manufacturing goods, ranging from agricultural
implements to such consumer products as clothing, pewter-ware and porcelain

8. According to paragraph 5. what ended England's position as a significant agricultural exporter?


A. Large increases in population
B. A decline in the nutritional value of its agricultural production
C. Increases in agricultural productivity in countries that bought its exports
D. A sudden drop in its agricultural productivity

Paragraph 5: ■In the meantime, the increasing productivity of English agriculture enabled it to feed a burgeoning
population at steadily rising standards of nutrition. ■ For about a century, from1660 to 1760. English farmers
produced surplus for export; after that period, the rate of population growth overtook the rate of increase of
productivity. ■The relatively prosperous rural population provided a ready market for manufacturing goods, ranging
from agricultural implements to such consumer products as clothing, pewter-ware and porcelain. ■

9. Look at the four squares that indicate where the following sentence could be added to the passage.
Indeed, for a long period the increases in productivity exceeded the growing demands of the domestic market.
Where would the sentence best fit? Click on a square to add the sentence to the passage.

10. Directions: An introductory sentence for a brief summary of the passage is provided below. Complete the
summary by s electing the THREE answer choices that express the most important ideas in the passage. Some
sentences do not belong in the summary because they express ideas that are not presented in the passage or are minor
ideas in the passage. This question is worth 2 points.

England was one of the first nations in the modern period to increase its agricultural productivity

第 244 页 共
373 页
2021 托福阅读真题

A. England improved its agricultural productivity by introducing new crops and new rotations that included the use
of croplands as temporary pastures
B. The creation of private fields through enclosure led to advances in productivity, by making it easier to innovate
and by making selective livestock breeding possible.
C. Agricultural productivity was also improved by an increase in the number of larger farms
D. Turnips, clover, and other fodder crops were replaced through trial-and-error experimentation
E. The enclosure process began through the action of Parliament in 1760 and ended in 1815
F. As farms became increasingly productive, the percentage of the population employed in agriculture grew

Evidence for the First Domesticated Rice in China

Paragraph 1: In 1988 evidence of domesticated rice (cultivated by humans) dating to at least 7500 BCE. was found
in an excavation near the Vanzetti River in China. This discovery led to a regional search for older sites that might
provide indications of the transition from wild to domesticated rice. Given the area’s wet climate, researchers had
little hope of finding preserved plants. Fortunately, however, living plants take up silica in the soil as they absorb
groundwater, and this silica is then redeposited as microscopic minerals known as monoliths when the plant decays.
Because they are inorganic, monoliths often survive in the soil long after all other traces of plants have disappeared,
which helps researchers study these plants. Moreover, different species plants-and indeed different parts of a single
plant-produce differently shaped monoliths.

第 245 页 共
373 页
2021 托福阅读真题

Paragraph 2: Just as some plants produce more pollen grains than others, some produce more monoliths. Grasses
are big producers, and while the pollen grains from different species are nearly identical, their monoliths are quite
distinct. Deborah Pearsall from the University of Missouri pioneered the study of those from rice, finding that the
most distinctive are produced in the cells of the glume and the husk of the grain, as these have larcie conical hairs or
peaks. The presence of such monoliths in an otherwise empty soil is a definite sign that rice plants once grew there.
The crux of Pearsall’s work, however, is that glume monoliths can be used to identify whether the long -gone
rice plants were wild or domestic--whether they grew in a swamp (an area of low-lying ground where water
collects) or were cultivated in a rice field--since the monoliths from domesticated plants are simply bigger
than those from wild varieties, matching the increase in grain size.

1, The word “pioneered” in the passage i closest in meaning to


A, contributed to
B, first started
C, supervised
D, influenced

2, Which of the sentences below best expresses the essential information in the highlighted sentence in the passage?
Incorrect choices change the meaning in important ways or leave out essential information.
A, Pearsall’s work showed that both the drains and the monoliths from domesticated rice plants cultivated in rice
fields are bigger than those from wild varieties grown in swamps
B, Central to Pearsall’s work is the theory that the monoliths from domesticated rice plants have become bigger over
time but used to be similar in size to those from wild rice varieties
C, Pearsall’s work with glume monoliths showed that long-gone wild rice plants were grown in swamps, whereas
long-gone domesticated rice plants were cultivated in rice fields.
D, Pearsall’s work showed that long-gone domesticated rice plants can be distinguished from wild ones by the larger
size of the glume monoliths they produced.
3, According to paragraph 2, phytolithy differ from pollen grains in that monoliths are
A, produced more often than pollen grains are
B, produced in the cells of the husk unlike pollen grains
C, more different across species than pollen grains are
D, more likely to be found in empty soil than pollen grains are

Paragraph 3: Cave sediments excavated by archaeologists Richard MacLeish and Van Wenming may hold the key
to identifying when and where rice cultivation began, given Pearsall’s discovery that domesticated plants are bigger
than wild varieties. Diaotonghuan Cave is located in the side of a small limestone hill within a region of swamps
known as the Dayuan basin, just the south of the Yangtze River-no more than 50 kilometers from recently discovered

第 246 页 共
373 页
2021 托福阅读真题

stands of wild rice. MacLeish and Wenming excavated a 5-meter deep trench in the center of the cave and revealed
at least sixteen occupation layers, neatly layered one on top of another. The eight uppermost layers covered the
period from 12,000 B.CE to 2000 B.C.E; the age of the lower layers was not determined. Each excavated layer
contained animal bones and stone tools; fragments of pottery were present in those dating back to 10,000 BCE

4, Paragraph 3 suggests which of the following about the excavated trench at the center of the cave?
A, It revealed the age of all sixteen layers found
B, It revealed that animal bones were present in all but the lowest undated occupation layers.
C, It revealed that pottery probably was not in use in all of the time periods represented by the layers.
D, It revealed that stone tools were not present beneath the uppermost layers.

Paragraph 4: While plant remains were entirely absent, the monoliths from each occupation layer showed that
several types of rice had once been gathered. Rice monoliths were very rare in the lower layers of the cave; the few
present might have arrived within dry leaves blown by the wind or on the hoofs or in the feces of animals that used
the cave for shelter. At about 12,000 B.C.E, however,there is a dramatic increase in the number of rice monoliths,
which must reflect rice being gathered and eaten by human occupants of the cave. The monoliths are small,
indicating that they had come from wild plants, most likely gathered from the edge of nearby swamps. The date
marks the particularly warm and wet conditions of the late glacial interstadial (brief periods of milder climate that
occurred during the last ice age), a time when stands of wild rice may have begun to flourish in the Yangtze basin.
After 12,000 B.C.E. rice monoliths remain abundant in the cave, except within layers dating to between 10,800 and
9600 B.CE-a period coinciding with the Young Drvas. During that period of cold and drought, wild rice, a
subtropical water- dependent plant, had evidently, and not surprisingly, become very scarce.

5, Which of the following statements best describes the relationship between paragraphs 3 and 4 of the passage?
A, Paragraph 3 describes an excavation site where stands of wild rice were discovered, while paragraph 4 discusses
how the wild rice became domesticated
B, Paragraph 3 explains the methods used by researchers to identify an important excavation site, while paragraph 4
presents what the researchers discovered at the site.
C, Paragraph 3 explains the organization and dating of an excavation site, while paragraph 4 presents contradictory
evidence for the organization and dating of the site.
D, Paragraph 3 describes the general characteristics of an excavation site, while paragraph 4 discusses specific
findings about rice at the site.

6, According to paragraph 4, why were rice monoliths far less abundant in cave layers dating to between 10,800 and
9600 5.C.E?
A, Because those layers mark a period in which fewer humans would have occupied the cave
B, Because the climate had become much less suitable for rice-plants

第 247 页 共
373 页
2021 托福阅读真题

C, Because the wild rice growing in the period was not a subtropical water dependent plant
D, Because during that period only small grained wild rice would have been available for gathering and eating

Paragraph 5: After the return of warm and wet conditions, rice was once again used as a major source of food.
Successive layers within Diaotonghuan Cave show a gradual increase in the proportion of large monoliths, reflecting
the appearance of the first domesticated plants. By 7500 BCE there was an equal use of wild and domesticated rice
plants; a thousand years later, all traces of wild rice had completely disappeared.

7, The word traces in the passage is closest in meaning to


A, forms
B, remains
C, supplies
D, uses

8, According to paragraph 5, which of the following statements about rice in the region around Diaotonghuan Cave
is true?
A, Rice had become fully domesticated by a thousand years after 7500 B.C.E
B, Wild rice increased in size with the return of warm, wet conditions.
C, The first domesticated rice appeared just before the return of warm, wet conditions.
D, Domesticated rice varieties were used more often than wild rice varieties by 7500 B.CE

Paragraph 1: In 1988 evidence of domesticated rice (cultivated by humans) dating to at least 7500 BCE. was found
in an excavation near the Yangtze River in China. This discovery led to a regional search for older sites that might
provide indications of the transition from wild to domesticated rice. Given the area’s wet climate, researchers had
little hope of finding preserved plants. Fortunately, however, living plants take up silica in the soil as they absorb
groundwater,and this silica is then redeposited microscopic minerals known as monoliths when the plant decays.
█Because they are inorganic, monoliths often survive in the soil long after all other traces of plants have disappeared,
which helps researchers study these plants. █Moreover, different species of plants-and indeed different parts of a
single plant-produce differently shape monoliths.█

Paragraph 2: Just as some plants produce more pollen grains than others, some produce more monoliths. █ Grasses
are big producers, and while the pollen grains from different species are nearly identical, their phytolith are quite
distinct. Deborah Pearsall from the University of Missouri pioneered the study of those from rice, finding that the
most distinctive are produced in the cells of the glume and the husk of the grain, as these have large conical hairs or
peaks. The presence of such monoliths in an otherwise empty soil is a definite sign that rice plants once grew there.
The crux of Pearsall’s work however, is that glume monoliths can be used to identify whether the long-gone rice
plants were wild or domestic-whether they grew in a swamp (an area of low-lying ground where water collects) or

第 248 页 共
373 页
2021 托福阅读真题

were cultivated in a rice field-since the monoliths from domesticated plants are simply bigger than those from wild
varieties, matching the increase in grain size.

9, Look at the four squares that indicated where the following sentence could be added to the passage.
These characteristics make it possible to use monoliths to identify plants that once grew in an area.
Where would the sentence best fit? Click on a square I to add the sentence to the passage

10, Directions: An introductory sentence for a brief summary of the passage is provided below. Complete the
summary by selecting the THREE answer choices that express the most important ideas in the passage. Some
sentences do not belong in the summary because they express ideas that are not presented in the passage or are minor
ideas in the passage.

Researchers have discovered that by 7500 B.C.E. rice was being domesticated in the Dayuan basin in China.

A, monoliths are much more likely to be preserved than other rice plant materials, and variations in their size reveal
whether they came from domesticated or wild rice varieties.
B, Rice in China was first cultivated in Diaotonghuan Cave, a very important archeological site characterized by
mostly dry soil that has been extensively excavated in recent times.
C, monoliths found in occupation layers from an excavated cave indicate that by around 12,000 BCE. humans had
began gathering and eating wild rice growing in nearby swamps
D, Before the discovery of monoliths, researchers relied on preserved pollen grains left in the soil long after all other
organic plant material had disappeared to identify what plants once grew there
E, The abundance of rice monoliths found in occupation layers before 10,800 B.CE and after 9600 B.CE. and their
absence in layers from the period in between is evidence of a prolonged cold, dry period
F, An analysis of excavated layers of the Diaotonghuan Cave suggests that the availability of wild rice varied with
climate changes, but domesticated rice eventually replaced wild rice.

Machines and Manufacturing

Paragraph 1: The tremendous growth in European industry in the eighteenth and nineteenth centuries was the result
of a number of changes, technology foremost among them. The accumulation and diffusion of technical knowledge
necessary for manufacturing began in the countryside, where handicraft operations were gradually enlarged and
mechanized. Often it was small- and medium-sized producers, and the occasional amateur experimenter in a
barn,who were the inventors and innovators. Numerous small inventions, applied are diffused on both sides of the
Atlantic, gradually built up a stock of technical knowledge and practice that was widely available.

1, In paragraph 1, which of the following is NOT mentioned as a factor that contributed to the growth in European

第 249 页 共
373 页
2021 托福阅读真题

industry during the eighteenth and nineteenth centuries?


A, The spread of information across the Atlantic
B, Inventions made by individuals and small producers
C, The growth of handicraft operations in the countryside
D, The emergence of manufacturing centers in large cities

Paragraph 2: Most famous of these inventors was James Watt (1739-1819) of Scotland. who succeeded in making
steam engines more efficient Steam engines burned coal to boil water, which condensed into steam that was used to
drive mechanized devices. Watt devised a way to separate steam condensers from piston cylinders so that pistons
could be kept hot, and therefor running constantly. This set the stage for a fuel efficient engine. Early prototypes of
Watt’s engine were used to pump water out of mines. After moving to Birmingham in 1774. Watt joined forces with
the industrialist Matthew Boulton (1728-1809). who marketed the steam engine won an extension of the patent for
another twenty-five years, and set up a special laboratory for Watt so that he could refine his devises. Thus, technical
exploration joined forces with the interests of business: collaboration between inventors and entrepreneurs was a
sign of the times. Moreover, perhaps the most important aspect of Watt’s engineering feat was that it was subject to a
stream of improvement and adaptation, not just from Boulton and company, but from its competitors as well.

2, The word “refine” in the passage is closest in meaning to


A, test
B, get assistance with
C, improve
D, assemble

3, According to paragraph 2, what is one of the roles Matthew Boulton played in the development of the steam
engine?
A, He made the steam engine more fuel efficient
B, He helped Watt to sell the steam engine.
C, He introduced the steam engine to miners.
D, He convinced Watt to move his laboratory to Birmingham.

Paragraph 3: The steam engine, driven by burning coal, provided vastly increased power and sparked a revolution
in transportation. Steam-powered ships and railroads, built once inventors were able to construct lighter engines that
required less coal to run. slashed the time and cost of long-distance travel. Steam power’s diffusion accelerated when
iron-making improved, allowing for the production of railroad track and cables used to hang suspension bridges. The
first public rail line opened in 1830 in England between Manchester and Liverpool. During the next twenty year,
railway mileage increased from less than 100 to almost 25,000 in England, France Russia. and the German-speaking
countries. Steamships appeared in the 1780s in France. Britain, and the United States. and in 1807 Robert Fulton

第 250 页 共
373 页
2021 托福阅读真题

inaugurated the first commercially successful route between New York City and Albany on the Hudson River. A
century of toying with boilers and pistons culminated in the radical reduction of distances. Moreover, steam-powered
engines also improved sugar refining, pottery making, and many other industrial processes. Mechanizing processes
that would have taken much longer and been subject to human error if done by hand enabled manufacturers to make
more products at cheaper cost.

4, According to paragraph 3. what helped steam power to spread more quickly?


A, The replacement of coal with other fuel sources
B, The construction of smaller, lighter ships
C, Better iron-making techniques
D, A decrease in the cost of coal

Paragraph 4: Textile production was one of the areas that benefited from both technical changes and the
consolidation of different stages of the work in a factory. With new machinery, a single textile operator could handle
many looms and spindles at once, and could produce bolts of cloth with stunning efficiency. Gone were the hand
tools, the family traditions, and the loosely organized and dispersed systems of households producing cloth in their
homes for local merchants to carry to markets. The material was also stronger, finer, and more uniform. Thanks to
such innovations. British cotton output increased tenfold between 1770 and 1790. leading to a 90 percent decline in
the price of cloth between 1782 and 1812.

5, The word “dispersed” in the passage is closest in meaning to


A, familiar
B, simple
C, inconvenient
D, scattered

6. In paragraph 4 why does the author refer to the hand tools, family traditions, and loose organization of household
systems of cloth production?
A, To express regret that a traditional way of life was disappearing
B, To help explain why home producers preferred not to use machinery to produce their cloth
C, To suggest that home producers put more care into cloth production than factory workers did
D, To provide a contrast to the methods and machinery used in factory textile production

7, According to paragraph 4, new machinery changed the cloth industry in all of the following ways EXCEPT:
A, The quality of cloth greatly improved.
B, Workers became increasingly productive.
C, Cloth became less expensive

第 251 页 共
373 页
2021 托福阅读真题

D, Home cloth-makers competed with or another.

Paragraph 5: Most raw cotton for the British cloth industry had come from colonial India until 1793, when the
American inventor Eli Whitney (1765-1825) patented a device called a cotton gin that separated cottonseeds from
fiber. Cotton farming quickly spread through the southern states-from South Carolina into Georgia. Alabama
Mississippi, and Louisiana-as the United States came to produce more than 80 percent of the world cotton supply by
the1850s Thus, the American south became a supplier of raw cotton to Britain.

8, Paragraph 5 suggests that which of the following occurred in Britain by the 1850s?
A, It obtained most of its cotton from the United States.
B, It began regularly using a device similar to Whitney’s cotton gin.
C, It had greatly reduced its dependence on foreign cotton.
D, It had moved much of its textile production to the United States.

Paragraph 3: The steam engine, driven by burning coal, provided vastly increased power and sparked a revolution
in transportation. Steam-powered ships and railroads, built once inventors were able to construct lighter engines that
required less coal to run. slashed the time and cost of long-distance travel. Steam power’s diffusion accelerated when
iron-making improved, allowing for the production of railroad track and cables used to hang suspension bridges. The
first public rail line opened in 1830 in England between Manchester and Liverpool. During the next twenty year,
railway mileage increased from less than 100 to almost 25,000 in England, France Russia. and the German-speaking
countries. █ Steamships appeared in the 1780s in France. Britain, and the United States. and in 1807 Robert Fulton
inaugurated the first commercially successful route between New York City and Albany on the Hudson Riv er. █A
century of toying with boilers and pistons culminated in the radical reduction of distances.█ Moreover,
steam-powered engines also improved sugar refining, pottery making, and many other industrial
processes.█Mechanizing processes that would have taken much longer and been subject to human error if done by
hand enabled manufacturers to make more products at cheaper cost.

9, Look at the four squares I that indicate where the following sentence could be added to the passage.
For example. a journey across the ocean that would have taken more than five weeks in 1800 took o nly
eighteen days by the mid-nineteenth century.
Where would the sentence best fit? Click on a square to add the sentence to the passage.

10, Directions: An introductory sentence for a brief summary of the passage is provided below. Complete the
summary by selecting the 3 answer choices that express the most important ideas in the passage. Some
sentences do not belong in the summary because they express ideas that are not presented in the passage or are minor
ideas in the passage. This question is worth 2 points.

第 252 页 共
373 页
2021 托福阅读真题

The invention and spread of new technology enabled industrial growth in the eighteenth and nineteenth
centuries.

A, The development of technology originated in urban European industrial areas and largely eliminated the smaller
handicraft operations in the countryside.
B, The creation of lighter engines enabled steam-powered ships and railroads to spread quickly and made
long-distance travel faster and cheaper
C, Many individuals who had been involved in household textile production moved to the cities, where they could
earn more money by handling many looms at once.
D, Collaboration between individual inventors and entrepreneurs was crucial to technical advances like the invention
of the steam engine
E, The expansion of steam-powered engines made sugar refining and pottery making two of Britain’s most
successful industries.
F, Mechanized textile production, which became increasingly reliant upon cotton from North America, was much
more efficient than the household system it replaced.

Fish and the Sense of Smell

Paragraph 1: The sense of smell is an organism's ability to detect chemical cues in the environment. Smells are
especially useful in water, where murky conditions make vision unreliable. Fish use smells for finding food and
mates, and for identifying danger. Unlike those of other vertebrates, fishes’ nostrils do not do double duty as both
organs of smell and openings for breathing; they are used exclusively for smell. The sophistication of fishes’
olfactory (smelling) organs varies greatly, but there is a basic design. Each nostril is populated by layers of
specialized cells composing the olfactory epithelium (lining), which is folded upon itself to save space. Thousands of
tiny cilia (hair-like structures on the cells) pulse in sequence to propel water into and out of the sense organ. Signals

第 253 页 共
373 页
2021 托福阅读真题

from the epithelium are sent to the olfactory part of the brain.

1:According to paragraph 1, all of the following statements about a fish's smelling organs are true EXCEPT:
A: They are used for sensing chemical cues.
B:They make breathing easier.
C:They contain folded layers of specialized cells.
D:Their cilia act together to push water in and out.

Paragraph 2: One of a fish's most useful adaptations is the production of an “alarm chemical in the presence of
danger, such as a predatory fish or a spear fisherman. We owe it to Austrian biologist Karl von Frisch (1886-1982)
for discovering this phenomenon in the world of fish senses. When he accidentally injured one of his captive
minnows, von Frisch noticed that other fishes in the tank began darting back and forth and freezing in place—classic
predator-evading behavior. Experiments by von Frisch and others showed that injured minnows (among other fish
species) release a pheromone—a secreted or excreted chemical that triggers a social response in members of the
same species. Detecting this particular pheromone causes agitated reactions by the minnows. Von Frisch coined the
term schreckstoff for these pheromones.

2: Which of the following caused the “classic predator-evading behavior’ of fish that von Frisch noticed?
A: An injury caused by another fish
B: The detection of a pheromone given off by a member of the same fish species
C: The sight of darting and freezing behavior in other fish
D: The discovery of the presence of predatory fish nearby

Paragraph 3: The cells that release schreckstoff are located in the skin, and they are fragile enough that they will
rupture and release the substance if a fish is placed on moist paper. And it is potent: a thousandth of a milligram of
chopped skin is enough to elicit a fright reaction from another fish in a 14-liter aquarium. That is like chopping a
piece of candy into 20 million pieces, dropping one piece into a sink full of water, and then trying to taste the
sweetness. Schreckstoff must have evolved long ago, for it is produced by several families of bony fishes. As a
freely available signal, schreckstoff acts like a fire alarm that can be used by other nearby fishes, including different
species that may also recognize it. Fathead minnows, when they smell the body waste of northern pikes that have fed
on other fathead minnows or on brook sticklebacks—both of which produce schreckstoff in their skin—immediately
flee to hiding places or form tighter swimming groups. But if the pikes have fed only on sword tail fishes—which do
not the minnows show no signs of fear. Thus, it is not the smell of the pike that minnows are reacting to—instead,
they detect and react to the schreckstoff from the pike’s victims. It is probably due to olfactory skills like the
minnow’s that pikes refrain from eliminating waste in their own hunting grounds.

3: Why does the author compare chopped fish skin in an aquarium to “chopping a piece of candy into 20 million

第 254 页 共
373 页
2021 托福阅读真题

pieces, dropping one piece into a sink full of water, and then trying to taste the sweetness’?
A: To support the claim that the cells that release schreckstoff are very fragile
B: To demonstrate how familiar the signal of schreckstoff is to certain species of fish
C: To emphasize how unusually powerful schreckstoff is
D: To suggest that schreckstoff likely evolved long ago

4: According to paragraph 3, all of the following are true about schreckstoff EXCEPT:
A: It is released from cells in the skin.
B: It is not produced by all species of fish.
C: It helps some prey fish avoid predatory fish.
D: It can only be recognized by other fish of the same species.

5: According to paragraph 3, in which of the following situations do minnows not react to the body waste of pikes?
A: When the body waste is eliminated in the same places where pikes hunt
B: When the body waste is produced by a species of pike different from northern pikes
C: When the body waste is produced from the consumption of swordtail fishes by pikes
D: When nearby brook sticklebacks show no signs of fear of the body waste of pikes

Paragraph 4: The schreckstoff reaction illustrates how fishes can extract subtle clues from waterborne chemicals.
But schreckstoff is not the only way to detect a fish foe by fragrance. There is the old-fashioned way of simply
recognizing the smell of the predator. Juvenile lemon sharks react to the odor of American crocodiles, who
sometimes prey on them. For an Atlantic salmon, it depends on what its predator has been eating. In a study
conducted at Swansea University in Wales, juvenile salmon that had never been exposed to predators were presented
with water containing traces of body waste from one of their natural enemies, the Eurasian otter. The salmon showed
a fear response only if the otter had been dining on salmon. In those cases they fled the smell and then remained still,
and they breathed faster. Salmon exposed to blank water or to body waste from otters on a nonsalmon diet were
unfazed. The scientist concluded that Atlantic salmon apparently do not innately recognize otters as a threat—they
perceive them as a danger only if salmon is on the menu. This generalized mode of predator detection works well
because it does not require learning the smell of different predators.

6: The word “subtle” in the passage is closest in meaning to


A: helpful
B: certain
C: significant
D: slight

7: The word “innately” in the passage is closest in meaning to

第 255 页 共
373 页
2021 托福阅读真题

A: by their nature
B: immediately
C: easily
D: specifically

8 :Paragraph 4 supports which of the following statements about predator detection by salmon?
A: Salmon recognize various predators as a threat when the predators have fed on salmon.
B: Salmon do not necessarily recognize predators as a threat unless they have been previously exposed to the
predators.
C: Salmon have to learn the smell of different predators in order to recognize them as a threat.
D: Salmon recognize otters that have not eaten salmon as a threat.

Paragraph 2: One of a fish's most useful adaptations is the production of an ‘alarm chemical’ in the presence of
danger, such as a predatory fish or a spear fisherman. █We owe it to Austrian biologist Karl von Frisch (1886-1982)
for discovering this phenomenon in the world of fish senses. █When he accidentally injured one of his captive
minnows, von Frisch noticed that other fishes in the tank began darting back and forth and freezing in place—classic
predator-evading behavior. █Experiments by von Frisch and others showed that injured minnows (among other fish
species) release a pheromone—a secreted or excreted chemical that triggers a social response in members of the
same species. █Detecting this particular pheromone causes agitated reactions by the minnows. Von Frisch coined the
term schreckstoff for these pheromones.

9: Look at the four squares that indicate where the following sentence could be added to the passage.
It was worth investigating what caused their response in the absence of clear danger.
Where would the sentence best fit? Click on a square to add the sentence to the passage.

10: Directions: An introductory sentence for a brief summary of the passage is provided below. Complete the
summary by selecting the 3 answer choices that express the most important ideas in the passage. Some sentences do
not belong in the summary because they express ideas that are not presented in the passage or are minor ideas in the
passage. This question is worth 2 points.

Fish rely on their sense of smell to avoid danger.

A. Because fish live in places where seeing can be difficult, the olfactory signals sent by their nostrils’ epithelium to
the brain are particularly important.
B. Schreckstoff produced by an injured fish or from the body waste of a predator that has eaten fish that release
schreckstoff can warn other fish, causing them to take defensive action.
C. Prey fish are able to stay away from the hunting grounds of predatory fish because they can smell the body waste

第 256 页 共
373 页
2021 托福阅读真题

that the predatory fish left in those areas.


D. Some fish release small amounts of schreckstoff so that predators cannot easily recognize them, which enables
these fish to avoid attacks from predators.
E. Some types of chemical signals released by fish in the presence of predators cause other prey fish to swim away,
while other chemicals cause prey fish to remain still.
F. Some fish recognize the smells of their predators, while others simply detect whether a predator has consumed
one of their own kind.

Plant Senses

Paragraph 1: Animals rely on various senses, such as vision, hearing, smell, and touch, to acquire information
about their environment and respond to it. Plants lack consciousness and are immobile, but, amazingly, they too have
senses that enable them to respond to the environment. The ability of organisms to use sensory cells in their nose,
mouth, and throat to detect and respond to airborne molecules (waterborne molecules for aquatic organisms) is
referred to as the sense of smell. That plants can smell was discovered accidentally about a century ago. Citrus
farmers had known that harvested fruit ripened faster if placed in heated shed. They assumed that heat facilitated
ripening and were surprised to discover that electric heaters, which replaced kerosene heaters in the early twentieth

第 257 页 共
373 页
2021 托福阅读真题

century, were ineffective at ripening fruit. Scientists have since determined that kerosene heaters emit ethylene,
molecules of which are also similarly produced by ripened fruit and stimulate maturation of unripe fruit. Presumably,
ethylene emission by fruit on a live tree serves to ensure that all the fruit on a tree will ripen at an appropriate time,
such as before the onset of frost. Plants may also use smell to communicate about dangers in the environment.
Leaves of trees that are being eaten by caterpillars give off an airborne molecule that causes undamaged leaves on
the same tree and on nearby trees to produce substances unpalatable to the insects. The substances, which are not
normally present in the leaves because they require a great deal of energy to produce, significantly reduce insect
infestation in leaves

1, The word "unpalatable" in the passage a closest in meaning to


A, useless
B, dangerous
C, unfamiliar
D, bad tasting

2, In paragraph 1, why does the author discusses the experience of farmers placing fruit in heated shed?
A, To explain how plants’ ability to smell were discovered
B, To illustrate changes in heating devices that took place in the twentieth century
C, To demonstrate an advantage of replacing kerosene heaters with electric heaters
D, To point out that farmers and scientists disagreed about the role of heat in ripening fruit

3, According to paragraph 1, why were kerosene heaters effective in ripening fruit?


A, They produced the right amount of heat
B, They produced a molecule that fruit could smell
C, They were placed in sheds that were near live trees
D, They were used only very late in the season

4, According to paragraph 1, what is the function of the airborne molecule produced


by the leaves of some plants?
A, The molecule helps leaves that have been eaten by caterpillars recover from the damage
B, The molecule causes other leaves to protect themselves from being eaten by caterpillars.
C, The molecule attracts caterpillars to neighboring plant species
D, The molecule forces caterpillars to stop eating the leaves that produce the molecule.

Paragraph 2: Plant physiologist Frank Salisbury set out to measure the growth rate of leaves on some plants he was
studying by periodically placing a ruler on them. In doing that he accidentally discovered that these leaves had a
sense of touch. He noticed that the leaves being measured stopped growing although surrounding leaves continued to

第 258 页 共
373 页
2021 托福阅读真题

grow. Tree on windy mountaintops are relatively short, and scientists speculate that feeling the wind may prevent the
trees from growing larger, an adaptation that may mitigate the risk of being blown over. Some plants can even move
in response to touch. The insect-eating Venus flytrap, which grows in nutrient-poor environments in the southeastern
United States, closes a jaw-like rap when an insect disturbs hairs located inside the trap. Movement of each hair
induces an electric current, and if the current reaches a threshold level, the trap will close. Movement of one hair
does not produce enough current to close the trap, but movement of two hairs within twenty seconds of each other
does. This ensures that the trap does not respond to false stimuli, such as raindrops or falling debris. Once an insect
is caught, the tree remains closed for about ten days to give the plant sufficient time to extract nitrogen and
phosphorus from its prey

5, Which of the following can be inferred from paragraph 2 about the growth of the leaves of some trees?
A, Their growth rate is influenced by the growth rate of surrounding leaves
B, They may stop growing in windy condition
C, Their growth can be aided by contact with solid objects.
D, They grow more rapidly on relatively short

6, According to paragraph 2, the Venus fly trap's sense of touch helps that plant which of the following?
A, Obtain nitrogen and phosphorus
B, Protect some of its parts from damage by insects
C, Protect some of its parts from damage by wind
D, Prevent its hairs from being disturbed

7, According to paragraph 2. all of the following statements about the Venus flytrap are true EXCEPT:
A, It uses its sense of touch to react to raindrops and falling debris
B, It lives in areas lacking in nutrients
C, It can produce an electric current inside its body.
D, It detects the presence of prey with its hairs.

Paragraph 3: Many plants can differentiate between red and blue light, which may be considered a sense of sight.
Phototropin, a protein found in plant cells, absorbs blue light and so regulates the direction of the plant's growth.
Directional growth occurs because molecules called auxins accumulate on the shaded side of a plant, causing the
cells on the shaded side to grow longer than the cells on the lit side. Thus making the plant bend toward the light. In
many plants, flowering is controlled by the red-light sensing system. Certain types of plants flower late in the season,
when nights become long. Flowering in such plants is typically regulated by a molecule that can switch between two
forms, an "on" form and an "off" form. Red light, which is abundant at sunrise, turns the molecule "off." but far-red
light which is the last light that plants are exposed to right after sunset, turns it "off." Plants flower only when the
molecule remains "off" for sufficiently long periods of time, meaning that what flowering plants measure is not the

第 259 页 共
373 页
2021 托福阅读真题

length of the day, but the uninterrupted periods of darkness. Florists exploit this mechanism to control the timing of
chrysanthemums' flowering: they subject the plants to bursts of red light at night to prevent the plants from flowering
before they want them to.

8, According to paragraph 3, blue light can have which of the following effects on a plant?
A, Blue light causes auxins to gather on the side of the plant that receives less sunlight
B, Blue light leads to an increase in the amount of phototropin in the plant's cells
C, Blue light causes the plant to produce molecules that speed up the growth of the entire plant
D, Blue light helps prevent the plant from flowering at the wrong time

Paragraph 2: Plant physiologist Frank Salisbury set out to measure the growth rate of leaves on some plants he was
studying by periodically placing a ruler on them. In doing that he accidentally discovered that these leaves had a
sense of touch. He noticed that the leaves being measured stopped growing although surrounding leaves continued to
grow. Tree on windy mountaintops are relatively short, and scientists speculate that feeling the wind may prevent the
trees from growing larger, an adaptation that may mitigate the risk of being blown over. Some plants can even move
in response to touch. █The insect-eating Venus flytrap, which grows in nutrient-poor environments in the
southeastern United States, closes a jaw-like rap when an insect disturbs hairs located inside the trap. █Movement of
each hair induces an electric current, and if the current reaches a threshold level, the trap will close. █Movement of
one hair does not produce enough current to close the trap, but movement of two hairs within twenty seconds of each
other does. █This ensures that the trap does not respond to false stimuli, such as raindrops or falling debris. Once an
insect is caught, the tree remains closed for about ten days to give the plant sufficient time to extract nitrogen and
phosphorus from its prey

9, Look at the four squares I that indicate where the following sentence could be added to the passage.
How is that plant able to perform such an animal-like motion?
Where would the sentence best fit? Click on a square to add the sentence to the passage.

10, Directions: An introductory sentence for a brief summary of the passage is provided below. Complete the
summary by selecting the THREE answer choices that express the most important ideas in the passage. Some
sentences do not belong in the summary because they express ideas that are not presented in the passage or are minor
ideas in the passage. This question is worth 2 points.

Like animals, plants use their various senses so adapt to their environment.

A, Scientists have discovered by accident that some plants have a sense of smell and others have a sense of touch
B, Some plants, like the Venus flytrap, possess several different senses, while other plants do not seem to have any
senses.

第 260 页 共
373 页
2021 托福阅读真题

C, Plants' senses serve a variety of purposes, such as providing information about threats in the environment, like
caterpillars and strong winds
D, Fruit-producing plants use the emission and smell of ethylene to protect their fruit
from being damaged by insects and other animals
E, The length of time that certain plants are able to flower increases with the amount as well as the duration of the
sunlight they receive.
F, Plants’ ability to distinguish between different colors of light may allow them to
grow in the right direction or to flower at the appropriate time.

Understanding Insects through Fossils

Paragraph 1: Although it has been estimated that insects account for roughly one-third of all animal species
alive today, insects are, on the whole poorly represented in the available fossil record, where many species are
known from just a single specimen, and a high proportion of fossil insects come from exceptional fossil
deposits that are sporadically distributed in time and space. Nonetheless, about 40 000 species of insects have
been described as fossils, with many more awaiting description. Foremost among insect-rich deposits are ambers in
which complete external preservation of insects is routine. Amber is the fossilized resin of a few particular kinds of

第 261 页 共
373 页
2021 托福阅读真题

trees. Oozing out of the bark, this resin had the ability to trap and surround insects as well as other small ani mals
protecting them from the normal processes of organic decay as it hardened into transparent yellow or orange amber
The chemical process of "amberization" could take up to 10 million years. During this time, it was common for
amber initially buried in the soil to be washed out by rivers and redeposited in the sea.

1. Paragraph1 supports all of the following statements about amber EXCEPT:


A. It is formed from a substance produced by certain types of trees
B. It is sometimes transported away from the place where it was produced.
C. It is fossilized through a quick chemical process.
D. It can preserve the outer body parts of organisms covered in it.

2. Which of the sentences below best expresses the essential information in the highlighted sentence in the passage?
Incorrect choices change the meaning in important ways or leave out essential information.
A. Insects, which account for roughly one-third of all animal species alive today, are also represented in the fossil
record.
B. The available fossil record offers a limited picture of early insects because it provides relatively few specimens.
C. Many insect species are represented in the available fossil record by just a single specimen.
D. Many fossil insects have been found in the exceptional fossil deposits that are associated with a variety of times
and places.

Paragraph 2: Although the oldest amber comes from the Carboniferous period (360-290 million years ago), the
great majority of amber deposits were formed between the start of the Cretaceous period (146 million years ago) and
the present. They provide priceless windows on the insects and other small animals living at the time in the forests
where amber-producing trees grew. Elsewhere in the fossil record, insects can be found in fine-grained sedimentary
rocks, such as clays and silts deposited in freshwater lakes and sluggish rivers. Unlike the insects in amber, these
fossils generally comprise only fragments, particularly of wings or wing cases, although more complete examples
can be found, such as the dragonflies of the famous Jurassic Solnhofen Limestone of Bavaria.

3. The word "priceless" in the passage is closest in meaning to


A. highly valuable
B. very clear
C. long-lasting
D. easy-to-use

4. Paragraph 2 suggests that most fossil insects found in a state of complete external preservation are insects that
A. died near the famous limestone deposits in Bavaria
B. died near a freshwater lake or river

第 262 页 共
373 页
2021 托福阅读真题

C. lived less than 146 million years ago


D. lived between 360 and 290 million years ago

Paragraph 3: The fossil record describes a multiplicity of insects that scientists have grouped according to their
features. The most basic categorization of insects is into a primitive group without wings called the Apterygota, and
the winged Pterygota. Surviving apterygotes include the springtails and silverfish. They are now relatively rare,
comprising less than 1 percent of all insect species Pterygotes are divided into those with wings that cannot be folded
which are called the Palaeoptera, and a larger, more advanced group, the Neoptera, capable of folding their wings
close to the body. Mayflies, dragonfies and damselflies are all palaeopteran insects, while the neopterans include
locusts, butterflies, and wasps.

5.According to paragraph 3,one difference between Pteygota and Apterygota is that Pterygota
A. currently represent the majority of all insect species
B. are a more primitive insect group
C. tend to be smaller in size
D. are harder to find in the fossil record

Paragraph 4: In spite of its imperfection, the fossil record holds a lot of useful information about the times of
origination of insect groups that are alive today. Primitive wingless insects-the apterygotes-appear to have undergone
an initial diversification during the Devonian period (416-359 million years ago), possibly even the Silurian period
(444-416 million years ago). Unfortunately, however, relatively few fossil insects of this age are known and there is
a great need for further discoveries. The oldest known fossil insect is currently Rhyniognatha hirsti from the early
Devonian fossils in Scotland. However, this species, preserved in sinter (mineral sediment) from an ancient hot water
spring that was active between 400and 412 million years ago, exhibits some advanced characteristics, implying that
there are more primitive, older insects still to be discovered.

6. In paragraph 4 the author mentions the need for further discoveries in order to
A. emphasize that the fossil record does not provide evidence about when today's insect groups originated
B. suggest that additional fossils might clarify the early evolution of primitive wingless insects
C. argue that currently known fossils do not preserve the characteristics of insects from the Devonian period
D. explain that insect fossils preserved ancient hot spring sediments are hard to date accurately

7. According to paragraph 4, what evidence indicates there might be primitive insects that have not yet been
discovered?
A. Many insect fossils available from the Devonian period have not yet been studied or classified.
B. Several known insect fossils from the Silurian period contain examples of the earliest insect groups.
C. The oldest known insect fossil was discovered in mineral sediment containing traces of even older insects.

第 263 页 共
373 页
2021 托福阅读真题

D. The fossil of Rhyniognatha hirsti has some advanced traits that imply there were once simpler insects.

Paragraph 5: Fossil insects with preserved wings (Pterygota) first occur in the mid Carboniferous. The evolution of
wings was accompanied by an increase in maximum body size. A remarkable dragonfly called Meganeura with a
wingspan approaching 70 cm has been described from the late Carboniferous. This inhabitant of the forests is one of
the largest insects ever to have lived. The huge size of Meganeura has led to speculations about the composition of
the atmosphere at the time, the powered flight of such a large insect perhaps demanding an atmosphere containing
higher levels of oxygen than that of the present day. Unfortunately, the fragmentary insect fossil record sheds little
light on the origin of flight, as the oldest winged insects already had fully formed wings.

8. According to paragraph 5, why does the fossil record shed little light on the origin of flight?
A. Most of the fossils of early winged insects are only fragments.
B. The earliest insects in the fossil record did not yet have wings.
C. The earliest winged insects were too large to have flown in the same way as later insects.
D. The oldest known winged insects already had wings that were fully-formed.

Paragraph 3: The fossil record describes a multiplicity of insects that scientists have grouped according to their
features. The most basic categorization of insects is into a primitive group without wings called the Apterygota, and
the winged Pterygota. ■Surviving apterygotes include the springtails and silverfish. ■They are now relatively rare,
comprising less than 1 percent of all insect species. ■Pterygotes are divided into those with wings that cannot be
folded which are called the Palaeoptera, and a larger, more advanced group, the Neoptera, capable of folding their
wings close to the body. ■Mayflies, dragonflies and damselflies are all palaeopteran insects, while the neopterans
include locusts, butterflies, and wasps.

9. Look at the four squares [■] that indicate where the following sentence could be added to the passage.
There are many examples of these two types of winged insects today.
Where would the sentence best fit? Click on a square [■] to add the sentence to the passage.

10. Directions: An introductory sentence for a brief summary of the passage is provided below. Complete the
summary by selecting the THREE answer choices that express the most important ideas in the passage. Some
sentences do not belong in the summary because they express ideas that are not presented in the passage or are minor
ideas in the passage. This question is worth 2 points.

Early insects are not very well represented in the fossil record except for those that lived in a few specific
times and places.

A. It is estimated that there have been 40.000 species of insects throughout the history of animal evolution and that

第 264 页 共
373 页
2021 托福阅读真题

only a small portion of them have survived to this date.


B. The richest known deposit of fossil insects that were not preserved in amber is in the Jurassic Solnhofen
Limestone in Bavaria, where complete examples of the most primitive dragonflies were found.
C. The fossil record shows that in the Carboniferous period wingless insects became smaller and smaller, while the
typical wingspan of winged insects reached 70cm before it began to decline again.
D. The best records of ancient insects come from later fossil insects in amber since the fossils in sedimentary rocks,
which are much earlier, tend to be fragmentary.
E. Fossil insects are divided into those without wings, which include the most primitive insects, and those with
wings.
F. The oldest known fossil insect is around 400 million years old, and fossils of winged insects date from the mid
Carboniferous, but fossils of the earliest winged and wingless insects have not been found.

The Cooperative Nesting of Fairy-wrens

Paragraph 1: Some birds feed other birds nestlings (birds that are too young to leave the nest). This behavior is
called "cooperative nesting.” As with all altruistic behavior, it seems to violate a basic principle of evolution-that the
need to survive and reproduce and thus pass down one’s genes, takes priority over all else. Many birds that do nest
cooperatively, including fairy-wrens, are concentrated in savanna (grassy plains in tropical and subtropical regions)
in Australia and Africa. An unpredictable environment, such as savanna that experiences sudden drought

第 265 页 共
373 页
2021 托福阅读真题

and rainy periods favors diversified team efforts. But geography cannot entirely clarify the cooperative behaviors of
fairy-wrens, which sometimes have nest helpers and sometimes do not. In a study conducted in Mornington,
Australia, researchers found that 90 percent of purple-crowned fairy-wren helpers live with at least one of their
parents. This means that these helpers are related to the nestlings they are helping to feed. By helping their relatives
survive, they are increasing the chances of some of their own genes being passed down to the next generation. But
what explains the behavior L the birds that help nonrelatives?

1, The word "principle" in the passage is closest in meaning to


A, rule
B, focus
C, consequence
D, preference

2, According to paragraph 1,which of the following is true about cooperative nesting savannas?
A, It helps some birds deal with unstable weather conditions
B, It is more common in dry areas than in rainy areas
C, It is required for building nests during rainy periods.
D, It was first discovered by researchers conducting a study in Australia.

Paragraph 2: The Mornington study also found that unrelated helpers are probably motivated by the prospect of
inheriting a good territory. Because the supply of waterside habitat is limited, almost all of it is occupied by
dominant adult fairy-wrens. Sometimes a young bird can carve out a place for itself only by joining an occupied
space contributing by helping the current owners raise babies until they either die or move on. It’s a win -win
agreement because the adults get extra help feeding their nestlings, and the helpers get chance to inherit a nice place
to live.

3, According to paragraph 2, which of the following is likely true about how unrelated fairy -wren helpers obtain
territory?
A, They help one another find a nice place to live.
B, They compete for territory with dominant adult fairy-wrens.
C, They search out unoccupied spaces next to dominant adult birds territories.
D, They obtain desirable waterside habitat in exchange for feeding other birds nestlings

Paragraph 3: It comes as no surprise that purple-crowns fairy-wren nests with helpers tend to be more productive.
Yet, in a closely related species the superb fairy-wren, which uses a similar system of nest helpers, researchers have
been unable to show that the extra help translates into healthier chicks (young birds). Study after study has found no
difference in fledging (feather development) success between nests with and without helpers. Scientists could only

第 266 页 共
373 页
2021 托福阅读真题

wonder whether their data were correct if helpers didn't increase nesting success, why did dominant adult
fairy-wrens allow your birds to remain in their territories?

4, Paragraph3 suggests that a difference fledging success between two nests would indicate a difference in
A, the healthiness of chicks in the nests
B, the number of helpers in the nests
C, how closely related chicks and helpers in the nests are
D, how dominant the adults in the nests are

Paragraph 4: Superb fairy-wrens have a different system from that of purple-crowned fairy- wrens. Males pluck
flower petals and display them to females for courtship, and females often sneak away before dawn to mate with
other males. This type of sexual behavior means that, unlike purple-crowned dads, male superb fairy-wrens are often
unrelated to the nestlings in their own nest, and so are the helpers (unless, as sometimes happens the helpers mate
secretly with the female). All this makes you wonder what motivates superb fairy-wrens to breed cooperatively.
Maybe helpers are doing it solely to inherit territory rather than to protect family. But are they even really helping?

5, The word "unless" in the passage is closest in meaning to


A, even though
B, except if
C, also
D, at the same time

6,According to paragraph4,all of the following statements about the mating behavior of superb fairy-wrens are true
EXCEPT
A, Males attract females with parts of flowers
B, Some females secretly leave their nests night to mate
C, Some females mate with helper birds
D, Protecting family is more important to males than mating

Paragraph 5: When a team of researchers led by an Englishman, Andrew Russell, compared the fledging success of
young superb fairy-wrens from nests with and without helpers in 2007, there was no measurable difference; although
chicks in nests with helpers were fed an average 19 percent more food, they weren't any healthier. Earlier studies had
reached the same conclusion. Russell couldn't make sense of this result, but he had a theory. If the chicks weren’t
benefiting from extra attention, maybe their mothers were. He careful measured eggs in a variety of nests and found
that eggs were more than 5 percent smaller in nests with helpers than in those tended to just two adults. Smaller eggs
also contained smaller proportion of yolk and nutrients. When females could rely on a bigger family to raise their
chicks, it seemed that they put less energy into their eggs. At hatching, those chicks emerged weak and underweight.

第 267 页 共
373 页
2021 托福阅读真题

But with additional fairy-wrens bringing them extra food, skinny chicks grew faster, and by the time they left the
nest they’d caught up to normal young birds with two parents. Then critically, Russell looked at the long-term
survival of the mothers. Adult female fairy-wrens who lived with just one male had a one in-three chance of dying
within one year. Even for females with helpers, those odds were reduced to one in five.

7,Which of the following best describes the relationship of paragraph 5 to paragraph 4?


A, Paragraph 5 challenges the research findings on superb fairy-wrens presented in paragraph 4.
B, Paragraph 5 contrasts the advantages of cooperative behavior in superb fairy-wrens with the disadvantages
described paragraph 4.
C, Paragraph 5 summarizes research evidence on superb fairy-wrens that addresses a question posed in paragraph 4
D, Paragraph 5 describes the behavioral differences between purple-crowned fairy-wrens and superb fairy-wrens
while paragraph4describes their similarities

8, According to paragraph 5,which of the following is true of superb fairy-wrens that have helpers as compared to
those that do not?
A, Their eggs are more nutritious even though they are smaller.
B, Females spend less energy to produce eggs
C, Young birds leaving their nests are comparatively less healthy.
D, Newly hatched female birds receive significantly more food than do newly hatched males.

Paragraph 1: Some birds feed other birds nestlings (birds that are too young to leave the nest). This behavior is
called "cooperative nesting.” As with all altruistic behavior, it seems to violate a basic principle of evolution -that the
need to survive and reproduce and thus pass down one’s genes, takes priority over all else. Many birds that do nest
cooperatively, including fairy-wrens, are concentrated in savanna (grassy plains in tropical and subtropical regions)
in Australia and Africa. ■An unpredictable environment, such as savanna that experiences sudden drought and rainy
periods favors diversified team efforts. ■But geography cannot entirely clarify the cooperative behaviors of
fairy-wrens, which sometimes have nest helpers and sometimes do not.■In a study conducted in Mornington,
Australia, researchers found that 90 percent of purple-crowned fairy-wren helpers live with at least one of their
parents. ■This means that these helpers are related to the nestlings they are helping to feed. By helping their relatives
survive, they are increasing the chances of some of their own genes being passed down to the next generation. But
what explains the behavior L the birds that help nonrelatives?

9, Look at the four squares that indicate where the following sentence could be added to the passage.
Research observations helped to produce a better understanding of the reasons for the behavior.

10, Directions: An introductory sentence for a brief summary of the passage is provided below. Complete the
summary by selecting the 3 answer choices that express the most important ideas in the passage. Some sentences do

第 268 页 共
373 页
2021 托福阅读真题

not belong in the summary because they express ideas that are not presented in the passage or are minor ideas in the
passage. This question is worth 2points.

Fairy-wrens are among the birds that nest cooperatively.

A, Although cooperative nesting seems to be in conflict with the helpers' need to survive and reproduce, it brings
helpers benefits, such as inheriting good territory.
B, In general, helpers who share genes with the birds they are helping are more motivated to help than those who do
not, and the chicks they feed develop faster.
C, Research shows that cooperative nesting in superb fairy-wrens benefits the mother birds, who are more likely to
live longer when they have helpers.
D, Cooperative nesting allows some male fairy-wrens to have more time and energy to mate with females, which
increases the chances of their genes being passed down
E, Although most purple-crowned fairy-wrens helpers are related to the chicks they feeding, many fairy-wrens help
with nests where they have no relatives.
F, Compared to superb fairy-wren females, purple-crowned fairy-wren females are probably more efficient at nesting
and thus benefit less from having helpers

Prairie Fires

Paragraph 1: Fire has played a critical part in shaping the structure and function of prairies (North American
grasslands). One of the most important roles of fire is to limit the number of trees that grow on the prairie. Some
trees, such as eastern red cedars, are extremely vulnerable to fire and are killed by any fire that scorches their
branches and needles. Most deciduous trees (those that drop their leaves in the fall), on the other hand, are not

第 269 页 共
373 页
2021 托福阅读真题

usually killed by fire, but can be weakened considerably-especially if they are burned during the growing season. If
those deciduous trees are small and their trunks are unprotected by thick bark a hot fire will cook the branches
sufficiently to kill all the living tissue above the ground. In that case the tree has to start over from the roots, leaving
it weakened and most vulnerable to being eaten by animals such as deer elk, and rabbits. Larger trees with thick
layers of bark, such as bur oaks, often survive the initial heat of a fire, although very hot fires can sometimes heat
less protected branches higher in a tree enough to severely stress or even kill it. Repeated fires can affect even large,
well-established trees by burning away enough bark to expose them to insects or diseases that can eventually kill
them.

1, According to paragraph 1, which of the following is true about bur oaks?


A, They are more easily killed by insects and diseases than by fires
B, They grow branches so high that animals cannot reach them
C, They have thick bark that helps protect them against damage by fire.
D, They suffer little or no damage from fires that bum their higher branches

Paragraph 2: In addition to their effect on trees, fires have contributed to the diversity and function of prairies in
many other ways. Because most prairie plants die back to the ground each fall they leave their dead stems and leaves
on the surface. Those fallen stems and leaves, over the years, can form a dense ground covering called thatch. That
thatch slows the warming of the soil in the spring and delays the beginning of the growing season.it can also shade
out many smaller plants and restrict the growth of larger plants. However, that accumulation of thatch also increases
the vulnerability of prairie to fire. When a fire burns through. It removes thatch and exposes bare soil again allowing
both established plants and new seedlings full sunlight to grow. In addition, some plant species have higher seed
germination rates (the rate at which seer grow into plants) following a fire because the smoke and/or heat of the
fire actually help the seeds start to grow.

2, Why does the author provide the information that "some plant species have higher seed germination rates (the
rate at which seeds grow into plants) following a fire"?
A, To point out a benefit that can occur when a fire burns through thatch
B, To emphasize the point that established plants need as much sunlight to grow as new seedlings do
C, To suggest that prairie plants recover very quickly from fires
D, To explain how the density of thatch delays the start of the growing season
3, According to paragraph 2, what is thatch?
A, A type of prairie plant that dies back to the ground every fall
B, A patch of shade produced by larger plants over smaller plants
C, A portion of the prairie with particularly dense plant growth
D, A thick collection of dead vegetation

第 270 页 共
373 页
2021 托福阅读真题

4, According to paragraph 2, thatch on the prairie performs all of the following functions EXCEPT
A, keeping the soil cool for longer
B, allowing plants to start growing earlier in the year
C. preventing small plants from getting enough sunlight to grow
D, burning easily

Paragraph 3: The season during which a fire occurs has a lot to do with its impact on the plant community. Fires
during the dormant season (late fall through early spring) but when most of the vegetation is not growing so
there is very little negative impact on most plants, and they can take advantage of the bare soil conditions
when the growing season begins. Cool-season plants benefit the most from dormant-season fires because they are
the first to emerge in the spring. Dormant-season fires can set wood plants back by making them start over a ground
level. However, some species, such as smooth sumac respond to that stress by forming multiple new shoots where
only one existed before. With that tactic, sumac can actually form a thicker patch following a dormant-season fire
than it had before Eventually, if plant-eating animals do n control them, those patches can become dense enough that
no grass grows underneath them, making them nearly invulnerable to future fires.

5, Which of the sentences below best expresses the essential information in the highlighted sentence in the passage?
Incorrect choices change the meaning in important ways or leave out essential information
A, Dormant-season fires occur from late fall through early spring when most of the vegetation is not growing.
B, Fires that burn when most vegetation is not growing often produce bare soil conditions at the beginning of the
growing season.
C. Dormant-season fires do comparatively little damage to most plants and clear the soil to help plants grow at the
start of the growing season.
D, Fires burn the most often when vegetation is not growing, so plant growth is usually not negatively affected by
fires in the dormant season

6, Paragraph 3 suggests which of the following about the dense patches of smooth sumac that form after dormant
season fires?
A, They burn more quickly and suffer more damage than plants that had to start over from ground level.
B, They are rarely burned by fire because of the lack of dead vegetation on the ground
C, They are usually avoided by animals that normally feed on grass.
D, They protect the grasses around them from future fires

Paragraph 4: Fires that occur during the growing season have much different impacts. Growing season fires are
usually slower-moving and less likely to burn large areas than dormant- season fires. The fires are fueled by the
dried grass from previous seasons, but the combination of higher humidity and abundant green vegetation causes
slow moving and very smoky fires. However because prairie vegetation is actively growing when these fires move

第 271 页 共
373 页
2021 托福阅读真题

through, the impact on vegetation can be significant. The plants growing most strongly at the time of the fire are
most affected. Fires during the late spring or early summer set back cool- season plants and allow warm-season
plants to begin their growth period with less competition for light and water. In contrast, fires during the summer
suppress the growth of warm-season plants and favor cool-season plants. Summer fires also tend to release many
annual and biennial plants because the suppression of the dominant warm-season grasses opens up space for new
seedlings to emerge. This often results in the abundance of showy flowers during the fall and through the next
growing season. However, it can also facilitate more vigorous growth of undesirable invasive (non-native)
cool-season grasses.

7, The word "vigorous" in the passage is closest in meaning to


A, strong
B, damaging
C, possible
D, widespread

8, According to paragraph 4, growing season fires are typically different from dormant-season fires in which of the
following ways?
A, Growing-season fires tend to cover larger areas.
B, Growing-season fires burn more quickly
C, Growing-season fires have more available fuel to burn.
D, Growing-season fires produce more smoke.

Paragraph 3: The season during which a fire occurs has a lot to do with its impact on the plant community. Fires
during the dormant season (late fall through early spring) but when most of the vegetation is not growing so
there is very little negative impact on most plants, and they can take advantage of the bare soil conditions
when the growing season begins. Cool-season plants benefit the most from dormant-season fires because they are
the first to emerge in the spring. Dormant-season fires can set wood plants back by making them start over a ground
level. ■However, some species, such as smooth sumac respond to that stress by forming multiple new shoots where
only one existed before. ■With that tactic, sumac can actually form a thicker patch following a dormant-season fire
than it had before.■Eventually, if plant-eating animals do n control them, those patches can become dense enough
that no grass grows underneath them, making them nearly invulnerable to future fires.■

9, Look at the four squares ■ that indicate where the following sentence could be added to the passage.
Other developments determine how later fires affect these plants.
Where would the sentence best fit? Click on a square to add the sentence to the passage.

10, Directions: An introductory sentence for a brief summary of the passage is provided below. Complete the

第 272 页 共
373 页
2021 托福阅读真题

summary by selecting the 3 answer choices that express the most important ideas in the passage. Some sentences do
not belong in the summary because they express ideas that are not presented in the passage or are minor ideas in the
passage. This question is worth 2points.

Fires play an important role in shaping the plant communities of prairie.

A, Although prairie fires cause harm trees, they limit the ability of animals such is deer, elk and rabbits to damage
plants as they grow back after fires.
B, Fires kill some trees outright and weaken or slow the growth of others, but they also help prairie plants grow by
burning away dead vegetation.
C, Warm-season plants are damaged by both dormant-season and growing-season fires, but cool-season plants are
damaged mainly by dormant-season fires.
D, Fires affect the structure of prairie by limiting the number and kinds of trees t grow in them and by creating
condition suitable for a variety of other plants
E, A fires impact on prairie plants depends significantly on when it occurs, with fires during the dormant season
generally doing less damage than fires during the growing season.
F, Prairie fires are often fueled by dried vegetation from earlier seasons but high humidity can result in very smoky
fires, with significant damage to emerging new vegetation.

Costs and Benefits of Dispersal

Paragraph 1: In order to move from one home base to another, animals must expand calories not only while moving
but even before the dispersal when they invest in the development of the muscles needed to move. For example, if a
cricket is to leave a deteriorating environment and move to a new and better place, it will need large flight muscles to
fly away. Presumably, the calories and materials that go into flight muscle development and maintenance have to

第 273 页 共
373 页
2021 托福阅读真题

come out of the general energy budget of the animal. This means that other organ systems cannot develop as rapidly
as they could otherwise, which may mean that the flight-capable individual is, in some other respects, less fit to
survive.

1,The word "Presumably” in the passage is closest in meaning to


A, it is possible
B, it is reasonable to assume
C, it can be argued
D, it is certainly true

2, In paragraph 1, the discussion of cricket dispersal is used to illustrate which of the following principles?
A, For dispersing animals, the benefits dispersal exceed the costs.
B, Dispersal always involves costs for the dispersing animal.
C, Only animals with the greatest fitness for survival are able to disperse
D, Dispersal, as much as organ development, requires energy.

Paragraph 2: Dispersing individuals not only have to pay energetic, developmental, and travel costs but are also
more often exposed to predators-alt of which raises the question, why are animals so often willing to leave home
even when this means leaving familiar resource-rich location? This question is particularly pertinent for species in
which some individuals disperse while others do not or do not disperse as far. One species in which some individuals
travel farther than others is Belding's ground squirrel Young male squirrels travel about 150 meters from the burrow
in which they were born, whereas young females usually settle down only 50 meters or so from where they were
born. Why should young Belding's ground squirrels disperse at all, and why should the males disperse farther than
their sisters?

3, According to paragraph 2, what is particular interest about the dispersal of Belding's ground squirrels?
A, Young squirrels disperse farther than adults.
B, Young squirrels disperse even from resource-rich areas
C, The males and females have different patterns of dispersal
D, They leave the mother's burrow at an earlier age than other squirrels.

4, It can be inferred from paragraph 2 that the phenomenon of dispersal


A, is difficult to observe in the wild
B, occurs for reasons that are not always immediately apparent
C, is motivated by the desire to obtain more or better resources
D, has few benefits for most species

第 274 页 共
373 页
2021 托福阅读真题

Paragraph 3: According to one argument dispersal by juvenile animals of many species may be an adaptation
against problems associated with inbreeding. When two closely related individuals mate, their offspring are more
likely to manifest genetic diseases than are the offspring of genetically unrelated individuals, and as a result,
inbreeding tend to produce animals that are less likely to survive to adulthood and reproduce. Dispersal of juveniles
makes inbreeding less likely.

Paragraph 4: If avoidance of inbreeding is the point of dispersing, then one might expect is man female ground
squirrels as males to travel 150 meters from their natal burrow. In fact, females do not disperse as far as males,
perhaps because the costs and benefits of dispersal differ for the two sexes. It has been suggested that the
reproductive success of female Belding's ground squirrels depends on their possession of a territory in which to rear
their young. Female group squirrels that remain near their birthplace enjoy assistance from their mothers in the
defense of their burrows against rival females. Thus, the benefits of remaining on familiar ground are greater for
females than for males.

5, According to paragraph 4, young female Belding's ground squirrels remain closer to where they were born than do
males in order to
A. avoid inbreeding with their male siblings
B. avoid competition with other females for territory
C. save energy they need to defend the burrows
D. benefit from their mothers" help

Paragraph 5: There may, however, be another reason why male mammals disperse greater distances than females.
The usual rule is that males, not females, fight with one another for access to mates, and, therefore, males that lose
such conflicts may find it advantageous to move away from same-sex rivals that they cannot subdue. Although this
hypothesis probably does not apply Belding's ground squirrels, since young males have not been seen fighting with
older ones around the time of dispersal, the idea is more plausible with respect to some other species, such as lions.

6. According to paragraph 5, the hypothesis that males disperse as a result of conflicts with other males is not
supported in the case of Belding's ground squirrels because
A, young male squirrels that lose fights to other males do not disperse
B, unlike most species, it is the females, not the males, that fight with each other
C, there is no evidence that young male squirrels fight with other males before dispersing
D, fights between males generally take place after the males have dispersed

Paragraph 6: Lions live in large groups or prides, from which young males disperse.in contrast, this daughters of
the resident lionesses usually spend their entire lives close to where they were born. The sedentary females benefit
from their familiarity with good hunting grounds and safe breeding dens in their natal territory, among other things.

第 275 页 共
373 页
2021 托福阅读真题

The departure of many young male lions coincides with the arrival of the new mature males that violently displace
the previous masters of the pride and chase off the males that are not yet adults in the pride as well. These
observations support the mate competition hypothesis for male dispersal. However, if young males are not evicted
after a pride takeover, they often leave anyway without any coercion from males and without ever having
attempted to mate with their female relatives. Moreover, mature males that have claimed a pride sometimes
disperse again, expanding their range to add a second pride of females, at a time when their daughters in the first
pride are becoming sexually mature. Inhibitions against inbreeding apparently exist in lions and cause males to
leave home.

7, According to paragraphs 5 and 6, the patterns of dispersal in Belding's ground squirrels and lions are similar in
which of the following ways?
A, Young males are forced to disperse by older males.
B, Avoiding inbreeding is probably not a factor in the explanation of the patter of dispersal.
C, Males disperse when new siblings are born
D, The males disperse farther than the females.

8. Why does the author provide the information that "if young males are not evicted after a pride takeover, they
often leave anyway without any coercion from adult males and without ever having attempted to mate with
their female relatives"?
A. To support the conclusion that male lions disperse to find mates to which they are not closely related
B. To indicate that the mate-competition hypothesis does not fully account for the dispersal of young male lions
C. To imply that young male lions voluntarily leave a pride that has been taken over because the adult males prevent
them from mating within the pride
D. To indicate that males, not females, leave a pride after it has been taken over by adult males

Paragraph 4: ■If avoidance of inbreeding is the point of dispersing, then one might expect is man female ground
squirrels as males to travel 150 meters from their natal burrow. ■In fact, females do not disperse as far as males,
perhaps because the costs and benefits of dispersal differ for the two sexes. ■It has been suggested that the
reproductive success of female Belding's ground squirrels depends on their possession of a territory in which to rear
their young. ■Female group squirrels that remain near their birthplace enjoy assistance from their mothers in the
defense of their burrows against rival females. Thus, the benefits of remaining on familiar ground are greater for
females than for males.

9, Look at the four squares ■ that indicate where the following sentence could be added to the passage
However, inbreeding can be avoided without both males and their female relatives dispersing the same
distance.
Where would the sentence best fit? Click on a square l to add the sentence to the passage.

第 276 页 共
373 页
2021 托福阅读真题

10, Directions: An introductory sentence for a brief summary of the passage is provided below. Complete the
summary by selecting the 3 answer choices that express the most important ideas in the passage. Some sentences do
not belong in the summary because they express ideas that are not presented in the passage or are minor ideas in the
passage. This question is worth 2points.

Many young animals disperse from where they are born.

A. There are a number of competing explanations of why animals disperse, but none of them is consistent with all
the observed patterns of dispersal.
B. The reason there is. little inbreeding among lions is that young males are driving away from their female relatives
by the adult males who forcibly take over a lion pride.
C. Inbreeding significantly lowers the likelihood that offspring will survive, and so avoiding inbreeding is a
significant benefit of dispersal
D. Since males of most species must establish a territory for mating dispersing of the males makes it more likely that
most males will be able to reproduce
E. There need to be significant benefits for a species to disperse because the requirements of dispersal involve energy
costs and dangers for the dispersing individuals.
F. In mammal species, young males often leave their family group while related females benefit from remaining in
close birthplace association with each other and the birthplace

Adaptations of Marine Mammals

Paragraph 1: Warm-blooded marine mammals and birds require some special adaptations to permit them to live in
ocean waters. One important set of adaptations is concerned primarily with maintaining the animal’s temperature.
Water has a higher thermal conductivity than air, which means that water is quick to extract heat from a warm body.

第 277 页 共
373 页
2021 托福阅读真题

Humans experience this property when they become chilled after a short time in water of even 80°F (27°C); in air
of the same temperature, they would be comfortable indefinitely. Marine mammals, which maintain elevated body
temperatures with respect to the surrounding water, must adapt to prevent their body heat from being drained away.

1, Why does the author discuss what happens to humans "after a short time in water of even 809F (27°C)"?
A, To explain the adaptations humans need to make in order to survive in ocean waters.
B, To contrast the effects of high and low water temperatures on humans
C, To suggest that humans and marine mammals have similar adaptations to cold water
D, To illustrate that water is a better conductor of heat than air

2, The word "indefinitely" in the passage is closest in meaning to


A, immediately
B, without effort
C, without exception
D, for an unlimited time

Paragraph 2: One means of slowing the rate of heat loss is to have a large body. The ratio of the surface area to the
volume of any body is lower for a large organism than for a small organism. The larger the body, the smaller the
surface area relative to the volume of the body. Since heat production is a function of the volume of a body,
whereas heat loss is a function of a wetted surface area, this means that large animals, which have a low ratio
of surface area to volume, can thermoregulate better than small animals can . Thermoregulation is aided by the
smaller surface area that is in contact with the environment through which heat may escape. All swimming marine
mammals are large. and it may be that the reason there are no mouse-sized marine mammals is that they would
simply die from chilling. There are small marine birds (petrels auklets), but these animals are completely immersed
in the water only for a short time during intermittent dives. Others, such as gulls, rarely dive and only a fraction of a
bird's body is in contact with the water at any time.

3, Which of the sentences below best expresses the essential information in the highlighted sentence in the passage?
Incorrect choices change the meaning in important ways or leave out essential information
A, The advantage in thermoregulation that large animals have over small ones is reduced when the surface area of
large animals becomes wet
B, Large animals maintain their body temperatures better than smaller animals because of their lower surface to
volume ratios
C, Large animals' bodies produce increased heat to replace heat that is last through the wetted parts of their large
surface areas.
D, Large animals can regulate heat production and loss better than smaller ones because their bodies have both larger
surface areas and volume

第 278 页 共
373 页
2021 托福阅读真题

4, According to paragraph 2, which of the is true of small marine following mammals?


A, They have a lower ratio of surface area to body volume than do large organisms.
B, They overheat easily due to their small surface area in contact with the environment
C, They have a slow rate of heat loss in comparison with their rate of heat production.
D, They lose body heat too easily to survive well in marine environments.

5, According to paragraph 2, which of the following explains the ability of petrels, auklets, and gulls to survive in
marine environments?
A, They do not spend enough time underwater to experience substantial heat loss.
B, They have developed special adaptations to prevent them from losing too much heat in cold water.
C, They have a lower ratio of surface area to body volume than would normally be expected for such small animals
D, They live in waters that are warm enough to prevent rapid heat loss from occurring.

Paragraph 3: A second adaptation that prevents or reduces heat loss is a thick insulating layer of blubber or fat just
beneath the skin. This layer reaches its greatest thickness in whales, where it may be 2 feet thick. In pinnipeds such
as the walrus and elephant seal, subcutaneous fat may constitute as much as 33 percent of the weight. Blubber and
fat are poor conductors of heat; hence they protect the animal from losing internal heat. The thicker the blubber or fat
layer the less the heat loss. Marine mammals inhabiting polar waters, therefore, have thicker layers of fat than
temperate and tropical species

6, Which of the following statements can be inferred from paragraph 3 about tropical marine mammal?
A, It loses body heat more easily than a polar marine mammal of the same size
B, It depends on blubber to maintain its body temperature
C, It holds more heat in its body than a temperate or polar marine mammal does.
D, It retains heat better than a polar mammal by staying in warm waters.

Paragraph 4: A final adaptation concerns the circulatory system. The areas of a marine mammal that offer the
greatest surface area to the water(and, therefore, the greatest opportunity for heat loss) and that also lack much of the
protective layer of fat are the fins, flippers and flukes. What adaptations prevent massive heat loss through these
extremities? In cetaceans (aquatic mammals),the answer is that the arteries that bring the warm blood out to these
extremities are surrounded by a number of smaller veins that bring blood back to the central core of the mammal.
Because of this arrangement. the heat of the blood in the arteries can be absorbed by the cooler blood returning in the
veins before it is lost to the external water through the thin flesh of the outer extremities. In effect, these animals
have a countercurrent system of circulation designed to save heat.

第 279 页 共
373 页
2021 托福阅读真题

7, According to paragraph 4, which of the following explains why marine mammals do not lose large amounts of
heat through their fins flippers, and flukes?
A, The fat and skin surrounding these areas prevents massive heat loss
B, The ratio of surface area to body volume is lowest at these areas
C, The number of veins and arteries in the areas is too small for massive heat loss to occur.
D, The veins near the arteries in these extremities prevent heat from escaping.

Paragraph 5: Because most of their adaptations conserve body heat, marine mammals (especially pinnipeds) may,
on occasion, become too warm. Hot, still days may mean considerable stress from overheating. On these uncommon
occasions, the animals must act to dissipate heat. They do so by waving their flippers in the air while increasing the
blood flow to the extremities and restrictiny the flow back to the core through the veins. The result is heat loss and
subsequent cooling. Seals and sea lions may also open their mouths and pant like a dog. lf they are resting on land,
they may go into the water to cool off.

8, In paragraph 5, which of the following is NOT mentioned as a strategy used by marine mammals to dissipate
heat?
A, Panting through their mouths
B, Moving from the land to the water
C, Reducing the amount of blood flowing to the extremities
D, Waving their flippers to increase heat loss through the extremities

Paragraph 3: A second adaptation that prevents or reduces heat loss is a thick insulating layer of blubber or fat just
beneath the skin. ■ This layer reaches its greatest thickness in whales, where it may be 2 feet thick. ■ In pinnipeds
such as the walrus and elephant seal, subcutaneous fat may constitute as much as 33 percent of the weight. ■
Blubber and fat are poor conductors of heat; hence they protect the animal from losing internal heat. ■ The thicker
the blubber or fat layer the less the heat loss. Marine mammals inhabiting polar waters, therefore, have thicker layers
of fat than temperate and tropical species

9, Look at the four squares ■ that indicate where the following sentence could be added to the passage
Its thickness varies from one marine mammal to another.
Where would the sentence best fit? Click on a square ■ to add the sentence to the passage.

10, Directions: An introductory sentence for a brief summary of the passage is provided below. Complete the
summary by selecting the THREE answer choices that express the most important ideas in the passage. Some
sentences do not belong in the summary because they express ideas that are not presented in the passage or are minor
idea-in the passage. This question is worth two points.

第 280 页 共
373 页
2021 托福阅读真题

Marine mammals have evolved special adaptations that allow them to conserve body heat.

A,Because the thermal conductivity water differs from that of air marine animals' adaptations for regulating body
temperature differ significantly from those of land animals.
B, Because large, fatty bodies lose less heat than small. thin ones, many marine mammals are adapted to reach
adulthood in a relatively short period of time
C, Because fat is a poor conductor of heat having a thick layer of blubber beneath the skin helps prevent heat loss.
D, Mammals that spend any significant amount of time in marine waters are large because a body with a large
volume relative to its surface area loses relatively little internal heat.
E, Marine mammals need to maintain high internal temperatures in part to prevent the blood in their extremities from
freezing when the extremities are in contact with water.
F, The amount of heat the extremities lose to the outside is reduced by having a circulatory system in which blood in
the veins absorbs heat from blood in the arteries.

The Sistine Ceiling

Paragraph 1: The term fresco refers to a number of techniques for painting images on a room surface. In buon
fresco, pigments (mineral colors) are mixed with water and then applied to a layer of wet, lime-based (alkaline)
plaster. Because the pigments become embedded in the plaster as it dries no glue or other binder is needed to hold

第 281 页 共
373 页
2021 托福阅读真题

them in place and buon frescoes are very durable. However, some pigments, especially shades of blue, cannot be
used in combination with wet plaster, and sin pigments can be applied only as long as the plaster remains damp
mistakes can be corrected only by replastering and then repainting the affected area.In contrast, a secco fresco is
done on a dry surface, thus requiring a binder. Sometimes artists combined the two techniques, beginning a work in
buon and then, once the plaster dried, using a secco to correct mistakes . add colors that could not be applied using
buon.

1, Paragraph 1 supports which of the following statements about secco frescoes?


A Images in them are easier to change than are images in buon frescoes
B They cannot be painted on a surface containing alkaline materials.
C They are more difficult to paint on a uneven surface than are buon frescoes
D They use a smaller range of colors than can be used in buon frescoes.

Paragraph 2: The biblical scenes created by Michelangelo for the ceiling of the Sistine Chapel in the Vatican City
are among the world's most famous frescoes. Commissioned in 1508. Michelangelo began his work reluctantly
considering himself more of a sculptor than a painter, and spent four years perched on scaffolding, painting some
300 figures across about 5000 square feet of ceiling. Despite his initial hesitation, Michelangelo followed the
purest fresco tradition of the time, applying his pigments mixed in water directly to fresh lime plaster and
choosing only those that are chemically compatible avoiding azurite, vermilion and lead-based colors, all of
which deteriorate under the alkaline conditions of the lime. The incredible scale of the task required him to
execute each scene in patchwork.with about five square meters worth of wet plaster applied per giornata, or"day's
work." use of plaster relatively low in lime which dries more slowly, bought him some extra time, but it is clear
that Michelangelo painted each giornata extremely quickly rarely, if ever resorting to finishing a section with
touch-up paint containing binder.

2,The word "reluctantly" in the passage is closest in meaning to


A nervously
B carefully
C with little experience
D unwillingly

3. Which of the sentences below best expresses the essential information in the highlighted sentence in the passage?
Incorrect choices change the meaning in important ways or leave out essential information
A. Michelangelo did not hesitate to follow the purest fresco method although he found that some traditional
pigments were not compatible with lime plaster.
B. Michelangelo at first hesitated to mix water with the pigments he directly applied to lime plaster, fearing that the
mixture might deteriorate under the alkaline conditions

第 282 页 共
373 页
2021 托福阅读真题

C. Michelangelo followed the purest fresco method of applying pigments to fresh lime plaster, using only pigments
that do not deteriorate in lime
D. With the exception of azurite, vermilion and lead-based colors Michelangelo mixed his pigments in water before
applying them to the lime plaster.

Paragraph 3: Over the years following their completion, the Vatican frescoes accumulated a coating of soot and dirt
that dulled and darkened the images. In 1787 the German writer Goethe made the following observation:"On 2nd
February we went to the Sistine Chapel to witness the ceremony of the blessing of the candles. I thought it is
precisely these candles that over three centuries have blackened these splendid frescoes; this is the incense that has
not only with its smoke covered over the sun itself of art, but with every year continues to dirty it and will finally
engulf it in darkness."

4, According to paragraph 3 what did Goethe believe would happen to the Sistine frescoes?
A,They would be destroyed by building fires started by candles.
B,They would be replaced with new frescoes.
C, The images in them would eventually no longer be visible
D, They would be destroyed if exposed to sunlight.

5. In paragraph 3 why does the author include Goethe's observations in 1787 about the Sistine frescoes?
A To explain how Goethe became interested in efforts to restore the frescoes
B To identify the year in which efforts to restore the frescoes began 个
C To describe the ceremonial function of the frescoes
D To support the claim that the frescoes darkened over time

Paragraph 4: While Goethe's dire prediction did not come completely to pass the slow darkening of the frescoes led
to a characterization of Michelangelo as a relatively somber artist with little appreciation for color. Deposits of salt
from rainwater let in by the leaking roof and from within the building materials themselves compounded the decline
leading to blistering of the paint. Eventually a full-time restorer was appointed to maintain the frescoes. Salt crystals
and dirt were removed by scrubbing with sponges dipped in wine, paints containing binder were used to touch up
sections, and varnish was applied to" renew" the colors.

6, According to paragraphs 3 and 4, all of the following contributed to the deterioration of the Sistine ceiling frescoes
EXCEPT
A the burning of candles
B the removal of salt crystals on the frescoes
C the leaking of the roof
D the images' exposure to salt from building materials

第 283 页 共
373 页
2021 托福阅读真题

7, According to paragraph 4, why did some people conclude that Michelangelo had "little appreciation for color"?
A Goethe had characterized Michelangelo’s frescoes as dull.
B Michelangelo was known to have had a somber personality.
C Michelangelo sometimes darkened colors by adding certain pigments to them.
D The buildup of layers of dark material made the fresco colors beneath seem dull.

Paragraph 5: The net effect of several hundred years of this cosmetic maintenance was a dark film consisting of
layers of dust, soot, and varnish entirely covering the frescoes. In1980 a massive effort began to restore
Michelangelo's work to its original glory. Restorers painstakingly removed the layers of grime and varnish, the
touch-up paint and the salt deposits with solution appropriate for each layer. A dilute solution of acrylic paint was
applied only when absolutely necessary to reinforce the existing paint.

Paragraph 6: Over the course of the next fourteen years the bright colors that emerged were almost shocking to
those accustomed to the dull earth tones that had been the status quo for centuries. Indeed. the work at the Sistine
Chapel has led to vigorous debate about the role of restoration. Restorers often cast themselves in the role of
interpreting the original artist's intent, using modern materials to override the effects of time. However, some critics
believe that aging is part of the natural evolution of art and, therefore, new is not necessarily better.

8,The word "net" in the passage is closest in meaning to


A practical
B final
C negative
D visual

Paragraph 1: The term fresco refers to a number of techniques for painting images on a room surface. ▇ In buon
fresco, pigments (mineral colors) are mixed with water and then applied to a layer of wet, lime-based (alkaline) plaster.
▇ Because the pigments become embedded in the plaster as it dries no glue or other binder is needed to hold them in
place and buon frescoes are very durable. ▇ However, some pigments, especially shades of blue, cannot be used in
combination with wet plaster, and sin pigments can be applied only as long as the plaster remains damp mistakes can
be corrected only by replastering and then repainting the affected area. ▇ In contrast, a secco fresco is done on a dry
surface, thus requiring a binder. Sometimes artists combined the two techniques, beginning a work in buon and then,
once the plaster dried, using a secco to correct mistakes . add colors that could not be applied using buon.

9, Look at the four squares ▇ that indicate where the following sentence could be added to the passage
Two methods of applying pigments, each with distinct advantages. were popular in the 1500s.
Where would the sentence best fit? Click on a square ▇ to add the sentence to the passage

第 284 页 共
373 页
2021 托福阅读真题

10, Directions: An introductory sentence for a brief summary of the passage is provided below. Complete the
summary by selecting the 3 answer choices that express the most important ideas in the passage. Some sentences do
not belong in the summary because they express ideas that are not presented in the passage or are minor ideas in the
passage. (This question is worth 2 points.)

A. Michelangelo used the purest fresco technique to cover the ceiling with imaged working quickly to complete as
much work as possible each day.
B. Early restorers used dull colors to touch up areas when making repairs to the Sistine frescoes, mistakenly
believing that Michelangelo had used such colors.
C. Restorers undertook detailed work to make the ceiling look as it did when fi completed, and this led to
disagreement among critics over the appropriateness of restoring old works of art.
D. Although Michelangelo was very knowledgeable about fresco technique, the pigments in his images quickly
began to lighten, with the result that many figures were no longer visible by Goethe's time.
E. The original colors revealed by the restoration beginning in 1980 had become obscured as a result of
accumulations of soot and dirt damage to the building, and other problems
F. The restoration of the ceiling started to debate about using certain types of material in art restorations and the
difficulty of knowing what an artist working centuries ago intended

Wool Manufacturing in Europe in the High Middle Ages

Paragraph 1: During the High Middle Ages(A.D.1000-1300, the Baltic Sea North Sea, and English Channel tied
together the peoples of Scandinavia, Lithuania, northern Germany. England, and Flanders (a Dutch-speaking region
along the North Sea).Scandinavian fish and timber, Baltic grain, English wool, and Flemish cloth circulated, linking
these lands in a common economic network. In this region, as in southern Europe, there developed urban merchant
and manufacturing communities linked by sea routes and distinguished by a distinctly urban commercial mentality.

第 285 页 共
373 页
2021 托福阅读真题

1, The word "distinguished" in the passage is closest in meaning to


A, supported
B, connected
C, influenced
D, marked

Paragraph 2: The earliest of those interrelated communities were the wool-exporting towns of England, particularly
London, and the cloth-producing towns of Flanders, Brabant (a region adjacent to Flanders), and northern France.
Chief among these cloth-producing towns were Ghent, Bruges, and Ypres.Both Flanders and England had bee known
for their cloth production since Roman times. In the eleventh century, Flanders, lacking the land for large-scale sheep
grazing and facing a growing population began to specialize in the production of high-quality cloth made from
English wool. At the same time, England, which experienced an economic and population decline following the
Norman Conquest in 1066, began to export the greater part of its wool to Flanders to be worked. The production of
wool cloth began to change from small-scale industry carried on in the home by family members using their own
equipment into Europe's first major industry.

2, It can be inferred from paragraph 2that Flanders did not specialize in wool production because
A, Flanders was too small to produce a great deal of wool
B, Flanders’ tradition of cloth production was too strong
C, Flanders was conquered by a foreign country
D, the population declined in rural areas of Flanders

Paragraph 3: Woolen manufacture was a natural activity for such a transformation. The looms required to
manufacture wool cloth were large and expensive. The skills needed to produce the cloth were complex. The need
for water both to power looms and to wash the cloth during production tended to concentrate cloth manufacture
along waterways. As competition increased only centralization and regulation of manufacture could ensure quality
control and thus enhance marketability.And Europe's growing population provided the first large-scale market for
manufactured goods since the disintegration of the Roman Empire For all of those reasons, by the late eleventh
century the traditional method of cloth production had been transformed. No longer did individual women sit in
farmhouses spinning and weaving. Now manufacture was concentrated in towns, and men replaced women at the
looms. Furthermore, production was closely regulated and controlled by a small group of extremely wealthy
drapers(cloth merchants).

3, According to paragraph 3, all of the following were required to successfully manufacture wool cloth EXCEPT
A, large, expensive looms
B, workers with complex skills

第 286 页 共
373 页
2021 托福阅读真题

C, accessible waterways for transporting wool and cloth


D, a water source that could provide enough energy to run machinery

4, All of the following are identified in paragraph 3 as ways in which the cloth industry changed EXCEPT:
A, The cloth produced became more consistent in quality
B, Men began performing aspects of cloth making previously associated with women.
C, Manufactured cloth goods became more varied to increase product marketability.
D, Cloth production began to take place mainly in urban settings.

Paragraph 4: Concentration of capital, specialization of labor, and an increased urban population created vibrant,
exciting cities essentially composed of three social orders. At the top were wealthy patricians-the drapers. Their
agents traveled to England and purchased raw wool, which they then distributed to weavers and other master artisans.
The master artisans using equipment rented from the patricians-directed less-skilled workers in the other tasks such
as carding dyeing, and spinning.Finally, the finished cloth was returned to the patricians, whose agents then
marketed it throughout Europe. Through their control of raw materials, equipment, capital, and distribution, the
drapers controlled the cloth trade, and thus the economic and political life of Flemish wool towns. Through their
closed trade groups, they controlled production and set standards, prices, and wages. They also controlled communal
government by monopolizing urban councils. In wealth and power, the drapers had become almost indistinguishable
from the great land- owning nobles.

5, According to paragraph 4, which of the following statements about the drapers is true?
A,They mostly belonged to the class of great nobles
B,They dominated the political life of the wool towns.
C,They were the highest-ranking master artisans.
D,They traveled throughout Europe to market finished cloth.

Paragraph 5: At the bottom of urban society were the unskilled and semiskilled artisans, called blue nails because
constant work with dye left their fingers permanently stained. Their existence was more uncertain than that of most
peasants (farmworkers). Employed from week to week without any guarantee of future employment, paid barely
living wages, and entirely dependent on the woolen industry for their livelihood, they often hovered on the edge of
subsistence. In the early fourteenth century, the temporary interruption of grain shipments from northern Germany to
Ypres left thousands dead of starvation. Unsurprisingly, from the thirteenth century on, these workers were
increasingly hostile to the patricians. Sporadic rebellions and strikes spread across Flanders, Brabant, and northern
France. Everywhere they were ruthlessly suppressed. The penalty for organizing a strike was death.

Paragraph 6: Between the patricians and the unskilled and semiskilled workers stood the masters the skilled
artisans who controlled the day- to- day production of cloth and lesser crafts. Masters organized into guilds, with

第 287 页 共
373 页
2021 托福阅读真题

which they regulated every aspect of their trades and protected themselves from competition. The masters often
leased their looms or other equipment from the drapers and received from them raw materials and wages to be
distributed to their workers.

6, The word “temporary” in the passage is closest in meaning to


A, dangerous
B, sudden
C, minor
D, lasting a limited time

7, According to paragraph 5, what was true of the unskilled and semiskilled artisans called blue nails
A, They accepted low wages to avoid the harsh punishments that would result from rebelling.
B, They were unsure whether their employment would continue beyond a given week.
C, They often took jobs outside the cloth industry to add to their income
D, They became increasingly difficult to suppress from the thirteenth century on,

8, In paragraph 5, why does the author include the information that the interruption of grain shipments left thousands
of workers dead of starvation?
A, To support the idea that survival was very uncertain for workers at the bottom of urban society
B, To help explain why the patricians decided to ruthlessly suppress rebellions and strikes
C, To give an example of the consequences suffered by workers for organizing strike
D, To explain why it was possible for strike and rebellions to spread across Flanders,Brabant, and northern France

Paragraph 3: Woolen manufacture was a natural activity for such a transformation. ■ The looms required to
manufacture wool cloth were large and expensive. ■ The skills needed to produce the cloth were complex. ■ The
need for water both to power looms and to wash the cloth during production tended to concentrate cloth manufacture
along waterways. ■ As competition increased only centralization and regulation of manufacture could ensure quality
control and thus enhance marketability.And Europe's growing population provided the first large-scale market for
manufactured goods since the disintegration of the Roman Empire For all of those reasons, by the late eleventh
century the traditional method of cloth production had been transformed. No longer did individual women sit in
farmhouses spinning and weaving. Now manufacture was concentrated in towns, and men replaced women at the
looms. Furthermore, production was closely regulated and controlled by a small group of extremely wealthy
drapers(cloth merchants).

9, Look at the four squares ■ that indicate where the following sentence could be added to the passage.
Woolen cloth had never been very suitable for home production because of the demands involved in its
manufacture.

第 288 页 共
373 页
2021 托福阅读真题

Where would the sentence best fit? Click on a square (■ ) to add the sentence to the passage.

10, Directions: An introductory sentence for a brief summary of the passage is provided below. Complete the
summary by selecting the 3 answer choices that express the most important ideas in the passage. Some sentences do
not belong in the summary because they express ideas that are not presented in the passage or are minor ideas in the
passage. This question is worth 2 points.

A, Although timber, fish, and grains were the first goods to be traded in the common economic network linking
northern European countries, raw wool and cloth came to be more important.
B, Europe's growing population provided a market for large-scale wool manufacture, which required specialized
labor and became concentrated along the waterways needed to power the looms.
C, A small group of wealthy patricians dominated the wool towns both politically and financially and controlled
every aspect of the cloth trade, from raw materials to the distribution of finished products.
D, Flanders had been known since Roman times for its production of high-quality cloth made from English wool,
which it had imported by way of the sea routes that linked the two lands.
E, Guilds were closed and often secret organizations through which drapers and artisans worked together to set
standards for cloth production and prices at which cloth was sold.
F, Masters-skilled artisans organized into guilds-directed the work of poorly paid unskilled and semiskilled workers
who were totally dependent on the wool industry for a living.

Silver in North America in the Late Eighteenth Century

Paragraph 1: Silver and silver objects had multiple meanings in late-eighteenth-century Nor America. Insights
about social attitudes and the buying habits of consumers come partly from the records kept by silversmiths such as
Paul Revere, the most politically active, artistically innovative, and successful silversmith of the period. His
customers came from a wide range of socioeconomic groups-artisans, middle-income merchant the business elite,
and government appointees. Revere produced more low-end goods--eating utensils, shoe or knee buckles, buttons,

第 289 页 共
373 页
2021 托福阅读真题

and harness fittings--than high-end. The elite preferred to buy more elaborate silver goods from England or other
European countries. Much of his success after the American Revolution- (1776-1783), however, lay in his ability
respond to demands for specific kinds of more elaborate silver objects, in particular sugar bowls, creamers, and
teapots. One expert notes that after the Revolutionary War, Revere dominated silver teapot production, taking
advantage of a return to popularity of a beverage that earlier symbolized a resistance to English taxes and the
consumption of which had thus diminished immediately prior to and during the war.

1, Paragraph 1 suggests that we would most likely learn about customer demand for "eating utensils, shoe or knee
buckle— buttons and harness fittings" from which of the following sources?
A, Records kept by silversmiths
B, Letters written by Paul Revere
C, Lists of objects from the homes of elite Americans
D, English tax documents

2, Which of the sentences below best expresses the essential information in the highlighted sentence in the passage?
Incorrect choices change the meaning in important ways or leave out essential information
A, Revere took advantage of high English taxes to gain a dominant position in the production of silver teapots and
other tea products.
B, Revere became a successful maker silver teapots when lower taxes after the Revolutionary War made tea more
affordable
C, The high quality of silver teapots produced by Revere helped tea to again become a popular beverage after the
Revolutionary War.
D, Revere became one of the primary producers of silver teapots after the Revolutionary War. when tea returned to
widespread popularity.

Paragraph 2: Before the war, inhabitants of the American colonies had preferred to purchase silver single
pieces, thus slowly acquiring complete tea sets; after the war the purchase of larger tea services as a complete set
became popular. Revere's production of such en suite tea services was facilitated by the availability of rolled sheet
silver, which was a more economical method of producing silver vessels— particular fluted teapots-which have
grooves or ridges-like the one at the center of the set he made for a wealthy Boston merchant and his wife, John and
Mehitable Templeman, in 1792. The rolled sheet metal also made it easier to achieve the fluting and straight edges
that mark these objects. Their clean lines and finely engrave—designs of draped fabric and tassels recoil neoclassical
forms (inspired by ancient Greek and Roman art) that were then popular in much of Europe and associated with
newly emerging republics on both sides of the Atlantic. These neoclassical attributes found on all manner of
goods and buildings in the immediate post-Revolution period, were the hallmarks what became known as the
Federalist style. Revere also engraved Templeman's initials-- JMT--on the spoon handles, stands, and sugar urn,
reinforcing the association between these objects and their owners, and serving as a kind of insurance, making it

第 290 页 共
373 页
2021 托福阅读真题

easier to track down the owner if the item was stolen

3, The word "attributes" in the passage is closest in meaning to


A, trends
B, innovations
C, features
D, images

4, According to paragraph 2, which of the following best describes the way in which people bought silver tea sets
after the Revolutionary War?
A, People began buying tea sets without grooves or ridges on them.
B, People stopped buying tea sets that were made from rolled sheet silver
C, People preferred to buy silver tea sets that were produced in Greece or Rome
D, People began buying entire silver tea sets at one time instead of buying one single piece at a time

5, According to paragraph 2. which of the following best describes how people responded to the neoclassical style
during the late eighteenth century?
A, They began demanding that rolled sheet silver be used to make objects other than tea services
B, They began preferring objects with grooves and fluting to objects with clean lines
C, They began buying copies of Greek and Roman artworks that inspired the style
D, They began associating the style with the new republican governments arising.

Paragraph 3: Much of the silver in the homes of the well- to-do in the eighteenth century took the form of coins
that were often melted down and transformed into domestic objects, and sometimes remelted to pay off debts or to
fund revolutionary armies. This transference from craft to currency and back suggest the particular place that silver
held in colonial and post-Revolutionary America According to historian Richard Lyman Bushman, silver was “a
powerful material for establishing identity and configuring hierarchical relationships... Everywhere, silver was used
to command assent, to assert authority, or to claim respect." Silver's power, notes Bushman, lay in the fact that could
signify three things at once: money; beauty, which was associated with divinity; and rank. In an era of dramatic
fluctuations in the value of paper currency silver maintained its value
6, The word "signify" in the passage is closest in meaning to
A, mean
B, bring
C, join
D, contrast

7, According to paragraph 3, in which TWO of the following ways was silver used in colonial and

第 291 页 共
373 页
2021 托福阅读真题

post-Revolutionary America? To receive credit, you must select TWO answers.


A, It was used to pay revolutionary soldiers.
B, It was used as money.
C, It was melted down for use in weapons.
D, It was used to create images of divine beings.

Paragraph 4: Yet the sheen of silver was tarnished—both its origins and its association with thievery and deceit.
■Much American silver originated in mines in Mexico and Peru and was extracted through a system of forced
indigenous labor marked by long hours and appalling conditions, conditions that were remarked upon in political
treatises and literary tracts available to American colonists. ■Many merchants, through then participation in the slave
trade, then acquired the coins created from this silver. ■Once acquired, owners of such silver had to worry about
thieves who recognized the wealth and status associated with this valuable metal and who were not averse to
counterfeiting it as well as stealing it. ■Thus, as Bushman writes, "silver figured in complex and ambiguous
narrative of power." Whether as coinage or as teapots, it represented both elite status and a more democratic
consumer revolution, as well as the ugly underside of the economic system that made both possible.

8, Which of the following best describe the relationship of paragraph 4 to paragraph 3?


A, Paragraph 4 describes negative aspects of the economic system that gave silver its importance, as described in
paragraph 3.
B, Paragraph 4 explains why silver was a popular material among the American elite as described in paragraph 3
C, Paragraph 4 explains how the power of silver as a symbol of wealth and status, described in paragraph 3,
increasingly diminished.
D, Paragraph 4 provides additional evidence for the claim in paragraph 3 that silver successfully maintained its value

9, Look at the four squares I that indicate where the following sentence could be added to the passage.
Problems noted included deep narrow shafts, weak wooden ladders, underground flooding, and generally
unsafe work environments.
Where would the sentence best fit? Click on a square I to add the sentence to the passage

10, Directions: An introductory sentence for a brief summary of the passage is provided below. Complete the
summary by selecting the 3 answer choices that express the most important ideas in the passage. Some sentences do
not belong in the summary because they express ideas that are represented in the passage or are minor ideas
in the passage. This question is worth 2 points.

Silver and silver objects held both economic and social significance in late-eighteenth- century North America.

A, While elites were the main North American consumers of high-end silver objects before the American Revolution,

第 292 页 共
373 页
2021 托福阅读真题

afterward a variety of socioeconomic groups were able to afford them


B, Silver was used both as money and to create practical and decorative goods, which made it an important source of
power and social rank.
C, Consumers enjoyed the greater availability of silver objects, but it had serious social consequences, as silver
was mined through forced labor and silver coins were often obtained through the slave trade.
D, Silversmith Paul Revere produced a variety of goods, but after the Revolutionary War he became the most
popular maker of silver teapots and tea sets, which were decorated in the neoclassical style.
E, Over time American colonists became somewhat more reluctant to acquire silver coins and objects because of the
growing threat of theft
F, Silver was more difficult to counterfeit than paper currency was which is largely why merchants preferred it for
their transactions and generally valued it more highly.

Building a Modern Japan

Paragraph 1: Japan's Meiji Revolution of the late nineteenth century replaced the conservative Tokugawa regime,
which had governed the country for some 250 years, with a new government determined to build a modern country.
One of the first steps the new administration took was to improve government revenues by means of a redesigned
land tax. The significance of this new tax system went beyond securing revenues. It changed the economic
relationship of the individual landowners the state and to each other. In the Tokugawa system, land ownership had

第 293 页 共
373 页
2021 托福阅读真题

been decided custom in the villages. Revenues were collected in lump sums from villages not individuals. There was
no state-supervised system of property titles or land registration, and no officially sanctioned market in the purchase
and sale of land. In addition taxes the state and to each other. In the Tokugawa system, land ownership had been
decided by custom in the villages. Revenues were collected in lump sums from villages not individuals. There was
no state-supervised system of property titles or land registration, and no officially sanctioned market in the purchase
and sale of land. In addition taxes were based on officially estimated grain yield, not assessed value. They were
usually collected in kind (in rice). This meant that the government stood to lose from fluctuating commodity prices:
if the price of grain fell, so did government revenue.

1, The word "sanctioned” in the passage is closest in meaning to


A, approved
B, created
C, operated
D, planned

2, According to paragraph 1, the amount money the Tokugawa regime obtained through taxation
A, was affected by customs in grain pricing that differed from village to village
B, was collected from each individual landowner
C, rose when a system of land registration was enforced
D, varied based on changing grain prices

Paragraph 2: The tax system of 1873 changed all this. It provided for a national land survey, conducted in the
mid-1870s, that matched owner to every piece of land and issue deeds. It also assessed the market value of all plots
of land. Finally, it set the land tax at 3 percent of assessed value. This new system gave the government a predictable
annual revenue. The new tax system also brought the national government into a direct economic relationship with
household heads. It shifted the risk and the opportunity of commodity price changes onto the taxpaying farmers, who
would have to sell a larger portion of their crop to pay the tax if grain prices fell. They would conversely profit from
higher prices. This made people aware of their economic and political ties to the state as they had never been before.
Not surprisingly, taxes and state budgets became two of the most contentious political issues of the Meiji era.

3,According to paragraph 2, what was the primary benefit of the Meiji tax system?
A, It increased the market value of privately owned land
B, It allowed the government to anticipate its yearly income.
C, It encouraged landowners to replace farming with other economic activities.
D, It removed the economic burden of changing prices from individual farmers.

4, Paragraph 2 suggests that a rise in grain values under the new tax system could result in which of the following?

第 294 页 共
373 页
2021 托福阅读真题

A, An increase in the assessed market value of farmer's land


B, An increase in the total amount of taxes owed by farmers
C, A decline in the percentage of assessed land value that farmers owed in tax
D, A decline in the amount of grain that farmers needed to sell in order to pay their tax

Paragraph 3: Over a period of decades, the new government used a portion of these new tax revenues for public
works and institution- building projects to create the infrastructure of an industrial economy. It dredged harbors and
built lighthouses to improve coastal shipping. It built telegraph lines beginning in 1869. and in 1871 it opened a
postal system modeled on British practices. It encouraged the founding of joint stock companies among private
investors. And by the mid-1880s. it had established a uniform national currency, the yen, backed by a central bank
modeled along European lines

5, The word "uniform" in the passage is closest in meaning to


A, stable
B, new
C, single
D, powerful

6, According to paragraph 3. the new regime used tax revenues for all of the following EXCEPT
A, improving coastal shipping
B, building telegraph lines
C, establishing a new postal system
D, creating joint stock companies

Paragraph 4: Most important of all, the government took the lead in building a railroad network. The first line
connected Tokyo to Yokohama and was completed in 1872. It was extended as far as Kobe by 1889 The government
also encouraged private investment in railroads Many former noblemen pooled their pensions to join wealthy
commoners in a railroad investment boom in the 1880s that helped bring a modern stock market into existence. By
1890, Japan boasted fourteen hundred miles of railroad, about 40 percent owned and operated by the
government, the rest in private hands.

7, According to paragraph 4. investment in railroads contributed to


A, a significant increase in government pensions
B, the rise of the modern stock market Japan
C, the end of government involvement Japanese railroads
D, the completion of the railway line from Tokyo to Yokohama

第 295 页 共
373 页
2021 托福阅读真题

Paragraph 5: As it did all over the world, the railroad had huge cultural as well as economic impact. It changed
people's sense of time, of distance, and of social behavior. As in the West several decades earlier, Japanese observers
in the 1870s and 1880s noted that the railroad made the world smaller with its speed, and that trains, with their exact
timetables, promoted punctuality. These changes came slowly. Complaints about slow and inefficient railway
performance were common into the 1890s. Nevertheless, the fact that customers were complaining itself indicates
that attitudes had changed. The advent of trains meant that for the first time in Japanese history, it became important
to calculate time to the minute-rather than the half hour. This promoted the uses of watches and clocks. Gradually,
increased attention to precise timekeeping spread among the entire population

8, Why does the author mention that“ increased attention to precise timekeeping spread among the entire
population”?
A, To illustrate the claim that the railroad h" a cultural impact in Japan
B, To suggest that watches and clocks became popular later in Japan than in the West
C, To explain why it was necessary for trains to follow exact timetables
D, To indicate the reason that complaints became more frequent among rail customers

Paragraph 1: Japan's Meiji Revolution of the late nineteenth century replaced the conservative Tokugawa regime,
which had governed the country for some 250 years, with a new government determined to build a modern country.
■ One of the first steps the new administration took was to improve government revenues by means of a redesigned
land tax. ■ The significance of this new tax system went beyond securing revenues. ■ It changed the economic
relationship of the individual landowners the state and to each other. ■ In the Tokugawa system, land ownership had
been decided custom in the villages. Revenues were collected in lump sums from villages not individuals. There was
no state-supervised system of property titles or land registration, and no officially sanctioned market in the purchase
and sale of land. In addition taxes the state and to each other. In the Tokugawa system, land ownership had been
decided by custom in the villages. Revenues were collected in lump sums from villages not individuals. There was
no state-supervised system of property titles or land registration, and no officially sanctioned market in the purchase
and sale of land. In addition taxes were based on officially estimated grain yield, not assessed value. They were
usually collected in kind (in rice). This meant that the government stood to lose from fluctuating commodity prices:
if the price of grain fell, so did government revenue.

9, Look at the four squares ■ that indicate where the following sentence could be added to the passage.
In order to build the infrastructure required for an industrialized economy, government needed secure
revenues.
Where would the sentence best fit? Click on a square ■ to add the sentence to the passage

10, Directions: An introductory sentence for a brief summary of the passage is provided below. Complete the
summary by selecting the 3 answer choices that express the most important ideas in the passage. Some sentences do

第 296 页 共
373 页
2021 托福阅读真题

not belong in the summary because they express ideas that are not presented in the passage or are minor ideas in the
passage. This question is worth 2 points.

Japan's Meiji Revolution of the late 19th century brought in government determined to transform Japan into
a modern country

A, In order to improve government revenues, a new system for recording land ownership was created along with a
tax on the value of land that was payable by individual landowners
B, Under the new tax system, farmers were able to reduce the amount hey paid in taxes by taking advantage of
variations in the price of grain and selling their grain crops at the highest possible price.
C, Reports of foreign observers at the time show that the arrival o railroads had much the same impact on people in
Japan during the 1870s and 1880s that it had earlier had on people in the West.
D, The government's primary aim in creating the new tax system was to change the traditional economic and
political relationship of individual landowners to the national state and to each other.
E, The government created a national currency backed by a central bank and us tax revenues for public works and
institution- building projects to create infrastructure of an industrial economy.
F, The development of a railroad network had an enormous economic impact, but it also changed people's
perceptions of their world an increased their attention to timekeeping.

Canals and Railroads in the Early Nineteenth Century

Paragraph 1: As the American early nineteenth the Jacksonian economy developed in the century--particularly
during Era (named for Andrew Jackson,president in the 1830s)-- transportation improvements played a crucial role,
with the construction of better roads, the creation of the first railroad lines, and the digging of canals. Of all these
developments, the Erie Canal had the greatest immediate impact because it provided the first all-water route directly
connecting the Great Lakes and the rapidly growing Midwest to the major eastern seaport at New York City. Th e

第 297 页 共
373 页
2021 托福阅读真题

canal proved so successful that it set off a wave canal building across the country: cities like Baltimore and
Philadelphia and states from Virginia to Illinois eagerly sought to imitate New York and tap into the profitable
trade that was flowing between its cities of Buffalo and Troy and down the Hudson River to New York City

1, The word "imitate" in the passage is closest in meaning to


A, copy
B, Join
C, get help from
D, compete with

2, According to paragraph 1, which of the following is true of the Erie Canal?


A, It was the first major transportation improvement during the Jacksonian Era.
B, It allowed trade to be conducted between New York City and the Midwest by water.
C, It had a smaller impact on the economy than the construction of better roads did.
D, It was soon linked with older canals that already existed across the country.

Paragraph 2: The Erie Canal not only carried manufactured goods westward and transported western
produce eastward; it also became the passageway for thousands of native-born white Americans and
European immigrations into the upper reaches of New Pennsylvania and beyond that, into the burgeoning
mid-western states Indiana, Illinois, and Michigan. Along its course, boom towns like Utica, Syracuse and
Rochester (cities in the state of New York) sprang up to supply the great migration and take advantage of the
growing trade that flowed back and forth along its heavily traveled canal towpaths. As New York's success invited
emulation, members of Congress and the Jackson administration were sent many requests for federal funding to help
build roads canals, and railroads in other parts of the country. Most of these requests were turned down with the
reminder that New York had constructed its canal system with its own state funds

3, Which of the sentences below best expresses the essential information in the highlighted sentence in the passage?
Incorrect choices change the meaning in important ways or leave out essential information.
A, The Erie Canal was not used for transporting goods as much as it was used for transporting European immigrants
into the upper reaches of New York and Pennsylvania
B, In addition to transporting goods east and west, the Erie Canal was an important passageway for transporting
immigrants and other Americans into New York and the mid-western states
C, Native-born white Americans and Europe immigrants began to use the Erie Canal in transporting manufactured
goods into the mid-western states and western produce in New York
D, Thousands of native-born white Americans and European immigrants began moving into the upper reaches of
New York and beyond as they realized the Erie Canal's importance in carrying goods

第 298 页 共
373 页
2021 托福阅读真题

4, According to paragraph 2, which of the following did NOT contribute to the rapid growth of Utica, Syracuse, and
Rochester?
A, Construction funding from the state of New York
B, Money provided by Congress and the Jackson administration
C, Increasing commerce along the Erie Canal
D, Large numbers of migrants traveling to regions in the West

Paragraph 3: The canal craze slowed appreciably by the late 1830s. Canals were costly to build ($20,000-$30,000
per mile and upward) and maintain. Both winter weather and recurrent flood damage interrupted the flow of traffic.
Financial returns were inadequate. Most significant of all, steam-powered railroads appeared just a few ears after
the canals. When boom times ended with the Panic of 837. followed by six years of economic depression,
Pennsylvania and Indiana stood virtually bankrupt, while states like Ohio found themselves in financial difficulty
because of their overextended investments in canal building. The canal boom halted around 1840 by which time the
United States had built 3,326 miles of canals at a cost of roughly $125 million, or $37,580 per mile. Railroad
construction, by contrast, had already reached 3,328 miles at an average cost of $17,000 per mile. Although most
lines existed in the Northeast, railroads could be found in all parts of the country by the 1840s, and no end was in
sight. By then, no one questioned that the Hiron horse represented the wave of the future.

5, The word "Inadequate” in the passage is closest in meaning to


A, lost
B, uncertain
C, getting lower
D, not enough

6, In paragraph 3, why does the author provide the information that the average cc of railroad construction was
$17,000 per mile
A, To emphasize that railroads were cheaper to build than canals
B, To suggest that railroad construction was one reason behind the financial difficulties of some states
C, To indicate that the government had spent more money or railroad construction expected
D, To demonstrate why more railroad were built in the Northeast compared other parts of the country
7, What can be inferred from paragraph 3 about steam-powered railroads?
A, They were introduced shortly after the canal boom ended.
B, They allowed states like Pennsylvania. Indiana, and Ohio to avoid economic depression
C, They were an attractive transportation alternative to canals
D, They spread to the Northeast after originating in another part of the country.

Paragraph 4: In addition to costing less to build, rail lines could serve areas that could not be reached by boats or

第 299 页 共
373 页
2021 托福阅读真题

barges in either the drought summer or the freezing of winter. Although canals and steamboats could ship goods
more cheaply, railroads provided much faster service. Whereas horse- and mule- drawn freight wagons and canal
boats averaged around two miles an hour, freight trains and steamboats averaged around twelve. More important,
because railroads, unlike steamboats and canals, could be built more directly from one geographic point to another,
they saved even more time than their traveling speeds suggest. During the 1850s, for example, a trip from Cincinnati
to New York City by (horse-drawn) freight wagon could take forty to fifty days or more depending s weather
conditions. The same goods shipped downriver by steamboat to New Orleans a then by packet ship to New York
require twenty-eight days, while those shipped on the Ohio canal system across Lake Erie to the Erie Canal and
down the Hudson took eighteen. Railroads, by contrast, required only six to eight days, thus cutting the time by more
than half that of their closest waterborne competitors.

8,According to paragraph 4, what advantage did canal transport have over railroad a steamboat transport?
A, Faster travel speeds during certain times of the year
B, More direct connections between major geographic points
C, Better access to some areas in extreme weather
D, A cheaper cost of shipping

Paragraph 1: ■As the American early nineteenth the Jacksonian economy developed in the century-particularly
during Era (named for Andrew Jackson,president in the 1830s)- transportation improvements played a crucial role,
with the construction of better roads, the creation of the first railroad lines, and the digging of canals. ■Of all these
developments, the Erie Canal had the greatest immediate impact because it provided the first all-water route directly
connecting the Great Lakes and the rapidly growing Midwest to the major eastern seaport at New York City. ■The
canal proved so successful that it set off a wave canal building across the country: cities like Baltimore and
Philadelphia and states from Virginia to Illinois eagerly sought to imitate New York and tap into the profitable
trade that was flowing between its cities of Buffalo and Troy and down the Hudson River to New York City. ■

9, Look at the four squares ■ that can where the following sentence could be added to the passage.
After the completion of the Erie Canal in 1825, the city of New York witnessed significant economic growth.
Where would the sentence best fit? Click on a square ■ to add the sentence to the passage.

10, Directions: An introductory sentence for brief summary of the passage is provided below. Complete the
summary by selecting the THREE answer choices that express the most important ideas in the passage. Some
sentences do not belong in the summary because they express ideas that are not presented in the passage or are
minor ideas in the passage. This question is worth 2 points

The United States economy in the early nineteenth century was largely influenced by the development of
transportation systems.

第 300 页 共
373 页
2021 托福阅读真题

A, During the Jacksonian era, states and cities were highly encouraged to follow the example of New York and apply
for funding to build more canals.
B, The economic success of the Erie Canal resulted in a canal craze that caused many cities throughout America,
like Baltimore a. Philadelphia, to build more canals.
C, Compared to canals and many other means of transportation railroads were a more efficient means of
transportation since trains traveled relatively quickly.
D, The building of the Erie Canal negatively affected some American cities by taking businesses away from those
cities and redirecting them to New York
E, Increased interest in building can throughout America slowed down during the late 1830s as a result of the
financia1 challenges associated with constructing canals
F, Because of increased economic pressures and the proven success o the canal system, it took a long time for many
American cities to adopt railroads as their main means of transportation.

Birdsong

Paragraph 1: Birdsong is the classic example of how genes (hereditary information) and environment both have a
crucial role to play in the behavioral development of animals. Since the pioneering work of W. H. Thorpe on
chaffinches (a common European bird), many species have been studied, and it has become clear both that learning
plays an important role for all species and also that there are constraints on what they are able to learn.

第 301 页 共
373 页
2021 托福阅读真题

Paragraph 2: Thorpe was able to show that learning from others was involved in chaffinch birds through a series of
experiments on hand-reared chicks (young birds). As in most other species, only the males sing. Thorpe found that, if
he raised young males in total isolation from all others, the song they produced was quite different from that of a
normal adult. It was about the right length and in the correct frequency range, it was also split up into a series of
notes as it should be. But these notes lacked the detailed structure found in wild birds, nor was the song split up into
distinct phrases as it usually is. This suggested that song development requires some social influence. Later
experiments in which researchers played recordings of songs to young birds showed just how precise this influence
was, many of them would learn the exact pattern of the recording they had heard. A remarkable feature here was in
the first few weeks of life, yet they did not sing themselves until about eight months old. They are thus able to store a
memory of the sound within their brain and then match their own output to their collection of it when they mature.

1. The word distinct in the passage is closest in meaning to


A. short
B. simple
C. separate
D. similar

2. According to paragraph 2, all of the following are characteristics of the songs of the young chaffinches in Thorpe’s
experiment EXCEPT:
A. They were not identical to the songs of normal adult chaffinches.
B. They lacked the complex form of the songs of wild chaffinches.
C. They were as long as the songs of normal adult chaffinches.
D. They were clearly different from each other.

3. According to paragraph 2, researchers discovered which of the following by playing recordings of songs to
chaffinches?
A. Chaffinches could no longer be taught to reproduce sounds after the first few weeks of life.
B. Chaffinches could not reproduce songs with exactly the same patterns of recorded songs.
C. Chaffinches at the age of eight months could recall and reproduce a song that they heard in the first few weeks of
life.
D. Chaffinches that learned a song from recordings in the first few weeks of life were later unable to copy the sounds
of mature chaffinches.

Paragraph 3: Young chaffinches normally learn only chaffinch song, though Thorpe found they could be trained to
sing the song of a tree pipit (another type of bird), which is very similar to that of their own species. ■ In general,
however, the constraints on learning which birds have ensure that they only learn songs appropriate to the species to
which they themselves belong. ■ These constraints may be in their brain’s circuitry, the young bird hatching with a

第 302 页 共
373 页
2021 托福阅读真题

rough idea of the sounds that it should copy. ■The crude song of a bird reared in isolation gives some clues as to
what this rough idea may be the length, the frequency range and the breaking up into notes are all aspects of
chaffinch song shared between normal birds and those reared in isolation. ■In other cases, the constraints are
more social, young birds only being prepared to learn from individuals with whom they have social interactions.
Thus, in a number of species, it has been found that they will not copy from recordings, but will do so from a liv e
tutor. In some cases, this may occur when they are young birds, but in others the main learning period is when they
set up their territories and interact with neighbors for the first time, enabling them to match their neighbor’s songs
and so countering with them. Whatever the nature of the learning rules in a particular species, there is no doubt that
they are effective, it is very unusual to hear a wild bird singing a song which is not typical of its own species despite
the many different songs which often occur in a small patch of woodland.

4. The word enabling in the passage is closest in meaning to


A. allowing
B. challenging
C. forcing
D. preparing

5. It can be inferred from paragraph 3 that one of the functions of songs in birds is to
A. bring together birds living in groups with birds living in isolation
B. help young birds distinguish other young birds from adults
C. make possible interactions between birds of different species
D. help birds to establish territories

6. Which of the sentences below best expresses the essential information in the highlighted sentence in the passage?
Incorrect choices change the meaning in important ways or leave out essential information.
A. Songs produced by chaffinches reared in isolation are cruder than the songs of wild birds.
B. The song of a bird reared in isolation suggests which aspects of chaffinch song may be inborn.
C. Comparing the crude songs of chaffinches reared in isolation to the songs of wild chaffinches suggests differences
as well as similarities.
D. By studying the song aspects of chaffinches reared in isolation, researchers have gained a better understanding of
the songs produced by wild birds.

7. According to paragraph 3, in some species, young birds do not copy songs from recordings because
A. they learn to sing only by live interactions with other birds
B. their ability to learn from recordings occurs later in life
C. they can only learn songs from the birds living in their area of woodland
D. they can only learn songs from other birds of their own species

第 303 页 共
373 页
2021 托福阅读真题

8. Why does the author mention that it is very unusual to hear a wild bird singing a song which is not typical of its
own species?
A. To explain why a variety of different bird songs are often heard in a relatively small area
B. To argue that social constraints have a greater impact upon learning than do genetic constraints
C. To provide an example of how the process of learning rules varies from one species to another
D. To illustrate how effective the different constraints upon learning are in young birds

Paragraph 4: However, not all birds show the same learning pattern as do chaffinches. There are some species
which produce normal sounds even if deaf, so that they cannot hear their own efforts, much less copy those of others.
The cooing of doves and the crowing of cocks are examples here. In other cases, such as parrots and hill mynahs,
birds can be trained to copy a huge variety of sounds, though those they learn in the wild are usually more restricted.
The amazing capability of mynahs has apparently arisen simply because birds in an area learn a small number of
their calls from each other, males from males and females from females, and these calls are highly varied in structure.
The ability to master them has led the birds, incidentally, to be capable of saying “hello” and mimicking a wide
variety of other sounds.

9. Look at the four squares [■] that indicate where the following sentence could be added to the passage.
Are these constraints genetic, environmental, or both?

10. Directions: An introductory sentence for a brief summary of the passage is provided below. Complete the
summary by selecting the 3 answer choices that express the most important ideas in the passage. Some sentences do
not belong in the summary because they express ideas that are represented in the passage or are minor ideas
in the passage. This question is worth 2 points

A combination of hereditary and environmental factors is believed to determine the sounds that birds are able
to produce.

A. Although only male chaffinches are able to sing in the wild, Thorpe found that he could teach hand-reared
females to copy songs from recordings and live tutors.
B. Chaffinches reared in isolation produce songs that differ significantly from those of normal birds, suggesting that
some social influence is important for learning a song precisely.
C. Birds vary a great deal with respect to both the variety of sounds they are able to learn and the conditions that
must be present for them to be able to learn their species’ song.
D. Chaffinches that are exposed only to other bird species for the first few weeks of their life are likely to learn the
songs of those species instead of the chaffinch song.
E. Researchers believe that both the circuitry or a bird’s brain and its interactions with other birds of the same

第 304 页 共
373 页
2021 托福阅读真题

species may prevent birds from learning the songs of other species.
F. The ability of deaf birds to produce their normal species’ song suggests that genes play a much greater role than
environment in determining the behavior of most bird species.

第 305 页 共
373 页
2021 托福阅读真题

The origin of life on Earth

Paragraph 1:The leading idea for the origin of life on Earth is that the right sort of chemicals, temperatures,
and energy source, which could have been lightning or heat from undersea volcanic vents, pro duced early
organic molecules that gradually organized into living organisms. However, there is one alternative some people
have proposed that life did not originate on Earth at all but instead migrated here from some other planet. This idea,
sometimes called panspermia, once seemed outlandish. After all, it's hard to imagine a more forbidding environment
than that of space, where there's no air, no water, and constant bombardment by dangerous radiation from the Sun
and stars. However, the presence of organic molecules in meteorites and comets tells us that the building blocks of
life can survive in the space environment, and some Earth microbes (such as D. radiodurans and anthrax are capable
of surviving at least moderate periods of time in space. It therefore seems possible that life could migrate from one
planet to another, if it could obtain a suitable ride. And rides are indeed available.

1. Which of the sentences below best expresses the essential information in the highlighted sentence in the passage?
Incorrect choices change the meaning in important ways or leave out essential information.
A. Most scientists think that lightning or heat from undersea volcanic vents transformed organic molecules into
living organisms.
B. Most scientists think that some early organic molecules on Earth were affected by undersea volcanic vents and
therefore only gradually organized into living organisms.
C. Most scientists think that the organic molecules that gave rise to living organisms were produced on Earth under
the right combination of conditions.
D. Most scientists think that as organic molecules organized into living organisms, they produced a number of
physical and chemical changes on Earth.

2. The word “forbidding" in the passage is closest in meaning to


A. strange
B. basic
C. harsh
D. empty

Paragraph 2:We know that meteorites can and do travel from one world to another. Among the more than 20,000
meteorites that scientists have identified and cataloged, careful chemical analysis has so far revealed about three
dozen with compositions that clearly suggest that they came from Mars; even more have been found that came from
the Moon. Apparently, these meteorites were blasted from their home worlds by large impacts and then followed
orbital trajectories that eventually caused them to land on Earth. Observations of these meteorites, along with
theoretical calculations based on the amount of material blasted into space by impacts, suggest that over time the
inner planets have exchanged many tons of rock. As the astronomer Nick Schneider puts it, Earth, Venus, and Mars

第 306 页 共
373 页
2021 托福阅读真题

have in effect been sneezing on each other for billions of years.

3. Why does the author discuss “the more than 20,000 meteorites that scientists have identified and cataloged"?
A. To support the idea that life could have arrived on Earth by means of meteorites
B. To explain what caused the meteorites to land on Earth
C. To argue that more meteorites came from Mars than from the Moon
D. To explain how scientists analyze the chemical composition of meteorites

4. According to paragraph 2, what do scientists do to study the meteorites that have been blasted onto Earth from
other planets?
A. They analyze where and how the meteorites landed.
B. They study the orbital trajectories the meteorites followed.
C. They study the composition of meteorites and do other calculations.
D. They observe the paths of meteorites in space.

5. Why does the author include Schneider's comment that "Earth, Venus, and Mars have in effect been sneezing on
each other for billions of years"?
A. To suggest that the impacts of meteorite crashes have not had significant effects on Earth, Venus, and Mars
B. To emphasize that the inner planets have exchanged tons of rock over time
C. To demonstrate how the orbits of Earth, Venus, and Mars have grown closer
together over time because of meteorite impacts
D. To imply that the amount of rock exchanged between the inner planets is not as high as previously calculated

Paragraph 3: The chance of surviving the trip between planets probably depends on how long the meteorite spends
in space. Once a rock is launched into space, it orbits the Sun until its orbit carries it directly into the path of another
planet. Most meteorites will orbit for many millions of years before reaching Earth even if they come from a world
as nearby as Venus or Mars, and it seems highly unlikely that living organisms could survive in space for such long
periods of time. However, a few meteorites are likely to be launched into orbits that cause them to crash to Earth
during one of their first few trips around the Sun. For example, calculations suggest that about 1 in 10,000meteorites
may travel from Mars to Earth in a decade or less-a short enough time that some known microbial species could
probably survive the journey. Note however, that the same considerations almost certainly rule out the possibility of
migration from other star systems. Under the best of circumstances, meteorites from planets around other stars
would spend millions of years in space before reaching Earth, and any living organisms would almost surely be
killed by exposure to cosmic rays during this time.

6. It can be inferred from paragraph 3 that a meteorite's ability to bring life to Earth from Venus or Mars would be
determined by

第 307 页 共
373 页
2021 托福阅读真题

A. how violently the meteorite crashes onto Earth


B. how quickly the meteorite's orbit brings it to a collision with Earth
C. the diversity of microbial species that the meteorite carries
D. the type of rock that makes up the meteorite

7. According to paragraph 3, why are living organisms unlikely to arrive on Earth from other star systems?
A. Meteorites from other star systems cannot reach Earth.
B. Planets around other stars are less likely to sustain microbial life than Venus and Mars are.
C. Planets around other stars are less likely than Venus and Mars to experience impacts that send meteorites into
space.
D. The journey from another star system would expose living organisms to cosmic rays for too long.

Paragraph 4: We therefore encounter the intriguing possibility that if Venus or Mars for some reason got life before
Earth, this life might have seeded our planet, avoiding the need for an indigenous origin of life on Earth. We do
indeed have good reason to think that the young Venus and young Mars may both have had condition s similar to
those on the young Earth, making this scenario plausible if not likely. Still, it would hardly seem to 不 change our
basic scenario for the origin of life, as it simply moves it to another world. But while migration to Earth might have
little impact on our ideas about the origin of life, migration from Earth raises some interesting issues. The numerous
impacts that have occurred over the past 4 billion years have surely offered many opportunities for microbes to ride
on meteorites blasted off Earth's surface, which means that if it were possible for Earth life to survive on any nearby
worlds, we should actually expect to find it there. If we someday find life on Mars, for example, we may be
hard-pressed to determine whether it originated there or migrated from Earth.

8. Paragraph 4 supports which of the following statements about the presence of life on Venus and Mars?
A. The life that was present once was likely destroyed by the numerous impacts over the past 4 billion years.
B. Life is more likely to be present now than it was when Venus and Mars were young.
C. If life is present, it may have originated on Earth.
D. If life is present, it proves that life is continually moving from one planet to another.

Paragraph 3: The chance of surviving the trip between planets probably depends on how long the meteorite spends
in space. Once a rock is launched into space, it orbits the Sun until its orbit carries it directly into the path of another
planet. ■ Most meteorites will orbit for many millions of years before reaching Earth even if they come from a world
as nearby as Venus or Mars, and it seems highly unlikely that living organisms could survive in space for such long
periods of time. ■ However, a few meteorites are likely to be launched into orbits that cause them to crash to Earth
during one of their first few trips around the Sun. For example, calculations suggest that about 1 in 10,000 meteorites
may travel from Mars to Earth in a decade or less-a short enough time that some known microbial species could
probably survive the journey. ■ Note however, that the same considerations almost certainly rule out the possibility

第 308 页 共
373 页
2021 托福阅读真题

of migration from other star systems. ■ Under the best of circumstances, meteorites from planets around other stars
would spend millions of years in space before reaching Earth, and any living organisms would almost surely be
killed by exposure to cosmic rays during this time.

9. Look at the four squares [■] that indicate where the following sentence could be added to the passage.
The length of time this journey takes varies widely.
Where would the sentence best fit? Click on a square[■] to add the sentence to the passage.

10. Directions: An introductory sentence for a brief summary of the passage is provided below. Complete the
summary by selecting the 3 answer choices that express the most important ideas in the passage. Some sentences do
not belong in the summary because they express ideas that are not presented in the passage or are minor ideas in the
passage. This question is worth 2 points.

A. Chemical analysis of more than 20,000 meteorites that have landed on Earth has revealed the presence of some
microbes from Mars and the Moon.
B. Tons of rock have moved between planets in meteorite impacts, but meteorites that spend lengthy periods of time
in space are less likely to successfully transport life between planets.
C. The fact that Earth's microbes can survive on nearby worlds indicates that life most likely originated on Earth
before migrating to nearby planets.
D. The idea that life came to Earth from another planet, once considered implausible, is being reconsidered because
the building blocks of life could have survived a journey through space.
E. The fact that Earth has been bombarded by more meteorites than any neighboring planet makes migration to Earth
more likely than to other planets.
F. The possibility of life-forms traveling among Earth, Venus, and Mars makes it difficult to determine where life
originally evolved.

第 309 页 共
373 页
2021 托福阅读真题

American Cultural Independence

Paragraph 1:At the beginning of the nineteenth century, the United States was a provincial culture, still
looking to Britain for values, standards, literature, and art, despite all the rapid improvements in
communication-for example, the great growth in the number of magazines and newspapers-that occurred in
the United States following independence from Great Britain. In a famous essay in the Edinburgh Review
in1820,Sydney Smith sarcastically inquired, “In the four quarters of the globe, who reads an American book? Or
goes to an American play? Or looks at an American picture or statue?"

1. Which of the sentences below best expresses the essential information in the highlighted sentence in the passage?
Incorrect choices change the meaning in important ways or leave out essential information.
A. At the beginning of the nineteenth century, the United States remained highly influenced by British culture, even
though improvements had been made in communication in the United States
B. After gaining its independence from Great Britain, the United States experienced rapid improvements in
communication
C. In the early nineteenth century, most magazines and newspapers in the United States dealt with British values
standards literature, and art
D. Communication improved rapidly in the nineteenth century, as demonstrated by the great growth in the number of
magazines and newspapers.

Paragraph 2:Yet interest in the arts and the cultural institutions that sustain them was rapidly growing in America.
To be sure, cultural institutions were difficult to create in the South,mainly because the population was so widely
dispersed: and in the West. dominated in this era by pioneers, emphasis was on the practical rather than on the
literary or artistic. But many eastern seaboard cities were actively building the cultural foundation that would nurture
a distinctively American art and literature. During these years, Philadelphia’s American Philosophical Society,
founded by Benjamin Franklin in 1743,boasted a distinguished roster of scientists, including Thomas Jefferson,
concurrently its president and president of the United States. Culturally Boston ran a close second to Philadelphia,
founding the Boston Athenaeum (1807). an impressive library and reading containing "works of learning and science
in all languages, particularly such rare and expensive publications as are not generally to be found in this country."
The North American Review. which became the most important and long-lasting intellectual magazine in the country,
was published in Boston. Devoted to keeping its readers in touch with European intellectual developments. it had a
circulation of 3.000 in 1862,about the same as similar British journals.

2. The word "distinguished" in the passage is closest in meaning to


A. varied
B. very large
C. highly respected

第 310 页 共
373 页
2021 托福阅读真题

D. newly formed

3. In paragraph 2,why does the author discuss the Boston Athenaeum?


A. To suggest that Boston in general was culturally superior to Philadelphia
B. To provide an example of a cultural organization founded by a president of the United States
C. To support the claim that cities in the East were building American cultural institutions
D. To argue that there were organizations founded to challenge the influence of Philadelphia's American
Philosophical Society

4. According to paragraph 2, which of the following is true of the North American Review?
A. It had about as many readers as similar British magazines had.
B. It was published by the Boston Athenaeum.
C. It was written by European intellectuals.
D. It focused on intellectual developments in the United States.

Paragraph 3:Of the eastern cities,New York produced the first widely recognized American writers. In 1809
Washington Irving published America’s first great book of comic literature-A History of New York-a humorous
account of life in New York both in colonial times and in Irving's own day. James Fenimore Cooper's Leather
stocking novels (of which The Last of the Mohicans, published in1826, is the best known)achieved wide success in
both America and Europe. In these novels, Cooper established the American experience of westward expansion as a
serious and distinctive American theme.

5. According to paragraph 3, which of the following is true of the Leather stocking novels?
A. They included America's first great book of comic literature.
B. They were widely read within the United States but were unknown outside that
C. They were more popular than comic literature
D. They began a tradition of literature dealing with America's westward expansion

Paragraph 4:It was New England, however, that saw itself as the home of American cultural independence from
Europe, a claim voiced in 1837 by Ralph Waldo Emerson in a lecture at Harvard: "Our day of dependence, our long
apprenticeship to the learning of other lands draws to a close, " Emerson announced. Rather than draw on British
models, American artists, Emerson argued, should seek inspiration in the ordinary details of daily American
Immensely popular,Emerson gave more than 1,500 lectures in 20 states between 1833 and 1860." The American
Scholar," his most famous lecture, carried a message of cultural self-sufficiency that Americans were eager to hear.

6. All of the following are mentioned in paragraph 4,All of the following are mentioned in paragraph 4 as being true
of Emerson’s view of art EXCEPT:

第 311 页 共
373 页
2021 托福阅读真题

A. It was based on ideas that Emerson first heard in a famous lecture when he was at Harvard
B. It focused on making day-to-day aspects of American life serve as sources of artistic inspiration.
C. It was popular with American readers and audiences.
D. It stressed that the art of American artists could contribute to cultural independence.

Paragraph 5: Artists in this period were as successful as writers in finding American themes. Thomas Cole, who
came to the United States from England in 1818,found great inspiration in the American landscape. the American
landscape. Cole founded the Hudson River school of American painting, whose romantic renderings of New York
landscapes were frankly nationalistic. Other painters-realists such as Karl Bodmer and George Catlin-drew on the
dramatic landscapes and peoples of the American West. Their art was an important contribution to the American
sense of the land and to the nation's identity. Catlin, driven by a need to document Native American life, spent eight
years among the tribes along the upper Missouri River. Then he assembled his collection of more than500 paintings
and toured the country in an effort to increase the public's awareness and appreciation of Native American lifeways.
George Caleb Bingham, an accomplished genre painter, produced somewhat tidied-up scenes of real-life American
workers, such as flatboat men on the Missouri River. All these painters found much to record and celebrate in
American life. Ironically, the inspiration for the most prevalent theme, the American wilderness, was profoundly
endangered by the rapid western settlement of which the nation was so proud. Despite such contradictions, the early
nineteenth century was the period in which writers and painters found the national themes that defined the first
distinctively American literature and art.

7. Paragraph 5 suggests which of the following about Thomas Cole's paintings?


A. They featured realistic portrayals of the peoples of the American West.
B. They portrayed the New York landscape as more impressive than it was.
C. They influenced realists such as Karl Bodmer and George Catlin
D. They often made comparisons between the United States and England.

8. According to paragraph 5,George Catlin did all of the following EXCEPT


A. establish the Hudson River School of American painting
B. live with and paint Native Americans for eight years
C. take his paintings around the country to teach people about Native Americans
D. create realistic portrayals of life along the upper Missouri River

Paragraph 5: Artists in this period were as successful as writers in finding American themes. █ Thomas Cole, who
came to the United States from England in 1818, found great inspiration in the American landscape. the American
landscape. █Cole founded the Hudson River school of American painting, whose romantic renderings of New York
landscapes were frankly nationalistic. █ Other painters-realists such as Karl Bodmer and George Catlin-drew on the
dramatic landscapes and peoples of the American West. █Their art was an important contribution to the American

第 312 页 共
373 页
2021 托福阅读真题

sense of the land and to the nation's identity. Catlin, driven by a need to document Native American life, spent eight
years among the tribes along the upper Missouri River. Then he assembled his collection of more than500 paintings
and toured the country in an effort to increase the public's awareness and appreciation of Native American life ways.
George Caleb Bingham, an accomplished genre painter, produced somewhat tidied-up scenes of real-life American
workers, such as flatboat men on the Missouri River. All these painters found much to record and celebrate in
American life. Ironically, the inspiration for the most prevalent theme, the American wilderness, was profoundly
endangered by the rapid western settlement of which the nation was so proud. Despite such contradictions, the early
nineteenth century was the period in which writers and painters found the national themes that defined the first
distinctively American literature and art.

9. Look at the four squares █ that indicate where the following sentence could be added to the passage.
New York was not the only source of artistic inspiration around this time.
Where would the sentence best fit? Click on a square █ to add the sentence to the passage.

10 Directions: An introductory sentence tor a brief summary of the passage is provided below. Complete the
summary by selecting the 3 answer choices that express the most important ideas in the passage. Some sentences do
not belong in the summary because they express ideas that are not presented in the passage or are minor ideas in the
passage. This question is worth 2 points. Drag your choices to the spaces where they belong. To review the passage,
click on View Text.

A. rapid improvements in communication in the United State: meant that readers in other countries, such as those of
the Edinburgh Review, could learn about American cultural life.
B. Authors Washington Irving and James Fenimore Cooper were widely recognized, and Ralph Waldo Emerson gave
popular lectures advocating for American cultural self-sufficiency.
C. The Hudson River School specialized in romanticized landscapes, while some other American painters produced
realistic portraits of American peoples and places, particularly of the West
D. The American Philosophical Society included many distinguished scientists, and Boston boasted an impressive
new library and a respectable and widely distributed intellectual journal.
E. Cultural institutions in the South and West could not compete with those on the eastern seaboard, which had
advantages in terms of greater wealth and connections to Europe.
F. Fearing the loss of the American wilderness, painters such as Cole, Catlin and Bingham criticized the rapid
western settlement of which the nation was so proud.

第 313 页 共
373 页
2021 托福阅读真题

Shift in Power Relations

Paragraph1:Beginning in the fifteenth century, monarchs (kings and queens) across Western Europe looked for
ways to increase their own power while diminishing the power of noblemen, or aristocrats. One development that
made such a shift in power relations possible was the introduction of gunpowder. In earlier centuries in Europe,
battlefields were dominated by noblemen who fought while mounted on horses. When firearms(weapons using
gunpowder) came into use, however, this way of fighting declined, and peasants (poor people who worked on the
land) came to play a significant role in armies. Because peasant foot soldiers were more numerous and could be
trained relatively quickly to use gun powder, armies rapidly grew larger. This meant that kings who wanted to remain
powerful had to gain control over the raising of troops, function formerly performed by noblemen. At the same time,
kings gained a new powerful ally in the power struggle with the aristocracy: a rising middle class of
townspeople and merchants, who opposed aristocrats by favoring a more meritocratic social structure in
which ability and hard work were better appreciated and rewarded. Merchants and townspeople also objected
to local nobles' intrusion both in their towns, which were situated near the nobles' lands. and in their long-distance
trade activities, which the nobles taxed.

1. Which of the sentences below best expresses the essential information in the highlighted sentence in the passage?
Incorrect choices change the meaning in important ways or leave out essential information.
A. At the same time that kings were fighting the aristocracy, middle-class townspeople and merchants were fighting
for a more meritocratic social structure.
B. Kings were supported in their struggle with the aristocracy by townspeople and merchants, who wanted more
appreciation and reward for hard work and ability.
C. At the same time that kings were fighting the aristocracy, middle-class townspeople and merchants were rising to
power through ability and hard work.
D. Kings allied themselves with townspeople and merchants rather than with the aristocrats because the middle class
was becoming better appreciated and rewarded.

2. According to paragraph 1, townspeople and merchants disliked aristocrats in part because aristocrats
A. competed with ordinary people for positions in the army
B. influenced activities in towns near the aristocrats' land
C. participated in the raising of troops
D. fought in battles while mounted

Paragraph 2: The French king Louis XIV centralized power more than any previous French monarch or any other
European monarch. Louis XIV's father,Louis XIII,and his grandfather Henry V, used direct methods to diminish the
power of aristocrats. At the turn of the seventeenth century, Henry IV had divided France into thirty-two districts and
appointed governors, known as intendants, to oversee the collection of taxes and the dispensation of justice.

第 314 页 共
373 页
2021 托福阅读真题

Functions formerly performed by high-ranking nobles. This consolidation of power enabled Henry's son, Louis XIIl,
guided by his chief advisor Cardinal Richelieu, to order the destruction of all castles in France that were owned by
aristocrats, thus removing the possibility of individual aristocrats resisting the kings’ military p ower by sheltering
behind their own military fortifications. Cardinal Richelieu and his successor Cardinal Mazarin further reformed the
French military and judiciary by controlling the funds allotted those institutions, and promoting talented men from
modest backgrounds to positions of authority. The intendants were always selected from the educated middle
classes, not the aristocracy.

3. Why does the author include the information that "talented men from modest backgrounds" were promoted to
"positions of authority"?
A. To explain why Cardinal Richelieu and Cardinal Mazarin thought that various reforms were needed
B. To point out a specific way that the intendants differed from other men Cardinal Richelieu and Cardinal Mazarin
promoted to positions of authority
C. To explain how the king was able to gain control of the funds for the military and the judiciary
D. To point out a specific way in which power was taken away from the aristocrats

4. According to paragraph 2,what was one power that was taken away from French aristocrats at the turn of the
seventeenth century?
A. The power to own land
B. The power to appoint district governors
C. The power to collect taxes
D. The power to write laws

Paragraph 3: By 1661 in the reign of Louis XIV, nearly every major political military, and religious office in France
was filled by direct appointment of the king. In 1682, the king strengthened royal control by moving the court and its
governmental councils to the newly built Palace of Versailles, which was then twenty kilometers from Paris.
Versailles, which was much larger than the previous royal residence at the Louvre Palace in Paris, served as a potent
visual expression of the king's increased power and prominence. It also had other benefits. The move allowed the
king to isolate his court from the influence of the population of Paris and of Parisian administrative and cultural
institutions.

5. The word "potent" in the passage is closest in meaning to


A. useful
B. familiar
C. powerful
D. respected

第 315 页 共
373 页
2021 托福阅读真题

6. According to paragraph 3, all of the following were true of the palace of Versailles EXCEPT:
A. It was bigger than the Louvre Palace in Paris.
B. It was highly influenced by Parisian administrative and cultural institutions
C. It was located in the countryside outside of Paris.
D. The move to Versailles strengthened Louis XIV's control over his court.

Paragraph 4: Louis required the highest ranking aristocrats of France to spend a large portion of every year in
residence in Versailles. How Louis managed the famous levee ceremony each morning, in which he was awakened,
bathed ,and dressed, illustrates his cleverness in strengthening his control over the aristocracy. The ceremony
involved high-ranking individuals attending the king as he prepared for the day: who was admitted and in what order
indicated one's status at court. Thus, making oneself agreeable and entertaining so as to first win access to the king,
and then his personal favor (and perhaps thereby gain a profitable appointment) became a major objective of the
Versaille residents. Because Louis the XIV loved to dance, aristocrats hired expensive dancing masters and spent
hours each day learning the latest steps. Louis was fond of clothes and favored courtiers dressed in the latest fashions.
When he greeted the Persian ambassador in 1715, he wore a gem-encrusted suit worth 12.5 million lives, an
incredible $255,000,000 in modern terms. Ultimately, the unceasing demands of court life exhausted the energies of
the French aristocracy and drained their funds, leaving little time for plotting against the king. The extended
absences from their hereditary estates meant that aristocrats were unable to successfully resist the rise of the
intendants in the countryside. The French aristocracy never recovered its former capabilities.

7. The word "objective" in the passage is closest in meaning to


A. goal
B. practice
C. challenge
D. responsibility

8. Paragraph 4 suggests that aristocrats spent most of their time at court engaging in which of the following
activities?
A. Trying to increase the power of the intendants
B. Spending the king's money
C. Plotting against the king
D. Trying to please the king

Paragraph 3: By 1661 in the reign of Louis XIV, nearly every major political military, and religious office in France
was filled by direct appointment of the king. █ In 1682, the king strengthened royal control by moving the court and
its governmental councils to the newly built Palace of Versailles, which was then twenty kilometers from Paris. █
Versailles, which was much larger than the previous royal residence at the Louvre Palace in Paris, served as a potent

第 316 页 共
373 页
2021 托福阅读真题

visual expression of the king's increased power and prominence. █ It also had other benefits. █ The move allowed
the king to isolate his court from the influence of the population of Paris and of Parisian administrative and cultural
institutions.

9. Look at the four squares that indicate where the following sentence could be added to the passage.
However, the king, dissatisfied with his level of control, continued to develop new ways of consolidating his
power.
Where would the sentence best fit? Click on a square to add the sentence to the passage.

10. Directions: An introductory sentence for a brief summary of the passage is provided below. Complete the
summary by selecting the THREE answer choices that express the most important ideas in the passage. Some
sentences do not belong in the summary because they express ideas that are not presented in the passage or are minor
ideas in the passage. This question is worth 2 points.

A. When firearms came into use in the fifteenth century, European monarchs were quick to limit the ability of the
aristocracy to access and use them.
B. The French cardinals Richelieu and Mazarin rewarded some members of the middle class by promoting them to
positions of power in government while reducing the power of the aristocrats.
C. Louis XIV often displayed his increased power by providing the aristocrats and intendants who visited his court
with clothes, jewels and dancing lessons.
D. Although the policies of Henry IV and Louis XIII reduce the influence of aristocrats, kings were unable to
appoint talented men to political and military offices until the reign of Louis XIN
E. Functions once performed by aristocrats, such as dispensing justice, were taken over by governors appointed by
Henry IV, whose son Louis XIII later destroyed aristocrats fortifications.
F. To further strengthen his political control,Louis XIV moved his court from Paris to Versailles and required
aristocrats to spend much time in court attempting to gain his favor.

第 317 页 共
373 页
2021 托福阅读真题

How Humans First Arrived in North America

Paragraph 1: One of the most prevalent theories of how humans first arrived in North America is that they crossed
the Bering Strait from Asia into what is now Alaska in northwestern North America about 12.000years ago. This was
near the end of the last Ice Age, when so much of Earth's water was locked up in glaciers that the Bering land bridge,
also known as Beringia, was left exposed. Beringia had, to be sure, an inhospitable climate-worse than that of Alaska
and Siberia today. But the climate had warmed up enough for humans to survive in the area. Fossils found there
indicate that woolly mammoths, steppe bison, wild horses, and caribou all lived in the region and their human
hunters might very well have followed them onto and across the land bridge. So the 12.000-year estimate made
sense; any earlier, and it would have been too cold for people to survive so far north; any later, and the water from
the melting glaciers would have submerged the land bridge and blocked the passage across.

1. According to paragraph 1, why would it have been difficult for humans to travel from Asia to Alaska more
recently than 12,000 years ago?
A. The food resources would have been too limited to support humans.
B. The Bering land bridge would have been covered by water.
C. It would have been too cold for humans to make the journey
D. The passage from Asia to Alaska would have been blocked by glaciers.

Paragraph 2: This geological logic was supported by archaeological evidence. In 1908 a cowboy named George
McJunkin was riding near the small United States town of Folson, New Mexico, searching for a lost cow. Instead, he
came across some bones with a stone spear point beside them. The bones were much too large to belong to a cow;
intrigued, McJunkin took them back to the ranch house. There they stayed until 1925.when they landed on the desk
of Jesse Figgins of the Colorado Museum of Natural History. Figgins easily identified the bones as those of a
long-extinct form of bison that had roamed the plains at the end of the Ice Age. But it was the stone spear points
McJunkin had found beside the bones that had the more far-reaching implications. If these spear points were
human-made weapons used to kill the bison, that meant humans had been hunting(and living) in America during the
Ice Age.

2. In stating that George McJunkin was "intrigued" when he came across some bones with a stone spear point
beside them, the author means that McJunkin was
A. concerned about the bones
B. hopeful about the bones
C. protective of the bones
D. curious about the bones

3. It can be inferred from paragraph 2 that, when he made his discoveries, George McJunkin

第 318 页 共
373 页
2021 托福阅读真题

A. concluded that large cows were common in the area


B. understood the importance of the artifacts and the need for them to be displayed in a museum
C. did not realize the importance of the spear points as evidence of humans hunting ancient animals
D. believed that Jesse Figgins would be able to identify the bones as belonging to the Ice Age

Paragraph 3: More indications of Ice Age hunters followed soon after. In 1932 two amateur collectors exploring
near Clovis, New Mexico, found the bones of some animals with stone spearheads between the ribs. In this case, the
animals turned out to be woolly mammoths, also extinct since the Ice Age. And with the spear points right between
the ribs, there could be little doubt that people were in America during the Ice Age.

4. What is the purpose of paragraph 3 in the passage?


A. To discuss the importance of a discovery mentioned in the previous paragraph
B. To introduce a competing theory of when humans first arrived in North America
C. To contrast an archaeological finding made in 1932 with a finding discussed in the previous paragraph
D. To provide further archaeological support for the theory already introduced in the passage

Paragraph 4: In the 1950s a University of Chicago scientist named Willard Libby developed radiocarbon
dating, which allowed scientists to determine the age of the Folsom and Clovis finds more precisely; they
turned out to date back between10,000 and 11,500 years ago-thus fitting in very neatly with the puzzle being
pieced together. So, too, did evidence that came from the prehistoric North Americans' teeth. After examining more
than 200,000 prehistoric teeth, anthropologist Christy Turner determined that all Native American teeth shared
certain genetic traits with northeast Asian teeth. These traits were unique-no other people had them besides
prehistoric North Americans and northeast Asians. From this, Turner concluded that North Americans' ancestors
must have come from northeast Asia. What's more, by examining the differences between northeast Asian and North
American teeth and by comparing them to general dental evolution patterns, Turner calculated that the two
populations must have separated in the vicinity of 12,000 years ago.

5. Which of the sentences below best expresses the essential information in the highlighted sentence in the passage?
Incorrect choices change the meaning in important ways or leave out essential information.
A. In the 1950s, the technique of radiocarbon dating developed at the University of Chicago needed to be more
precise in order to determine whether the Folsom and Clovis finds were 10,000 or11,500 years old.
B. The dating of the Folsom and Clovis finds, made possible by radiocarbon dating in the 1950s, was consistent with
the dominant theory of when humans first arrived in North America.
C. Willard Libby of the University of Chicago first used radiocarbon dating to solve the puzzle of why there were
different dates for the Folsom and Clovis finds.
D. The precision of radiocarbon dating helped scientists to determine that archaeological finds such as those
discovered at Clovis and Folsom were more puzzling than once believed.

第 319 页 共
373 页
2021 托福阅读真题

6. The phrase “in the vicinity of" in the passage is closest in meaning to
A. about
B. before
C. only
D. beginning

7. According to paragraph 4, why did the anthropologist Christy Turner conclude that prehistoric North Americans
came from northeast Asia?
A. The teeth of North Americans and northeast Asians that he discovered were dated using radiocarbon dating
methods and were found to be similar in age.
B. The teeth of prehistoric North Americans and northeast Asians have sometimes been discovered together in the
same areas.
C. Neither northeast Asian nor North American teeth had genetic traits that appeared in the general population
until12,000 years ago.
D. The teeth of Native Americans and northeast Asians shared genetic traits that were found in no other prehistoric
populations.

Paragraph 5: If people first entered the American continent about 12,000 years ago, that explained another mystery
as well: the rapid extinction of mammoths, mastodons, saber-toothed cats, and other large animals that roamed North
America until about 10.000 years ago. When the bones of these animals were found at sites dated 12.000 years or
older, there were no human artifacts nearby. When human artifacts less than 10,000 years old were found at similar
sites, none of these animals' bones were around. Only in finds dating back between12.000 and 10,000 years ago,
such as those at Clovis and Folsom, did human artifacts and these bones appear together. From this, paleo-ecologist
Paul Martin formulated his hypothesis of "Pleistocene overkill"in1967. Martin was convinced that human hunters
crossed the Bering land bridge about 12,000 years ago and then moved south. Here they found mammals so
completely unadapted to human predators that humans quickly hunted them to extinction.

8. According to paragraph 5, Paul Martin's hypothesis of "Pleistocene overkill" provided an answer to which of the
following questions?
A. Why did prehistoric North Americans hunt certain large animals but not others?
B. Why were there so many large animals in North America by the time humans arrived
C. Why did many large North American animals become extinct between 12,000 and 10.000 years ago?
D. Why did it take so long for large North American mammals to become extinct?

Paragraph 1: One of the most prevalent theories of how humans first arrived in North America is that they crossed
the Bering Strait from Asia into what is now Alaska in northwestern North America about 12.000years ago. This was

第 320 页 共
373 页
2021 托福阅读真题

near the end of the last Ice Age, when so much of Earth's water was locked up in glaciers that the Bering land bridge,
also known as Beringia, was left exposed. Beringia had, to be sure, an inhospitable climate-worse than that of Alaska
and Siberia today. █ But the climate had warmed up enough for humans to survive in the area. █ Fossils found there
indicate that woolly mammoths, steppe bison, wild horses, and caribou all lived in the region and their human
hunters might very well have followed them onto and across the land bridge. █So th e 12.000-year estimate made
sense; any earlier, and it would have been too cold for people to survive so far north; any later, and the water from
the melting glaciers would have submerged the land bridge and blocked the passage across. █

9. Look at the four squares that indicate where the following sentence could be added to the passage.
In fact, there is evidence showing that sources of meat were even available.
Where would the sentence best fit? Click on a square to add the sentence to the passage.

10. Directions: An introductory sentence for a brief summary of the passage is provided below. Complete the
summary by selecting the THREE answer choices that express the most important ideas in the passage. Some
sentences do not belong in the summary because they express ideas that are not presented in the passage or are minor
ideas in the passage. This question is worth 2 points.

A. It is unlikely that humans crossed Beringia more recently than 12,000 years ago because by that time most large
mammals were extinct.
B. Archaeological evidence from Folsom and Clovis has taught scientists a great deal about the lifestyle and hunting
practices of the first North Americans.
C. Radiocarbon dating and genetic evidence provided more precise information about when prehistoric North
Americans arrived and where they came from.
D. Spear points discovered in and around the bones of extinct mammals suggest that humans were present in North
America during the last Ice Age.
E. The skeletons of Ice Age bison and woolly mammoths suggest very different dates for the arrival of the first
human than does the evidence from prehistoric teeth.
F. Archaeological evidence suggests that the first North Americans may have hunted many large animals to
extinction soon after the humans had arrived.

第 321 页 共
373 页
2021 托福阅读真题

Insect Colony Movement

Paragraph 1: In general, colonies of social insects such as bees or ants move either to change nesting sites when the
current site deteriorates in quality, or when the mother colony splits to start a new colony. Deterioration of nest
quality can occur in a variety of ways. Perhaps the most common is for the nest to come under attack by a predator,
forcing the colony to abandon the nest on short notice and reassemble elsewhere. Emergency moves may also be
elicited by physical disasters such as flood or fire. The technique of pacifying honeybees with smoke, which has long
been practiced by beekeepers as well as honey hunters, is effective because the smoke induces the workers to begin
preparations for moving to escape fire, and to become less likely to sting those disturbing the nest.

1. According to paragraph 1, deterioration of nest quality probably most often occurs when
A. there is a physical disaster such as a flood or fire
B. part of an insect colony splits off to form a new colony
C. the nest is disturbed by one of the insects' predators
D. beekeepers and honey hunters fill a nest with smoke

2. Which of the following can be inferred from paragraph 1 about the use of smoke by beekeepers and honey
hunters?
A It causes bees to respond as they would when a physical disaster is possible.
B It is most often used in emergencies encourage bees to move.
C It is risky because bees tend to sting when disturbed.
D It is an effective means of maintaining the health of a bee colony.

Paragraph 2: Colony movement may also be precipitated by more gradual deterioration of nest quality. For
example,an increase in populations of parasites or microorganisms affecting the brood (young insects that hatched
together) may induce the colony to establish a clean nest elsewhere. Also, changes in the nest that increase the
colony's vulnerability to predators may lead the colony to search for a more secure nest. A response to elevated
predation risk has been documented in one of the Asian honeybee species, Apis florea. Colonies of this species
construct their nests in the open, suspending a wax comb the size of a dinner plate from a thin branch in dense
vegetation and then protecting the comb with a curtain of interlinked worker bee. Because the workers and the
colonies are very small, and can mount only a relative' weak stinging defense when attacked, the cover of vegetation
plays a crucial role in their defense against predation. Loss of leaf cover as seasons change is sufficient to trigger a
move. In some species, colonies may also change their nesting to trigger a move. In some species, colonies may also
change their nesting sites in response to a deterioration of food resources or climatic conditions in the habitat.
Relocation can be beneficial if it brings the colony nearer to rich food resources or allows it to occupy a nest with a
more favorable microclimate.

第 322 页 共
373 页
2021 托福阅读真题

3. The word "dense” in the passage is closest in meaning to


A. thick
B. tall
C. suitable
D. specific

4. According to paragraph 2, Apis florea protect their nests from predators by which TWO of the following ways?
To receive credit, you must select TWO answers.
A By hiding their nest in vegetation
B By forming a protective wall of interconnected bees
C By building a protective curtain made of thin branches
D By building their nest on the ground and covering it with leaves

Paragraph 3: Any benefits that arise must, of course, balanced against potential costs. Costs of colony movement
might include, for example, the energetic and time costs of traveling and transporting brood and nest materials;
reduction of the workforce as individuals are disoriented or lost to predators; loss of any food stores that have
predators; loss of any food stores that have been left behind; the energetic costs of constructing the new nest; and
interruption of colony growth during the time required to move, establish a new nest, and find food sources in the
new location. Unsurprisingly, many social insects have evolved strategies that reduce these costs, thus enhancing the
net benefit of the move.

5. In paragraph 3, all of the following are mentioned as potential costs of moving an insect colony EXCEPT
A. the resources required to move materials and build a new nest
B the possibility that some insects will be lost or killed during the move
C the inability to move all stored food from the old location to the new
D the time spent identifying a suitable location for the new nest

Paragraph 4: Even with minimal time for preparation, colonies are often able to take steps that increase the
probability of re-forming the colony and returning to the tasks of acquiring resources, growing in size, and
reproducing. ■ Colony-specific odors and other means allow individuals to find nest-mates quickly. ■ In ant colonies
that are forced into an emergency move (easily observed if one simulates a drastic predatory attack by exposing a
nest inside a rotten log or under a flagstone), workers can often be seen carrying brood as they escape. ■ In species,
such as honeybees, that hoard food in the nest, workers try to load themselves to capacity with food before they
escape the nest. ■ In the Asian honeybee Apis florea, foragers from colonies forced to abandon a nest return to it in
the first few days after the move to retrieve honey that may have been left behind and to collect wax for constructing
the new comb.

第 323 页 共
373 页
2021 托福阅读真题

6. Why does the author provide the information that an emergency move by ants is "easily observed if one
simulates a drastic predatory attack by exposing a nest inside a rotten log or under a flagstone"?
A To illustrate a type of attack that is most dangerous to insects
B To argue how easy it is for predators to locate and attack insect t colonies
C To suggest a simple experiment to observe a colony's behavior during an emergency move
D To demonstrate that individuals have difficulty making emergency moves during sudden predatory attacks

7. The phrase "to capacity" in the passage is closest in meaning to


A. immediately
B. equally
C. repeatedly
D. fully

8. According to paragraph 4, why might some honeybees return to a nest after abandoning it?
A To collect food and materials for building the new nest
B To search for worker bees that may have remained at the nest
C To determine if it is possible to repair the nest
D To collect brood that may have been left behind

9. Look at the four squares ■ that indicate where the following sentence could be added to the passage.
In addition to their young, insects can also carry other items from their old nest to the new.
Where would the sentence best ft? Click on a square to add the sentence to the passage.

10.Directions: An introductory sentence for a brief summary of the passage is provided below. Complete the
summary by selecting the THREE answer choices that express the most important ideas in the passage. Some
sentences do not belong in the summary because the express ideas that are not presented in the passage or are minor
ideas in the passage. This question is worth 2 points.

A. When a natural disaster or predation disturbs a nest, its insects may split up into different groups and form several
colonies in new locations
B. Insects that are able to hide their nests in vegetation or other objects rarely experience predation and do not often
move their nests.
C. Insects' strategies of limiting the loss of resources when moving may be especially effective when the insects have
more time to prepare for the move
D. Colony movement sometimes occurs in emergencies because of sudden events, but more gradual changes in
condition may also lead insects to relocate.
E. Moving the nest to a new location may lead to better safety, access to food, and climatic conditions for the insects

第 324 页 共
373 页
2021 托福阅读真题

but also involves a loss of energy, time, and other resources.


F. The size of the new nest and the distance to it determine the preparation time that insects require to move their
nest and reform their colony successfully.

第 325 页 共
373 页
2021 托福阅读真题

Competition for Life

Paragraph 1: However successful their adaptations to their physical environment, organisms must also adapt to the
demands of the biological world around them, either to avoid being eaten or to compete for space or food supply
with other organisms. There can be no final solution to any of these problems for, as quickly as new adaptations
appear that reduce predation or allow more successful competition with other species, the predator or competitor will
in its turn adapt. The herbivorous (plant-eating) group that becomes able o run faster and escape from its predators,
itself provides the stimulus that leads to the evolution of fast predators. The plant that evolves spines or
unpleasant-tasting biochemicals in order to avoid being eaten by herbivores similarly stimulates the appearance of
herbivores insensitive to these defenses. For example, milkweeds are rich in poisonous glycosides and are avoided
by most caterpillars. The caterpillars of the monarch butterfly, however, not only feed on these plants, but also in
turn use the biochemicals to make the adult butterfly unpalatable to birds.

1, According to paragraph 1, what happens when plant-eaters evolve to be able to run away from their predators?
A, Their nutritional needs increase, and so competition over food increases.
B, Their predators evolve the ability to run faster.
C, They gain a competitive advantage over slower plant-eaters.
D, They become more vulnerable to other such as poisons found in types of danger, plants.

2, In paragraph 1, why mention that caterpillars does the author of the monarch butterfly feed on milkweeds that are
rich poisonous glycosides?
A, To provide an example of two very different organisms that have evolved a mutually beneficial relationship
B, To argue that unpleasant-tasting biochemicals are better at deterring birds than at deterring caterpillars
C, To support the point that as organisms evolve adaptations to reduce predation or improve competition, predators
and competitors evolve that are not affected by the adaptations
D, To show how the behavior of an organism when it is very young can have a significant effect on its chances of
survival as an adult

Paragraph 2: One method by which the competition can be reduced is for the two competing groups gradually to
become specialized to different ways of life. They may then be able to exist together in the same area without
competing with one another. Dolph Schluter, of the University of British Columbia in Vancouver, has studied this
aspect of evolution in sticklebacks, small fish that became isolated in small lakes in British Columbia at the end of
the last Ice Age, about 12,000 years ago. Where there is only one species in a lake, it is a generalist, feeding both on
the bottom and in the open water of the lake. But, where there are two species in the lake, one has become
specialized to feed on the bottom and has a larger, more bulky body, a wide mouth, and a coarse food-filtering
system in the gills. The other instead feeds only in the open waters and has a more slender body, a narrower mouth,
and a finer filter system. Schluter designed an experiment with generalists, open-water feeders, and bottom feeders.

第 326 页 共
373 页
2021 托福阅读真题

He created artificial ponds simulating the environment from which each group came. In each pond he placed a
sample from each of the three groups: generalists, bottom feeders, and open-water feeders. He found that those
generalists better adapted to feeding on the bottom grew more rapidly than the generalists that were more similar to
open-water feeders. Schluter speculated that there was greater Competition for growth and survival between the
open-water feeders and the generalists similar to them because of greater predation compared to that experienced by
the bottom feeders

3, The word "artificial" in the passage is closest in meaning to


A, fairly small
B, containing limited resources
C, new and unfamiliar
D, made by humans

4, In paragraph 2, what is the author's purpose in discussing the work done by Dolph Schluter?
A, To illustrate how living in the same environment tends to make different species develop similar features
B, To indicate the effect of the Ice Age on the present location of some small fish
C, To show how two different species within one habitat may specialize in such a way that they do not compete for
the same food resources
D, To confirm that evolution can be studied in artificial environments

5, According to paragraph 2, what happened when Schluter placed generalists, bottom feeders, and open-water
feeders in the same pond together?
A, The generalists that were most like bottom feeders grew faster than those that resembled open-water feeders
B, The open-water feeders began to disappear because they could not compete with the generalists.
C, All of the species maintained their existing feeding habits and were able to grow as a result.
D, Open-water feeders learned to become generalists.

Paragraph 3: Finally, a part of the adaptation of any population is to ensure that its numbers are approximately
adjusted to the food supply of the area. The territorial behavior of some birds, such as the Scottish red grouse, does
this very effectively. Each male takes possession of an open area of flowering plants large enough to provide an
adequate food supply for its family and defends it against other members of its species. In a year when food is scarce,
the territory claimed is larger. The males compete for these territories by display behavior, and this system
therefore not only ensures that it is the weakest birds that are excluded from the territory (and are frequently
killed by predators or starvation) but also ensures that an adequate food supply is available for the successful
birds. This type of social competition has parallels in other species. As a result of social competition, those that are
successful receive a variety of advantages-sexual, nutritive, and environmental. The red grouse society no longer
contains a group of moderately successful males sometimes adequately fed and at other times weakened by

第 327 页 共
373 页
2021 托福阅读真题

malnutrition. Instead, it is permanently divided into the Hhaves Hassured of the necessities of life and of the
opportunity, by reproduction, to transmit their characteristics to the next generation and the "have nots,” of whom
about sixty percent die during the winter.

6, The word "scarce” in the passage is closest in meaning to


A, abundant
B, nearby
C, insufficient
D, undesirable

7. Which of the sentences below best expresses the essential information in the highlighted sentence in the passage?
Incorrect choices change the meaning in important ways or leave out essential information
A, Display behavior means that the least successful birds are not necessarily those that are physically weak, but
rather those that do not do well in competing for territory.
B, By competing for these territories, a male displays his strength and his ability to ensure that an adequate food
supply will be available for himself and a potential mate.
C, This system ensures that at least some birds will have an adequate food supply, which is important in a population
that is frequently threatened by predators and starvation
D, The males use display behavior to compete for these territories. which both excludes the weakest birds and
ensures an adequate food supply for the successful birds.

8, Which of the following can be inferred from paragraph 3 about male grouse who do not acquire territory in a
particular year?
A, They must depend on other males to provide food for their families.
B, They will be forced to kill another grouse to obtain food
C, They will most likely succeed in finding territory in the next year.
D, They may starve during the winter.

Paragraph 2: ■One method by which the problem of competition can be reduced is for the two competing groups
gradually to become specialized to different ways of life. ■ They may then be able to exist together in the same area
without competing with one another. ■ Dolph Schluter, of the University of British Columbia in Vancouver, has
studied this aspect of evolution in sticklebacks, small fish that became isolated in small lakes in British Columbia at
the end of the last Ice Age, about 12,000 years ago. ■ Where there is only one species in a lake, it is a generalist,
feeding both on the bottom and in the open water of the lake. But, where there are two species in the lake, one has
become specialized to feed on the bottom and has a larger, more bulky body, a wide mouth, and a coarse food -
filtering system in the gills. The other instead feeds only in the open waters and has a more slender body, a narrower
mouth, and a finer filter system. Schluter designed an experiment with generalists, open-water feeders, and bottom

第 328 页 共
373 页
2021 托福阅读真题

feeders. He created artificial ponds simulating the environment from which each group came. In each pond he placed
a sample from each of the three groups: generalists, bottom feeders, and open-water feeders. He found that those
generalists better adapted to feeding on the bottom grew more rapidly than the generalists that were more similar to
open-water feeders. Schluter speculated that there was greater Competition for growth and survival between the
open-water feeders and the generalists similar to them because of greater predation compared to that experienced by
the bottom feeders

9, Look at the four squares ■ that indicate where the following sentence could be added to the passage
Evolution also allows species to survive in environments in which they must compete for limited resources.
Where would the sentence best fit? Click 01 square ■ to add the sentence to the passage.

10, Directions: An introductory sentence for a brief summary of the passage Improvised below. Complete the
summary by selecting the THREE answer choices that express the most important ideas in the passage. Some
sentences do not belong in the summary because they express ideas that are not presented in the passage or are minor
ideas the passage. This question is worth 2 points.

Organisms must continuously adapt to avoid being eaten and to compete for space and food.

第 329 页 共
373 页
2021 托福阅读真题

Hermit Crabs in Snail Shell

Paragraph 1:Hermit crabs depend on a properly fitting snail shell for protection from predators. Crabs in shells that
are too small grow more slowly, are less tolerant of air exposure and drying out,and are more likely to be eaten by a
predator than those that can withdraw completely into their shell. When a crab has outgrown its snail shell, it has to
look for a bigger one that fits properly. They locate shells by smell, either when the preceding owner (the snail) dies
and begins to decay or by detecting calcium,the major component of snail shells. The crabs do not generally kill
snails to find new shells, although they may fight with other hermit crabs over empty shells, especially when empty
shells are scarce. After locating a new shell, the crab investigates its surface and internal size by rolling it over and
exploring it with its claws and walking legs. If it looks like a good fit, the crab withdraws its abdomen from its old
shell and inserts it into the new one so quickly that it can be difficult to see. Switching shells is risky- -the crab
could be attacked by a predator or lose one or both shells to other hermit crabs.

1.The word "preceding" in the passage is closest in meaning to


A sick
B earlier
C similarly sized
D natural

2.Why does the author point out that "Switching shells is risky" for hermit crabs?
A To suggest that hermit crabs rarely switch shells
B To illustrate the consequences of moving into a poorly fitting shell
C To provide an explanation for why many hermit crabs remain in a shell that is too small for them
D To explain why hermit crabs insert themselves into their new shell very quickly

3.According to paragraph 1, all of the following are disadvantages a hermit crab may experience when occupying a
shell that is too small EXCEPT:
A The crab is more likely to suffer from drying out.
B Predators can reach the crab more easily.
C The crab has less access to fresh air.
D The crab grows more slowly.

4, According to paragraph 1, what is the first thing a hermit crab does after locating a new shell?
A, Force the current owner out of the shell
B, Insert its abdomen into the shell quickly
C, Determine whether the shell is the right size
D, Roll over to make switching shells easier

第 330 页 共
373 页
2021 托福阅读真题

Paragraph 3: Particular species of hermits tend to prefer certain species of snail shells, but size is the critical factor.
Occupying larger and heavier shells gives them greater protection, but it takes more energy to lug a large house
around. Shells often become covered with other organisms, such as barnacles or algae, that make them heavier and
more difficult to carry. Some hermit species live in shells that get entirely covered by bryozoan colonies that
eventually dissolve the snail shell, leaving the crab with a house composed entirely of the bryozoans. As the
bryozoan colony grows, so too do the crabs, so these crabs have no need to change their homes.

5, Paragraph 3 discusses which TWO of the following as possible effects of organisms that may cover a hermit crab's
shell? To receive credit, you must select TWO answers.
A. Organisms on the shell force the crab to spend more energy in carrying the shell around.
B. Organisms that dissolve the shell leave the crab at risk from predators.
C. Organisms can form colonies that replace the shell.
D. Organisms on the shell allow the crab to obtain food without leaving its home.

Paragraph 4: Hermit crabs with poor-fitting shells are chemically attracted to dying gastropods(snails and related
animals) and other hermit crabs where a shell may become available. In the Mediterranean species Clibanarius
erythropus, gastropod predation sites attract dozens of hermit crabs. Researcher Elena Tricarico and colleagues
observed that these aggregations function observed that these aggregations function as shell exchange markets: the
first crab to arrive takes the empty shell, and a chain of shell exchanges among the crabs follows. They found that
simulated snail predation sites quickly attracted a larger number of hermit crabs than other types of sites; therefore,
aggregation is the most efficient tactic for this species to acquire new shells. In Belize, the land hermit Coenobita
clypeatus has an even more organized shell exchange in what is called a "synchronous vacancy chain" by Randi
Rotjan and colleagues, who discovered this unique behavior. When a large, empty shell becomes available, many
crabs gather around it, which can take hours. As they gather, the crabs arrange themselves into a line of decreasing
size, starting with the largest crab holding onto the empty shell. As though choreographed, the crabs begin shell
swapping, one after the other, smaller crab climbing into a new shell, after it is vacated by the slightly larger crab
ahead of it. What makes the synchronous chain possible is that smaller crabs linger near a too-large shell, perhaps
attracting others, waiting until a bigger crab comes along, which increases their chances of getting a good-fitting
hand-me-down.

6,The word "tactic" in the passage is closest in meaning to


A, strategy
B, type of cooperation
C, movement
D, action

7, According to paragraph 4, which of the following is true of the shell exchanges of the Mediterranean hermit crab

第 331 页 共
373 页
2021 托福阅读真题

species Clibanarius erythropus?


A They are not as organized as the synchronous vacancy chains of Coenobita clypeatus are.
B, They are not as competitive as the shell exchanges of Coenobita clypeatus are.
C They begin after the crabs arrange themselves into a line of decreasing size.
D, They are more advantageous to larger crabs than to smaller ones.

Paragraph 5: Most hermit crabs and most snails are right-handed and spiral clockwise. Some species, however, are
left-handed and spiral counterclockwise, and they need to find snail shells that are coiled that way. Since the majority
of snail species are right- handed, left-handed hermits may have trouble finding left- handed snails. Adults of the
left-handed Pefrochirus Diogenes the largest hermit crab in the Caribbean, are often found in shells of the queen
conch Eustrombus gigas. As an exception to the general rule, they will attack and eat the conch, thus obtaining both
a meal and a shell.

8, Which of the following can be inferred about "Eustrombus gigas"?


A, They often attack and eat hermit crabs.
B, They sometimes have trouble finding a suitable shell to use as a home.
C, Their shells are too small for large adult Petrochirus diogenes to live in.
D, The species includes individuals whose shells coil in a counterclockwise direction.

Paragraph 3: Particular species of hermits tend to prefer certain species of snail shells, but size is the critical factor.
Occupying larger and heavier shells gives them greater protection, but it takes more energy to lug a large house
around. ■Shells often become covered with other organisms, such as barnacles or algae, that make them heavier
and more difficult to carry. ■Some hermit species live in shells that get entirely covered by bryozoan colonies that
eventually dissolve the snail shell, leaving the crab with a house composed entirely of the bryozoans. ■As the
bryozoan colony grows, so too do the crabs, so these crabs have no need to change their homes. ■

9,Look at the four squares ■that indicate where the following sentence could be added to the passage.
However, one type of organism can benefit crabs when it covers their shells.
Where would the sentence best fit? Click a square to add the sentence to the passage.

10. Directions: An introductory sentence for a brief summary of the passage is provided below. Complete the
summary by selecting the THREE answer choices that express the most important ideas in the passage. Some
sentences do not belong in the summary because the express ideas that are not presented in the passage or are minor
ideas in the passage. This question is worth 2 points.

A,As a hermit crab grows, it typically needs to find a new shell that fits it in size and spirals in the same direction as
its body.

第 332 页 共
373 页
2021 托福阅读真题

B, Hermit crabs may fight each other for empty shells, but, apart from adult crabs of one species that eat their shells'
original occupants, they do not normally kill gastropods to obtain shells.
C,Several hermit crab species use synchronous vacancy chains to compete with other crab species for shells at
gastropod predation sites.
D, Organisms that cover and dissolve the shell are dangerous to hermit crabs. But bryozoan colonies are beneficial
because they provide the crab with a source of food.
E,Hermit crabs are attracted to available shells by smell, and extensive shell exchanges occur after a large crab
vacates its earlier shell to move into a bigger one.
F, Certain hermit crab species do not use snail shells at all, which puts them at greater risk of predation but makes it
easier for them to survive during storms and floods.

第 333 页 共
373 页
2021 托福阅读真题

Forest Management

Paragraph 1: In the Mediterranean, the Roman Empire (27 B.C.E--394 C.E) focused on agriculture in the lowlands
and protected the wooded uplands. But this broke down as the Empire declined, starting around 300 C.E. as
large-scale lowland agriculture was abandoned in favor of small upland farms. Some historians have claimed that
this caused the subsequent increase in forest clearing (removal of trees), with a need for more small farms, and more
grazing animals like sheep and goats (which eat young trees and green shoots). However, others consider this
conclusion something of an exaggeration, as it overstates the extent to which widespread forest use meant forest
clearance. Even in the absence of government control and supervision, many forces limited the exploitation of
woodlands. Wooded land is valuable and likely to attract the interest and protection of powerful people
whether they are figures of authority or not, so dense forests and sizable woodlands would typically have been
off-limits to random exploitation. Jobs like building ships and roofing large structures require big forest trees.
Demand was low, since shipbuilding was pointless when commerce was virtually nonexistent, and large structures
are more usually built when population is increasing and cities are growing. Even where demand existed, transport
remained a problem. The high cost and difficulty of cutting down a great tree was negligible compared with the
expense and challenge of moving wood overland from the forest where it was harvested to the shipyard or building
site where it was to be used.

1, The word "focused" in the passage is closest in meaning to


A depended on
B concentrated
C expanded
D authorized

2, In paragraph 1, why does the author provide the information that sheep and goats "eat young trees and green
shoots"?
A To suggest that grazing animals like sheep and goats were better adapted to small upland farms than they were to
large-scale lowland agriculture
B To argue that grazing animals increased in number on small upland arms because there was an unlimited supply of
food for them there
C To indicate why some historians believe that the increase in small upland farms led to more forest clearing
D To explain why large- scale Roman agriculture was more productive than small upland farming

3, Which of the sentences below best expresses the essential information in the highlighted sentence in the passage?
Incorrect choices change the meaning in important ways or leave out essential information
A Powerful people helped the authorities protect valuable trees in dense forests and sizable woodlands from random
exploitation.

第 334 页 共
373 页
2021 托福阅读真题

B Powerful people probably protected dense forests and sizable woodlands from random exploitation because of
their value.
C The random exploitation of wooded land caused powerful people to authorize the protection of dense forests and
sizable woodlands
D Powerful people found dense forests and sizable woodlands attractive and interesting, so they tried to protect them

4, According to paragraph 1, all of the following factors limited the use of big forest trees in large-structure projects
EXCEPT
A lack of population growth
B lack of commerce
C difficulty of finding forests from which big trees could be cut
D cost of transporting wood from the forest to the building site

5, Which of the following can be inferred from paragraph 1 about the probable effect that the decline of the Roman
Empire had on forests?
A It made forests more valuable to rich landowners.
B It had little impact on the extent of tree removal
C It resulted in closer government supervision of forest clearance.
D It increased the demand for trees as building materials.

Paragraph 2: A factor that is harder for us to appreciate in an age when chain saws and wood splitters are widely
available is the technological limitation on harvesting forest giants. Big trees contain many times the potential heat
energy of small trees,but harvesting them and reducing them to wood that is useful for burning is a formidable task
the small hand tools that most villagers possessed were hardly up to the job. A householder was better off gathering
and using small pieces of wood. For fires, fallen or hanging branches that had dried out over the course of a year or
so were preferable.

Paragraph 3: Two forest management techniques that Roman writers or farming advocated continued to be widely
used after the fall of Rome. They survived not because they were mandated but because they were practical methods
of producing renewable supplies of the small-dimension wood that met the needs of farmers. These techniques,
known as coppicing and pollarding, are occasionally still practiced today, and many Mediterranean forests still bear
the marks of this kind of cultivation. The purpose of coppicing is to transform a single-trunked tree into a producer
of multiple fast-growing branches that can be harvested periodically. Mature trees are cut to the ground then
allowed to sprout multiple trunks. A coppiced woodland is typically divided into sections that are harvested on a
rotating basis. The length of the rotation depends on the speed with which the trees grow and the use to which the
branches are to be put. Fast-growing trees might be harvested every four years; slower-growing trees, like oak, might
require a fifty-year rotation. Instead of big trunks difficult to harvest and slow to regrow, coppiced woods produce

第 335 页 共
373 页
2021 托福阅读真题

multiple small diameter sticks and poles. Pollarding works on the same principle as coppicing, but trees that are
pollarded are cut back to a main trunk rather than to the ground. Where wild or domestic animals graze in forests, the
new branches that grow from coppiced trees at grouno level are likely to be eaten. With pollarded trees the
productive source of new growth is raised above the level where animals browse.

6, The word "periodically" in the passage is closest in meaning to


A at regular intervals
B many times
C profitably
D without much effort

7, According to paragraph 3, what do the coppicing and pollarding methods have in common?
A They both rely on cutting back trees produce multiple small-diameter branches.
B They both cut back mature trees to the ground
C They both raise the productive growth above the ground to prevent its being eaten by grazing animals.
D They were both required by law after the fall of Rome.

8, According to paragraph 3, which of the following is an advantage that pollarded trees have over coppiced trees?
A They produce more branches
B They have bigger trunks.
C They regrow more quickly.
D They are not damaged by grazing animals.

Paragraph 2: ■A factor that is harder for us to appreciate in an age when chain saws and wood splitters are widely
available is the technological limitation on harvesting forest giants. ■ Big trees contain many times the potential heat
energy of small trees,but harvesting them and reducing them to wood that is useful for burning is a formidable task
the small hand tools that most villagers possessed were hardly up to the job.■ A householder was better off gathering
and using small pieces of wood. ■ For fires, fallen or hanging branches that had dried out over the course of a year
or so were preferable.

9, Look at the four squares■ that indicate where the following sentence could be added to the passage.
Even if those problems could be overcome, there was a more practical obstacle to...

10.Directions: An introductory sentence for a brief summary of the passage is provided below. Complete the
summary by selecting the THREE answer choices that express the most important ideas in the passage. Some
sentences do not belong in the summary because the express ideas that are not presented in the passage or are minor
ideas in the passage. This question is worth 2 points.

第 336 页 共
373 页
2021 托福阅读真题

A The protection and supervision of upland forests and woodlands that existed under the Romans continued
uninterrupted by those who continued to live in the area
B Exploitation of lowland fields was taken over by villagers and householders after the fields were abandoned by the
Romans.
C Harvesting large trees and preparing their wood for use was difficult with the limited tools available at the time.
D Use of large, valuable trees was restricted to large- scale projects, for which there was little demand, and
transportation costs were high.
E Useful Roman techniques for the use and regrowth of small trees continued to be popular after the decline of the
Roman Empire.
F The Romans began growing grain in many small upland plots to supplement an inadequate lowland harvest.

第 337 页 共
373 页
2021 托福阅读真题

Grains, Cities, States, and Armies

Paragraph 1: Ancient human societies developed cuisines (styles of prepared food) that were based on specific
grains or roots. Roots and grains have many advantages over other food sources. They are rich in calories and
nutrients because they provide sustenance for the next generation of plants. They often grow abundantly in the wild.
They also can be easily harvested, as American botanist Jack Harlan demonstrated in the 1970s by harvesting two
kilograms of wheat in just an hour with a sickle (simple harvesting tool). Roots could often be harvested
year-round or left in the ground without rotting, and grains could be stored in granaries (storage buildings) to
provide food in the difficult seasons of cold,drought,or heavy rain, depending on the region.

1, Why does the author point out that "Roots could often be harvested year-round or left in the ground without
rotting, and grains could be stored granaries"?
A, To explain why grain was harvested faster and in larger quantities than roots
B, To indicate that early human societies harvested as much wild-growing food as possible
C, To suggest that roots were superior to grains in certain ways
D, To support the claim that roots and grains have advantages in comparison to other foods

Paragraph 2: Few other candidates for plant food measured up. Although some fruits, particularly bananas and
breadfruit, also provided substantial calories, most were small, sour or bitter, seasonal, and hard to stockpile. Nuts,
such as acorns, chestnuts, pine nuts, coconuts.and hazelnuts, are rich in calories but many are so oily that they upset
the stomach when eaten in large quantities Because nut-bearing trees often take years to fruit, migrants who needed
to reproduce cuisines quickly in a new place would have been more likely to move roots and grains. Leaves and
shoots, low in calories, often bitter, and difficult to store were used mainly as medicines. Most societies,
therefore, came to depend on two or three favorite roots or grains as their staples-that is as the food resources they
depended on to provide most of their calories. Other foodstuffs, such as meat, fruits, and vegetables, provided flavor
variety and nutritional balance. The staples that humans had picked out centuries before 1000 B.C.E. still provide
most of the world's human food calories. Only sugarcane, in the form of sugar, was to join them as a major food
source.

2, The word "substantial" in the passage is closest in meaning to


A, necessary
B, adequate
C, considerable
D, valuable

3, Why does the author mention that "Leaves and shoots,low in calories, often bitter, and difficult to store. were
used mainly as medicines"?

第 338 页 共
373 页
2021 托福阅读真题

A, To emphasize the importance of medicinal plants in ancient societies


B, To support the idea that few plants could compare to roots and grains as staples
C, To compare the uses of low-calorie plants with those of calorie-rich plants such as nuts
D. To explain why ancient people grew leaves and shoots despite the difficulties involved

Paragraph 3: Only cuisines consisting of grain were able to support cities in 1000 B.C.E., a generalization that
would hold true until the end of the nineteenth century CE. This phenomenon has to do with the difficulty of
provisioning large conglomerations of people, notably the cities and the armies. which in the ancient world often
rivaled cities in size. To sustain an individual on a staple diet of roots required consuming a large weight daily, up to
sixteen pounds one estimate suggests, though it seems improbably high. Whatever the exact figure, grains had a
much better nutrient-to-weight ratio:only about two pounds of grain, on average, were needed to provide 2,500 to
3,000 calories per person per day. When everything had to be carried on the backs of men or animals, in
lumbering oxcarts with a maximum speed of three miles an hour, by sea, this difference between moist, heavy
roots (as much as 80 percent water) and dry, relatively light grains (10 percent water) was crucial. Grains
stored well too, keeping at least a year and often longer, unlike wet roots, which began to rot once out of the ground.
Not until the cheap, fast steamships and railroad transport of the nineteenth century could roots compete with grains
as provisions for cities.

4, Which of the sentences below best expresses the essential information in the highlighted sentence in the passage?
Incorrect choices change the meaning in important ways or leave out essential information.
A, The difference in weight between moist roots and lighter dry grains was very important when everything had to
be carried by people or animals, pulled in carts, or transported by sea.
B, Because of their water content, roots and grains had to be transported slowly, either by land or by sea
C, Transporting roots and grains was a slow process, given the limitations of human, animal, and sea-based means of
carrying heavy goods
D, A crucial difference between grains and roots is the fact that grains are dry while roots are moist.and this causes
roots to be heavier and more difficult to carry than relatively light grains

5, According to paragraph 3, which TWO of the following reasons explain why cuisines that supported cities in 10u
B.CE. were based on grains rather than roots? To receive credit, you must select TWO answers
A, Grains could be eaten in smaller amounts than roots and still provide enough nutrients.
B, Keeping enough roots to feed each person sixteen pounds per day required too much storage space
C, The supply of roots was often set aside to provision the large armies of the ancient world
D, Grains could be eaten long after they were harvested while roots could not.

Paragraph 4: That is not to say feeding cities or armies even with grains was easy. A packhorse could carry 200 to
250 pounds of grains enough for ten people for ten days. The problem was that a horse ate ten pounds of grain (and
ten of grass) every day, so unless grain could be obtained along the route, it consumed rations equivalent to those of

第 339 页 共
373 页
2021 托福阅读真题

five men every day, and within three weeks had eaten its entire load. Water transport was more efficient. A merchant
ship in the ancient Mediterranean Sea could carry as much as 400 tons. It cost no more to ship grain from one end of
the Mediterranean to the other than to cart it 75 miles overland. Hence grains were rarely moved more than about 5
miles by land, since land transport cost 7 times as much as river transport and 25 to 30 times as much as going by sea.
Cities, not surprisingly, were usually located on navigable rivers or good harbors(places where ships could safely
stop).

6, According to paragraph 4, supplying enough grain to feed cities and armies was difficult for which of the
following reasons?
A, Animals used to transport grain often ate large portions of the grain they carried along the way
B, Packhorses were rarely able to carry heavy loads of grain on journeys lasting more than ten days
C, Horses needed to eat more grass each day than could be found along many transportation routes.
D, Cities and armies were often located far from the places most favorable for growing grain

7, According to paragraph 4, all of the following are true of grain transportation in the ancient Mediterranean region
EXCEPT:
A, Shipping grain in the Mediterranean Sea was worth the cost only when loads were heavier than 400 tons
B, Moving grain 75 miles on land cost the same as or more than shipping it across the entire Mediterranean Sea
C, Grains were typically carried overland only if they were being moved 5 or fewer miles
D, Shipping grains by sea was much less expensive than transporting them by river

8, Paragraph 4 suggests that many cities were located on rivers or harbors for which of the following reasons?
A. Many cities needed access to grain growth at the other end of the Mediterranean Sea.
B, The grain grown by people living in cities required a reliable water source
C, The cost of transporting food supplies on land was often too expensive
D, People living in cities lacked enough horses to transport their food supplies overland.

Paragraph 2: Few other candidates for plant food measured up. Although some fruits, particularly bananas and
breadfruit, also provided substantial calories, most were small, sour or bitter, seasonal, and hard to stockpile. Nuts,
such as acorns, chestnuts, pine nuts, coconuts.and hazelnuts, are rich in calories but many are so oily that they upset
the stomach when eaten in large quantities. ■Because nut-bearing trees often take years to fruit, migrants who
needed to reproduce cuisines quickly in a new place would have been more likely to move roots and grains. ■Leaves
and shoots, low in calories, often bitter, and difficult to store were used mainly as medicines. ■Most societies,
therefore, came to depend on two or three favorite roots or grains as their staples-that is as the food resources they
depended on to provide most of their calories. Other foodstuffs, such as meat, fruits, and vegetables, provided flavor
variety and nutritional balance. ■The staples that humans had picked out centuries before 1000 B.C.E. still provide
most of the world's human food calories. Only sugarcane, in the form of sugar, was to join them as a major food

第 340 页 共
373 页
2021 托福阅读真题

source.

9, Look at the four squares■ that indicate where the following sentence could be added to the passage.
Because these were fast-growing crops, they could provide valuable sources of nutrition soon after planting.
Where would the sentence best fit? Click on a square to add the sentence to the passage.

10. Directions: An introductory sentence for a brief summary of the passage is provides below. Complete the
summary by selecting the THREE answer choices that express the most important ideas in the passage. Some
sentences do not belong in the summary because they express ideas that are not presented in the passage or are minor
ideas in the passage. This question is worth 2 points.

Ancient human societies often based their cuisines on certain roots or grains.

A, Many of the staples that humans selected before 1000 B.CE were small and difficult to store, so ancient people
eventually began to rely on roots and grains, which were more nutritious and easier to store.
B, All cities depended on grain-based food from 1000 B.C.E until the end of the nineteenth century C.E, when faster,
less expensive transportation modes brought greater variety to urban cuisines.
C, In the ancient world, the supply of roots was needed to feed large armies, whose diets were insufficiently
nutritious, and that made roots unavailable in cities.
D, Roots and grains were not the only nutritious, calorie-rich plant foods, but they were plentiful, easy to harvest,
and often relatively easy to make available as a food source throughout the year.
E, Ancient people often relied on oxcarts of packhorses to transport enough grain to feed large populations, and oxen
and horses frequently carried large loads of grain for many miles.
F, Feeding cities posed challenges for ancient people, who relied on themselves or animals to carry supplies on land,
and ship provided a means of transporting large amounts of grain over long distances

第 341 页 共
373 页
2021 托福阅读真题

Science and Nineteenth-Century Visual Arts

Paragraph 1: In the middle of the nineteenth century, rapid developments in the sciences and popular fascination
with recent technological advances were being reflected in European art. Many artists believed they could portray
reality with greater accuracy if they adopted a scientific approach, depicting their subjects with a strict and detached
objectivity. New technologies and ideas about how the mind perceives the world inspired completely new artistic
methods that abandoned traditional standards and often surprised the public.

1, According to paragraph 1, which of the following statements best describes the effect of science on European art
in the middle of the nineteenth century?
A, Scientists successfully advocated new uses of technology in art.
B, Scientific developments led artists to represent their subjects in a more objective way.
C,Technology and scientists became popular subjects in new works of art
D, Artists competed with scientists to describe reality most accurately

Paragraph 2: Camera photography, based on the exposure of chemically coated film to light, had a profound impact
on the art of the nineteenth century. Improvements by the Frenchman Louis Daguerre made the camera a useful tool
by the 1830s, though it remained large and cumbersome until the dry plate and miniature camera were introduced in
the 1870s. Photographic services were already in high demand by mid-century; in the 1860s 30,000 people in Paris
were making their living from photography and allied fields. The use of celluloid film and George Eastman’s
invention of the Kodak camera in 188 which quickly became mass-produced and affordable, gave wide public access
to the practice of photography. The ability of photography to depict a scene with exactitude had a significant impact
on art. On the one hand, it encouraged many artists to be true to reality, to reproduce or canvas a visual image akin to
that of a photograph. On the other hand, other artists now felt that such realism was no longer necessary in their
sphere. However, the great majority of the public, which now had wide access to museum exhibitions, was
accustomed to photographic accuracy and desired art that was representative and intelligible. Realistic works of art
met the need, at least superficially.

2, The word profound"in the passage is closest in meaning to


A, gradual
B, immediate
C, surprising
D, significant

3, Which of the following can be inferred about works by artists who felt that "realism was no longer necessary" in
art?
A, They were displayed in museums more often than other works.

第 342 页 共
373 页
2021 托福阅读真题

B, They depicted objects with great exactness than did most other works of art.
C, They were less popular with the viewing public than realistic paintings.
D, They were generally considered representative and intelligible

4, According to paragraph 2, advances in camera technology had which of the following consequences?
A, Camera use became specialized and was practiced by professionals only
B, Photographers began to create photographs that looked like traditional paintings.
C, Photography became less expensive and more available to ordinary people.
D,Most painters gave up their craft and made their living from photography

Paragraph 3: Many artists began to reject myths and symbols, choosing to portray the world as it actually was, or at
least as it appeared to them-a world without illusions, everyday life in all its grimness. The realist painter Gustave
Courbet proclaimed himself to be "without ideals and without religion." His fellow Frenchman Jean-Francois Millet
held a similar opinion. Instead of romanticizing peasants in the manner of earlier artists, he painted the harsh
physical conditions under which they labored. In England, the so-called Pre-Raphaelites took as their model painters
who came before Raphael Renaissance Italy and who had depicted the realistic simplicity of nature. In painting
historical scenes, these artists meticulously researched the landscape, architecture, fauna,and costumes of their
subjects. William Holman Hunt, one of the movement's founders, traveled all the way to Palestine before painting.
The Scapegc in order to guarantee accurate portrayal of the Dead Sea. In the past, artists had been concerned about
composition and perspective (the representation of three-dimensional objects in a two-dimensional space), but under
the influence of photography they began to paint incomplete off-center pictures. Orchestra the Opera, by French
painter Edgar Deg looks as if its edges have been cut off, with the musicians only partially visible and the dancers'
heads out of the frame.

5, The word "proclaimed"in the passage is closest in meaning to


A, declared
B, believed
C, wished
D, forced

6, In paragraph 3, why does the author discuss William Holman Hunt's trip to Palestine?
A, To illustrate the importance some artists placed on correctly reproducing the world as it actually is
B, To explain why some nineteenth-century English artists looked to Renaissance Italy for models
C, To explain why Pre-Raphaelites were less concerned than earlier artists about composition and perspective
D, To show that English painters were more enthusiastic than French painters about applying the methods of
photography.

7, According to paragraph 3, the Pre-Raphaelites were a group of painters who

第 343 页 共
373 页
2021 托福阅读真题

A, modeled their work on that of Gustave Courbet and Jean-Francois Millet


B, devoted much effort to depicting the subjects realistically
C,preferred to paint historical scenes situated in distant places
D, had a very strong attachment to ideals and religion

8, According to paragraph 3,the unusual framing of Degas's Orchestra of the Opera indicates
A, the artist's rejection of the ideas of the Pre-Raphaelites
B, the artist's concern with composition and perspective
C, the problem of representing three-dimensional objects in two-dimensional space
D, the influence of photography

Paragraph 4: On April 15,1874, six French artists--Edgar Degas, Claude Monet, Camille Pissarro, Auguste Renoir,
Alfred Sisley, and Bert Morisot--opened an exhibition in Paris that a critic scornfully called "impressionist." after the
title of one of Monet's paintings. Impression: Sunrise. The Impressionists were influenced by new theories of physics
that claimed images are transmitted to the brain as small light particles that the brain then reassembles. The
Impressionists wanted their paintings to capture what things look like before the brain "distorts" them. Many of these
painters left their studios to paint objects exactly as they look outdoors when light hits them at a certain angle. Monet,
for example, emphasized outdoor painting and the need for spontaneity--for reproducing subjects without
preconceptions about how earlier artists had depicted them--and seeking to show exactly how the colors and shapes
struck the eve. Monet was particularly interested in creating multiple paintings of the same scene-from different
viewpoints, under different conditions, at different times of day-to underscore that no single "correct" depiction
could possibly capture a subject

Paragraph 2: Camera photography, based on the exposure of chemically coated film to light, had a profound impact
on the art of the nineteenth century. ■Improvements by the Frenchman Louis Daguerre made the camera a useful
tool by the 1830s, though it remained large and cumbersome until the dry plate and miniature camera were
introduced in the 1870s. ■Photographic services were already in high demand by mid-century; in the 1860s 30,000
people in Paris were making their living from photography and allied fields. ■The use of celluloid film and George
Eastman’s invention of the Kodak camera in 188 which quickly became mass-produced and affordable, gave wide
public access to the practice of photography. ■The ability of photography to depict a scene with exactitude had a
significant impact on art. On the one hand, it encouraged many artists to be true to reality, to reproduce or canvas a
visual image akin to that of a photograph. On the other hand, other artists now felt that such realism was no longer
necessary in their sphere. However, the great majority of the public, which now had wide access to museum
exhibitions, was accustomed to photographic accuracy and desired art that was representative and intelligible.
Realistic works of art met the need, at least superficially.

9, Look at the four squares■ that indicate where the following sentence could be added to the passage.

第 344 页 共
373 页
2021 托福阅读真题

This influence on art grew steadily, thanks to periodic advances in photographic technology
Where would the sentence best fit? Click on a square to add the sentence to the passage.

10. Directions: An introductory sentence for a brief summary of the passage is provided below. Complete the
summary by selecting the THREE answer choices that express the most important ideas in the passage. Some
sentences do not belong in the summary because the express ideas that are not presented in the passage or are minor
ideas in the passage. This question is worth 2 points.

By the mid-nineteenth century in Europe, a new era of technological and scientific advances was being
reflected in the arts

A, As cameras and camera images became more familiar,people grew to value photographic exactitude in the painted
image.
B, Many painters abandoned symbolic and idealized images, preferring to depict the world as they experienced it
directly.
C, The Pre-Raphaelites' concern with composition and perspective was rejected by realists such as Degas,who
painted incomplete, off-center pictures.
D, Access to photographic services and Kodak cameras caused the public to conclude that realism was no longer
necessary in the artistic sphere.
E, Photographs of grim peasant labor led the Pre-Raphaelites to give up romantic illusions about peasant life and
turn to historical subjects.
F, New theories about how the mind perceives images led the Impressionists to experiment with daylight and other
variables.

第 345 页 共
373 页
2021 托福阅读真题

Determining the Source of a Meteorite

Paragraph 1:Asteroids are small, planet-like bodies that orbit the Sun, with most orbiting in an area between Mars
and Jupiter known as the asteroid belt. Meteorites are asteroids or fragments of asteroids that have fallen to Earth.
Only rarely are scientists able to tell from direct observation which asteroid was the source of a given meteorite. In
October 2008, astronomers were lucky enough to observe a tiny asteroid, named 2008tc3, about 20 hours before it
hit Earth The 4-meter-diameter object appeared as a brilliant fireball over Sudan before disintegrating in the Nubian
Desert, where scientists later collected 280 meteorite fragments. These meteorites tell us that the parent of 2008 TC3
was a highly unusual jumble of rocky chunks from many different bodies that were later assembled into single
asteroid.

1. The word “disintegrating" in the passage is closest in meaning to


A. coming to rest
B. breaking apart
C. descending
D. landing

2. According to paragraph 1, all of the following are true of the asteroid 2008 TC3 EXCEPT:
A. It left at least 280 meteorite pieces on Earth.
B. It was observed by scientists before it fell to Earth.
C. It was found about 20 hours after it hit Earth.
D. It was composed of rocks from multiple bodies.

Paragraph 2:One way to work out where meteorites come from is to measure their spectrum, the particular groups
of colors that they reflect, and compare it with the spectrum of light reflected by different asteroids viewed through a
telescope. In one case, this connection is easy to make. The HED meteorites--short for howardite, eucrite, and
diogenite--are a group of about 1,000 objects with a uniquely characteristic spectrum that is almost identical to that
of Vesta, the largest body in the inner part of the asteroid belt. Astronomers have also found a nearby cluster of small
asteroids with similar spectra that appear to be fragments from an ancient impact on Vesta. Some of these fragments
lie close to a zone where the gravitational effects of celestial bodies on each other can make the orbits of the bodies
unstable. These fragments are probably the immediate parents of the HED meteorites, while Vesta itself is the
grandparent.

Paragraph 3: Despite this success, it has been surprisingly difficult to find other matches. Many meteorites have a
spectrum that doesn't correspond to any known asteroid, and most asteroid spectra don’t match known meteorites.
Fortunately, we now understand the reason why. Space is a very harsh environment. Asteroids and other objects
without an atmosphere are constantly bombarded by cosmic rays (high-powered radiation that originates in outer

第 346 页 共
373 页
2021 托福阅读真题

space), particles in the solar wind, and other space debris. This continuous assault transforms to outermost layer of
an asteroid, breaking chemical bonds, and freeing metal atoms which then coat the surface of the surrounding rock.
The dust grains returned by spacecraft visiting asteroid Itokawa contained numerous microscopic particles of
metallic iron and iron sulfide that changed the grains' outward appearance. Astronomers call this effect space
weathering.

3. In saying that high-powered- radiation “originates in” outer space, the author means that the radiation
A. passes through outer space
B. is fast-moving in outer space
C. is found everywhere in outer space
D. comes from outer space

4. The author discusses the process of space weathering in paragraph 3 in order to explain why
A. asteroids and other objects eventually lost their atmospheres
B. it is difficult to identify the asteroid that a meteorite came from by comparing light spectra
C. dust grains from the asteroid Itokawa contain numerous iron particles
D. it is difficult to measure the spectrum of light reflected by an asteroid

Paragraph 4: Space weathering makes an asteroid appear redder in color and washes out features in its spectrum.
Meteorites are generally chunks that have broken off asteroids relatively recently, and their newly exposed surfaces
have experienced space weathering for only a short time. This makes meteorites look fresh compared to asteroids.
In a similar way, rocks on Earth change their appearance as they become old and weathered, although the weathering
process is different. When geologists want to identify rocks, they usually look for a surface that has been exposed
recently, or break open the rock to look at the pristine interior. Breaking open an asteroid is not a practical way of
figuring out what it is made of. Instead, scientists take the opposite approach and artificially weather meteorites in
the laboratory When this is done, the match between meteorites and asteroids becomes much better, and it is possible
to estimate which of the several types of asteroid-C-type, S-type, or U-type-that a particular meteorite comes from.

5. Why does the author include the information that the surfaces of meteorites “have experienced space weathering
for only a short time" ?
A. To provide evidence that meteorites are broken off parts of asteroids
B. To present a reason why meteorites look different from asteroids
C. To explain how meteorites can be distinguish from Earth rocks
D. To explain why it is not practical to open an asteroid to find out what it is made of

6. What can be inferred from paragraph 4 about meteorite surfaces?


A. They are older than the surfaces of many Earth rocks.

第 347 页 共
373 页
2021 托福阅读真题

B. They tend to resemble the surfaces of their parent asteroids.


C. They look similar to the surfaces of weathered rocks found on Earth.
D. They appear less red in color than the surfaces of their parent asteroids.

7. According to paragraph 4, why do scientists artificially weather meteorites in the laboratory?


A. To make it possible to match such meteorites to asteroids
B. To make it possible to examine the interior of such meteorites
C. To compare the weathering process on Earth to the weathering process in space
D. To determine the similarities between two different meteorites

Paragraph 5: Ordinary chondrites, the most common type of meteorite, come from a class of bright asteroids called
S-types that make up much of the inner part of the asteroid belt. ltokawa and several other asteroids visited by
spacecraft belong to this class, and their prevalence in the inner asteroid belt-the region closest to Earth- may explain
why so many meteorites are ordinary chondrites.

8. According to paragraph 5, which of the following is true of ordinary chondrites?


A. They are brighter than other types of meteorites.
B. They come from type of asteroid that is common in the inner asteroid belt.
C. They travel quickly through the inner asteroid belt.
D. Many come from asteroids that have been visited by spacecraft.

Paragraph 6: Carbonaceous chondrites appear to come from dark C-type asteroids in the middle and outer regions
of the asteroid belt, many of which contain hydrated, clay-like minerals judging from their spectra. Curiously, these
dark asteroids seem to be very common, and yet carbonaceous meteorites are quite rare, perhaps because they tend
to break apart when entering Earth's atmosphere and the pieces never make it to the ground.

Paragraph 1:Asteroids are small, planet-like bodies that orbit the Sun, with most orbiting in an area between Mars
and Jupiter known as the asteroid belt. Meteorites are asteroids or fragments of asteroids that have fallen to Earth.
Only rarely are scientists able to tell from direct observation which asteroid was the source of a given meteorite. ■
In October2008, astronomers were lucky enough to observe a tiny asteroid, named2008tc3, about 20 hours before it
hit Earth. ■ The 4-meter-diameter object appeared as a brilliant fireball over Sudan before disintegrating in the
Nubian Desert, where scientists later collected 280 meteorite fragments. ■ These meteorites tell us that the parent of
2008 TC3 was a highly unusual jumble of rocky chunks from many different bodies that were later assembled into
single asteroid. ■

9.Look at the four squares [■] that indicate where the following sentence could be added to the passage.
One such event recently occurred in Africa.

第 348 页 共
373 页
2021 托福阅读真题

Where would the sentence best fit? Click on a square to add the sentence to the passage.

10.Directions: An introductory sentence for a brief summary of the passage is provided below. Complete the
summary by selecting the THREE answer choices that express the most important ideas in the passage. Some
sentences do not belong in the summary because the express ideas that are not presented in the passage or are minor
ideas in the passage. This question is worth 2 points.

A. Scientists know the source of the 280meteorite fragments recovered in the Nubian Desert because they contain
the same unusual mixture of rock later detected in distant asteroid.
B. The weathering that an asteroid usually experiences in space affects its spectrum, making it difficult to match the
surface of a meteorite to the surface of its parent asteroid.
C. By understanding the processes of weathering in Earth rocks, scientists have been able to identify the parent
bodies from which some meteorites have descended.
D. If the spectrum of a meteorite is the same as the spectrum reflected by an asteroid, then the meteorite likely came
from that asteroid.
E. Meteorites form when asteroids and other objects are heavily bombarded by cosmic rays, particles in solar wind,
and other space debris.
F. Most meteorites come from S-type asteroids, and while C-type asteroids are common in some parts of the asteroid
belt, meteorites from them are rare.

第 349 页 共
373 页
2021 托福阅读真题

Songbirds

Paragraph 1: About 350 species of songbirds occur in North America, and about 250 of them are Neotropical
migrants. These Neotropical migrants spend winters in the New World tropics, which extend from southern Mexico
through Central and South America and into the West Indies. United States bird watchers have a nationwide system
of local bird censuses, and, starting in the 1970s and 1980s, these counts revealed dramatic decreases in the numbers
of some Neotropical migrants such as wood thrushes and cerulean warblers in certain areas-as much as 1 percent
annually for the past 30 years. The pattern of decline is not consistent across species on regions of Populations of
some species of Neotropical songbirds a declining, but others are increasing. Also, populations of songbirds in the
forest areas of the eastern United States a) declining, but those in the West are not.

1, The word "consistent" in the passage is closest in meaning to


A, clear
B, present
C, significant
D, the same

2, Paragraph 1 suggests that which of the following statements is true of the nationwide system of local bird
censuses in the United States?
A, It existed before the 1970s
B, It was extended beyond the United States in the 1970s and 1980s
C, It is more effective in the eastern United States than it is in the western United States.
D, It was replaced in the 1980s

Paragraph 2: This complex picture results from the variety of factors that affect migratory birds in their
summer and winter ranges and in stopover points on their routes of migration, although changes in land use
are probably the primary reason for declining bird populations. Neotropical migrants breed in their summer
ranges, and the cutting up of forests into smaller and smaller patches has reduced the total amount of breeding
habitat available and changed the nature of the habitat that remains. Nests near the edges of woodlands generally
suffer higher rates of failure than do those nearer the center, and as forest patches grow smaller a greater proportion
of the habitat is near an edge.

3, Which of the sentences below best expresses the essential information in the highlighted sentence in the passage.
Incorrect choices change the meaning in important ways or leave out essential information.
A, A variety of factors affect the summer and winter environments that migratory birds choose to live in as well as
the many points on their migration.
B, This complex picture of increasing and declining bird populations is caused by many factors, but primarily by

第 350 页 共
373 页
2021 托福阅读真题

changes in land use.


C, Many factors affect migratory birds in stopover points on their migration routes between their summer and winter
ranges, which results in a complex picture
D, The picture is complex because of the variety of factors that contribute to the decline in migratory bird
populations.

Paragraph 3: Part of the increased rate of nest failure near the edges of woodlands may result from nest parasitism by
brown-headed cowbirds, which are nest parasites. Cowbirds do not build their own nests but Cowbirds do not
build their own nests but lay their eggs in the nests of other species; some 200 species of birds have been reported to
be parasitized by cowbirds. A female cowbird lays 20 to 40 eggs, one or two in each nest, and a few pairs of
cowbirds can parasitize all the nests in a small woodland. A female cowbird often removes an egg of the host species
from a nest when she lays her own, and cowbird eggs develop and hatch more rapidly than the eggs of the host
species, giving the cowbird nestling an advantage. Larger and pushier than the nestlings of the host species, cowbird
nestlings take so much of the food their unwitting foster parents bring to the nest that the hosts' nestlings may starve.

4, Which of the following is true about "some 200 species of birds"?


A, The woodland edges are the most preferred nesting places for all of these species.
B, Cowbirds lay their eggs in all of these species' nests.
C, All of these species typically lay 20 to 40 eggs in their nests
D, None of these species has suffered from as much nest failure as cowbirds have

5, According to paragraph 3, cowbirds have all of the following reproductive advantages over the birds they
parasitize EXCEPT:
A, Cowbirds lay their eggs earlier than their host birds do
B, Cowbird eggs hatch sooner than the hosts' eggs do.
C, Cowbirds lay a large number of eggs.
D, Cowbird nestlings eat more than the nestlings of other species do.

6, According to paragraph 3, in which of the following TWO ways is cowbird parasitism damaging to the parasitized
species? To receive credit, you must select 2 answer choices
A, The host birds may refuse to raise the cowbird's young and may leave their nest as a result.
B, A cowbird may leave up to 40 eggs in the hosts' nest, which can make it impossible for the hosts' own eggs to
hatch.
C, The cowbird frequently destroys an egg in the nest of the host birds
D, Cowbird nestlings take much of the food the hosts bring to their nest, and the hosts' young may not get enough
food as a result.

第 351 页 共
373 页
2021 托福阅读真题

Paragraph 4: Cowbird parasitism is insidious. When a nest with eggs or fledglings is lost to a predator, the parent
birds usually build another nest and start a second clutch (set of eggs). They may succeed in reproducing that season
despite the loss of their first clutch. In contrast, the parent birds that serve as hosts for the cowbirds do not
distinguish the cowbird eggs or nestlings from their own. Thus, when a pair of birds has raised a cowbird to fledgling,
they behave as if they had successfully fledged their own young and do not nest again that year.

7, In paragraph 4, why does the author contrast songbirds' response to losing their eggs or offspring to a predator
with their response to losing their eggs nestlings to cowbird parasitism?
A, To suggest that cowbird parasitism may be more damaging to songbirds’ reproduction than predation is
B, To show that songbirds are often unable to distinguish between damage caused by predators from damage caused
by cowbirds
C, To explain why the reproductive season is necessarily longer for songbirds than for cowbirds
D, To explain why songbirds are more likely to lose their eggs or offspring to predators than to cowbirds

Paragraph 5: Cowbirds are insect-eating and feed primarily in open fields Because their movements are not limited
by the need to care for their young, they can travel long distances between the open fields where they feed and the
woodland edges where they find the nests of other birds. Populations of cowbirds have increased dramatically as
open fields have replaced woodland habitats in North America, and the small remaining woodlands offer songbirds
too little interior space to conceal their nests from cowbirds. Thus habitat fragmentation in the summer breeding
range of Neotropical migrants appears to be responsible directly and indirectly for some of the decrease in their
populations.

8, The word "conceal" in the passage is closest in meaning to


A, protect
B, free
C, hide
D, separate

Paragraph 5: Cowbirds are insect-eating and feed primarily in open fields. Because their movements are not limited
by the need to care for their young, they can travel long distances between the open fields where they feed and the
woodland edges where they find the nests of other birds.■ Populations of cowbirds have increased dramatically
as open fields have replaced woodland habitats in North America, and the small remaining woodlands offer
songbirds too little interior space to conceal their nests from cowbirds. ■ Thus habitat fragmentation in the summer
breeding range of Neotropical migrants appears to be responsible directly and indirectly for some of the decrease in
their populations.

Paragraph 6: ■ Habitat change in the winter range probably also responsible for some population declines. The

第 352 页 共
373 页
2021 托福阅读真题

huge landmass of North America funnels down to a much smaller area in Central America, where many migrant
birds overwinter. Competition for food and space in the small landmass of Central America may be one of the
reasons that migrants do not breed in their winter ranges. ■ Because the land area in the winter range is small, habitat
changes caused by agricultural practices even on a relatively small scale could affect large numbers of birds.

9 , Look at the four squares ■ that indicate where the following sentences could be added to the passage
In recent decades, more and more of this winter habitat has been cleared for cultivation of various crops.

10, Directions: An introductory sentence for a brief summary of the passage is provide below. Complete the
summary by selecting the THREE answer choices that express 甘 . most important ideas in the passage. Some
sentences do not belong in the summary because they express ideas that are not presented in the passage or are minor
ideas in the passage. This question is

A, Data collected by bird-watching groups are useful, but we need additional studies to find out exactly how serious
the declines in bird populations are
B, Cowbirds do not raise their own young but lay eggs in the nests of other songbirds, which later raise cowbirds'
young at the expense of their own young
C, Cowbird parasitism is dangerous to songbirds because foster parents spend so much time gathering food for the
cowbird nestlings that the songbirds' nests are often lost to predators.
D, The cutting of forests into smaller patches reduces the available habitat for songbirds and leads to increase
parasitism by cowbirds at the woodland edges.
E, Cowbirds select nests containing eggs that look similar to cowbird eggs to make it more difficult for the host birds
to distinguish between parasitic cowbird eggs and their own
F, Relatively small losses of habitat in migratory birds' winter ranges car seriously affect the birds' populations
because the is not much total land available in those ranges.

第 353 页 共
373 页
2021 托福阅读真题

Dead Zones in the Sea

Paragraph 1: Many cities depend on nearby rivers to carry away waste. It eventually reaches the sea, where it
provides nutrients for plankton (small organisms that drift in water), causing plankton blooms (rapid and excessive
increases in plankton populations). When plankton die and sink into deeper water, they decompose in a process that
uses oxygen. As a result,the water beneath plankton blooms often becomes anoxic (lacking oxygen) and thus devoid
of life. These dead zones" now occur permanently or seasonally at over four hundred places worldwide, and one of
the largest forms annually where the Mississippi River meets the sea. Dead zones form more readily where water is
stagnant (moves little and separated into nonmixing layers. Stagnant pools near the seabed eventually lose their
oxygen as the sinking plankton built into piles. Despite their massive nutrient flows to the ocean, the Orinoco and
Amazon Rivers have no dead zones, because water is pushed offshore too fast for oxygen to be lost. Where the flows
of once great rivers have been greatly reduced by dams, cities, and crops along their course, areas where rivers flow
into seas may hold water long enough for dead zones to form. This is the case, sadly, for rivers like the Loire in
France and Po in Italy, when their lack of oxygen suppresses life in thousands of square kilometers of seabed every
summer.

1, Paragraph 1 suggests which of the following about dead zones?


A, They could be reduced by providing more nutrients for plankton.
B, They often move great distances along with water flowing from one part of the ocean to another.
C, They are more numerous now than they were in the past.
D, They are more common on the surface of the ocean than they are beneath the surface.

2, According to paragraph 1, which of the following is true about the area where the Mississippi River meets the sea
as compared to the areas where the Orinoco and Amazon Rivers meet the sea?
A, Water is more stagnant there
B, This area receives fewer nutrients from the river.
C, There is a relatively high amount of oxygen in the water there
D, Surface water mixes relatively easily with water near the seabed there

Paragraph 2: Enclosed seas like the Baltic, Adriatic, and Black also suffer problems of stagnation. Shaped like a
deep bowl that is cut off from the Mediterranean by a shallow underwater ridge in the Bosporus Strait, the Black Sea
was a freshwater lake when sea levels went down fairly dramatically during the last glaciation (an extremely cold
period during which much of Earth's water was locked in ice sheets). It refilled eventually, with saltwater, in a
catastrophic flood seven thousand years ago, when the Mediterranean rose high enough to break through the
Bosporus. Massive floods triggered a tremendous human migration and the event may be recorded in the ancient
tale of Gilgamesh. Today only a warm, less salty, surface layer is well oxygenated and able to support abundant life.
This low density layer sits over the cooler, saltier waters of the deep basin like a lid and has suffocated life below.

第 354 页 共
373 页
2021 托福阅读真题

Deeper than about 150 meters, the Black Sea is devoid of oxygen.

3, The word triggered" in the passage closest in meaning to


A, occurred at the same time as
B, stopped
C, followed
D, caused

4, According to paragraph 2, why is life in the Black Sea abundant only at the surface?
A, Most life in the deep parts of the Black Sea was destroyed in a catastrophic saltwater
flood seven thousand years ago.
B, Below the surface layer, the water in the Black Sea is too cold for life
C, High salt concentrations in the Black Sea kill off any life that enters the deep water.
D, The deep water in the Black Sea lacks enough oxygen to support life.

Paragraph 3: The Black Sea owes its stagnant depths to nature, but other enclosed seas have us to blame. Northern
Europe's Baltic Sea connects to the North Sea only by a shallow, tortuous channel that curls around Denmark. The
rivers that feed it drain from highly populous countries and run through croplands and intensive pig farms.
They receive substances flowing out of heavy industry and paper-pulp mills and sewage from cities great and small.
By the time the rivers reach the coast they are loaded with nutrients and organic waste. Sediment samples from one
bay of the Baltic Sea tell a similar story to those from the Mississippi dead zone. Land was cleared and trees felled
for agriculture and industry at increasing rates from 1800 onward, which led to an ever-growing rise in nutrient
runoff. Problems from plankton blooms and anoxic bottom water emerged after the 1950s, when artificial fertilizers
came into widespread use. The number of people in areas surrounding the Baltic Sea has more than doubled since
then, and sewage has compounded the effects of fertilizers.

5,Why does the author provide the information that certain rivers' drain from highly populous countries and run
through croplands and intensive pig farms"?
A, To help explain how human activities have contributed to the creation of dead zones
B, To show how substances released by heavy industry and paper-pulp mills can enter people's food
C,To identify Denmark as a source of pollution in the channel that connects the Baltic and North Seas
D, To suggest that agriculture is less harmful to the ocean than industry is

6, The word “artificial" in the passage is closest in meaning to


A, harmful
B, various
C, modern

第 355 页 共
373 页
2021 托福阅读真题

D, human-made

7, Which of the following can be inferred from paragraph 3 about the Mississippi River dead zone?
A, It is nearly the same size as the dead zone of the Baltic Sea.
B, It was caused by nutrients that spread into the area from faraway seas.
C, It is primarily caused by runoff from agriculture and industry.
D, It came into existence in the early 1800s.

Paragraph 4: The Baltic is a huge brackish water lake that occasionally takes in saltwater from the denser North
Sea. When conditions are right, the North Sea pours in over the underwater ridge around Denmark, staying close to
the seabed as it moves. Brackish water (a mix of salt and freshwater) is less dense and thus flows out at the same
time along the surface. This density difference restricts mixing between the salty deep and fresher surface layers and
so promotes a loss of oxygen when dead plankton sink to the bottom of the sea. Species that need saltwater to thrive
are thus confined to the deep layers with dwindling oxygen. One reason why Baltic codfish stocks fell so
catastrophically in recent years was that their eggs could not survive in these deep pockets of low-oxygen saltwater.

8, According to paragraph 4, which of the following happens to some of the water in the Baltic Sea when water from
the North Sea pours in?
A, The Baltic water flows toward the North Sea on top of the denser water flowing into the Baltic.
B, The Baltic water mixes freely with the water flowing into the Baltic from the North Sea.
C, The Baltic water's salt content decreases significantly
D, Most of the plankton present in the Baltic water die off.

Paragraph 2: Enclosed seas like the Baltic, Adriatic, and Black also suffer problems of stagnation. ■ Shaped like a
deep bowl that is cut off from the Mediterranean by a shallow underwater ridge in the Bosporus Strait, the Black Sea
was a freshwater lake when sea levels went down fairly dramatically during the last glaciation (an extremely cold
period during which much of Earth's water was locked in ice sheets). ■ It refilled eventually, with saltwater, in a
catastrophic flood seven thousand years ago, when the Mediterranean rose high enough to break through the
Bosporus. Massive floods triggered a tremendous human migration and the event may be recorded in the ancient tale
of Gilgamesh. ■ Today only a warm, less salty, surface layer is well oxygenated and able to support abundant life. ■
This low density layer sits over the cooler, saltier waters of the deep basin like a lid and has suffocated life below.
Deeper than about 150 meters, the Black Sea is devoid of oxygen.

9. Look at the four squares that indicate where the following sentence could be added to the passage.
With the Bosporus now open, salty Mediterranean water continues to flow into the bottom of the Black Sea,
which has increased the density contrast between Black Sea layers.
Where would the sentence best fit? Click on a square to add the sentence to the passage.

第 356 页 共
373 页
2021 托福阅读真题

10.Directions: An introductory sentence for a brief summary of the passage is provided below. Complete the
summary by selecting the THREE answer choices that express the most important ideas in the passage. Some
sentences do not belong in the summary because the express ideas that are not presented in the passage or are minor
ideas in the passage. This question is worth 2 points.

A, The decomposition of plankton following plankton blooms removes oxygen from deeper seawater and causes
many areas to have little life, especially where the water does not move much.
B, Because the Baltic and North Seas are connected, polluted waters have spread to the North Sea, contributing to
the appearance of plankton blooms and dead zones there.
C, The deep waters of the Black Sea lack oxygen because of how the sea formed, but the Baltic lost oxygen because
human activities have increased plankton-supporting nutrient runoff from rivers.
D, Some rivers suffer from oxygen loss even more than seas do because rivers often canal waste from cities,
industries, and cropland, which can cause large plankton blooms in river water.
E,Seawater that loses oxygen becomes denser and thus sinks, and this kills the plankton piled up on the seabed,
leading to fewer nutrients available for other organisms.
F, Both the Black and Baltic seas have saltier deep waters and fresher surface waters, and because the layers do not
mix much, oxygen cannot reach the deeper layers.

第 357 页 共
373 页
2021 托福阅读真题

Grinding Grain

Paragraph 1:It now seems that humans began to grind grain into flour earlier than was originally thought. Grinding
stones have been found at African and Asian sites dating from 200,000-50,000 years ago It was presumed at first that
these stones were used primarily to grind plant and animal materials, or minerals, to make pigments, rather than for
the preparation of foodstuffs. However, new findings from the Middle East raise the intriguing possibility that some
human groups may have been using grinding stones to process cereal grains, such as wheat, and maybe other types
of edible plant, as early as the Middle Paleolithic Era(i.e. 50,000 years ago and earlier). But why is it such an
advantage to grind cereal grains before eating them? The main reason is that grinding breaks down the hard, fibrous
cereal grain to release the easily digestible starch granules contained within This served two purposes. Firstly, it
enabled people to save enormously on the wear and tear of their teeth, compared to eating raw unprocessed grains.
Unlike the teeth of grazing animals, human teeth do not continue to grow after childhood. Tooth wear due to a diet
high in fiber and raw plants can result in the substantial erosion of molars (back teeth by early adulthood. People
with worn or absent teeth faced starvation unless they could find alternative types of food that did not require
chewing. Alternatively, they could try to find another way of grinding the fibrous plant material before eating it.
Perhaps it was one of these incentives that led to the use of stone grinding tools for seed processing.

Q1:Why does the author make the statement that "People with worn or absent teeth faced starvation unless they
could find alternative types of food that did not require chewing"?
A. To help explain why people needed to eat high-fiber food and raw plants
B. To introduce the second reason why starch granules were an important part of the human diet
C. To support the claim that humans used cereal grains for food as early as the Middle Paleolithic Era
D. To emphasize an advantage of grinding cereal grains before eating them Paragraph 1 is marked with

Q2:The word "incentives" in the passage is closest in meaning to


A. Ideas
B. efforts
C. motivations
D. Problems

Q3:Which of the following does paragraph 1 suggest about grinding stones found at African and Asian sites?
A. They were first used for purposes other than the preparation of foodstuffs
B. They are clearly unsuitable for the production of pigments.
C. They may have been used to process grain and other edible plants T
D. They provide evidence that humans in Africa and Asia learned grinding techniques from humans in the Middle
East.

第 358 页 共
373 页
2021 托福阅读真题

Q4:According to paragraph 1, eating raw, unprocessed grains was problem for humans because
A. the grains contained few nutrients compared to other foods
B. the grains caused the teeth to wear down and eventually be lost
C. children were incapable of chewing the grains
D. the grains needed to be saved to feed grazing animals

Paragraph 2:The development of grinding technology would have been socially advantageous to a human group.
People would tend to keep their teeth for much longer, and thus older, more experienced individuals could have lived
longer, despite the ultimate loss of their teeth. Such people could then serve their clan either as "grandparent"
childcare givers or by acting as instruments for the innovation and transmission of oral culture. The latter role was a
key adaptation in preliterate societies (societies without a writing system particularly in relation to strategies for food
acquisition and technology in an era of considerable climatic flux. The remembered knowledge of how their
grandparents dealt with the last arid period, including alternative food acquisition strategies, would have
enabled such surviving elders to greatly enhance the ability of their clan to deal with such difficult situations.

Q5: Which of the sentences below best expresses the essential information in the highlighted sentence in the passage?
Incorrect choices change the meaning in important ways or leave out essential information.
A. With their knowledge of how difficult conditions were dealt with in the past elders could have helped their clan
deal with similar situations.
B. Clans helped their grandparents survive by having them remember the difficult situations during the last arid
period.
C. The clan members who survived arid periods were generally elders who developed alternative methods of finding
food.
D. Grandparents helped their clans develop new food acquisition strategies in difficult situations, such as arid
periods

Paragraph 3:Unfortunately, grinding seed to make flour could be a mixed blessing. Depending on the type of stone
used, the prolonged and laborious process of grinding cereal grains would produce small chips of stone that could
get into the flour. People eating the products of such flour every day would be repeatedly exposed to stone chips as
hey chewed their food, and eventually their teeth might become chipped and worn. This problem was partially
alleviated many millennia later by the invention of pottery, which enabled a porridge to be made from grains mixed
with water and boiled without grinding.

Q6:According to paragraph 3, which of the following could be a problem with grinding seeds to make flour?
A. Grinding could be so time-consuming that people did not consider it worth the effort.
B. The type of stone that was effective for grinding could be difficult to find.
C. Grinding could remove certain important nutrients from the seeds.

第 359 页 共
373 页
2021 托福阅读真题

D. Pieces of stone could get into the flour and cause damage to teeth.

Paragraph 4: ■The second, and more immediate, reason for grinding cereal grains is that it enables us to produce a
more attractive. sweeter tasting, more nutritious, and calorie-rich foodstuff. ■Rather than a hard, dry, indigestible,
tooth-destroying- cereal grain, people could enjoy foods such as seed cakes, biscuits, and all the various forms of
bread that we still relish so much today. Cereal grains that have been ground and processed into flour can be much
more easily digested due to the higher surface area that is available for digestive enzymes (molecules that break
down food). ■This means that not only the plentiful starches but also the grain proteins and the much less abundant
micronutrients, are more easily absorbed from processed cereals. ■In the cold, dry climate of the Last Glacial
Maximum, plants of the grass family, such as cereals. would have been a more reliable source of food than woodland
plants (e.g. nuts and berries. Many of these woodland plants would have died out as the weather worsened, and
edible animals would have also become increasingly unavailable as they migrated to warmer climates, leaving
cereals as one of the few remaining options for the people who chose. or were obliged. to remain in the Middle East.

Q7:According to paragraph 4, why does grinding cereals make them more nutritious?
A. Grinding allows mixing of the grain with different types of nutrients in the process of making cakes, biscuits, and
bread.
B. Grinding removes certain types of indigestible starches from the grain, which makes the protein in it easier to
absorb.
C. Grinding breaks starches and proteins down into micronutrients that are otherwise unavailable in the grain. 、
D. Grinding exposes larger part of the grains's nutrients to digestive enzymes.

Q8:According to paragraph 4, why did cereal is increase in importance for people in the Middle East during the Last
Glacial Maximum?
A. Large animal populations migrated into the area and consumed most of the other available food options.
B. Woodland plants and animals died or moved to other areas as the climate cooled.
C. The woodland plants that became common in the Middle East were difficult for humans to digest.
D. During periods of cold weather, humans have a greater need for the micronutrients contained in cereals.

Q9:Look at the four squares[ ⬛]that indicate where the following sentence could be added to the passage Where
would the sentence best fit?
Not only are cereal grains nutritious, but they can also grow under conditions in which other plants cannot.
Click on a square [⬛] to add the sentence to the passage. To select a different location, click on a different square.

Q10: Drag your answer choices to the spaces where they belong. To remove an answer choice, click on it. To review
the passage, click VIEW TEXT

第 360 页 共
373 页
2021 托福阅读真题

Humans in the Middle East may have begun to grind grain 50,000 years ago or earlier.

A. By observing patterns of tooth wear researchers have


B. Grinding grain allowed people who had lost teeth to eat it, and, determined when human groups in Africa, Asia,
and the Middle especially if the right kind of grinding stone was used, teeth East began to grind grain. remained
healthy longer.
C. Grain provided a relatively reliable source of food in a cold climate, and grinding grain improved its taste and
made it easier to digest.
D. The development of grinding technology freed many individuals from hunting and allowed them to develop oral
culture and arts such as pottery
E. People who kept their teeth lived longer and befitted their society by caring for children and transmitting
knowledge.
F. Woodland plants are less nutritious than cereals are, and humans in the Middle East began to eat large amounts
of grain as soon as climate conditions became suitable for its growth.

第 361 页 共
373 页
2021 托福阅读真题

Origins of Earth's Salty Oceans

Paragraph 1: Scientists have long been interested in discovering the origin of Earth's water and establishing why
Earth's oceans are so salty There has been speculation that earliest Earth was so hot that no liquid water existed, and
all light elements (such as hydrogen and oxygen) were rapidly stripped away from Earth by the solar wind (a stream
of charged particles emitted by the Sun). If this were true, then the elements needed to form water on Earth would
not have been freely available. As a consequence, it was proposed that collisions with icy comets or similar gas-and
water-rich materials brought water to Earth after the planet had sufficiently cooled to retain it. This concept was
supported by comparisons of the gas compositions of meteorites with those of rocks from beneath Earth's surface,
notably using krypton and xenon, nonmetallic gases that do. not react with other materials. There certainly is enough
ice in space to have supplied our water (and atmosphere)in this manner.

1. The word "notably" in the passage is closest in meaning to


A. simply
B. expertly
C. in particular
D. by necessity

2. According to paragraph 1 the proposal that Earth water was likely related to comets depends on which TWO of
the following beliefs? To receive credit, you must select TWO answer choices.
A. Earth was too hot for water to exist.
B. Earth had cooled too rapidly for the presence of the elements needed for water.
C. The elements that made up meteorites and rocks could not have produced water.
D. Elements required for water to form were stripped away by solar wind.

Paragraph 2: In July 2015, the space probe Philae. which landed on comet Churi, discovered not only ice and dust,
but also 16 types of organic compounds, present not in a loose distribution but in discrete clumps. Suddenly, the idea
gained lots of traction that comets brought not only water, but also the ingredients for life, even in ready-made
clumps. Intriguingly, in October 2015 it was reported. that-as this comet slowly thaws-molecular oxygen(02) escapes
in a constant and high proportion(1% to 10%) relative to water which suggests that the comet also contains a
surprising amount of primordial (ancient) oxygen, which was incorporated during the comet's formation.

3. The phrase "gained lots of traction" in the passage is closest in meaning to


A. was quickly resisted
B. became much more popular
C. was frequently suggested
D. created much argument

第 362 页 共
373 页
2021 托福阅读真题

4. According to paragraph 2. the space probe Philae landing on Churi led to all of the following findings EXCEPT:
A. Ice, dust and a large variety of organic compounds are present on the comet.
B. There are distinct clumps of organic compounds necessary for life on the comet.
C. The oxygen contained in the comet likely has been there since very ancient times.
D. The comet is able to keep large proportions of its oxygen from escaping as it thaws.

Paragraph 3: Other work favors an alternative explanation. This work found that the hydrogen isotope ratio(the
proportion of different forms of hydrogen) of ice in comets may be different from that of water on Earth. It instead
emphasizes that the chemical composition of water on Earth resembles that of the small percentage of water
contained within rocky meteorites, and thus in asteroids, which are essentially very large meteorites. Thus, a theory
was developed that the asteroids planetesimals. and protoplanets that clumped together to form Earth had carried
enough water in their rock minerals to explain our oceans. It would have escaped from the planet's interior as steam,
which in turn would have condensed into water at the surface and in the early atmosphere. Calculations indicate that
this mechanism can also provide plenty of water to explain Earth's observed water content.

5. Why does the author discuss "the hydrogen isotope ratio" of ice in comets?
A. To provide evidence to challenge the idea that icy comets were the source of Earth's water
B. To suggest that ice in comets underwent significant changes after reaching Earth
C. To support the claim that asteroids are large meteorites
D. To explain how water from icy comets resembles that found in meteorites

6. The explanation in paragraph 3 suggests which of the following about Earth's water?
A. Water in Earth's early atmosphere later became trapped in rock minerals.
B. Water on Earth originated in ice on meteorites that turned into steam as the meteorites fell through Earth's
atmosphere.
C. Water present on Earth today was inside Earth when the planet formed.
D. Water escaped from the early atmosphere when meteorites and asteroids collided with Earth.

Paragraph 4: We have a more complete understanding of the origin of salt in our oceans. It represents an
accumulation of dissolved minerals over tens of millions to hundreds of millions of years. These minerals were
broken up and dissolved during chemical weathering We are all familiar with this process from limestone buildings
that become pitted or smoothed by the action of water, wind, and weather; this is where the term weathering comes
from. The key process at work is one of chemical reactions between the rock and the water, with an important role
for gases that are dissolved in water, such as carbon dioxide or sulfur dioxide, since these make the water more
corrosive. The chemical weathering reactions break up rock minerals into charged atoms or molecules, called ions,
which are removed in solution by river water and groundwater. This is exactly what happens when you dissolve table
salt in water: the mineral salt breaks down into sodium and chloride ions that are held in a solution.

第 363 页 共
373 页
2021 托福阅读真题

7. Paragraph 4 suggests which of the following about the process by which limestone buildings become pitted or
smoothed?
A. lt occurs because of the presence of salt in limestone
B. lt causes the stones that the buildings are made of to release dissolved salt.
C. lt generally occurs in structures that were built using corrosive materials.
D. lt is the same process by which minerals made the oceans salty.

Paragraph 5: The early atmosphere contained high levels of carbon dioxide, or CO2. This gas is easily dissolved in
water, forming a mildly acid solution. In the CO2-rich early atmosphere, this resulted in a corrosive acid rain that
was highly effective at chemically weathering rocks, and fresh volcanic rocks are especially easily weathered.The
intense weathering released dissolved minerals in the form of ions into river water and groundwater. From early
times onward, river and groundwater flow has transported the dissolved minerals to their final collection point, the
ocean basins. Given the extremely slow input and removal of salts, it becomes clear that the oceans' vast store of salt
has accumulated because the oceans have for ages been the end station for salt transport. Meanwhile water itself
continually evaporates from the oceans-concentrating- its salts-and- the evaporated fresh water continues the
weathering cycle.

8. According to paragraph 5, high CO2 levels in the early atmosphere contributed to which of the following?
A. Increased mineral levels in river water and groundwater
B. Increased numbers of fresh volcanic rocks
C. Reduced flow of river water and groundwater into the oceans
D. Decreased rates of chemical weathering

Paragraph 3: Other work favors an alternative explanation. This work found that the hydrogen isotope ratio(the
proportion of different forms of hydrogen) of ice in comets may be different from that of water on Earth. It instead
emphasizes that the chemical composition of water on Earth resembles that of the small percentage of water
contained within rocky meteorites, and thus in asteroids, which are essentially very large meteorites. ⬛Thus, a
theory was developed that the asteroids planetesimals. and protoplanets that clumped together to form Earth had
carried enough water in their rock minerals to explain our oceans.⬛It would have escaped from the planet's interior
as steam, which in turn would have condensed into water at the surface and in the early atmosphere.⬛ Calculations
indicate that this mechanism can also provide plenty of water to explain Earth's observed water content. ⬛

9. Look at the four squares [■] that indicate where the following sentence could be added to the passage.
Although this evidence is promising, scientists are still uncertain which explanation for the oriain of Earth's
water is correct.
Where would the sentence best fit? Click on a square [■] to add the sentence to the passage.

第 364 页 共
373 页
2021 托福阅读真题

10. Directions: An introductory sentence for a brief summary of the passage is provided below. Complete the
summary by selecting the 3 answer choices that express the most important ideas in the passage. Some sentences do
not belong in the summary because they express ideas that are not presented in the passage or are minor ideas in the
passage. This question is worth 2 points.

A. Some evidence suggests that Earth's water was brought to Earth by icy comets or similar objects that collided
with Earth.
B. Earth's water may have originated in the rock minerals from which Earth formed, been released from the rock as
steam, and collected in the atmosphere and oceans.
C. Salt in Earth's oceans is a result of chemical weathering of rocks and the continuous transport of ions into the
oceans.
D. The presence of ice and organic compounds on an ancient comet. visited by a space probe demonstrated that
water is likely to have existed on Earth from the time it was formed.
E. The presence of salt in early Earth's ocean basins has significantly influenced the accumulation and evaporation
pattern of ocean water.
F. Earth had more salt in its oceans in its early history. when the action of wind and water was more intense.

第 365 页 共
373 页
2021 托福阅读真题

Characteristics of Sixteenth-Century European Towns

Paragraph 1: Size, legal status, or presence of fortifications (walls or other defensive barriers)might seem natural
criteria in defining the distinction between town and village in sixteenth-century- Europe. Ultimately, however, it
was none of these, but rather "urban" functions that distinguished even the smallest towns from villages. Some
villages could be relatively large. Some had their own walls. On the other hand, many towns were unfortified and
lacked legal status as a town.

1. In paragraph 1, why does the author mention that some villages in sixteenth-century Europe could be relatively
large?
A. To help explain why some villages needed their own fortifications as much as towns did
B. To show why towns cannot be distinguished from villages on the basis of size
C. To suggest that the distinction between villages and towns is an artificial one
D. To argue that legal status is a better criterion for distinguishing between towns and villages than size is

Paragraph 2: Early modern towns were multifunctional. Whether they were large or small this was what they
shared. and this in turn is what distinguished them most from rural settlements. Not all of these functions were
economic, but the economic functions were the foundations upon which all others were constructed. Only newly
established military towns, such as Palmanova in northeastern Italy or Neuf-Brisach on the Franco-Imperial border,
and the occasional ecclesiastical center were an exception to this pattern. Without the money and demographic
momentum generated by economic activity, towns were not selected as the location for administrative centers, law
courts, capital cities, colleges and universities. Cathedrals, or the sites for religious orders. They might be
transformed in the process, as were Madrid, when it was chosen definitively by Philip ll in 1561 as the capital of
Spain, and Weilburg in Hesse, which was reconstructed for a similar purpose in the late seventeenth century by
Count Johann Ernst of Nassau, but the preconditions for growth were already there.

2. The word "momentum" in the passage is closest in meaning to


A. Foundation
B. Stability
C. Push
D. Diversity

3. According to paragraph 2, what made Palmanova and Neuf-Brisach exceptions to the usual pattern for
establishing towns?
A. They were both ecclesiastical centers rather than economic centers.
B. Their foundations were military rather than economic.
C. They could not be clearly distinguished from rural settlements.

第 366 页 共
373 页
2021 托福阅读真题

D. Neither was selected as the location for an administrative center.

4. According to paragraph 2, for a town to be chosen to fulfill an important function, such as being the site for a
cathedral, it had to
A. be long-established
B. be an administrative center
C. have significant economic activity
D. have a central location

Paragraph 3: Five economic functions distinguished towns from the countryside: their location as centers for
exchange; the presence of artisans; occupational diversity; regular links with other centers of exchange; and
influence over a hinterland, or rural area far from any town. These distinguishing characteristics were then reinforced.
by social and cultural indicators, such as more complex forms of government, the presence of a stratified society
with an identifiable elite, the presence of processionals, such as lawyers or schoolteachers. and of religious orders
and educational establishments. As time passed, new cultural indicators were added, such as discussion groups and
charitable societies.

5. According to paragraph 3, each of the following is a social or cultural indicator of an early modern town
EXCEPT:
A. a decrease in the power of local government
B. professionals among the permanent population
C. the presence of religious orders
D. clear divisions in social class

Paragraph 4: By using these urban characteristics and indicators, it is possible to suggest that centers of all sizes in
western Europe shared common urban identity. Consider two Italian urban centers: Venice and the Sicilian town of
Gangi, which, while no quite at the opposite ends of the urban spectrum, are usually considered belong to totally
different spheres. Gangi had a population of some 4,000 in the mid-sixteenth century. Two-thirds of its inhabitants
were engaged in agriculture. The others were artisans shopkeepers, merchants, servants, rentiers (people whose
earnings are derived from rent on properties), and members of religious orders. At the height of its prosperity at the
end of the sixteenth century. on the other hand. Venice had a population of 190,000. It was a major international
trading center, supported by a substantial industrial sector, and was the capital of an extensive maritime and
land-based empire. It had a significant number of merchants, renters, and professionals among its permanent
population. As a point of departure for pilgrims and a point of arrival for tourists, it also functioned as a center of
hospitality. Servants and others engaged in the business of food, drink, and hotels swelled the already large numbers
who worked in the households of the permanent residents. In spite of these substantial contrasts both Gangi and
Venice shared the basic economic functions and the social and cultural indicators that have already been discussed.

第 367 页 共
373 页
2021 托福阅读真题

Both functioned above all as centers of exchange, which linked their rural hinterlands to long-distance trading
networks. Gangi lay on the old Roman road between Palermo and Messina and was an important center for the
export of livestock and wine. Venice's strategic location at the head of the Adriatic Sea enabled it to link the eastern
and western Mediterranean with the producers and consumers of central Europe. It was also a center of exchange for
goods emanating from northern Italy. Each town had its important public buildings and churches, a central square,
and large houses belonging to an elite. Each had its cultural identity. Comparisons of similar kind could be made
from any region of Europe.

6. The phrase "emanating from" in the passage is closest in meaning to


A. going to
B. originating in
C. transported through
D. sold in

7. Paragraph 4 suggests which of the following about Venice as an urban center?


A. Its physical layout was very similar to that of Gangi.
B. Its population was not primarily involved in agriculture.
C. It originally became prosperous from providing hospitality to pilgrims and tourists.
D. Its land-based empire developed as a direct result of its maritime empire.

8. According to paragraph 4, which of the following was factor that contributed to Gangi's becoming an urban
center?
A. Its status as a base
B. Its role as a destination
C. Its links to central-Europe
D. Its location as land-based- connection for trading exchange

Paragraph 1: Size, legal status, or presence of fortifications(walls or other defensive barriers)might seem natural
criteria in defining the distinction between town and village in sixteenth-century- Europe. ■ Ultimately, however, it
was none of these, but rather "urban" functions that distinguished even the smallest towns from villages. ■ Some
villages could be relatively large. ■ Some had their own walls. On the other hand, many towns were unfortified and
lacked legal status as a town. ■

9.Look at the four squares [■] that indicate where the following sentence could be added to the passage.
Stratford-upon-Avon, for example, was a fully-fledged town long before it was legally recognized as one.
Where would the sentence best fit? Click on a square to add the sentence to the passage.

第 368 页 共
373 页
2021 托福阅读真题

10. Directions: An introductory sentence for a brief summary of the passage is provided below. Complete the
summary by selecting the THREE answer choices that express the most important ideas in the passage. Some
sentences do not belong in the summary because the express ideas that are not presented in the passage or are minor
ideas in the passage. This question is worth 2 points.

Towns and villages in sixteenth-century western Europe cannot be distinguished by seemingly obvious factors
such as legal status.

A. What primarily distinguished towns from villages was that towns were centers for exchange with links to other
such centers and that they had occupational diversity, artisans, and influence over a hinterland.
B. Ecclesiastical centers, which had the social and cultural indicators characteristic of towns without being centers of
exchange, went against the rule that economic functions were the basis for all others.
C. A consideration of Venice and Gangi suggests the important role played by the hospitality industry in a town's
ability to function as a center of population and commerce.
D. Because many towns were unfortified and thus had no legal status as towns, military towns that lacked certain
basic economic functions were often selected for administrative centers and law courts.
E. Besides the defining economic functions, towns had other characteristics such as stratified societies, complex
forms of government, and religious and educational establishments.
F. A comparison of Venice and Gangi supports the idea that centers of all sizes in sixteenth-century western Europe
shared a common urban identity of basic economic functions and social and cultural indicators.

第 369 页 共
373 页
2021 托福阅读真题

The Meaning of Upper Paleolithic Art

Paragraph 1: The period beginning 40,000 years ago (the Upper Paleolithic) witnessed a marked increase in human
artistic and symbolic expression. At about this time, a large number of statues carved from bone or stone begin to
appear in the archaeological record, as do magnificent paintings of animals that were hunted and animals that were
not, as well as other images on Cave walls and ceilings. It is difficult for modern viewers to remain unmoved by
these images, but what did these works mean to their creators and why did they create them?

1 According to paragraph 1, which of the following happened about 40.000 years ago?
O People started living in caves in Europe.
O Bone and stone tools began to appear in the archaeological record.
O Humans developed new hunting methods.
O People started to create more works of art.

Paragraph 2: Some researchers regard Paleolithic artwork as part of a system of communication of ideas-a system
that uses animals and geometric patterns as symbols, the specific meaning of which may be lost forever.
Anthropologist Meg Conkey views the 1,200 bones engraved with abstract geometric patterns at Altamira Cave,
Spain, as the identifying symbols-the "flags"-of different groups of people who came together at the cave during
certain periods. Archaeologist Michael Joachim views the cave paintings of northern Spain and southern France (the
so-called Franco- Cantabrian region) as symbols marking territory. Social stresses that accompanied the population
influx into the region during the period beginning 25,000 years ago may have resulted in the need to mark territory
with symbols of ownership. Painting animals-probably the most important resources of a territory-within a
sacred place in the territory, like a cave, might have served to announce to intruders the rightful ownership of
the surrounding lands. Archaeologist Clive Gamble views the small stone statues of female figures, known as
Venus figurines, as a symbolic social glue, helping to maintain social connections between geographically distant
groups through a common religion and art style.

2. The word "influx” in the passage is closest in meaning to


O an arrival in large numbers
O a change in the way something is organized
O a state of uncertainty
O a series of conflicts

3. Which of the sentences below best expresses the essential information in the highlighted sentence in the passage?
Incorrect choices change the meaning in important ways or leave out essential information.
O Painting animals in a particular cave may have marked it as sacred to the owners.
O Animal paintings may have been one of the most important resources of a territory

第 370 页 共
373 页
2021 托福阅读真题

O Animal paintings in sacred places may have marked a claim on the surrounding land and its resources.
O Only the rightful owners of a territory could produce paintings in its sacred places.

4. Paragraph 2 suggests which of the following about the small stone statues known as Venus figurines?
O Some of the statues represent males and some represent females.
O Similar statues have been found in locations far apart from one another.
O This style of statue was first described by archaeologist Meg Conkey
O Clive Gamble believes these statues did not serve a religious unction

Paragraph 3: More recently, researchers Patricia Rice and Ann Paterson have returned to a more economic
perspective. Their statistical analysis of the numbers and kinds of animals seen on cave walls in the European Upper
Paleolithic shows interesting correlations with the collections of animal remains found at habitation sites in Spain
and France. Small, non-aggressive animals such as reindeer and red deer were important in the diet of the cave
painters and seem to have been depicted on cave walls in proportion to their economic importance. In addition,
animals whose remains are found less often at archaeological sites, but that were impressive, dangerous, and
produced large quantities of meat when they were successfully hunted, were commonly included in the artwork as
well. So it would appear that cave painters wanted to depict animals that were important food sources. However, the
relatively recently discovered Chauvet Cave contradicts this pattern, with its stunning depictions of animals not
known to have been exploited for food by Paleolithic Europeans, including carnivores like lions, bears, and panthers,
as well as woolly rhinoceroses.

5. Which of the following is NOT mentioned in paragraph 3 as an observation made by Patricia Rice and Ann
Paterson as part of their analysis of cave art?
O Cave artists frequently painted animals that they regularly used for food.
O Cave artists frequently painted impressive or large animals that were hunted only occasionally Cave art was
O likely part of an economic transaction, with artists being paid in meat or other food.
O Unaggressive animals made up an important part of the diet of Paleolithic people living in Spain and France.

6. In paragraph 3, why does the author point out that there are paintings of lions, bears, panthers, and woolly
rhinoceroses in the Chauvet Cave?
O To suggest that these animals may have been important food sources for Paleolithic Europeans
O To support the claim that Paleolithic artists mainly painted animals that were impressive and dangerous
O To provide examples of the types of animals that may have threatened Paleolithic people
O To present a piece of evidence that goes against Patricia Rice and Ann Paterson's economic interpretation of cave
art

Paragraph 4: A neuron-psychological approach has been applied by researchers J. D. Lewis-Williams and T. A.

第 371 页 共
373 页
2021 托福阅读真题

Dowson to explain at least some of the less naturalistic cave art. They note that there are six basic geometric forms
that people who are placed into an altered state of consciousness (for example, through hypnosis) under experimental
conditions report seeing: dots, wavy lines, zigzags, cross-hatching or grids, Concentric circles or U- shaped lines,
and parallel lines. Interestingly, these geometric forms are precisely those seen in some ancient cave art dating to
more than 30,000 years ago.
7 According to paragraph 4. what did J. D Lewis-Williams and T A. Dowson note about people in an altered state of
consciousness?
O People could more easily recreate ancient cave art patterns when in such a stat
O People in such a state reported feeling more personally connected to ancient cave art
O People reported seeing six basic geometric patterns while in such a state
O People in such a state preferred looking at geometric patterns to looking at organic patterns

Paragraph 5: Lewis-Williams and Dowson's approach is cross-cultural-in other words, they surveyed a wide variety
of historical and archaeological cultures, finding common images in artwork all over the world. Lewis-Williams and
Dowson point out ethnographic records of shamans (priests) who, in an attempt to communicate with spirits or see
into other worlds, fall into a trance like state by fasting, dancing, hyperventilating, going into isolation in absolute
darkness, undergoing sleep deprivation, or even ingesting natural hallucinogens. When these shamans produce an
artistic representation of what they have seen in their trances, they often include geometric shapes that are also seen
in Upper Paleolithic artwork. These images from trances are not culturally controlled but result, in part, from the
structure of the optic system itself and are therefore universal. Perhaps through sleep deprivation, staring at a
flickering fire or the ingestion of drugs, ancient shamans or priests produced these images in their own optic systems.
They then translated these images to cave walls as part of religious rituals.
8 According to the information in paragraph 5, why did Lewis-Williams and Dows on think the geometric shapes
seen in Upper Paleolithic artwork might have been created by artists who were in a trancelike state?

O The geometric images in Upper Paleolithic art are unrelated to normal everyday life.
O There are records of shamans in some cultures who produce similar geometric images after being in trancelike
states.
O The geometric images seen in Upper Paleolithic artwork are typical of images in dreams
O The geometric images seen in Upper Paleolithic artwork are specific to the culture of the Upper Paleolithic people.

Paragraph 3: More recently, researchers Patricia Rice and Ann Paterson have returned to a more economic
perspective. Their statistical analysis of the numbers and kinds of animals seen on cave walls in the European Upper
Paleolithic shows interesting correlations with the collections of animal remains found at habitation sites in Spain
and France. Small, non-aggressive animals such as reindeer and red deer were important in the diet of the cave
painters and seem to have been depicted on cave walls in proportion to their economic importance. ■ In addition,
animals whose remains are found less often at archaeological sites, but that were impressive, dangerous, and

第 372 页 共
373 页
2021 托福阅读真题

produced large quantities of meat when they were successfully hunted,were commonly included in the artwork as
well. ■ So it would appear that cave painters wanted to depict animals that were important food sources. ■ However,
the relatively recently discovered Chauvet Cave contradicts this pattern, with its stunning depictions of animals not
known to have been exploited for food by Paleolithic Europeans, including carnivores like lions, bears, and panthers,
as well as woolly rhinoceroses. ■

9. Look at the four squares[ ■ ] that indicate where the following sentence could be added to the passage
Where would the sentence best fit?
The economic explanation did not seem to apply to all sites.

10. Directions: Drag your answer choices to the spaces where they belong. To remove an answer choice, click on to
review the passage, click VIEW TEXT

It can be difficult for modern observers to determine the meaning of ancient art such as Paleolithic cave
paintings and carved figures.

A. Stone statues of women and animal figures carved from bone or horn were widely traded, suggesting that
economic ties among various groups may have been present.
B. Ancient artwork might have served primarily symbolic purposes, communicating ideas such as the ownership of
territory or shared religious and artistic practices.
C. Certain abstract geometric forms, such as wavy lines, dots, cross-hatching, grids, and concentric circles, may have
represented elements of nature such as waves, Clouds, or rocks.
D. Some cave markings may have been used to keep track of population numbers or food supplies, or they may have
been used as maps or messages.
E. Paleolithic artists may have preferentially painted animals that were significant to them either as sources of food
or as threats to their safety.
F. Some of the geometric patterns found in Paleolithic art likely result from features of the human optic system and
may re present experiences in an altered state of consciousness.

第 373 页 共
373 页

You might also like